You are on page 1of 354

GATE – SLP

EC / EE / IN / CS / ME / CE
Engineering Mathematics
Module 1 : Linear Algebra, Calculus,
Probability and Statistics

Edition : 0617/0619800

Copyright © reserved with Vidyalankar Classes and Publications.

All rights reserved. No part of this work herein should be reproduced or used either
graphically, electronically, mechanically or by recording, photocopying, taping, web
distributing or by storing in any form and retrieving without the prior written permission of
the publisher. Anybody violating this is liable to be legally prosecuted.

Pearl Centre, S.B. Marg, Dadar (W), Mumbai 400 028.


Tel. No. : 022 – 4232 4232 / 022 – 2430 63 67
web site : www.vidyalankar.org
e-mail : gate@vidyalankar.org
Dear GATE Aspirant,

The GATE program will help you to overcome the common feeling of confusion about the
exact scope of the syllabus. The course content will enable you to cover the course
smoothly in the limited time at your disposal without wasting time on unnecessary details,
without omitting any useful part.

The program provides graded questions in every topic leading you to deeper and more
intricate and basic concepts. This will help you to stimulate your own thinking and also
makes the process of learning, enjoyable and highly efficient.

The quick methods suggested to tackle questions and the practice that one has to put in
while solving the problems develops in you the required skill of tackling tricky problems
independently and confidently. We are sure you will be in a position to deal with every
and any problem most successfully.

I wish you all the best for your GATE.

Thank You

Rashmi Deshpande
Director
Vidyalankar Group of Educational Institutes
GATE Syllabus
Engineering Mathematics
Electronics & Communications (EC)
Linear Algebra: Vector space, basis, linear dependence and independence, matrix algebra, eigen values and eigen vectors,
rank, solution of linear equations – existence and uniqueness.
Calculus: Mean value theorems, theorems of integral calculus, evaluation of definite and improper integrals, partial derivatives,
maxima and minima, multiple integrals, line, surface and volume integrals, Taylor series.
Probability and Statistics: Mean, median, mode and standard deviation; combinatorial probability, probability distribution
functions - binomial, Poisson, exponential and normal; Joint and conditional probability; Correlation and regression analysis.
Electrical Engineering (EE)
Linear Algebra: Matrix Algebra, Systems of linear equations, Eigenvalues, Eigenvectors.
Calculus: Mean value theorems, Theorems of integral calculus, Evaluation of definite and improper integrals, Partial
Derivatives, Maxima and minima, Multiple integrals, Fourier series, Vector identities, Directional derivatives, Line integral,
Surface integral, Volume integral, Stokes’s theorem, Gauss’s theorem, Green’s theorem.
Probability and Statistics: Sampling theorems, Conditional probability, Mean, Median, Mode, Standard Deviation, Random
variables, Discrete and Continuous distributions, Poisson distribution, Normal distribution, Binomial distribution, Correlation
analysis, Regression analysis.

Instrumentation Engineering (IN)


Linear Algebra: Matrix algebra, systems of linear equations, Eigen values and Eigen vectors.
Calculus: Mean value theorems, theorems of integral calculus, partial derivatives, maxima and minima, multiple integrals,
Fourier series, vector identities, line, surface and volume integrals, Stokes, Gauss and Green’s theorems.
Probability and Statistics: Sampling theorems, conditional probability, mean, median, mode and standard deviation, random
variables, discrete and continuous distributions: normal, Poisson and binomial distributions.

Computer Science and Information Technology (CS)


Linear Algebra: Matrices, determinants, system of linear equations, eigenvalues and eigenvectors, LU decomposition.
Calculus: Limits, continuity and differentiability. Maxima and minima. Mean value theorem. Integration.
Probability: Random variables. Uniform, normal, exponential, poisson and binomial distributions. Mean, median, mode and
standard deviation. Conditional probability and Bayes theorem.

Mechanical Engineering (ME)


Linear Algebra: Matrix algebra, systems of linear equations, eigenvalues and eigenvectors.
Calculus: Functions of single variable, limit, continuity and differentiability, mean value theorems, indeterminate forms;
evaluation of definite and improper integrals; double and triple integrals; partial derivatives, total derivative, Taylor series (in one
and two variables), maxima and minima, Fourier series; gradient, divergence and curl, vector identities, directional derivatives,
line, surface and volume integrals, applications of Gauss, Stokes and Green’s theorems.
Probability and Statistics: Definitions of probability, sampling theorems, conditional probability; mean, median, mode and
standard deviation; random variables, binomial, Poisson and normal distributions.

Civil Engineering (CE)


Linear Algebra: Matrix algebra; Systems of linear equations; Eigen values and Eigen vectors.
Calculus: Functions of single variable; Limit, continuity and differentiability; Mean value theorems, local maxima and minima,
Taylor and Maclaurin series; Evaluation of definite and indefinite integrals, application of definite integral to obtain area and
volume; Partial derivatives; Total derivative; Gradient, Divergence and Curl, Vector identities, Directional derivatives, Line,
Surface and Volume integrals, Stokes, Gauss and Green’s theorems.
Probability and Statistics: Definitions of probability and sampling theorems; Conditional probability; Discrete Random
variables: Poisson and Binomial distributions; Continuous random variables: normal and exponential distributions; Descriptive
statistics - Mean, median, mode and standard deviation; Hypothesis testing.

For EC/EE/IN/ME/CE : Topics in Vector Calculus are covered in different module.


Pearl Centre, S.B. Marg, Dadar (W), Mumbai  400 028. Tel. 4232 4232

EC / EE / IN / CS / ME / CE
ENGINEERING MATHEMATICS
MODULE  1
INDEX
Page
Contents Topics
No.
Chapter  1 : Linear Algebra
1.1 Determinants 1
1.2 Matrices 4
1.3 Rank of a Matrix 12
Notes
1.4 System of Linear Equations 14
1.5 Eigen Values and Eigen Vectors 18
1.6 Vectors 21
Assignment  1 24
Assignment  2 27
Assignment  3 30
Assignment  4 34
Assignment  5 37
Assignments Assignment  6 40
Assignment  7 43
Assignment  8 46
Assignment  9 48
Assignment  10 51
Assignment  11 54
Test Paper  1 57
Test Paper  2 60
Test Paper  3 62
Test Papers
Test Paper  4 64
Test Paper  5 66
Test Paper  6 68
Page
Contents Topics
No.
Chapter  2 : Calculus
2.1 Function of single variable 71
2.2 Limit of a function 71
2.3 Continuity 73
2.4 Differentiability 76
2.5 Mean Value Theorems 79
2.6 Maxima and Minima 80
2.7 Integration 84
2.8 Definite Integration 93
Notes 2.9 Double Integrals 97
2.10 Triple Integrals 104
Change of Variables in Double and Triple Integrals
2.11 106
and Jacobians
2.12 Application of Integration 111
2.13 Partial and Total Derivatives 114
2.14 Taylor's Series and Maclaurin's Series 121
2.15 Fourier Series 123
List of Formulae 129
Assignment  1 134
Assignment  2 136
Assignment  3 138
Assignment  4 141
Assignments Assignment  5 144
Assignment  6 146
Assignment  7 148
Assignment  8 150
Assignment  9 152
Test Paper  1 154
Test Paper  2 156
Test Papers Test Paper  3 158
Test Paper  4 160
Test Paper  5 162
Test Paper  6 164
Page
Contents Topics
No.
Chapter  3 : Probability and Statistics
3.1 Basic Terms 166
3.2 Definition of Probability 167
3.3 Complement of an event 167
3.4 Independent Events 168
3.5 Theorems of Probability 168
3.6 Random Variables 172
Notes
3.7 Probability Distribution Function 173
3.8 Expectation(Mean), Variance and Standard Deviation 173
3.9 Standard Distributions 174
3.10 Mean, Median, Mode and Standard Deviation 178
3.11 Correlation and Regression Analysis 180
List of Formulae 181
Assignment  1 184
Assignment  2 187
Assignment  3 190
Assignments
Assignment  4 193
Assignment  5 195
Assignment  6 198
Test Paper  1 200
Test Papers Test Paper  2 202
Test Paper  3 204
Solutions  Linear Algebra
Answer Key 206
Assignment
Model Solutions 209
Answer Key 240
Test Paper
Model Solutions 241
Solutions  Calculus
Answer Key 255
Assignment
Model Solutions 258
Answer Key 291
Test Paper
Model Solutions 293
Page
Contents Topics
No.
Solutions  Probability and Statistics
Answer Key 308
Assignment
Model Solutions 310
Answer Key 332
Test Paper
Model Solutions 333
Chapter - 1 : Linear Algebra
1.1 Determinants
a b
If a, b, c and d are any four terms then the representation   is called a determinant
c d

and is denoted by D.
A determinant of order 2 is evaluated as follows:

a b
D=   = ad  bc
c d
A determinant of order 3 can be evaluated as follows:

a1 a 2 a3
b b3 b b3 b b2
D = b1 b2 b 3  = a1  2  a2  1  a 3  1 
c2 c3 c1 c3 c1 c2
c1 c2 c3

1 2 3
1 3 2 3 2 1
For example: 2 1 3 = 1   2   3  
1 2 3 2 3 1
3 1 2

= 1(2  3)  2(4  9) + 3(2  3) = 1 + 10  3 = 6

Properties of Determinants
Interchange of rows and columns (Ri  Ci)

The value of determinant is not affected by changing the rows into the corresponding
columns, and the columns into the corresponding rows. Thus
a1 a 2 a3 a 1 b1 c1
b1 b2 b3   a 2 b2 c2 
c1 c2 c3 a3 b3 c3

Identical rows and columns (Ri = Rj , Ci= Cj).


If two rows or two columns of a determinant are identical, the determinant has the value
zero. Thus,
a1 a 2 a3 a 1 a1 a 3
a1 a 2 a 3   0 b1 b1 b3   0
c1 c2 c3 c1 c1 c3

1
Vidyalankar : GATE – Engineering Mathematics

Interchange of two adjacent rows and columns (Ri  Ri+1, Ci  Ci+1)

If two adjacent rows or columns of the determinant are interchanged, the value of the
determinant so obtained is the negative of the value of the original determinant. Thus
a1 a 2 a3 b1 b 2 b3
b1 b2 b3    a1 a 2 a3 
c1 c2 c3 c1 c2 c3

Multiplication of row or column by factor [Ri (m), Cj(n)]


If the elements of any row or column are multiplied by the same factor, the value of the
determinant so obtained is equal to the value of the original determinant multiplied by that
factor. Thus,
ma1 ma 2 ma 3 a1 a 2 a3
 b1 b2 b3   m b1 b2 b3 
c1 c2 c3 c1 c2 c3

Sum of determinants
If any element in any row (or column) consists of the sum of two terms, the determinant
can be expressed as the sum of two other determinants whose other rows (or columns)
remain the same, while the remaining row (or column) consists of these terms
respectively. Thus,

a1   1 a 2 a3 a1 a 2 a3 1 a 2 a3
b1  1 b2 b3   b1 b2 b3   1 b2 b3 
c1   1 c2 c3 c1 c2 c3 1 c2 c3

Change of row or column by multiples of other rows and columns Rij(p), Cij(p)
As the consequence of the properties 5, 4 and 2 we have the result.
a1 a 2 a3 a1  pa 2 a 2  qa 3 a3
b1 b2 b3   b1  pb2 b2  qb3 b3 
c1 c2 c3 c1  pc2 c2  qc3 c3

where care must be taken to leave at least one row or column unaltered in such changes
P and q being any positive or negative factors.

2
Notes on Linear Algebra

Note : 1. Area of a triangle whose vertices are (x1, y1) (x2, y2) and (x3, y3) is
x1 y1 1
1
given by the absolute value of x 2 y 2 1
2
x3 y3 1

2. Area of a quadrilateral can be found by dividing it into two triangles.


3. If the area of a triangle obtained from the three given points is zero,
then the three points lie on a line.
 The condition for three points to be collinear is
x1 y1 1
1
x2 y 2 1 = 0
2
x3 y3 1

Solved Example 1 : Solved Example 2 :


Find the area of a triangle whose vertices Find if the three points (1, 1), (5, 7) and
are (2, 1), (4, 3), (2, 5). (8, 11) are collinear.
Solution : Solution :
The area of the triangle is If the points are collinear, area of the
x1 y1 1 2 1 1 triangle formed by the three given points
1 1
A = x 2 y2 1 =  4 3 1 should be zero.
2 2
x3 y3 1 2 5 1 1 1 1
1
1 Area A=  5 7 1
= × abs [2(3  5)  4(1  5)  2(1 + 3)] 2
2 8 11 1

1 1
= × abs [16 + 16  8] = 4 sq. units = [1(7  11)  5(1  11)
2 2
+ 8 (1  7)]
1
= [4 + 60  64] = 0
2
The given points are collinear.

3
Vidyalankar : GATE – Engineering Mathematics

1.2 Matrices
A Matrix is a rectangular array of elements written as
 a11 a12 . . . . a1n 
a a 22 . . . . a2n 
 21
 . . . . 
 
A=  . . . . 
 . . . . 
 
 . . . . 
a am2 . . . . amn 
 m1
The above matrix A has m rows and n columns. So it is a m × n matrix or it is said that
the size of the matrix is m × n.

Types of Matrices
Square Matrix :
It is a Matrix in which number of rows = number of columns
 1 2 3
For example: 4 5 6  is a square matrix of order 3.
 
7 8 9 

Diagonal Matrix :
It is a square matrix in which all non diagonal elements are zero.
 1 0 0
For example: 0 2 0 
 
0 0 4 

Scalar Matrix :
It is a diagonal matrix in which all diagonal elements are equal
4 0 0 
For example: 0 4 0 
 
0 0 4 

Unit Matrix :
It is a scalar matrix with diagonal elements as unity. It is also called Identity Matrix.
 1 0
Identity matrix of order 2 is I2 =  
0 1

4
Notes on Linear Algebra

 1 0 0
Identity matrix of order 3 is I3 = 0 1 0 
0 0 1

Note : For any matrix A, AI = IA = A

Upper Triangular Matrix :


It is a square matrix in which all the elements below the principal diagonal are zero.
 1 2 0
For example: 0 0 3 
 
0 0 2 

Lower Triangular Matrix :


It is a square matrix in which all the elements above principal diagonal are zero.
0 0 0 
For example: 2 1 0 
 
 1 2 4 

Column Matrix :
It is a matrix in which there is only one column.
1 
For example: 3 
 
1 

Row Matrix :
It is a matrix in which there is only one row.
For example: [2 3 4]

Transpose of a Matrix :
It is a matrix obtained by interchanging rows into columns
 1 3 5
For example: If A =  
2 3 8 

 1 2
A = transpose of A = 3 3 
5 8 

5
Vidyalankar : GATE – Engineering Mathematics

Symmetric Matrix :
If for a square matrix A, A = A then A is symmetric
 1 4 5
For example: 4 2 8 
 
5 8 3 

Skew Symmetric matrix :


If for a square matrix A, A = A then it is skew  symmetric matrix.
 0 5 7
For example:  5 0 3 
 
 7 3 0 

Note : For a skew symmetric matrix, diagonal elements are zero.

Orthogonal Matrix :
A square matrix A is orthogonal if AA = AA = I
 cos  sin  
For example: A=  
  sin  cos  
Here AA = I

Note : For orthogonal matrix A, A1 = A

Conjugate of a Matrix :
Let A be a complex matrix of order m  n. Then conjugate of A is the matrix obtained by

taking conjugate of every element in the matrix and denoted by A


 7  i 3  3i 4 
For example: if A= 
 9  2i i 8  4i

 7  i 3  3i 4 
then conjugate of A = A  
9  2i i 8  4i

Matrix A :
The transpose of the conjugate of a matrix A is denoted by A
 7  i 2  4i 4 
For example: Let A = 
3  2i i 1  2i

 7  i 2  4i 4 
then A  
3  2i i 1  2i

6
Notes on Linear Algebra

 7  i 3  2i

and 
A = A    2  4i i 
 4 1  2i 

Unitary Matrix :
A square matrix A is said to be unitary if A A = I
1 i 1  i 
 2 
For example: A=  2 
1 i 1 i 
 2 2 

 1 0
Here AA =   =I
0 1
Hermitian Matrix :
A square matrix A is called Hermitian matrix if aij = a ji

 4 1  i 2  5i
For example: A =  1  i 3 1  2i 
2  5i 1  2i 8 

The necessary and sufficient condition for a matrix A to be Hermitian is that A = A.

Skew  Hermitian Matrix :


A square matrix A is skew Hermitian matrix if aij =  a ji

 2i 2  8i 1  2i
For example:  (2  8i) 0 2i 

 (1  2i) 2i 4i 

The necessary and sufficient condition for a matrix A to be skewHermitian is that A = A

Note : All the diagonal elements of a skew Hermitian matrix are either zeroes or
pure imaginary.

Idempotent Matrix :
Matrix A is called idempotent matrix if A2 = A
 2 2 4 
For example: A =  1 3 4 
 1 2 3 
Here A2 = A

7
Vidyalankar : GATE – Engineering Mathematics

Periodic Matrix :
A matrix A is called a periodic matrix. AK+1 = A where K is a +ve integer; if K is the least
+ve integer for which AK+1 = A, then K is the period of A.

Note : If K = 1, we get A2 = A and it is idempotent matrix.

Nil potent Matrix :


A matrix is called a Nilpotent matrix, if AK = 0 where K is a positive integer. If K is the
least positive integer for which AK = 0, then K is the index of the nil potent matrix.
 ab b2 
For example: A=  2  has index 2
 a ab 

Involutory Matrix :
A matrix A is called involutory matrix if A2 = I
 0 1 1
For example: A = 4 3 4 
 3 3 4 

Here A2 = I

Note : I2 = I.  Identity matrix is always involutory.

Determinant of a square matrix


Let A be a square matrix, then | A | = determinant of A.
 1 2 3
For example: A = 2 1 3 
 3 1 2 

1 2 3
|A| = 2 1 3
3 1 2

1 3 2 3 2 1
= 1   2   3  
1 2 3 2 3 1

= 1(2  3)  2(4  9) + 3(2  3) = 1 + 10  3


=6
 If | A |  0 then matrix A is called as nonsingular.
 If | A | = 0, A is called singular.

8
Notes on Linear Algebra

Adjoint and Inverse of a Square Matrix


Minor : Consider the determinant
a11 a12 a13
 = a 21 a 22 a 23 
a 31 a 32 a 33

To find minor leave the row and column passing through the element aij.
a a13
The minor of the element a21 = M21 =  12 
a 32 a 33

a a13
The minor of the element a32 = M32 =  11 
a 21 a 23

a a 23
The minor of the element a11 = M11 =  22 
a 32 a 33

Cofactor : The minor Mij multiplied by (1)i+j is called the cofactor of the element aij.

a a13
The cofactor of the element a21 = A21 = (1)2+1 M21 =   12 
a 32 a 33

a a13
The cofactor of the element a32 = A32 = (1)3+2 M32 =   11 
a 21 a 23

a a 23
The cofactor of the element a11 = A11 = (1)1+1 M11 =  22 
a 32 a 33

And so on.

Adjoint of a Matrix :
Adjoint of a square matrix A is the transpose of the matrix formed by the cofactors of the
elements of the given matrix A.
 a11 a12 a13 
If A = a 21 a 22 a 23 
a 31 a 32 a 33 

 A11 A 21 A 31 
Then adj (A) =  A12 A 22 A 32 
 A13 A 23 A 33 

9
Vidyalankar : GATE – Engineering Mathematics

Inverse of a Square Matrix :


For a nonsingular square matrix A
1
A1 = adj(A)
|A|

where A1 is called the inverse of square matrix.

Note : A A1 = A1 A = I

Solved Example 3 : A31 = the cofactor of a31


Calculate the adjoint of A, 1 1
in | A | =   = 5
1 1 1  2 3
where A =  1 2 3 
A32 = the cofactor of a32 in
 2 1 3 
1 1
Solution : |A|=    =4
1 3
A11 = the cofactor of a11 in
A33 = the cofactor of a33 in
2 3
|A|=   =3 1 1
1 3 | A |=   =1
1 2
A12 = the cofactor of a12 in
Adj (A) = transpose of the matrix formed
1 3 by cofactor
|A|=   = 9
2 3  A11 A 21 A 31   3 4 5 
=  A12 A 22 A 32    9 1 4 
A13 = the cofactor of a13 in
 A13 A 23 A 33   5 3 1 
1 2
|A|=   = 5
2 1 Solved Example 4 :
A21 = the cofactor of a21 in Find the inverse of the matrix by finding its
1 1 1 3 3
| A |=    = 4
1 3 adjoint where A = 1 4 3 
1 3 4 
A22 = the cofactor of a22 in
Solution :
1 1
| A |=   =1 |A| = 1  0
2 3
A1 exists
A23 = the cofactor of a23 in
 1 1 1
1 1
| A |=    =3 Now A = 3 4 3 
2 1
 3 3 4 

10
Notes on Linear Algebra

The cofactors of the elements of A are  150 98 18 


(1) = 7 ; (1) = 3 ; (1) = 3 (adj B) (adj A) =  80 70 10  ,
 15 14 1 
(3) = 1 ; (4) = 1 ; (3) = 0
(3) = 1 ; (3) = 0 ; (4) = 1  150 98 18 
adj (AB) =  80 70 10 
 7 3 3 
 Adj (A) =  1 1 0   15 14 1 

 1 0 1  Hence we verify (adj B) (adj A) = adj (AB)

 7 3 3 
1
 A = 1
adj(A)   1 1 0  Solved Example 6 :
A
 1 0 1  Find the inverse of the matrix finding its
2 1 3 
Solved Example 5 : adjoint where A = 3 1 2 
 1 2 3 
Find the adjoints of the matrices A and B
Solution :
1 2 3 
where A = 1 3 4  , 2 1 3 
1 4 3  | A | = 3 1 2  = 6  0  A1 exists
 1 2 3 
 0 4 5
B =  1 2 3  .  2 3 1
 1 1 7  transpose of A = A =  1 1 2 
 3 2 3 
Verify the formula adj(AB) = (adj B ) (adj A)
Solution : The cofactors of the elements of A are
We can find that (2) = 1 ; (3) = 3 ; (1) = 1

 7 6 1 (1) = 7 ; (1) = 3 ; (2) = 5


adj A =  1 0 1 , (3) = 5 ; (2) = 3 ; (3) = 1
 1 2 1 
 1 3 1
17 33 2  adj (A) =  7 3 5 
adj B = 10 5 5   5 3 1
 1 4 4 
1
 A1 = adj(A)
 1 5 32  |A |
AB =  1 6 42   1 3 1
 1 9 38  1 
=  7 3 5 
6
 5 3 1

11
Vidyalankar : GATE – Engineering Mathematics

Solved Example 7 : Solved Example 8 :


Show that Show that
 cos  sin    8 4 1 
A=   is orthogonal 1
  sin  cos  A =  1 4 8  is orthogonal and
9
 4 7 4 
Solution :
find A1.
cos   sin  
A =   Solution :
 sin  cos  
 8 4 1   8 1 4 
 cos  sin   cos   sin   1  1
AA =     AA =  1 4 8  4 4 7 
  sin  cos    sin  cos   9 9
 4 7 4   1 8 4 
 cos 2   sin2  0 
=  2 2 
 81 0 0   1 0 0 
0 sin   cos  1
  = 0 81 0   0 1 0  = I
81 
 1 0  0 0 81 0 0 1
=   =I
0 1 Similarly A A = I
Similarly A A = I  A is orthogonal and therefore
Hence AA= AA = I
 8 1 4 
1
 A is orthogonal A = A =  4 4 7 
1
9
 1 8 4 

1.3 Rank of a Matrix


Sub-matrix
Any matrix obtained by omitting some rows and columns form a given m  n matrix A is
called a sub-matrix of A.

 a1 b1 c1 
For example: a b contains
 2 2 c 2 

 a b1   a1 c1   b1 c1 
three 2 x 2 sub-matrices  1 b

 a 2 b2 
a
 2 c 2   2 c 3 

two 1  3 sub matrices namely [a1 b1 c1] and [a2 b2 c2] and

 a1  b1  c1 
three 2  1 sub matrix namely       and so on
 a 2  b2  c 2 
The rank of a matrix is r if :
i) It has atleast one non-zero minor of order r
ii) Every minor of A of order higher than r is zero

12
Notes on Linear Algebra

The rank of a matrix in Row-Echelon form is equal to the number of non-zero rows.
The rank of a matrix is also given by the number of linearly independent rows.

Note : 1. If A is zero matrix, then r(A) = 0


2. IF A is not a zero matrix, r(A)  1
3. IF A is a nonsingular n  n matrix then r(A) = n (  | A |  0)
4. r(In) = n
5. If A is an m  n matrix then r(A)  minimum of m and n

Solved Example 9 : Solved Example 10 :


4 2 1 3  Find the rank of the matrix
Find the rank of the matrix 6 3 4 7  .  3 2 1 5 
 2 1 0 1 A = 5 1 4 2  .
Solution :  1 4 11 19 

This matrix contains four 3  3 matrices. Solution :

 4 2 1 4 1 3  The matrix contains four 3  3 matrices


 6 3 4 6 4 7 
     3 2 1 3 1 5 
 2 1 0  2 0 1  5 1 4 5 4 2 
   
 1 4 11  1 11 19 
4 2 3  2 1 3 
 6 3 7 3 4 7 
    3 2 5   2 1 5 
 2 1 1  1 0 1  5 1 2  1 4 2 
   
The determinants of all these are zero.  1 4 19   4 11 19 

Then consider 2nd order sub matrices. It The determinants of all these are zero.
2 1 Then consider 2nd order sub matrices.
can be seen that   has determinant
3 4  3 2 
It can be seen that   has a value 7
whose value is 5. i.e. not zero. Hence the  5 1
rank of the matrix is 2. i.e. not zero.
Hence the rank of the matrix is 2.

13
Vidyalankar : GATE – Engineering Mathematics

1.4 System of Linear Equations


Consider a set of equations
a1x + b1y + c1z = d1
a2x + b2y + c2z = d2
a3x + b3y + c3 z = d3

The equations can be written in the matrix form as

 a1 b1 c1   x  d1 
a b c 2   y   d 
 2 2    2
a 3 b3 c 3  z  d3 
which is of the form AX = B
 a1 b1 c1 : d1 
Augmented matrix, [A : B] = a 2 b2 c 2 : d2 
a 3 b3 c 3 : d3 

Consistency conditions
After reducing [A : B] to Row-Echelon form, find the ranks of A and [A : B]

Case 1 : r(A)  r (A : D), then the system is inconsistent.


i.e. it has no solution.

Case 2 : r(A : D) = r(A) then the system is consistent and if


(i) r(A : D) = r(A) = Number of unknowns then the system is consistent
and has unique solution.
(ii) r(A : D) = r(A) < Number of unknowns then the system is consistent
and has infinitely many solutions.

Solution of Linear Equations


Cramer’s Rule
The solutions of the equations
a1x + b1y + c1z = d1
a2x + b2y + c2z = d2
a3x + b3y + c3z = d3

14
Notes on Linear Algebra

a1 b1 c1
Let  = a 2 b2 c2 
a 3 b3 c3

The solution is given by


x y z 1
  
d1 b1 c1 a1 d1 c1 a1 b1 d1 a1 b1 c1
d2 b2 c2  a 2 d2 c2  a 2 b2 d2  a 2 b2 c2 
d3 b3 c3 a3 d3 c3 a3 b3 d3 a3 b3 c3

x y z 1
   [  0]
x y z 

i.e.
x y 
x= ,y= ,z= z
  

Method of Inversion
If the matrix form of the given equations is AX = B and if A  0, then the solution can be

obtained as

X = A1B

Non Zero Solutions of Linear Homogenous Equation


The homogenous equation in x, y, z are
a1x + b1y + c1z = 0
a2x + b2y + c2z = 0
a3x + b3y + c3z = 0
A system of simultaneous linear equations is said to have zero or trivial solutions if all the
unknowns have zero values, and is said to have nonzero solution if at least one of the
unknowns has the nonzero value.
The necessary condition that the equations have nonzero solutions is
a1 b1 c1
a 2 b2 c2  = 0
a3 b3 c3

15
Vidyalankar : GATE – Engineering Mathematics

Solved Example 11 : 3 1 5
Solve 3x + y = 19 y = 2 6 0
3x  y = 23 1 5 1

Solution : = 3(6  0) + 1(2  0)  5(10  6)


3 1 = 18 + 2  20 = 0
Here =   = 6
3 1 3 0 1
By Cramer’s rule z = 2 7 6 
1 1 5
19 1
 
x 23 1 = 3 (35  6)  0 (10  6)  1(2  7)
x= 
 3 1 = 87  0 + 5 = 92
 
3 1
x 138
Now x=  =3
19  23  46
= =7
3  3
y 0
y=  =0
3 19  46
 
y 3 23 69  57 z 92
and y =   = 2 z=  =2
 3 1 3  3  46
 
3 1

Solved Example 12 : Solved Example 13 :


Solve 3x  5z = 1 Test whether the following equations have
2x + 7y = 6 nonzero solution. If they have such
x+y+z=5 solution obtain the solutions.
Solution : x + y  3z = 0, 3x  y  z = 0,
3 0 5 2x + y  4z = 0
Here  = 2 7 0 Solution :
1 1 1
1 13
= 3(7  0)  0(2  0)  5(2  7) Now 3 1 1 = 0
= 21 0 + 25 = 46 2 1 4

1 0 5 and hence the equations have nonzero


x =  6 7 0 solution. Solving the first two equations
5 1 1 we get
= 1(7  0)  0(6  0)  5(6  35) x y z
 
= 7  0 + 145 = 138 4 8 4

16
Notes on Linear Algebra

so that x = 4, y =  8, z = 4, where  1 1 1


is a non-zero constant. These values Rank (A) = 2 since 2 4 3 = 0
satisfy the third equation and hence they 3 5 2

are non-zero solutions. 1 1


and   =2 0
2 4
Solved Example 14 :
Rank [A : B] = 3
Test whether the following equations have
1 1 1
non zero solution
since 4 3 9  = 7  0
2x + 3y + 4z = 0
5 2 11
x  2y  3z = 0
r(A)  r(A : B) thus the system is
3x + y  8z = 0
inconsistent i.e. it has no solution.
Solution :
2 3 4
Solved Example 16 :
Now 1 2 3 = 63  0
3 1 8 Examine for consistency
3x + y + 2z = 3
Hence the equations have no nonzero
2x  3y z = 3
solution. The only solution is x = 0, y = 0,
x + 2y + z = 4
z=0
Solution :
Solved Example 15 : In the matrix form
Discuss the consistency of
3 1 2   x  3 
x+y+z = 1 2 3 1  y    3 
     
2x + 4y  3z = 9  1 2 1  z  4 
3x + 5y  2z = 11
3 1 2 
Solution : A = 2 3 1
In the matrix form  1 2 1 

1 1 1  Rank of A = r(A) = 3 since | A | = 8  0


A = 2 4 3  and
3 1 2 3 
3 5 2 
[A : B] = 2 3 1 3 
1 1 1 1   1 2 1 4 
[A, B] = 2 4 3 9  Similarly r(A : B) is also 3.
3 5 2 11
r(A) = r(A : B)
 System is consistent and has unique
solution.

17
Vidyalankar : GATE – Engineering Mathematics

Solved Example 17 : 5 3 7 : 4 
Examine the consistency of [A : B] = 3 26 2 : 9 
5x + 3y + 7z = 4 7 2 10 : 5 

3x + 26y + 2z = 9 [A : B] contains four 3 × 3 sub matrices:


7x + 2y + 10z = 5 5 3 7  5 7 4 
3 26 2 3 2 9 
Solution :    
In the matrix form 7 2 10  7 10 5 

5 3 7   x  4  5 3 4   3 7 4
3 26 2  y   9  3 26 9 26 2 9 
        
7 2 10  z  5  7 2 5   2 10 5 

5 3 7  All the above 4 sub matrices have


A = 3 26 2  determinant value 0
7 2 10 
 r (A) = r(A : B)
Rank of A = r(A) = 2 since | A | = 0 = 2 < 3 (number of unknowns)
5 3  System is consistent and has infinitely
and   = 121  0
3 26 many solutions.

1.5 Eigen Values and Eigen Vectors


Characteristic Equation
Let A be an n × n square matrix. Then [A  I] is characteristic matrix of A, where I is
identity matrix.
|A  I| is characteristic polynomial.
|A  I| = 0 is characteristic equation of A.

Eigen values
The roots of the characteristic equation of a matrix are called its Eigen values.

Eigen vectors
If  is an Eigen value of A, then a non-zero vector X such that
AX = X or [A  I] [X] = 0 is called the Eigen vector of A corresponding to Eigen value .

18
Notes on Linear Algebra

Properties of Eigen values and Eigen vectors


 Sum of Eigen values of a matrix is equal to the trace of the matrix.
 Product of Eigen values of a matrix is equal to its determinant.
 Eigen values of A and AT are same.
 If A is a triangular or diagonal matrix, Eigen values are the diagonal elements.
1
 If an Eigen value of A is , then is an Eigen value of A1.

 If Eigen values of A are 1, 2….., then Eigen values of Ak are 1k ,  k2 ………

 Eigen vectors of a real symmetric matrix corresponding to different Eigen values are
orthogonal.
 If X is an Eigen vector of A corresponding to an Eigen value, the kX is also an Eigen
vector of A corresponding to Eigen value X, where k is a non-zero scalar.

Cayley  Hamilton Theorem


Every square matrix satisfies its characteristic equation i.e.,
|A  I| = 0
 n + a1n1 + a2n2 …….. + an = 0
is satisfied by A,
 An + a1An1 + a2An2 + …… anI = 0

Solved Example 18 : Instead of evaluating the determinant


1. Find the eigen values of the matrix. directly we use the formula for its

2 1 1 expansion which is as follows :


2 3 2 
  3  (sum of diagonal elements of A) 2 +
3 3 4  (sum of minors of diagonal elements of A)
Solution :   |A| = 0
Characteristic equation of A in  is 3  92 + 15  7 = 0
| A  I | = 0  (  1) (  7) (  1) = 0
2 1 1  1 = 7, 2 = 1, 3 = 1
 2 3 2  =0
3 3 4

19
Vidyalankar : GATE – Engineering Mathematics

Solved Example 19 : 0 
Find the eigen vectors for the matrix :  X2 = 0 
0 
2 1 1
2 3 2 
  But by definition we want nor zero X2. So
3 3 4 
we proceed as follows,
Solution : Expanding R1
From example (1) we get x1 + x 2 + x 3 = 0
1 = 7, 2 = 1, 3 = 1 We assure any element to be zero say x1
Matrix equation of A in  is (A  I) x = 0 and give any convenient value say 1 to x2

2 1 1 x1   0  and find x3.


 
  2 3 2   x 2   0  Let x1 = 0, x2 = 1 x3 = 1
3 3 4    x 3  0  0
x2 = 1 
Case (1) for 1 = 7, matrix equation is
1 
 5 1 1   x1  0 
     Case 3 : for 3 = 1
 2 4 2  x 2   0 
 3 3 3   x 3  0   1 1 1  x1  0 
2 2 2  x  0 
By Cramer’s rule    2  
3 3 3   x 3  0 
x1 x 2 x
   3
6 12 18 By Cramer’s rule we get

x1 x x x1 x2 x
  2  3   3
1 2 3 0 0 0

1  0 
 x1 = 2   x3 = 0 
3  0 

Case (2) for 2 = 1 Matrix equation is Again consider x1 + x2 + x3 = 0


Now let x2 = 0 x1 = 1 and x3 = 1
 1 1 1  x1  0 
2 2 2  x   0  1 
   2  
3 3 3   x 3  0   x3 = 0 
 1
By Cramer’s rule, we get
x1 x 2 x
  3
0 0 0

20
Notes on Linear Algebra

Solved Example 20 : 1 1 8
2
 1 2 1 Now A = 5 5 2
Verify whether the matrix A =  1 0 3  5 3 0
 2 1 1
16 6 12
satisfies its characteristic equation. A = 14 8 8
3

Solution : 2 10 14

1  2 1 0 0 0 
| A   I | =  1  3   A  2A + 4A  18 I = 0 0 0 
3 2

2 1 1   0 0 0 

= 3  22 + 4  18

1.6 Vectors
An ordered set of n number is called an nvector or a vector of order n.
For example: X = (x1 , x2, …..xn ) is an nvector.
The numbers x1, x2, …..xn are called as the components of X.
The components x1, x2, ……..xn of a vector may be written in a row or a column.
i.e. X = (x1, x2, x3, ………xn) and
 x1 
 
x2 
X = :  are n vectors
 
: 
x 
 n

Operation on Vectors
Inner product of two vectors
Let X = (x1, x2, ……..xn) and
Y = (y1 , y2, …….yn) be two nvectors
Then the product XY = x1y1 + x2y2 + …..+ xn yn is called the inner product of two
nvectors

Length of a vector
Let X = (x1 , x2, ……xn ) be a vector. Then the length of a vector is the positive square
root of the expression
x12  x 2 2  .........xn2

21
Vidyalankar : GATE – Engineering Mathematics

Note : Length of the vector is also called as Norm of the vector

Normal vector
A vector whose length is 1 is called a normal vector
i.e. if X = (x1 , x2, ……xn) is a normal vector then x12 + x22 + ……..xn2 = 1
If a vector is not a normal one, then it can be converted to a normal vector as follows.
Let X = (1, 3,  7) be a 3vector.

Let d = 12  32  (7)2  59

 1 3 7 
Then x   , ,  is a normal vector.
d d d 

Orthogonal vector
A vector X is said to be orthogonal to Y if the inner product of X and Y is zero i.e. XY = 0
i.e. if X = (x1, x2, …….xn) and
Y = (y1, y2, ……..yn) and if X and Y are orthogonal then
x1y1 + x2y2 + ……….+ xn yn = 0

Linear dependence and independence of vectors


A system of n vectors x1, x2, ….xn of the same order are said to be linearly dependent if
there exists n numbers K1, K2, ….Kn (where all of them are not zero) such that
K1X1 + K2X2 + …. + KnXn = 0
(where 0 is a null vector of the same order)
The vector are linearly independent only if K1 = K2 = ………Kn = 0

Solved Example 21 : 1  3  0 
Examine for linear dependence i.e. c1 2 + c2  7 
  =  0 
   
T
X1 = (1 2 4) , X2 = (3 7 10) T  4  10   0 

Solution :  c1  3c 2  0 
  2c  7c  =  0 
We have  1 2   
1  3   4c1  10c 2   0 

X1 =  2  X2 =  7  c1 + 3c2 = 0
 4  10  2c1 + 7c2 = 0
Let c1x1 + c2x2 = 0 4c1 + 10c2 = 0
(Note 0 on R. H. S. is Zero Vector) Consider first two equations in Matrix form.

22
Notes on Linear Algebra

 1 3   c1   0   c1  2c 2  0c1 2c1  c 2  3c 3
2 7  c   0   3c  3c  3c 2c1  4c 2  0c 3
  2    1 2 3
 c1  2c 2  0c 3 3c1  2c 2  4c 3
|A| = 1  0
 system has zero solution 3c1  3c 2  3c 3  0 0 0 
4c1  2c 2  6c 3  0 0 0 
 
 c1 = c 2 = 0
2c1  c 2  3c 3   0 0 0 
 X1, X2 are linear independent.
 c1 + 2c2 = 0
2c1 + c2 + 3c3 =0
Solved Example 22 :
c1 + c 2 + c 3 = 0
Show that
Solving we get
 1 2 3  2 1 3  0 3 3 
3 2 4  3 4 2  3 0 6  c1 = 2c2 and c2 = c3
     
 1 3 2   2 2 1 0 4 3  Let c2 = c3 = 1
 c1 = 2
are linearly dependent.
We see that x1 x2 x3 are linearly dependent
Solution :
Consider  1 2 3  2 1 3  0 3 3 
2  3 2 4    3 4 2   3 0 6  = 0
 1 2 3 2 1 3 
 1 3 2   2 2 1 0 4 3 
c1  3 2 4   c 2  3 4 2 
 
 1 3 2   2 2 1

0 3 3  0 0 0 
c 3  3 0 6   0 0 0 
 0 4 3  0 0 0 



23
Vidyalankar : GATE – Engineering Mathematics

Assignment  1
Duration : 45 Min. Max. Marks : 30

Q 1 to Q 10 carry one mark each (B) All the eigen values are positive.
1. Given that the determinant of the (C) All the eigen values are distinct.

 1 3 0 (D) Sum of all the eigenvalues is zero.


matrix  2 6 4  is 12, the
5. With reference to the conventional
 1 0 2 
Cartesian (x, y) coordinate system, the
 2 6 0 vertices of a triangle have the following
determinant of the matrix  4 12 8  coordinates: (x1, y1) = (1, 0); (x2, y2)
 2 0 4 
= (2, 2); and (x3, y3) = (4, 3). The area
is [ME  2014] of the triangle is equal to [CE  2014]
(A) 96 (B) 24 3 3
(A) (B)
(C) 24 (D) 96 2 4

2. Consider a 3 x 3 real symmetric matrix 4 5


(C) (D)
5 2
S such that two of its eigen values are
a  0, b  0 with respective eigen 6. At least one eigen value of a singular
matrix is [ME  2015]
 x1   y 1 
vectors  x 2  , y 2  . If a  b then x1y1 + (A) positive (B) zero
 x 3   y 3  (C) negative (D) imaginary

x2y2 + x3y3 equals [ME  2014] 7. We have a set of 3 linear equations in


(A) a (B) b 3 unknowns. 'X  Y' means X and Y
(C) ab (D) 0 are equivalent statements and 'X ≢ Y'
3. A real (4 X 4) matrix A satisfies the means X and Y are not equivalent
equation A2 = I, where I is the (4 X 4) statements. [EE  2015]
identity matrix. The positive eigen P : There is a unique solution.
value of A is ______. [EC  2014] Q : The equations are linearly
independent.
4. Which one of the following statements
R : All eigen values of the coefficient
is true for all real symmetric matrices?
matrix are nonzero.
[EE  2014]
S : The determinant of the coefficient
(A) All the eigen values are real.
matrix is nonzero.

24
Assignment on Linear Algebra

Which one of the following is TRUE? 12. A system matrix is given as follows.
(A) P  Q  R  S 0 1 1
(B) P  R ≢ Q  S A =  6 11 6 
 6 11 5 
(C) P  Q ≢ R  S
(D) P ≢ Q ≢ R ≢ S The absolute value of the ratio of the
maximum eigen value to the minimum
8. Consider a system of linear equations:
eigen value is________. [EE  2014]
x  2y + 3z = 1,
x  3y + 4z = 1, and  1 5 3 7  T
13. If A =   and B = 8 4  . AB is
2x + 4y  6z = k. [EC  2015]  6 2   
The value of k for which the system equal to [CE  2017]
has infinitely many solutions is _____. 38 28   3 40 
(A)   (B)  
32 56  42 8 
9. The larger of the two eigen values of
43 27  38 32 
(C)   (D)  
4 5  34 50  28 56 
the matrix   is ____.[CS  2015]
2 1
14. The value of x for which all the eigen
10. A real square matrix A is called skew- values of the matrix given below are
symmetric if [ME  2016] real is [EC  2015]
T T 1
(A) A = A (B) A = A 10 5  j 4 
T T 1 x 2 
(C) A = A (D) A = A + A  20
 4 2 10 

Q 11 to Q 20 carry two marks each (A) 5 + j (B) 5  j


11. The system of linear equations (C) 1  5j (D) 1 + 5j
2 1 3 a  5 
     15. The two Eigen values of the matrix
 3 0 1  b  4  has
 1 2 5   c   14  2 1
    
 1 p  have a ratio of 3:1 for p = 2.
[EC  2014]  
(A) a unique solution What is another value of p for which
(B) infinitely many solutions the Eigen values have the same ratio
(C) no solution of 3:1? [CE  2015]
(D) exactly two solutions (A) 2 (B) 1
(C) 7/3 (D) 14/3

25
Vidyalankar : GATE – Engineering Mathematics

16. Let the eigen values of a 2 x 2 matrix 19. Given that


A be 1, -2 with eigen vectors x1 and x2  5 3   1 0
A  and I , the
respectively. Then the eigen values 2 0 0 1
and eigen vectors of the matrix value of A3 is [EC, EE, IN  2012]
2
A  3A+ 4I would, respectively, be (A) 15 A + 12 I (B) 19 A + 30 I
(A) 2, 14; x1, x2 [EE  2016] (C) 17 A + 15 I (D) 17 A + 21 I
(B) 2, 14; x1 + x2, x1  x2
20. Consider the matrix
(C) 2, 0; x1, x2
(D) 2, 0; x1 + x2, x1 – x2  1 1 
 0 
 2 2
17. Consider the matrix P=  0 1 0 .
 
2 1 1  1 1 
  0
A =  2 3 4  whose eigen values  2 2 
  1 1 2 
  Which one of the following statements
are 1, 1 and 3. Then Trace of about P is INCORRECT? [ME  2017]
3 2
(A  3A ) is _______. [IN  2016] (A) Determinant of P is equal to 1
(B) P is orthogonal
18. Consider the following system of
(C) Inverse of P is equal to its
equations:
transpose
2x1 + x2 + x3 = 0,
(D) All eigen values of P are real
x2  x3 = 0,
numbers
x1 + x2 = 0.
This system has [ME  2011]
(A) a unique solution
(B) no solution
(C) infinite number of solutions
(D) five solutions



26
Assignment on Linear Algebra

Assignment  2
Duration : 45 Min. Max. Marks : 30

Q 1 to Q 10 carry one mark each  3 2 1


1. The matrix form of the linear system 4. Given the matrices J = 2 4 2  and
 1 2 6 
dx dy
= 3x  5y and = 4x + 8y is
dt dt  1
K =  2  , the product KT JK is
[ME  2014]  
 1
d x   3 5  x 
(A)    
dt y   4 8  y  _______. [CE  2014]

d x  3 8  x 
(B)    
dt y  4 5  y  5. The determinant of matrix
0 1 2 3
d x   4 5  x  1
(C)     0 3 0 
dt y   3 8  y   is ______. [CE  2014]
2 3 0 1
d x  4 8  x   
(D)     3 0 1 2
dt y  3 5  y 

6. The lowest eigen value of the 2  2


2. Which one of the following equations
4 2 
is a correct identity for arbitrary 3 x 3 matrix   is ______. [ME  2015]
 1 3
real matrices P, Q and R? [ME  2014]
(A) P(Q + R) = PQ + RP 7. For what value of p the following set of
2 2 2
(B) (P  Q) = P  2PQ + Q equations will have no solution?
(C) det (P + Q) = det P + det Q 2x + 3y = 5 [CE  2015]
2 2 2
(D) (P + Q) = P + PQ + QP + Q 3x + py = 10

3. The determinant of matrix A is 5 and 8. The value of p such that the vector

the determinant of matrix B is 40. The  1


 2  is an eigen vector of the matrix
determinant of matrix AB is  
 3 
_________. [EC  2014]
4 1 2
p 2 1  is ______. [EC  2015]

14 4 10 

27
Vidyalankar : GATE – Engineering Mathematics

9. The solution to the system of 14. Consider the following 2  2 matrix A


 2 5  x   2  where two elements are unknown and
equations       is
 4 3  y  30  are marked by a and b. The eigen
[ME  2016] values of this matrix are 1 and 7.
(A) 6, 2 (B) 6, 2 What are the values of a and b?

(C) 6, 2 (D) 6, 2  1 4


A=   [CS  2015]
b a 
10. Let M4 = I, (where I denotes the (A) a = 6, b = 4 (B) a = 4, b = 6
identity matrix) and M  I, M2  I and (C) a = 3, b = 5 (D) a = 5, b = 3

M3  I. Then, for any natural number k,


15. The number of linearly independent
M1 equals: [EC  2014]
eigen vectors of matrix
(A) M4k+1 (B) M4k+2
2 1 0 
(C) M4k+3 (D) M4k
A = 0 2 0  is ______. [ME  2016]
0 0 3 
Q 11 to Q 20 carry two marks each
11. The maximum value of the 16. Let A be a 4 × 3 real matrix with rank

determinant among all 2 x 2 real 2. Which one of the following

symmetric matrices with trace 14 is statement is TRUE? [EE  2016]


T
_______. [EC  2014] (A) Rank of A A is less than 2.
(B) Rank of AT A is equal to 2.

12. The rank of the matrix [CE  2014] (C) Rank of AT A is greater than 2.
(D) Rank of AT A can be any number
6 0 4 4 
 2 14 8 18  is _________. between 1 and 3.
 
14 14 0 10 
17. The system of equations
x+y+z=6
13. The smallest and largest eigen values
x + 4y + 6z = 20
of the following matrix are:
x + 4y + z = 
3 2 2 
4 4 6  [CE  2015] has NO solution for values of  and 
 
2 3 5  given by [EC  2011]
(A) 1.5 and 2.5 (B) 0.5 and 2.5 (A)  = 6,  = 20 (B)  = 6,   20
(C) 1.0 and 3.0 (D) 1.0 and 2.0 (C)   6,  = 20 (D)   6,   20

28
Assignment on Linear Algebra

18. Let A be the 2  2 matrix with elements 0 x  y x2  y2


a11 = a12 = a21 = +1 and a22 = 1. Then (C) 0 y  z y 2  z2 
the eigen values of the matrix A19 are 1 z z2

[CS  2012] 2 x  y x2  y2
(A) 1024 and 1024 (D) 2 y  z y 2  z2 
1 z z2
(B) 1024 2 and 1024 2

(C) 4 2 and 4 2
50 70 
(D) 512 2 and 512 2 20. Consider the matrix A =  
70 80 
whose eigen vectors corresponding to
19. Which one of the following does NOT
eigen values 1 and 2 are
equal [CS  2013]
 70    80 
1 x(x  1) x  1 x1 =   and x2 =  2 ,
(A) 1 y(y  1) y  1 1  50   70 
1 z(z  1) z  1 respectively. The value of x1T x 2 is

1 x  1 x2  1 _________. [ME  2017]


2
(B) 1 y  1 y  1
1 z  1 z2  1



29
Vidyalankar : GATE – Engineering Mathematics

Assignment  3
Duration : 45 Min. Max. Marks : 30

Q 1 to Q 10 carry one mark each 4. The sum of eigen values of the matrix,
1. One of the eigen vectors of the matrix [M] is [CE  2014]
 5 2  215 650 795 
 9 6  is [ME  2014]
  Where [M] = 655 150 835 
485 355 550 
1 2 
(A)   (B)  
 1 9 (A) 915 (B) 1355
(C) 1640 (D) 2180
2 1
(C)   (D)  
1 1 5. If any two columns of a determinant
2. For matrices of same dimension M, N 4 7 8
and scalar c, which one of these P = 3 1 5 are interchanged,
properties DOES NOT ALWAYS hold? 9 6 2

(A) (MT )T = M [EC  2014] which one of the following statements


(B) (cM)T = c(M)T regarding the value of the determinant
T T T
(C) (M + N) = M + N is CORRECT? [ME  2015]
(D) MN = NM (A) Absolute value remains
3. Given a system of equations: unchanged but sign will change
x + 2y + 2z = b1 (B) Both absolute value and sign will
5x + y + 3z = b2 change
Which of the following is true regarding (C) Absolute value will change but

its solutions? [EE  2014] sign will not change

(A) The system has a unique solution (D) Both absolute value and sign will

for any given b1 and b2 remain unchanged

(B) The system will have infinitely


6. The eigen value of the matrix
many solutions for any given b1
 1 1 5 
A = 0 5 6  are
and b2
[IN  2017]
(C) Whether or not a solution exists 0 6 5 
depends on the given b1 and b2
(A) 1, 5, 6 (B) 1, 5  j6
(D) The system would have no
(C) 1, 5  j6 (D) 1, 5, 5
solution for any values of b1 and b2

30
Assignment on Linear Algebra

 1 1 1 Q 11 to Q 20 carry two marks each


7. Let P = 2 3 4  and 11. Which one of the following statements
3 2 3  is NOT true for a square matrix A?
 1 2 1 [EC  2014]
Q =  6 12 6  be two matrices. (A) If A is upper triangular, the eigen
 5 10 5  values of A are the diagonal
Then the rank of P + Q is _________. elements of it
[CS  2017] (B) If A is real symmetric, the eigen
values of A are always real and
 1 tanx  positive
8. For A =  , the
  tanx 1  (C) If A is real, the eigen values of A
T
determinant of A A is 1
[EC  2015] and AT are always the same
(A) sec2 x (B) cos 4x (D) If all the principal minors of A are
(C) 1 (D) 0 positive, all the eigen values of A

9. Consider the following simultaneous are also positive.

equations (with c1 and c2 being


constants): 4  3i i 
12. For given matrix P =  ,
 i 4  3i
3x1 + 2x2 = c1
4x1 + x2 = c2 where i = 1 , the inverse of matrix P
The characteristic equation for these is [ME  2015]
simultaneous equations is [CE  2017] 1 4  3i i 
(A) 
2
(A)   4  5 = 0 24  i 4  3i
(B) 2  4 + 5 = 0 1  i 4  3i
(B) 
2
(C)  + 4  5 = 0 25 4  3i i 
(D) 2 + 4 + 5 = 0 1 4  3i i 
(C) 
24  i 4  3i
10. The value of for which the matrix 1 4  3i i 
(D)
3 2 4  25  i 4  3i

A= 9 7 13  has zero as an
 6 4 9  x 

eigen value is _______. [EC  2016]

31
Vidyalankar : GATE – Engineering Mathematics

13. If the characteristic polynomial of a  5 1


17. Consider the matrix   . Which
3 × 3 matrix M over  (the set of real 4 1 
numbers) is 3  42 + a + 30, a   , one of the following statements is TRUE
and one eigen value of M is 2, then the for the eigen values and eigen vectors of
largest among the absolute values of this matrix? [CE  2017]
the eigen values of M is ________. (A) Eigen value 3 has a multiplicity of
[CS  2017] 2, and only one independent eigen

14. Perform the following operations on vector exists.


(B) Eigen value 3 has a multiplicity of
 3 4 45 
the matrix  7 9 105  . 2, and two independent eigen
13 2 195  vectors exist.

(i) Add the third row to the second row. (C) Eigenvalue 3 has a multiplicity of

(ii) Subtract the third column from the 2, and no independent eigen

first column. vector exists.

The determinant of the resultant matrix (D) Eigenvalues are 3 and 3, and two

is ____________. [CS  2015] independent eigen vectors exist.

a 0 3 7
2 5 1 3  5 3 
 18. For the matrix A    , ONE of the
15. The matrix A = has  1 3
0 0 2 4
 
0 0 0 b normalized eigen vectors is given as

det (A) = 100 and trace (A) = 14. [ME  2012]

The value of a  b is ___. [EC  2016]  1   1 


  
2  2
(A)  (B) 
16. Consider the following linear system.  3  1 
   
x + 2y  3z = a  2   2
2x + 3y + 3z = b  3   1 
   
5x + 9y  6z = c 10  5
(C)  (D) 
 1   2 
This system is consistent if a, b and c    
 10   5
satisfy the equation [CE  2016]
(A) 7a  b  c = 0
(B) 3a + b  c = 0
(C) 3a  b + c = 0
(D) 7a  b + c = 0

32
Assignment on Linear Algebra

19. The minimum eigen value of the 20. The eigen values of the matrix given
below are
following matrix is
0 1 0 
3 5 2  0 0 1  [EE  2017]
 5 12 7  
  [EC  2013]  0 3 4 
 2 7 5 
(A) (0, 1, 3) (B) (0, 2, 3)
(C) (0, 2, 3) (D) (0, 1, 3)
(A) 0 (B) 1
(C) 2 (D) 3



33
Vidyalankar : GATE – Engineering Mathematics

Assignment  4
Duration : 45 Min. Max. Marks : 30

Q1 to Q6 carry one mark each 5. A matrix A has x rows and x + 5


1. A square matrix each of whose columns, matrix B has y rows and
diagonal elements are ‘1’ and 11y columns. Both AB and BA exists.
nondiagonal elements are '0’ is called Then values of x and y respectively
(A) Null matrix are
(B) Skew symmetric matrix (A) 3, 8 (B) 5, 11
(C) Identity matrix (C) 5, 11 (D) 3, 8
(D) row matrix
6. For what values of x, the matrix

2. If product of two matrices AB exists  3  x 2 2 


 2 4x 1  is singular
does BA exists ? 
 2 4 1  x 
(A) Always exist (B) May exist
(C) Never exist (D) None of these (A) 0, 3 (B) 0, 3
(C) 1, 4 (D) 3, 4
3. If a square matrix A is real and
Q7 to Q18 carry two marks each
symmetric, then the eigen values
7. Evaluate
(A) are always real
1  2
(B) are always real and positive
 =  2 1  where  is one of
(C) are always real and non-negative
2
 1 
(D) occur in complex conjugate pairs
the imaginary cube root of unity
4. If A and B are two matrices such that (A) 0 (B) 1
AB and A + B are both defined then A, (C) 2 (D) None of these
B are
8. If a given matrix [A] mn has r linearly
(A) Scalar matrices of same order
independent vectors (rows or columns)
(B) square matrices of same order
and the remaining vectors are
(C) matrices of different order
combination of these r vectors. Then
(D) cannot predict
rank of matrix is
(A) m (B) n
(C) r (D) m  n

34
Assignment on Linear Algebra

9. Are following vectors linearly dependent 8 7 9 


x1 = (3, 2, 7) x2 = (2, 4, 1) 12. The inverse of a matrix 5 10 15  is
 1 2 3 
x3 = (1, 2, 6)
(A) dependent (B) independent 15 7 8 
1
(C) can’t say (D) none of above (A) 10 5 3 
2
 5 2 1
10. If A is nonzero column matrix and B  15 7 8 
is a nonzero row matrix then rank of (B)  10 5 3 
 5 2 1
matrix AB is
(A) always 1  15 7 8 
(B) always = No. of elements in row (C)  10 5 3 
 5 2 1
(C) always = No. of elements in
column (D) none of these
(D) does not depends on rows &
columns 0 2 2
13. Rank of matrix  7 4 8  is
a h g  7 0 4 
11. If A = [x y z], B =  h b f  ,
 g f c  (A) 3 (B) 2
(C) 1 (D) none of these
x 
C =  y 
z  14. Eigen values of matrix

Then ABC =  1 0 0
A = 2 3 1
(A) ABC is not possible
0 2 4 
(B) [ax2 + by2 + cz2 + 2hxy + 2gzx
(A) 1, 2, 4 (B) 1, 3, 4
+ 2fyz]
(C) [a2x + b2y + c2z + 2abc + 2xyz (C) 1, 2, 5 (D) 1, 3, 4

+ 2fgh]
ax 2  byz  fcz2 
(D)  
2hxy  2gzx  2fyz 

35
Vidyalankar : GATE – Engineering Mathematics

15. Whether the following equations are  cos  sin  


17. If A =   &
inconsistent   sin  cos  
x + y + z = 3  sin   cos 
B=   then
3x + y  2z = 2 cos  sin  
2x + 4y + 7z = 7 cos  (A) + sin  (B) =
(A) Yes 1 0   1 0
(A)   (B)  
(B) No 0 1 0 1
(C) Can’t say
0 1  0 1
(D) Can’t be determined. (C)   (D)  
 1 0  1 0 

 5 4
 1 6 3   3 5 
16. If A =  3 2  & B =   18. If A =   & A2 = kA + 14I, then
 2 5 1  4 2 
 2 3 
k=....
& A + B  X = 0 then X =
4 2  4 2  (A) 3 (B) 5
(A) 9 7  (B) 9 7 
(C) 1 (D) 5
 1 4   1 4 

4 2  4 2 
(C) 9 7  (D) 9 7 
 1 4   1 4 



36
Assignment on Linear Algebra

Assignment  5
Duration : 45 Min. Max. Marks : 30

Q1 to Q6 carry one mark each 5. Inverse of a matrix


1. If A be any matrix, then matrix B if it (A) exist if matrix is singular
exists such that AB = BA = I, then it is (B) is unique
called A
(C)
(A) Transpose of A adj A
(B) Inverse of A (D) is not unique
(C) Cofactor matrix A
(D) Adjoint of A.  p q  r s
6. If P =   , Q =   , then
 q p   s r 
2. The eigen vector corresponding to an
PQ is equal to
eigen values are
 pr  qs ps  qr 
(A) different (B) unique (A)  
 qr  ps qs  pr 
(C) nonunique (D) none of these
 pq  rs pr  qs 
(B)  
3. Rank of matrix which is in echelon qr  ps qs  pr 

form is equal to pr  qs ps  qr 


(C)  
(A) No. of non-zero rows in the matrix qr  ps qs  pr 
(B) No. of columns in the matrix  pq  rs pr  qs 
(D)  
(C) It is independent of No. of rows  qp  rs qs  pr 
and columns
(D) Can’t say Q7 to Q18 carry two marks each
0 1 1
7. Is the matrix 4 3 4  an involutory
4. Cayley Hamilton theorem is
(A) A matrix can be expressed as sum 3 3 4 
of symmetric and skewsymmetric
matrix ?
matrices
(A) No
(B) Every square matrix satisfies its
(B) Yes
own characteristic equation.
(C) Can’t say
(C) Inverse of a matrix exists if it is
(D) Cannot be determined
singular
(D) None of these

37
Vidyalankar : GATE – Engineering Mathematics

8. The inverse of the matrix 4 2 


11. For the matrix   the eigen value
 1 2 3  2 4 
 2 1 0  is
  corresponding to the eigen vector
 4 2 5 
101
101 is
 5 4 3   
(A) 10 7 6  (A) 2 (B) 4
 8 6 5 
(C) 6 (D) 8
 5 4 3
(B) 10 7 6  12. Characteristic root of matrix
 8 6 5   1 2 5
A = 0 3 5  are
10 7 6 
0 0 2 
(C)  8 6 5 
 5 4 3  (A) 1, 3, 2 (B) 1, 3, 2
(C) 1, 2, 3 (D) 3, 1, 2
 5 4 3 
(D) 10 7 6 
1 1 1
 8 6 5 
13. The eigen values of 1 1 1 are
1 1 1
0 1 2 2 
9. The rank of the matrix 4 0 2 5  (A) 0, 0, 0 (B) 0, 0, 1
2 1 3 1  (C) 0, 0, 3 (D) 1, 1, 1
(A) 4 (B) 3
(C) 2 (D) 1 1 1 2 3 
1 3 0 3 
14. The rank of matrix   is
10. The following equations have solutions 1 2 3 3 
 
x + 2y  z = 3 1 1 2 3 
2x  2y + 3z = 2 (A) 3 (B) 4
3x  y + 2z = 1 (C) 2 (D) 1
x y+z = 1
(A) x = 1, y = 4, z = 4
(B) x = 4, y = 1, z = 1
(C) infinite solutions
(D) it is consistent

38
Assignment on Linear Algebra

15. The following equations have  1 0


17. If A =   , then the value of k for
solutions.  1 7 
x + 2y + 3z = 6 which A 2 = 8A + kI is . . . .
3x  2y + z = 2 (A) 5 (B) 5
4x + 2y + z = 7
(C) 7 (D) 7
(A) x = 2, y = 2, z = ½
(B) x = y = z = 1 3 2   1 0
18. If Y =   & Y  2X =  ,
(C) They are inconsistent  1 4  3 2 
(D) Infinite solutions then X = . . . .
2 1 1 2 
5 1  4 3  (A)   (B)  
16. If A =   &B=  1 2  then  1 1 1 1
3 2   
1 1  1 1
the matrix X which satisfies the (C)   (D)  
1 2  2 1
equation 3A + X = 2B is given by X =
23 9   23 9 
(A)   (B)  
 7 10   7 10 

 23 9 
(C)   (D) None of these
 7 10 



39
Vidyalankar : GATE – Engineering Mathematics

Assignment  6
Duration : 45 Min. Max. Marks : 30

Q1 to Q6 carry one mark each 3. If a square matrix A be such that A2 = I


 1 2  then it is called as
1. Eigen values of Matrix   are
 5 4  (A) nilpotent matrix

(A) 6, 1 (B) 6, 1 (B) idempotent matrix

(C) 6, 1 (D) 6, 1 (C) involutary matrix


(D) none of these
2. Given an orthogonal matrix
1 1 1 1  4. If 1 , 2 , ……..n are the latent roots
 1 1 1 1 of matrix A then A3 has latent roots
A 
 1 1 0 0  (A) 1 …….n
 
0 0 1 1 1 1 1
(B) , , ........
l 1  2 n
 AA T  is
(C) 13, 23 , …….n3
1/ 4 0 0 0 
 0 1/ 4 0 0  1 1 1
(D) , , ........
(A)  13 2 3
 n3
 0 0 1/ 2 0 
 
 0 0 0 1/ 2 

1/ 2 0 0 0  5 2  1
 0 1/ 2 0 5. If A =   , then A is equal to
0  3 1
(B) 
 0 0 1/ 2 0 
   1 2   1 2 
 0 0 0 1/ 2  (A)   (B)  
 3 5   3 5 
1 0 0 0
0  1 2   1 2
1 0 0  (C)   (D)  
(C)   3 5  3 5 
0 0 1 0
 
0 0 0 1

1/ 4 0 0 0  2x 0   1 0
6. If A =   and A1 =  ,
 0 1/ 4 0 0   x x  1 2 
(D) 
 0 0 1/ 4 0  then value of x is
 
 0 0 0 1/ 4 
(A) 1 (B) 2
(C) 1/2 (D) none of these

40
Assignment on Linear Algebra

Q7 to Q18 carry two marks each 3 2 


12. The eigen value of A =   are
7. A matrix ‘P’ which diagonalises ‘A’ is 6 5 
called (A)  3 33 (B) 3, 1
(A) Spectral matrix (B) Modal matrix
(C) 1  2 7 (D) None of these
(C) Square matrix (D) None of these

 cos  sin  0 
 1 1 0 13. Given A =   sin  cos  0  , indicate
8. If A = 0 1 1 , then A1 is equal to  0 0 1
0 0 1
the statement which is not correct for A,
(A) A2 (B) A2  3A + 3I3 (A) It is orthogonal
(C) A (D) A2 + 2A  2I3 (B) It is non singular
(C) It is singular
9. If the rank of the matrix
(D) A1 exists
 1  1
A=   is 1, then the value of
 1 1  14. The eigen values and the
 is corresponding eigen vectors of a 2  2
(A) 1 (B) 1 matrix are given by
(C)  1 (D) None of these Eigen value Eigen vector
1
10. Rank of a unit matrix of order n is 1 = 8 v1 =  
1
(A) 1 (B) 2
1 
(C) 0 (D) n 2 = 4 v2 =  
 1
11. For what value of , the system of The matrix is
equations 6 2  4 6 
(A)   (B)  
3x  y + z = 0 2 6  6 4 
15x  6y + 5z = 0 2 4  4 8 
(C)   (D)  
x  2y + 2z = 0 4 2  8 4 
has non  zero solution.
(A) 1 (B) 2
(C) 6 (D) None of these

41
Vidyalankar : GATE – Engineering Mathematics

15. If A is orthogonal (A AT = I = AT A) then 17. For a second order matrix A if A 2 = I


| A | is then A is equal to
(A)  0 i 0  1 0 
(A)   (B)  
(B) 1 0 1 0 i
(C) 1 or 1  i 0  1 0 
(C)   (D)  
(D) can be any value 0 i  0 i

2 0.1
16. Let A =  and
0 3   5 4 0 
18. If A = [1, 2, 3], B =  0 2 1 , then
1/ 2 a 
A 1     1 3 2 
 0 b
AB =
Then (a + b) =
 2 
(A) 7/20 (B) 3/20
(A)  1 (B)  2 1 4
(C) 19/60 (D) 11/20
 4 

(C)  4 1 2 (D) None of these



42
Assignment on Linear Algebra

Assignment  7

Duration : 45 Min. Max. Marks : 30

Q1 to Q6 carry one mark each 6. Find the eigen values of the matrix and
1. Eigen value of an inverse of matrix is state that whether it is diagonal or not
(A) Same as the matrix  8 8 2 
(B) Negative of matrix values A = 4 3 2 
(C) Inverse of the matrix values  3 4 1 

(D) No any relation between them (A) Eigen value 1, 2, 3 & diagonal
(B) Eigen value 1, 2, 3 and
2. Rank ‘n’ of nonzero matrix
non diagonal
(A) may be n = 0 (B) may be n > 1
(C) Eigen value 1, 1, 3 and
(C) may be n = 1 (D) may be n  1
non diagonal
(D) Eigen value 1, 1, 3 and diagonal
 1 0
3. The rank of matrix   is
0 1
Q7 to Q18 carry two marks each
(A) 3 (B) 2
7. The inverse of matrix
(C) 1 (D) 0
2 1 3 
4. For what value of  do the equations x A =  1 1 1 is
 1 1 1
+ 2y = 1, 3x + y = 3
have unique solution  1 1 2 
(A)  = 6 (B)   6 (A)  0 1/ 2 1/ 2 
 1 1/ 2 3 / 2 
(C)  = 5 (D)   5
 1 0 1 
1
5. The formula for A is given by (B)  1 1/ 2 1/ 2 
 2 1/ 2 3 / 2 
(A) A1 = Adj A | A |
(B) A1 = Adj A  | adj A |  1 1 2 

(C)  1 0 1/ 2 
|A|
(C) A1 =  1 1/ 2 3 / 2 
adjA
(D) None of these  1 1 2 
(D)  1/ 2 1/ 2 1/ 2 

 3 / 2 1/ 2 3 / 2 

43
Vidyalankar : GATE – Engineering Mathematics

8. The eigen vectors of the matrix 11. The eigen values of a matrix
0 0   5 4 
0 0 0  ,   0 is (are)] A=   are
   1 2
0 0 0 
(A) 5, 2 (B) 1, 6
(i) (a, 0, ) (ii) (, 0, 0) (C) 4, 5 (D) 1, 5
(iii) (0, 0, 1) (iv) (0, , 0)
(A) (i), (ii) (B) (iii), (iv) 1 2 3 0
2 4 3 2 
(C) (ii), (iv) (D) (i), (iii) 12. The rank of matrix  is
3 2 1 3
 
9. The rank of the following 6 8 7 5
(n + 1)  (n + 1) matrix, where a is a (A) 1 (B) 2
real number is (C) 3 (D) 4
1 a a2 . . . an 
 
1 a a2 . . . an  13. The following set of equations
  3x + 2y + z = 4
. . . . 
. . . .  xy+z =2
 
. . . . 
2x + 2z = 5 have
1 a a2 . . . an 
 (A) No solution
(A) 1 (B) Unique solution
(B) 2 (C) Multiple solution
(C) n (D) An inconsistency
(D) Depends on the value of a

14. The matrix form of quadratic equations


10. The characteristic equation of matrix 6x12 + 3x22 + 14 x32 + 4x2x3 + 18x3x1 +
0 h g  4x1x2 …is
A = h 0 f  is 6 2 9 
g f 0 
(A) 2 3 2 
(A) 3   (f2 + g2 + h2)  2fgh = 0 9 2 14 

(B) 2   (f2 + g + h)  2fgh = 0  6 4 18 


3 2 2
(C)  +  (f + g + h )  2f gh = 0 2 2
(B)  4 3 4 
18 4 14 
(D) 2 +  cf2 + g2 + h2)  2fgh = 0

44
Assignment on Linear Algebra

 6 2 9   x 1   y 1  1 2
17. If  1 y   3 x  = 2 1 , then
(C)  2 3 2       
 9 2 14 
(x, y) = . . . .
 6 4 18  (A) (1, 0) (B) (1, 1)
(D)  4 3 4  (C) (0, 1) (D) (2, 1)
18 4 14 

15. If P and Q are both unitary matrices 0 a b


then product PQ is 18. If  = a 0 c , then . . .
b c 0
(A) nonunitary (B) unitary
(C) can’t say (D) none of these (A)  = 2abc (B)  = 0

(C)  = abc (D)  = a 2  b2  c 2


 1 2
16. If A =   , then adj A is
2 1

 1 2  2 1
(A)   (B)  
 2 1  2 1

 1 2   1 2 
(C)   (D)  
 2 1  2 1



45
Vidyalankar : GATE – Engineering Mathematics

Assignment  8
Duration : 45 Min. Max. Marks : 30

Q1 to Q6 carry one mark each (C) A is any matrix


1. Matrix A and B are square matrices of (D) It's inverse is unique
order n. Then B is said to be similar to 3 4 5 
4 1 2   
A if there exists a nonsingular matrix 6. If 
0 5 3   1 0 2 
  3 4 7 
P such that  
(A) B = PAP1 (B) P AP 8x  3y 6z 32 
=  , the
1
(C) B = P A P (D) B = PAP  4 12 26x  5y 

2. For m number of equations, n number values of x, y, z are

of columns, let r be rank of m  n (A) x = 3, y = 4, z = 1

coefficient matrix, then for r < n, the (B) x = 0, y = 1, z = 4

solution of matrix is (C) x = 1, y = 3, z = 4

(A) zero matrix (D) x = 1, y = 3, z = 4

(B) unit matrix Q7 to Q18 carry two marks each


(C) infinite number of solutions 7. Find the rank of matrix
(D) solution not possible 1 3 4 3
A =  3 9 12 9 
1 bc a(b  c)
 1 3 4 3 
3.  = 1 ca b(c  a) is equal to
1 ab c(a  b) (A) 1 (B) 2 (C) 3 (D) 4

(A) 0 (B) 1 8. If the characteristic roots of matrix A


(C) a + b + c (D) ab + bc + cd are 1, 2, 3, 4….n, then the

4. The multiplication of matrix is characteristic roots of A2 are


(A) Squares of eigen values of A
(A) always commutative
(B) Square roots of eigen values of A
(B) may be commutative
(C) Reciprocal of eigen values of A
(C) not always commutative
(D) Does not have any relation.
(D) can’t say

5. If the matrix A has inverse, then which 9. Are following vectors linearly
is wrong dependent
(A) A is non singular x1 = (1, 1, 1, 3) , x2 = (1, 2, 3, 4)
(B) | A |  0 x3 = (2, 3, 4, 9)

46
Assignment on Linear Algebra

(A) Yes (B) No 14. The value of determinant of matrix


(C) Can’t say (D) None of these 1 a a2 a 3  bcd 
 
10. Consider the system of equations 1 b b2 b3  cda 
  is
1 c c2 c 3  dab 
x + 2y + z = 6
 
1 d d2 d3  abc 
2x + y + 2z = 6
x+y+z=5 (A) a3 . b3 .c3 . d3  abcd
This system has (B) a2 b2 c2 d2  abcd
(A) Unique solution (C) 0
(B) Infinite no. of solution (D) 1
(C) No solution  8 29 18 
15. Eigen values of matrix  0 7 29  are
(D) exactly two solutions
 0 0 18 
11. The eigen value of a matrix
(A) 8, 7, 29 (B) 8, 7, 18
5 8 5 
0 7 12  are (C) 18, 7, 8 (D) 29, 18, 29
 
0 0 13  1 1 1
(A) 5, 8, 5 (B) 6, 7, 13 16.  4 3 2 =.....
2 2
(C) 5, 7, 13 (D) 5, 12, 13 4 3 22

(A) 2 (B) 2
12. Determine the eigen values of the
(C) 1 (D) 0
matrix
1 1 1
 2 1  2i
A =  17.  a b c  =....
1  2i 2 
2 2 2
a  bc b  ca c  ab
(A) 2, 2, (B) 1, 1
(A) 1 (B) 0
(C) 3, 3 (D) 1, 3
(C) 1 (D) None of these
13. Find the rank of matrix 18. The value of the determinant
 2 2 0 6 1 2 3
4 2 0 2 
A=  0 sec x tanx  is . . . .
 1 1 0 3 0 tan x sec x
 
 1 2 1 2
(A) 1 (B) 0
(A) 1 (B) 2
(C) tan2 x (D) sec 2 x
(C) 3 (D) 4



47
Vidyalankar : GATE – Engineering Mathematics

Assignment  9
Duration : 45 Min. Max. Marks : 30

Q1 to Q6 carry one mark each cos   sin  


(C)  
1. A square matrix U is called unitary if  sin  cos  
(A) U = U1 (B) U = U1  sin  sin  
(D)  
 sin  cos  
1 1
(C) U = U (D) U = U

2. If A is Hermitian matrix, then IA is


1 0 5 3
(A) Symmetric  2 1 6 1
6. If A =  , then value
(B) Hermitian 3 2 7 1
 
(C) Skew  Hermitian  4 4 2 0
(D) None of these of A + A
 2 2 8 7 
3. If A A = I, then | A | is equal to  2 2 8 3 
(A)  
(A) +1 (B) 1  8 8 14 1
 
(C)  1 (D) 0  7 3 1 0 

 2 2 8 7 
4. The matrix A and B are given below :  2 2 8 3 
(B)  
1 1  1 0  8 8 14 1
A =  ,B=   . The value of  
1 1  1 0   7 3 1 0 
AB is  2 8 2 7 
 2 8 2 3 
0 0   1 0 (C)  
(A)   (B)    8 14 8 1
0 0   0 1  
 7 1 3 0 
1 1  1 0 
(C)   (D)  
1 1  1 0   2 2 7 2 
 2 2 3 2 
(D)  
14 8 1 8 
 cos  sin    
5. Inverse of a matrix   sin  cos   is
   0 7 0 3 

  sin  cos  
(A)  
 cos  sin  

  sin  sin  
(B)  
 cos  cos  

48
Assignment on Linear Algebra

Q7 to Q18 carry two marks each (A) x = y = z = 1


a  ic b  id  (B) x = 1, y = z = 0
7. A =   is unitary matrix if
b  id a  ic  (C) x = y = z = 0

and only if (D) none of these


2 2 2
(A) a + b + c = 0
10. The matrix form of quadratic
(B) b2 + c2 + d2 = 0
expression
(C) a2 + b2 + c2 + d 2 = 1
2x12 + x22  3x32  8x2x3 4x3x1
2 2 2 2
(D) a + b + c + d = 0
+ 12x1x2 is

8. The quadratic expression in matrix  2 12 4 


form are (A) 12 1 8 
x2 + 4y2 + 9z2 + t2  12yz + 6zx  4xy  4 8 3 

 2xt  6zt  2 6 2 
 1 2 3 1 (B)  6 1 4 
 2 4 6 0 
 2 4 3 
(A)  
 3 6 9 3 
   2 6 4 
 1 0 3 1  (C)  6 1 4 
 1 4 6 2   4 4 3 
 4 4 12 0 
(B)    2 6 2 
 6 12 9 6 
  (D)  6 1 4 
 2 0 6 1  2 4 3 
 1 2 3 1
 2 4 6 0  11. Find the rank of matrix
(C) 
 3 6 9 3  1 1 1
 
1 0 3 1 A = 2 3 4 
8 2 3 
 1 4 6 2 
 4 4 12 0 
(D)  (A) 1 (B) 2
 6 12 9 6 
  (C) 3 (D) 4
 2 0 6 1
12. Find eigen value of the matrix
9. The solution of the equation  4 3 
 1 2 
x + 2y + 3z = 0  
3x + 4y + 4z = 0 (A) 1, 5 (B) 1, 5
7x + 10y + 12z = 0 (C) 1, 5 (D) 1, 5

49
Vidyalankar : GATE – Engineering Mathematics

 3 1 where A  square matrix,


13. If A =   , the value
 1 2  L  lower triangular matrix
A2  5A + 7I U  Upper triangular matrix
9 1 1 9 
(A)   (B)  
 1 4 1 4  16. Tp , Tq , Tr are the pth, qth & rth terms of

0 0   1 0 Tp Tq Tr
(C)   (D)  
0 0  0 1 an A.P. then  p q r  equals
1 1 1
 cos  sin  
14. If A =   , then value of (A) 1 (B) 1
  sin  cos  
(C) 0 (D) p + q + r
An is
 ncos  nsin   17. If 1, , 2 are three cube roots of unity,
(A)  
 nsin  ncos  
1 2 
  cosn  sinn 
(B)   then   1 2  is equal to
 sinn  cosn 
2  1
 cosn sinn 
(C)   (A) 1 (B) 
  sinn cosn  2
(C)  (D) zero
cosn  sinn 
(D)  
 sinn cosn 
18. If 1, , 2 are three cube roots of unity,

1 n 2n
15. Which of the following statement is
then 2n 1 n  has the value....
wrong ?
n 2n 1
1 1 1 1
(A) (ABC) =C B A
(A) 1 (B) 2
(B) (ABC)T = CT AT BT
(C)  (D) zero
1 1
(C) A = (A + A) + (A  A)
2 2
provided A is square matrix
(D) A = LU,



50
Assignment on Linear Algebra

Assignment  10
Duration : 45 Min. Max. Marks : 30

Q1 to Q6 carry one mark each 3 1 2


3 2 9  5. P = 2 3 1 = 8 then
1. Rank of matrix  6 4 18  is
 1 2 1
12 8 36 
1 3 2
(A) 2 (B) 3 Q = 3 2 1 = ?
2 1 1
(C) 2 (D) 1
(A) 8 (B) 8
2. The value of the determinant for an
(C) 16 (D) 16
upper triangular matrix is equal to
6. The rank of matrix
(A) sum of elements along principal
1 2 3 0
diagonal 2
 4 3 2 
(B) product of the elements along A = is
3 2 1 3
principal diagonal  
6 8 7 5
(C) sum of all elements
(A) 1 (B) 2
(D) product of all elements
(C) 3 (D) 4
3. The skew matrix always have rank ‘r’ Q7 to Q18 carry two marks each
equal to  2 1 
 0 
(A) r = 1 (B) r > 1  6 3
(C) r = 0 (D) can’t say  1 1 1 
7. The matrix   is
 2 6 3
4. The quadratic from of the symmetrical  1 1 1 
 
matrix  2 6 3

diag. [1, 2, … n ] is (A) orthogonal (B) not orthogonal

(A) 1 + 2 + …. n (C) singular (D) none of these

(B) 12x1 + 22x2 + ….. n2xn 8. The eigen values of matrix

(C) 1x12 + 2x22 + …. + nxn2 2 3 4 


 0 2 1 are
(D) 1x1 + 2x2 + …… + nxn  
 0 0 1

(A) 2, 2, 1 (B) 2, 2, 1


(C) 3, 4, 1 (D) 3, 4, 1

51
Vidyalankar : GATE – Engineering Mathematics

9. Write quadratic equation in matrix form  5 2 4 


2x12 + x22  3x32  8x2x3  4x3x1 + 12. The inverse of a matrix  2 1 1
 4 1 0 
12x1x2
 2 6 2  is
(A)  6 1 4   1 4 6 
1 
 2 4 3  (A) 4 16 13 
37 
 6 13 1 
 2 12 4 
(B) 12 1 8   1 4 6 
1 
 4 8 3  (B)   4 16 13 
37
 6 13 1 
 2 6 2 
(C)  6 1 4   1 4 6 
1 
 2 4 3  (C) 4 16 13 
36 
 6 13 1 
 2 12 4 
(D)  12 1 8   1 4 6 
1 
 4 8 3  (D) 4 16 13 
36 
 6 13 1
10. The characteristic roots of the matrix
 6 2 2   0 2 3
A =  2 3 1 are 13. For A =  2 0 5  matrix which of
 2 1 3   3 5 0 

(A) 2, 0, 8 (B) 2, 2, 8 following statement is false


(C) 2, 2, 8 (D) 0, 2, 8 (A) A is square matrix
(B) A is skew symmetric matrix
11. The rank of the matrix (C) A is symmetric matrix
1 4 8 7  (D) | A | = 0
0 0 3 0 
  is
4 2 3 1 
   1 2 3
 3 12 24 21 14. The rank of matrix A =  4 5 8 
(A) 1 (B) 2  3 2 1
(C) 3 (D) 4 is
(A) 1 (B) 2
(C) 3 (D) 4

52
Assignment on Linear Algebra

15. The value of the determinant 17. The roots of the equation
1 2 3 4 5 ax b c
2 3 4 5 1  0 bx a  = 0 are
1 2 3 4 5 is 0 0 cx
4 5 1 2 3 (A) a & b (B) b & c
5 1 2 3 4
(C) a & c (D) a, b & c
(A) 1, 00, 000 (B) 10,000
(C) 1,000 (D) 0
1 x 2 3
18. If  0 x 0 = 0, then its roots
16. If a + b + c = 0, one root of the
0 0 x
ax c b
are . . . .
equation  c bx a  =0
(A) 1 only (B) 0 only
b a cx
(C) 0 & 1 (D) 0, 1 & 1
is . . . .
(A) x = 1 (B) x = 2
(C) x = 0 (D) x = a 2  b2  c 2



53
Vidyalankar : GATE – Engineering Mathematics

Assignment  11
Duration : 45 Min. Max. Marks : 30

Q1 to Q6 carry one mark each 4. If A & B are arbitrary square matrices


1  1  1 17  of the same order, then
1. If x 1  y  1  z 0   15  ,
     
(A) (AB) = AB
1 0   1 13 
(B) (A) (B) = B A
x  (C) (A + B) = A  B
then  y  = . . . . . (D) (AB) = B A
 z 
5. Which of the following matrices is not
4 2
(A) 11 (B)  4  invertible
 2  11  1 1  1 1
(A)   (B)  
0 1  1 2 
11
(C)  4  (D) None of these 2 3   2 2 
(C)   (D)  
 2  4 6  1 1 

2. If A & B are matrices such that A + B 6. If A is a square matrix, then AA + AA

and AB are both defined then : is a

(A) A & B can be any matrices (A) Unit matrix

(B) A & B are square matrices, not (B) Null matrix

necessarily of the same order (C) Symmetric matrix

(C) A & B are square matrices of the (D) Skewsymmetric matrix

same order
Q7 to Q18 carry two marks each
(D) No. of columns of A = no. of rows
7. A = (aij) is a 3  2 matrix, whose
of B.
elements are given by
3. If the matrix AB = 0, then we must aij = 2i  j if i > j
have = 2j  i if i  j
(A) Both A & B should be zero Then the matrix A is
(B) At least one of A & B should be  1 0  1 3
zero (A)  3 2  (B)  3 2 
(C) Neither A nor B may be zero  1 1  1 1

(D) A + B = 0

54
Assignment on Linear Algebra

 1 3  1 0  1 0 0
(C)  3 2  (D)  3 2  (C)  2 3 1
 5 4   1 1  2 1 0 

(D) None of these


7 0  3 0 
8. If X + Y =   & X  Y = ,
2 5  0 3 
p 0 0 0
then the matrices X & Y are a q 0 0
11. The value of   is . . .
2 0  5 0  b c r 0
(A) X =   , Y =  1 4
 1 1   d e f s

5 0  2 0  (A) p + q + r + s (B) 1
(B) X =   ,Y=  
 1 1  1 4 (C) ab + cd + ef (D) pqrs

5 0  2 0
(C) X =  , Y =
1 
 12. The determinant
 1 4 1
a b a  b
2 0  5 0 
(D) X =   ,Y=    b c b  c = 0 if
 1 4  1 1 a   b b  c 0

 1 0  0 1 (A) a, b, c are in A.P.


9. If I =   ,J=   &
 0 1  1 0  (B) a, b, c are in G.P. or (x  ) is a

 cos  sin   factor of ax 2  2bx  c


B=   then B equals
  sin  cos  (C) a, b, c are in H.P.
(A) I cos  + J sin  (D)  is a root of ax 2  2bx  c = 0
(B) I sin  + J cos 
(C) I cos   J sin  13. If a1, a2, . . . . form a G.P. & ai > 0 for
(D) I cos  + J sin  all i  1, then
1 log am logam 1 logam  2
 1 2 3 
logam  3 logam  5 
10.  0 1 2 
logam  4
=.....
 0 0 1  logam  6 logam  7 logam  8

is equal to
1 2 5 
(A)  0 0 2  (A) log am  8  log am
 0 0 1  (B) log am  8  logam
 1 2 7  (C) zero
(B)  0 1 2 
(D) log am  4 2
 0 0 1 

55
Vidyalankar : GATE – Engineering Mathematics

14. If , ,  are real numbers, then The eigen vector corresponding to this

1 cos(   cos() eigen value is . . .


cos( - ) 1 cos() is 3 4
cos( -  ) cos( -  ) 1 (A)  2  (B)  2 
equal to  3   4 

(A) 1 5
(B) cos  cos  cos  (C)  2  (D) None of these
 5 
(C) cos  + cos  + cos 
(D) None of these
2 22 23 1 2 4
15. The characteristic equation of the 17. If 1 = 3 32 3  &  = 1 3
3
9 ,
2 3
0 2 4 4 4 4 1 4 16
matrix A =  1 1 2  is . . .
then
 2 0 5 
(A) 1 = 2 (B) 1 = 3
3 2
(A) x  6x  11x  6  0 (C) 1 = 4 (D) 1 = 24
3 2
(B) x  6x  11x  6  0

(C) x 3  6x 2  11x  6  0 18. Let a, b, c, d u, v be integers. If the

(D) None of these system of equations ax + by = u,


cx + dy = v has a unique solution in

16. The characteristic equation of the integers, then

 3 10 5  (A) ad  bc = 1
matrix A =  2 3 4  has a (B) ad  bc = 1
 
 3 5 7  (C) ad  bc = ±1
repeated root. (D) ad  bc need not be equal to ±1



56
Test Paper on Linear Algebra

Test Paper  1
Duration : 30 Min. Max. Marks : 25
Q1 to Q5 carry one mark each 4 1
5. For the matrix   , the eigen
1. A square matrix having determinant  1 4
equal to zero is called values are
(A) scalar matrix (A) 3, 3 (B) 3, 5
(B) singular matrix (C) 3, 5 (D) 5, 0
(C) nonsingular matrix
(D) skew symmetric matrix Q6 to Q15 carry two marks each
 0  tan  / 2 
2. The values of x, y, z and a which 6. If A =   then
 tan  / 2 0 
satisfies the matrix equations
 x  3 2y  x  0 7  cos   sin  
 z  1 4a  6   3 2a  are  sin  cos   = . . . .
     

(A) x = 3, y = 2, z = 4, a = 3 (A) (I  A)2 (B) (I  A)2

(B) x = 3, y = 2, z = 4, a = 1/3 I A


(C) I  A 2 (D)
(C) x = 3, y = 2, z = 4, a = 3 I A

(D) x = 3, y = 2, z = 4, a = 1/3 7. The relationship between following


vectors
3. Transpose of the product of two
x1 = (1, 3, 4, 2), x2 = (3, 5, 2, 2 ),
matrices is
x3 = (2, 1, 3, 2) is
(A) product of matrices in reverse
order (A) 2x1 + x2  2x3 = 0

(B) product of transposes of matrices (B) x4 + x2 + x3 = 0

in same order (C) x1 + x2  2x3 = 0

(C) product of transpose of matrices in (D) x1  2x2 + x3 = 0


reverse order
8. The points (x1, y1), (x2, y2), (x3, y3) are
(D) product of matrices in same order
collinear if and only if the rank of
4. Any matrix is said to be orthogonal if matrix
its determinant is  x1 y1 1
x y 2 1 is
(A) 0 (B) 1  2
(C)  2 (D) None of these  x 3 y 3 1

57
Vidyalankar : GATE – Engineering Mathematics

(A) equal to 3 11. Given that f(x) = x2  5x + 6,


(B) less than 3  3 1
A=   then f(A) = ?
(C) equal to or less than 3  1 2 
(D) can’t say
1 0   1 0 
(A)   (B)  
9. Consider the following equation 0 1  0 1

x+ y+z=9  1 0  1 0 
(C)   (D)  
2x +5y + 7z = 52  0 1  0 1

2x + y z = 0
(A) They have a unique solution  2 2 4 
(B) They posses multiple solutions 12. Is the given matrix  1 3 4 
 1 2 3 
(C) They have no solution
(D) The coefficient matrix is idempotent matrix ?
orthogonal (A) Yes
(B) No
0 c b 
(C) Can’t say
10. If A =  c 0 a  ,

(D) Cannot be determined
 b a 0 

 a 2 ab ac 
   cos   sin  0 
B = ab b2 bc  Then AB is 13. Inverse of a matrix  sin  cos  0 
 2
 ac bc c   0 0 1

 2bc abc abc   cos   sin  0 


(A) abc abc ac  (A)  sin  cos  0 
abc ac abc   0 0 1

 2bc ab ac   cos  sin  0 



(B)  ab 2ac ab  (B)   sin  cos  0 
 ac ab 2ba   0 0 1

0 0 0   cos  sin  0 
(C)  0 0 0  (C)  sin  cos  0 
 0 0 0   0 0 1

 0 ab ac   sin  cos  0 
(D)  ab 0 bc  (D)  cos  sin  0 
 
 ac bc 0   0 0 1

58
Test Paper on Linear Algebra

1 1 1  8 1 3 6
14. The eigen values of 1 1 1 are 15. The rank of matrix  0 3 2 2
 
1 1 1  8 1 3 4 

(A) 0, 0, 0 (B) 0, 0, 1 is
(C) 0, 0, 3 (D) 1, 1, 1 (A) 1 (B) 2
(C) 3 (D) 4



59
Vidyalankar : GATE – Engineering Mathematics

Test Paper  2
Duration : 30 Min. Max. Marks : 25

Q1 to Q5 carry one mark each  4 9 12 


(B)  
1. Every square matrix can be  8 8 28 
represented as  6 9 12 
(C)  
(A) sum of lower and upper triangular  8 8 28 
matrix
4 9 8 
(D)  
(B) sum of symmetric and skew  8 8 28 
symmetric matrix
(C) product of symmetric and skew 5. Consider the following determinant

symmetric matrix. 1 a bc
 = 1 b ca which of the following
(D) product of Hermitian & skew
1 c ab
hermitian matrix.
is a factor of  ?
2. Sum of eigen values of a matrix is
(A) a + b (B) a  b
equal to
(C) a + b + c (D) abc
(A) sum of all elements
(B) sum of principal elements Q6 to Q15 carry two marks each
(C) sum of diagonal elements 6. Consider the following equations
(D) none of these 2x + 6y + 11 = 0

x 5  3 4   7 16  6x + 20y  6z + 3 = 0
3. If 2    
 7 y  3   1 2  15 14  6y  18z + 1 = 0

then (A) They have a unique solution


(A) x = 2, y = 9 (B) x = 2, y = 5 (B) No solution
(C) x = 4, y = 6 (D) None of these (C) They have multiple solutions
(D) The coefficient matrix is idempotent
2 3 4 
4. If A =   and  cos  sin  
 1 2 5  7. If A =  ,
  sin  cos  
0 1 2 
B=   , then 2A + 3B will be  cos  sin  
2 4 6  B=   . Then AB = ?
  sin  cos  
 4 9 12 
(A)   cos(  ) sin(  ) 
 8 8 28  (A)  
 sin(  )  cos(  )

60
Test Paper on Linear Algebra

 cos(  ) sin(  )  (C) It is singular


(B)  
  sin(  ) cos(  ) (D) A1 exists

 cos(  ) sin(  )  1 3 5 
(C) 
 sin(  ) cos(  )
 12. If A =  0 2 1 , the eigen values
 0 0 3 
 cos(  ) sin(  ) 
(D)  
  sin(  ) cos(  ) of the matrix I + A + A2 are
(A) 1, 2, 3 (B) 3, 7, 13
1 1 3
5 2 6 (C) 1, 7, 13 (D) 1, 1, 1
8. Given matrix   is a
 2 1 3 
 cos   sin  
13 The matrices  
nilpotent matrix of index . . .  sin  cos  
(A) less than 3 a 0 
and  commute under
(B) 3 0 b 
(C) higher than 3 multiplication
(D) can not be determined (A) if a = b2 ,  = n, in integer
 1 2 3  (B) always
9. The rank of the matrix  2 4 1 is (C) if a = b
 1 2 7 
(D) never
(A) 3 (B) 2
14. The rank of the matrix given below is
(C) 1 (D) 0
1 4 8 7 
10. Which of following is correct ? 0 0 3 0 
 
(A) (AB)1 = A1 B1 4 2 3 1 
 
(B) (AB)t = At Bt  3 12 24 21
(C) (A1)1 = At (A) 3 (B) 1
(D) adj(AB) = (adj B) (adj A) (C) 2 (D) 4

 cos  sin  0 
15. For a square matrix A, if A2 = A then A
11. Given A =   sin  cos  0  . Indicate
 0 is called
0 1
(A) idempotent matrix
the statement which is not correct for A.
(B) nilpotent matrix
(A) It is orthogonal
(C) involuntary matrix
(B) It is non singular
(D) none of above



61
Vidyalankar : GATE – Engineering Mathematics

Test Paper  3
Duration : 30 Min. Max. Marks : 25

Q1 to Q5 carry one mark each (C) there are infinite number of


1. For triangular matrix eigen values are solutions
(A) same as that of row elements (D) solution is possible is in the form
(B) same as of column elements of complex number
(C) same as of that principal diagonal 5. Consider the following four possible
elements properties of a matrix A.
(D) can not predict (i) It is square matrix
(ii) aij = +aji
2. Rank of a matrix A is said to be ‘r’ if
(iii) aij = aji (iv) All leading diagonal
(A) atleast one square matrix of A of
elements are zero.
order ‘r’ whose determinant is not
If A is skew-symmetric, which of the
equal to zero.
following is True ?
(B) the matrix A contains any square
(A) (i), (ii) (iv) (B) (iii) & (iv)
submatrix of order r + 1, then the
(C) (i) , (iii), (iv) (D) None of these
determinant of every square sub
matrix of order r + 1 should be Q6 to Q15 carry two marks each
zero. 6. If aij = 1 for all i , j then rank (A) is
(C) both (A) and (B) true (A) 1 (B) 0
(D) only (B) true (C) n (D) anything

3. If A is m  n matrix and B is   n 7. | Adj A | is equal to


matrix, then product AB is
(A) | A | n (B) | A | n1
(A) m  n matrix
(C) |A|n+1 (D) |A1|
(B) n   matrix
where n is the order of the matrix
(C) n  n matrix
(D) product not possible 8. A and B are idempotent then AB is
also idempotent if
4. The equations are said to be
inconsistent if (A) A = BT

(A) the equations have no solution (B) A and B are nonsingular

(B) the solution is zero vector (C) AB is symmetric


(D) AB = BA

62
Test Paper on Linear Algebra

0 0 0 0 13. Consider system of equations


4 2 3 0  2x + y + 2z =1
9. The rank of A =  is
1 0 0 0 x+y =0
 
4 0 3 0
x  ky + 6z = 3 for the system to
(A) 0 (B) 1 have unique solution, the value of k is
(C) 2 (D) 3 (A) k = 2 (B) k  2
10. What are values of  and  so that the (C) k = 1 (D) k = 0
equations have no solution
14. The adjoint of matrix
2x + 3y + 5z = 9
7x + 3y 2z = 8  1 2 3 
A =  2 1 1 is
2x + 3y + z = 
 4 5 2 
(A)  = 9,  = 5 (B)  = 5,  = 9
 7 8 6 
(C)   9,  = 5 (D)   9,   5
(A)  11 14 13 
 5 5 5 
 5 3 14 4 
11. The rank of matrix  0 1 2 1 is 7 4 5 
 1 1 2 0  (B)  5 14 6 

 5 11 8 
(A) 1 (B) 2
(C) 3 (D) 4  7 11 5 
(C)  8 14 5 
12. The characteristic equation of the  6 13 5 

 1 0 2  7 11 13 
matrix A =  0 2 1 is (D)  5 9 11 
 2 0 3   2 14 20 
(A) 3 + 62  7 2 = 0
(B) 3  62 + 7 + 2 = 0 15. Find the value of x if
3 2
(C) 2 +   6 + 7 = 0 x2 2x  3 3x  4
3 2
(D) 2   + 6  7 = 0 2x  3 3x  4 4x  5  = 0
3x  5 5x  8 10x  17

(A) 1, 1, 2 (B) 1, 1, 2


(C) 1, 2 (D) 1, 1



63
Vidyalankar : GATE – Engineering Mathematics

Test Paper  4
Duration : 30 Min. Max. Marks : 25

Q1 to Q5 carry one mark each Q6 to Q15 carry two marks each


1. The matrix A is said to be skew 6. Find the rank of matrix
Hermitian if  1 2 1
(A) A = A (B) A =  A A =  6 12 6 
 5 10 5 
(C) A = A1 (D) A = Adj A
(A) 1 (B) 2
2. If  is eigen value of a matrix and also
(C) 3 (D) 4
1/ is eigen value of same matrix then
that matrix is 7. If A & B are 3  3 matrices then AB = 0
(A) Skew  Hermitian matrix implies :
(B) Hermitian matrix (A) Both A = 0 and B = 0
(C) Orthogonal matrix (B) | A | = 0 and | B | = 0
(D) None of these (C) Either | A | = 0 or | B | = 0

3. If matrix A has rank ‘r’, then A (D) Either A = 0 or B = 0

(transpose of A) will have rank


8. The characteristic equation of the
(A) r + 1 (B) r  1
matrix
(C) r (D) can’t say
 2 1 1 
4. A square matrix A be such that AK = 0 A =  1 2 1 is
then it is called  1 1 2 
(A) Hermitian matrix (A) 63 + 92  4 + 1 = 0
(B) idempotent matrix for index K (B) 3 + 62  9  4 = 0
(C) nilpotent for index K
(C) 3  62 + 9 4 = 0
(D) singular matrix for index K
(D) 63 + 92 + 4 1 = 0
5. For x12  18x1x2 + 5x22 quadratic form,
9. If In is the identity matrix of order n,
matrix is
1
then In 
 1 9   1 9 
(A)   (B)  
9 5   9 5  (A) does not exist (B) = In

 1 18   1 18  (C) = 0 (D) = nIn


(C)   (D)  
18 5   18 5 

64
Test Paper on Linear Algebra

10. If A is the transpose of a square 12. If a matrix is singular then the


matrix A, then characteristic root of the matrix is
(A) | A |  | A | (A) 0
(B) | A | = |A | (B) diagonal matrix

(C) | A | + | A  | = 0 (C) can’t say

(D) | A | = | A | only when A is (D) not possible

symmetric
13. Find the rank of matrix

 0 1 1  1 3 4 3
 3 9 12 9 
11. Inverse of matrix  1 0 1 is  
 1 1 0   1 3 4 1

(A) 1 (B) 2
 1 1 1 
1
(A) 1 1 1  (C) 3 (D) 4
2
 1 1 1
3 6 
 1 1 1  14. If A =   , then A is
(B) 2  1 1 1  2 4 
 1 1 1 (A) Singular (B) Nonsingular

 1 1 1 1 2 
1 (C) I (D) 6  
(C)  1 1 1  1 2 
2
 1 1 1

 1 1 1 15. If A is 3  4 matrix and B is a matrix


(D) 2  1 1 1  such that both AB & BA are defined,
 1 1 1 then B is a . . .
(A) 4  3 matrix (B) 3  4 matrix
(C) 3  3 matrix (D) 4  4 matrix



65
Vidyalankar : GATE – Engineering Mathematics

Test Paper  5
Duration : 30 Min. Max. Marks : 25

Q1 to Q5 carry one mark each 5. If the eigen values of an n  n matrix


1. If for a square matrix A, At = A, then are all distinct then it is always similar
the matrix A is to a
(A) Symmetric (A) square matrix
(B) Orthogonal (B) triangular matrix
(C) Hermitian (C) diagonal matrix
(D) Skew symmetric (D) none of these

2. The values of ‘a’, & ‘b’ from the matrix


Q6 to Q15 carry two marks each
equation
6. If A & B are square matrices of order 3
 a  3 b2  2  2a  1 3b 
     such that | A | = 1, | B | = 3 then
2
 0 2   0 b  3b 
determinant of 3AB is equal to
are (A) 9 (B) 27
(A) a = 1, b = 1 (B) a = 2, b= 1 (C) 81 (D) 81
(C) a = 2, b = 2 (D) a = 1, b = 2
7. Find the rank of matrix
3. Find the index for which the matrix
1 2 3 
 ab b 
2
A =  2 4 7 
 2  is nilpotent
 3 6 10 
 a ab 

(A) 3 (A) 1 (B) 2


(B) 2 (C) 3 (D) 4
(C) 1
(D) It is not nil potent 8. Eigen values of the matrix
a h g 
4.  is characteristic root of matrix A if
A =  0 b 0  are
and only if these exists a nonzero  0 0 c 
vector X such that
(A) a, b, c (B) 0, a, b
1
(A) A  A = X (B) AX = X (C) b, a, g (D) h, b, g
(C) XA = X (D) A  A1 = XX

66
Test Paper on Linear Algebra

9. If following vectors are linearly 3 4 2 


dependent then what is relation 12. Is the given matrix  4 3 1
between x1, x2, x3, x4 ?  2 1 3 

x1 = (1, 2, 4); x2 = (2, 1, 3); idempotent ?


x3 = (0, 1, 2); x4 = (3, 7, 2) (A) No
(A) 9x1 + 12x2 + 15 x3 + x4 = 0 (B) Yes
(B) 9x1  12x2 + 5x3  5x4 = 0 (C) Can’t say

(C) 9x1 + 12x2 + 5x3 + 5x4 = 0 (D) Insufficient Data

(C) None of these


13. The rank of the matrix

10. If A is a nonsingular matrix & A is the 1 1 1 


transpose of A then A =  0 2 5  is
 0 2 5 
(A) | A . A |  | A2 |
(B) | A . A |  | A |2 (A) 1 (B) 2
(C) 3 (D) 4
(C) | A | + | A | = 0
(D) | A | = | A |
14. The determinant of the matrix

11. The inverse of the matrix 6 0 0 0


 8 2 0 0 
 is
 a  ib c  id  2 2 2 2 1 4 4 0
 c  id a  ib  if a + b +c + d = 1  
  6 8 1
1
is
(A) 11 (B) 24
a  ib c  id  (C) 48 (D) 0
(A)  
 c  id a  ib 

 a  ib c  id 15. If a  b  c, one value of x which


(B)  
 c  id a  ib  satisfies the equation
 a  ib c  id  0 x a x b
(C)  
 c  id a  ib  x  a 0 x  c = 0 is given by
a  ib c  id  xb xc 0
(D)  
 c  id a  ib  (A) x = a (B) x = b
(C) x = c (D) x = 0



67
Vidyalankar : GATE – Engineering Mathematics

Test Paper  6
Duration : 30 Min. Max. Marks : 25

Q1 to Q5 carry one mark each  1  i 1 2  3i


1. The value of the determinant of matrix (C)  2  i 2i 1  i 
 3i 0 3  4i
a h g f
0 b c e 
A=  is  1  i 1 2  3i
0 0 d k (D)  2  i 2i 1  i 
 
0 0 0   3i 0 3  4i 
(A) abd  (B) abd 
(C) abck  abde (D) abd  abge 3 1 2 
4. If A = 6 2 4  Then rank of matrix A
3 1 2 
0 1 2   1 2 
2. If A =  1 2 3  , B =  1 0  , then
  is
2 3 4   2 1 (A) 1 (B) 2
the product BA is (C) 3 (D) 0

3 2   2 3 
(A) 5 5  (B)  5 8  5 3 
5. Eigen value of matrix   are
7 8   8 7  3 3 

0 0  (A) 6, 4 (B) 5, 3
(C) 0 0  (D) does not exist. (C) 3, 5 (D) 4, 6
0 0 
Q6 to Q15 carry two marks each
6. The value of determinant
 1  i 1 2  3i
3. Let A =  2  i 2i 1  i  , then the 3 1 1 1
 3i 0 3  4i 1 3 1 1
  is
1 1 3 1
matrix A is given by 1 1 1 3
 i  1 1 2  3i (A) 1 (B) 1
(A)  2  i 2i 1  i 
(C) 0 (D) 4
 3i 0 3  4i

 1 i 1 2  3i
(B)  2  i 2i 1  i 

 3i 0 3  4i

68
Test Paper on Linear Algebra

7. The rank of matrix 10. Determine eigen values of matrix


 2 3 1 1 a h g 
 4  is
A =  1 1 2 A =  0 b 0  are
3 1 3 2   0 0 c 
 
6 3 0 7 
(A) g, b, c (B) g, h, c
(A) 1 (B) 2
(C) a, b, g (D) b, a, c
(C) 3 (D) 4

11. If A + B + C = , then the value of


 1 2 3
8. Adjoint of matrix A = 0 5 0  is sin(A  B  C) sinB cos C
2 4 3   sinB 0 tanA  is
cos(A  B)  tan A 0
 15 0 10 
(A)  6 3 0  equal to
 
 15 6 5  (A) 0
(B) 1
 15 6 15 
(B)  0 3 0  (C) 2 sin B tan A cos C
 
 10 0 5  (D) None of these

 15 3 5 
(C)  10 1 2 1 2
6 0  1
 3 1 2 
 0 15 0  12. The rank of matrix A = 
2 4 3 4
15 10 0   
3 7 4 6
(D)  3 6 15 

 5 0 0 
is
(A) 1 (B) 2
(C) 3 (D) 4
9. The characteristic roots of the matrix
0 1 2  13. If a3  b 2  c  d
 1 0 1 are
 
2 1 0  3  1  1
=   3 2   2 be an identity
(A) 1, 2  3 (B) 3, 1  2   3   4 5
(C) 2, 1  3 (D) None of these in , where a, b, c, d are constants,
then the value of d is
(A) 5 (B) 6
(C) 9 (D) 0

69
Vidyalankar : GATE – Engineering Mathematics

3 4  15. If the system of equations


2 1 2 
14. If A =  1 1  & B =   then x + ay + az = 0, bx + y + bz = 0,
2 0   1 3 4
cx + cy + z = 0 where a, b, c are
AB = B A non zero & non unity has a non trivial
(A) Yes solution then the value of
(B) No a b c
  is
(C) Can’t say 1 a 1 b 1 c

(D) Insufficient Data (A) 0 (B) 1


abc
(C) 1 (D)
a 2  b2  c 2



70
Chapter - 2 : Calculus
2.1 Function of single variable
A real valued function y = f(x) of a real variable x is a mapping whose domain S and
co-domain R are sets of real numbers. The range of the function is the set y  f(x) : x  R ,

which is a subset of R.

2.2 Limit of a function


The function f is said to tend to the limit  as x  a, if for a given positive real number
 > 0 we can find a real number  > 0 such that
f(x)     whenever 0 < |x  a| < 

Symbolically we write lim f(x)  


x a

Left Hand and Right Hand Limits


Let x < a and x  a from the left hand side.
If |f(x)   1 | < , a<x<a or lim f(x)   1
x a

then  1 is called the left hand limit.

Let x > a and x  a from the right hand side.


If |f(x)   2 | < , a < x < a +  or lim f(x)   2
x a

then  2 is called the right hand limit.

If  1   2 then lim f(x) exists. If the limit exists then it is unique.


x a

Properties of Limits
Let f and g be two functions defined over S and let a be any point, not necessarily in S
And if lim f(x)   1 and lim g(x)   2 exist, then
x a x a

 lim Cf(x)  C lim f(x)  C 1 C a real constant.


x a x a

 lim  f(x)  g(x)  lim f(x)  lim g(x)   1   2


x a x a x a

 lim  f(x)g(x)   lim f(x)  lim g(x)   1 2


x a  x a   x a 

71
Vidyalankar : GATE – Engineering Mathematics

 f(x)   xlim f(x)  1


 lim    a  2  0
x  a g(x)


  lim g(x) 
x a  2

g x 
lim  f  x     1 
2

x a

Standard Formulae

sinx ax  1
lim =1 …x in radians lim = na (a > 0)
x0 x x0 x
tanx e x 1
lim = 1 …x in radians lim =1
x0 x x0 x
1
limcosx = 1
x0 lim1  x) x =e
x0

xn  an n(1  x)
lim  nan1 lim 1
xa x  a x0 x

Solved Example 1 :
Solution :
 1 Let h > 0, we have
Show that lim sin   does not exist.
x 0
x
lim f(x  h)   4  h  1
2

Solution : h 0  
For different values of x in the interval = 17  h  h  8  
 1 = 17 if h (h + 8) < 1
0 < | x | <  the function sin   takes
x
 (h  4)2  17  h  17  4
values between 1 and 1.
and lim f(x  h)  (4  h)2  1
 1 h 0
Since lim sin   is not unique limit does
x 0
x = 17  h(h  8)
not exist.
= 16 if h ( h  8) >  1

or (h  4)2  15 or h  4  15
Solved Example 2 :
 lim f(x)  17 and lim f(x)  16
Show that lim  x 2  1 does not exist, x  4 x 4
x 4

The limit does not exist.


where  is the greatest integer function.

72
Notes on Calculus

2.3 Continuity
Let f be a real valued function of the real variable x. Let x0 be a point in the domain of f
and let f be defined in some neighbourhood of the point x0. The function f is said to be
continuous at x = x0 if
i) lim f(x)   exists and
x  x0

ii) lim f(x)  f(x 0 )


x  x0

Types of discontinuity
A point at which f is not continuous is called a point of discontinuity.
If lim f(x)   exists, but lim f(x)  f(x 0 )
x  x0 x  x0

then x0 is called the point of removable discontinuity.


In this case we can redefine f(x), such that f(x0) =  , so that the new function is
continuous at x = x0.
For example :
 sin x  / x ; x0
The function, f(x) = 
4 ; x0

has a removable discontinuity at x = 0, since lim f(x)   and a new function can be
x 0

defined as

 sin x  / x ; x0


f(x) = 
1 ; x0

If lim f(x) does not exist, then x0 is called the point of irremovable discontinuity.
x  x0

For Example :
1
f(x) = at x = 0
x
Here limf(x) does not exist. Also f(0) is not defined.
x 0

1
 f(x) = has an irremovable discontinuity at x = 0.
x

Note : In case of an irremovable discontinuity, it does not matter whether or


how the function is defined of the point of discontinuity.

73
Vidyalankar : GATE – Engineering Mathematics

Properties of continuous functions


 Let the functions f and g be continuous at a point x = x0 Then,
i) cf, f  g and fg are continuous at x = x0, where c is any constant.
ii) f/g is continuous at x = x0, if g(x0)  0
 If f is continuous at x = x0 and g is continuous at f(x0), then the composite function
g(f(g)) is continuous at x = x0.
 A function f is continuous in a closed interval [a, b] if it is continuous at every point in
(a, b)
lim f  x   f  a  and lim f  x   f  b 
x a x b

 If f is continuous at an interior point c of a closed interval [a, b] and f(c)  0, then there
exists a neighbourhood of c, throughout which f(x) has the same sign as f(c).
 If f is continuous in a closed interval [a, b] then it is bounded there and attains its
bounds at least once in [a, b].
 If f is continuous in a closed interval [a, b], and if f(a) f(b) are of opposite signs, then
there exists at least one point c  [a, b] such that f(c) = 0.
 If f is continuous in a closed interval [a, b] and f(a)  f(b) then it assumes every value
between f(a) and f(b).

Solved Example 3 : x
 The function f(x) = is discontinuous
Determine the point of discontinuity of the x

x at x = 0.
function f(x) =
x Note : |x| = x when x > 0 and |x| = x
Solution : when x < 0.
x
For x < 0, = 1 Solved Example 4 :
x
Find the limit if it exists, as x approaches
x zero for the function f(x) given by
For x > 0, = +1
x
if x < 0, f(x) = x
 f(0) = 1 x = 0, f(x) = 1
and f(+0) = +1 x > 0, f(x) = x2
Also the function is not defined at x = 0 Solution :
If x < 0, f(0) = lim f  x   lim x  0
x 0 x 0

74
Notes on Calculus

If x > 0, f(+0) = lim f  x   lim x 2  0 Solution :


x 0 x 0
lim f(x)  2  (1)  3 for x < 2,
Thus f(0)  f(0) each being zero. x 2

Since the limit on the left = the limit on the x2


= 1
|x2|
right at x = 0 the function f(x) has limit 0 at
x = 0. lim f(x)  2  (1)  1 for x > 2,
x 2

Note : The limit at x = 0 is not equal to


x2
f(0) = 1, and hence the function is =1
|x2|
discontinuous at x = 0.
 The one sided limits do not coincide.
Solved Example 5 : Thus the function is not defined at x = 2.
x 3  x 2  16x  20  x = 2 is the only point of discontinuity.
  x  2
2
Let f(x) = 
The jump at this point = 1  (3) = 2
k ifx  2

Solved Example 7 :
if x  2
x  2 for x2
Find k if f(x) is continuous at x = 2. Show that f(x) =  2 is
x  1 for x2
Solution :
discontinuous at x = 2 and determine the
For x  2 we have
jump of the function at this point.
x 3  x 2  16x  20
f(x) = Solution :
x  2
2

lim (x  2)  4
x  2
 x  2  x  5
2

= =x+5
 x  2
2
x  2

lim x 2  1  3
 lim f(x)  lim(x  5)  7  The limit on the left  the limit on the right.
x 2 x 2

 The function has discontinuity at the


We have f(2) = k
point x = 2
 f(x) is continuous at x = 2.
 The jump of the function at x = 2 is
Now lim f(x)  f(2)
x 2
f(2 + 0)  f(2  0) = 3  4 = 1
i.e. 7 = k
Note : If f(x0  0)  f(x0 + 0), then
Solved Example 6 : f(x0 + 0)  f(x0  0) is called a jump
x2 discontinuity of the function f(x) at x0.
For the function f(x) = x  find the
x2

point of discontinuity and determine the


jump of the function at this point.

75
Vidyalankar : GATE – Engineering Mathematics

2.4 Differentiability
Let a real valued function f(x) be defined on an I and let x0 be a point in I. Then, if
f  x   f  x0  f  x 0  x   f  x 0 
lim or lim
x  x0 x  x0 x  0 x

exists and is equal to  , then f(x) is said to be differentiable at x0 and  is called the
derivative of f(x) at x = x0.
If f(x) is differentiable at every point in the interval (a, b) then f(x) is said to be
differentiable in (a, b).

Properties of differentiation
 Let the functions f and g be differentiable at a point x0. Then
i) (cf) x0 = cf(x0), c any constant.
ii) (f  g)x0 = f(x0)  g(x0)
iii) (fg)(x0) = f(x0) g(x0) + f(x0) g(x0)

 f  g  x 0  f   x 0   f  x 0  g  x 0 
iv)    x 0   , g  x0   0
g g2  x 0 

 If f is differentiable at x0 and g is differentiable at f(x0) then the composite function


h = g(f(x)) is differentiable at x0 and
h(x0) = g(f(x0)) f(x0)
 If the function y = f(x) is represented in the parametric form as
x = (t) and y = (t), and if (t), (t) exist, then

dy / dt    t 
f(x) =    t   0
dx / dt   t 

Note : If a function is differentiable at x = x0, then it is continuous at x = x0.


However the converse need not be true.

76
Notes on Calculus

Standard formulae

Function Derivative Function Derivative

x 1
k(constant) 0 sin1
a a  x2
2

x 1
xn nxn1 cos 1
a a2  x2
x a
logx 1/x tan1
a a  x2
2

x a
ex ex cot 1
a a  x2
2

x a
ax axloga sec1
a x x2  a2
x a
sin x cos x cosec 1
a x x2  a2

cos x  sin x sin h x cos h x

tan x sec2x cos h x sin h x

1
cot x  cosec2 x sin h1 x 2
x 1
1
sec x sec x tan x cos h1 x
x2  1
1
cosec x  cosec x cot x tan h1 x 2
x 1

Solved Example 8 :  f(x) is continuous at x = 0


Show that the function f(x)  f(0)
Now f(0) = lim
x cos(1/ x),
2
x0
x 0 x
f(x) = 
0 x0   1 
 lim  x cos     0
x 0
  x 
is differentiable at x = 0 but f(x) is not
continuous at x = 0. Hence f(x) is differentiable at

Solution : x = 0 and f (0) = 0

We have lim f(x)  0  f(0) for x  0 we have


x 0

77
Vidyalankar : GATE – Engineering Mathematics

 1   1   1  At x = 0, we have y = | x |
f(x) = 2x cos    x 2   sin      2 
x   x   x 
y x
 
 1  1 x x
 2x cos    sin  
x x
y
lim  1
 1 x  0 x
Now lim f   x  does not exist as lim sin  
x 0 x 0
x y
lim 1
does not exist. Therefore, f(x) is not x  0 x
continuous at x = 0. Since the right and the left side derivative
are not equal, the function f(x) = |x| is not
Solved Example 9 : differentiable at the point zero.
y  x is defined and continuous for all x
3

investigate whether this function has a Solved Example 11 :


derivative at x = 0. d n
Show that
dx
 
x  nxn1
Solution :

y = 3
x  x  3 x at x = 0 Solution :
Let y = xn
y = 3
x Let x receive a small increment x and let

y 3 x 1 the corresponding increment in y be y


  
x x 3
 x 
2 then
y + y = (x + x)n
y 1
 lim  lim   By subtraction, y = (x + x)n  xn
x  0 x x  0 3
 x 
2

y  x  x   x  x  x   xn
n n n

 There is no finite derivative.   


x x x  x  x

 x  x   xn
n
y
Solved Example 10 :  lim  lim  nxn1
x  0 x x x  x x  x  x
Investigate the function f(x) = | x | for
dy d n
differentiability at the point x = 0 
dx
 nxn1 or
dx
 
x  nxn1

Solution :

y = f(x) = | x |
y = | x + x |  | x |

78
Notes on Calculus

2.5 Mean Value Theorems


Rolle's Theorem
The theorem states that
 if f(x) is continuous in the closed interval [a, b] and
 if f(x) exists in open interval (a, b) and
 if f(a) = f(b)
Then there exists at least one value c in (a, b) such that f(c) = 0

Geometric Interpretation
y
f(c) = 0

y = f(x)
f(a) = f(b)

a c b x

There exists at last one point at which slope of the tangent is 0 or the tangent is parallel
to x-axis

Lagrange's Mean Value Theorem


The theorem states that
 if f(x) is continuous in the closed interval [a, b] and
 if f(x) exists in open interval (a, b)
Then there exists one value c such that

f(b)f(a)
f(c) =
ba

79
Vidyalankar : GATE – Engineering Mathematics

Geometric Interpretation
f(b)f(a)
f(c) =
ba

f(b)

f(a)

a c b
There exists at last one point at which the tangent is parallel to the secant through the
end points.

Cauchy's Mean Value Theorem


The theorem states that
 if f(x) and g(x) are both continuous in closed interval [a, b] and
 if f(x) and g(x) both exist in open interval (a, b)
Then there exists at least one value c such that
f (c) f(b)f(a)

g(c) g(b)g(a)

2.6 Maxima and Minima


A function f(x) is said to have a maximum value at x = a if f(a) is larger than any other
values of f(x) in the immediate neighbourhood of ‘a’. It has a minimum value if f(a) is less
than any other value of f(x) sufficiently near ‘a’.

Finding Maximum and Minimum values of y = f(x)


dy d2 y dy
 Get and 2 . Solve = 0 and consider its roots. These are the values of x
dx dx dx
dy
which make = 0.
dx
 For each of these values of x, calculate the corresponding value of y and examine
d2 y
the sign of .
dx 2
 If the sign is ve the corresponding value of y is a maximum.
If the sign is +ve, the corresponding value of y is minimum.

80
Notes on Calculus

Note : Maximum and minimum values of a function occur alternately in a


continuous function. (The graph of a continuous function has no break
gaps)
A maximum is not necessarily the greater value and a minimum is not
necessarily the least value of the function.

Stationary Points and Turning Points


dy dy
A stationary point on a graph is any point at which = 0. A value of x for which =0
dx dx
does not necessarily give either a maximum or minimum. A turning point is a maximum or
minimum point.

Solved Example 12 : 1
Hence x = gives a minimum value of y
Find the maximum and minimum points on 3

the curve y = 2x3 + 5x2  4x + 7 and x = 2 gives a maximum value of y


Solution : 1 8
for x  , y6 ; for x   2, y  19
Differentiating the equation we have 3 27

dy 1 8 
 6x 2  10x  4   3 , 6 27  is a minimum point
dx  

d2 y (2, 19) is a maximum point.


 12x  10
dx 2
Solved Example 13 :
Now
Find the maximum and minimum values of
dy
 0 if6x 2  10x  4  0 x 2  2x  4
dx the function
x 2  2x  4
or 3x 2  5x  2  0
Solution :
i.e. (3x  1) (x + 2) = 0
x 2  2x  4
dy 1 Let y =
i.e.  0, when x  or  2 x 2  2x  4
dx 3
(x 2  2x  4)(2x  2) 
1 d2 y  2 
for x  , 2  14 and  is positive dy (x  2x  4)(2x  2)
3 dx Then =
dx (x 2  2x  4)2
d2 y
for x  2,  14 and  is negative dy
dx 2  = 0 when x 2  4
dx
or x = 2

81
Vidyalankar : GATE – Engineering Mathematics

In this case it is much more convenient to dy


Note : If the sign changes from
dy dx
find the changes of sign of than to
dx + to  the point is a maximum point; if
d2 y the sign changes from  to +, the point
work out the value of .
dx 2 is a minimum point.
The denominator must be positive and
therefore only the sign of the numerator Solved Example 14 :
need be examined. Find the maximum and minimum values of
Near x = 2 the function 4 cos x  3 sin x.
2
If x is slightly less than 2, x < 4; Solution:
dy Let y = 4 cos x  3 sin x
is ve
dx
dy
If x is slightly greater than 2, x2 > 4; Then = 4 sin x  3 cos x
dx
dy
is +ve d2 y
dx = 4 cos x + 3 sin x
dx 2
dy
Here changes from ve to +ve dy
dx = 0 when  4 sin x  3 cos x = 0
dx
Near x = 2
3
If x is slightly less than 2, x2 > 4; or tan x =
4
dy
is +ve 3 4
dx and then (i) sin x = , cos x 
5 5
If x is slightly greater than 2, x2 < 4;
3 4
dy (ii) sin x = , cos x 
is ve 5 5
dx
 tan  is negative.
dy
Here changes from +ve to ve
dx d2 y 16 9
In the first case, 2
=   5
 x = 2 gives a minimum value of y, dx 5 5

1 and  is +ve
y ;
3 d2 y 16 9
In the second case, 2
=   5
and x = 2 gives a maximum value of y, dx 5 5
y=3 and  is ve
This the maximum value is 3 and the  The minimum value
1  4  3
minimum is . = 4    3    5
3  5  5

82
Notes on Calculus

and the maximum value Let ABCD by rectangular sheet and let a
4  3  square of edge x m be cut from each
= 4  3    5
5  5  corner.
When the flaps are folded up, the
Solved Example 15 :
dimensions of the rectangular box
Find the maximum and minimum values of
obtained are 8  2x, 3  2x and x meters.
f(x, y) = 7x 2  8xy  y 2 where x, y are
Let V cubic meters be the volume of the
2 2
connected by the relation x  y  1 . box then
Solution : V =  8  2x  3  2x  x
Since x 2  y 2  1,  4x 3  22x 2  24x ;
we can put x  cos  , y  sin  dV
 12x 2  44x  24
dx
f(x, y) = 7 cos2   8 cos  sin   sin2  = F()
d2 V
1 cos 2 1 cos 2 and  24x  44
F(  ) = 7  4 sin2  dx 2
2 2
dV
= 4  3 cos 2  4 sin2
dx
 
 0 if 4 3x 2  11x  6  0
= 4  5 sin  cos 2  5 cos  sin2
(3x  2) (x  2) = 0
= 4  5 sin (2  )
i.e. when x = 2/3 or 3
Since sine functions takes its maximum
2 d2 v
for x  ,  28 and
and minimum value as +1 and 1 3 dx 2
respectively.  is negative
Fmax = 4 + 5 ; Fmin = 4  5 2
Hence x  gives a maximum value of V
 Fmax = 9 Fmin = 1 3
2 11
Solved Example 16 : For x = , V= 7
3 27
A rectangular sheet of a metal is 8 meters 11
Thus the maximum volume is 7 cm.
by 3 meters; equal squares are cut out at 27
each of the corners and the flaps are then x = 3 is inadmissible, since the breadth
folded upto form an open rectangular box. itself of the sheet is 3m.
Find its maximum volume.
Solution: D C

x 3m

A B
8m
83
Vidyalankar : GATE – Engineering Mathematics

2.7 Integration
Integrand and element of integration
The function under the sign of integration is called integrand. For e.g. in  x 3 dx ; x3 is

called integrand. In the integral  f(x) dx , dx is known as the element of integration and it

indicates the variable with respect to which the given function is to be integrated.

Constant of integration :
We know that
d 2
(x )  2x  2x dx  x
2

dx
d 2
  2x dx  x
2
Also (x  c)  2x  c , where c is any constant
dx
So we notice that x2 is an integral of 2x, then x2 + c is also an integral of 2x. In general if

 f(x)dx = (x) then

 f(x)dx = (x) + c

Standard formulae
xn  1 1 x
 x dx    sin1
n
 c , (n  1) c
n1 a2  x2 a

1 dx x
 x dx  log x  c  2 2
  cos1
a
c
a x
dx 1 x
e  a2  x2 tan1  c
x
dx  e x  c 
a a
ax dx 1 x
a
x
dx 
log e a
+c  a2  x2 
a
cot 1  c
a
dx 1 x
 cos xdx  sin x  c  2 2

a
s ec1  c
a
x x a
dx 1 x
 cos ecx cot xdx   cos ecx  c  2 2

a
cos ec 1  c
a
x x a

 sin x dx   cos x  c  sinh x dx = cos h x + c

84
Notes on Calculus

 sec x tan x dx  sec x  c  cosh x dx = sin h x + c


dx
 sec x dx  tan x  c
2  2
= sin h1 x + c
x 1

dx
 cos ec x dx = cot x + c
2  2
= cos h1 x + c
x 1

dx
 cot xdx = log sin x + c  x2  1 = tan h1 x + c

 tan xdx = log sec x + c  tan xdx =  log (cos x) + c

 x
 sec xdx  log(sec x  tan x)c  sec xdx  logtan  4  2 c
 x
 cosecxdx   log(cosecx  cot x)c  cosec xdx log  tan 2  +c

Important Trigonometric Identities


 sin2 A + cos2 A =1
 sin (A + B) = sin A cos B + cos A sin B
 cos (A + B) = cos A cos B  sin A sin B
tanA  tanB
 tan (A + B) =
1  tanA tanB
 sin (A  B) = sin A cos B  cos A sin B

 cos (A  B) = cos A cos B + sin A sin B


tan A  tanB
 tan (A  B) =
1  tan A tanB
 sin2 A  sin2 B = sin (A + B) sin (A  B)

 cos2 A  sin2 B = cos (A + B) cos (A  B)

 2 sin A cos B = sin (A + B) + sin (A  B)

 2 cos A sin B = sin (A + B)  sin (A  B)

 2 cos A cos B = cos (A + B) + cos (A  B)

85
Vidyalankar : GATE – Engineering Mathematics

 2 sin A sin B = cos (A  B)  cos (A + B)


CD CD
 2 sin cos = sin C + sin D
2 2
CD CD
 2 cos sin = sin C  sin D
2 2
CD CD
 2 cos cos = cos C + cos D
2 2
CD DC
 2 sin sin = cos C  cos D
2 2

1  tan2 A
 cos 2A = cos2 A  sin2 A = 1  2sin2 A = 2 cos2A  1 =
1  tan2 A
2 tanA
 sin 2A = 2 sin A cos A =
1  tan2 A

2 tanA
 tan 2A =
1  tan2 A

 sin 3A = 3 sin A  4 sin3A

 cos 3A = 4 cos3 A  3 cos A


3 tanA  4 tan3 A
 tan 3A =
1  3 tan2 A

 sin
m
Note : Integration of x cosn xdx where m and n are positive integers

(i) If m be odd and n be even, for integration put t = cos x


(ii) If m be even and n be odd, for integration put t = sin x
(iii) If m and n are odd, for integration put either t = cos x or sin x
(iv) If m and n are even, for integration put either t = cos x or sin x

Solved Example 17 :  4  sin x dx =  (3 sin x  sin3x)dx


3

Evaluate  sin x dx 3
 3  sinxdx   sin3xdx
Solution :
1 cos 3x 
 sin x dx  3 cos x  c
3
sin 3x = 3 sin x  4 sin3x 4  3 
4 sin3x = 3 sin x  sin 3x

86
Notes on Calculus

Solved Example 18 : Solved Example 21 :

 sin3x cos 2x dx  sin


3
Evaluate Evaluate x cos3 x dx

Solution : Solution :
1 We have
 sin3x cos 2x dx  2
(sin5x  sinx)dx
sin3 x cos3 x = (sin x cos x)3
1 1 1
= cos 5x  cos x = (2 sin x cos x)3
10 2 8
1
= sin3 2x
Solved Example 19 : 8

Evaluate  sin2x sin3x dx =


1 1
 (3 sin 2x  sin 6x)
8 4
Solution :
[ 4 sin3x = 3 sin x  sin 3x]
1
 sin2xsin3xdx  2  cos x  cos 5xdx =
1
(3 sin 2x  sin 6x)
32
1 1
= sinx  sin5x
  sin
3
2 10 x cos3 xdx

1
32 
 (3 sin2x  sin6x)dx
Solved Example 20 :
dx 3 1
Integrate  cos2 x sin2 x =
32  sin2xdx 
32 
sin6xdx

Solution : 3   cos 2x  1   cos 6x 


=
32  2   32  6 c
1 cos2 x  sin2 x   
Now, 2 2

cos x sin x cos 2 x sin2 x
 sin
3
Hence x cos3 xdx
1 1
  3 1
sin x cos2 x
2
 cos 2x  cos 6x  c
64 192
= cosec2 x + sec2x
[ 1 = sin2 x + cos2 x] Solved Example 22 :
dx 1
  cos2 x sin2 x Evaluate  1  sin x dx
=  (cos ec 2 x  sec 2 x)dx Solution :
1 1 1  sinx
 
 cos ec x dx   sec
2 2
= x dx
1  sinx 1  sin x 1  sinx
=  cot x + tan x + c 1  sin x
=
1  sin2 x

87
Vidyalankar : GATE – Engineering Mathematics

1  sinx 1 sin x Solved Example 24 :


 2
 
cos x cos x cos2 x
2

 sin
6
Evaluate x cos3 x dx
1
or = sec2x  tan x sec x Solution :
1  sinx
 sin x cos3 xdx   sin6 x(1  sin2 x)cos xdx
6
1
  1  sin x dx
=  (sin6 x  sin8 x)cos x dx
  (sec 2 x  tanx sec x)dx
=  (t 6  t 8 )dt
 sec xdx   tanx sec xdx
2
=

 tanx  sec x  c  by putting


1 7 1 9 1 1
sin x = t = t  t = sin7 x  sin9 x
7 9 7 9
Solved Example 23 :
Solved Example 25 :
 sin
7
Evaluate x cos6 x dx
 sin
4
Evaluate x cos2 x dx
Solution :
Solution :
 sin
7
x cos6 xdx
sin4 x cos2 x
  sin6 x cos6 x sinxdx 1
= (2 sin2x)2 (1+cos2x)
8
=  (1  cos2 x)3 cos6 x sin x dx
1
= (1cos2x  cos22x + cos3 2x)
=  (1  3 cos x  3 cos x  cos x)
2 4 6
8

cos6 x sinxdx 1  1  cos 4x


=
8  1  cos 2x  2

=  (cos6 x  3 cos8 x  3 cos10 x  cos12 x)
cos 6x  3 cos 2x 
 
sinxdx 4 
=   (t 6  3t 8  3t10  t12 )dt 1 1 1 1 1 
=
8  2  4 cos 2x  2 cos 4x  4 cos 6x 
 
by putting t = cos x
1 1 1 
t7 t9 t11 t13 = 1  cos 2x  cos 4x  cos 6x 
= 3 3  16  2 2 
7 9 11 13
 sin
4
1 1 3 x cos2 xdx
=  cos7 x  cos9 x  cos11 x
7 3 11 1 1 1 
= 1  cos 2x  cos 4x  cos 6x  dx
1 16  2 2 
 cos13 x
13
1 1 1 1 
=  x  sin2x  sin4x  sin6x 
16  4 4 12 

88
Notes on Calculus

t
3
Solved Example 26 : = (1  t 2 ) dt

 sin
3
Evaluate x cos5 x dx by putting sin x = t
Solution : =  (t 3  2t 5  t 7 ) dt

 sin
3
x cos5 x dx t4 t6 t8
=  
4 3 8
 sin
3
= x (cos4 x)cos x dx
sin4 x sin6 x sin8 x
=  
 sin x(1  sin x) cos x dx
3 2 2
= 4 3 8

Integration by Substitution
Some of the general substitutions are
Type Substitution

a2  x 2 x = a sin  or a cos 
a2 + x 2 x = a tan  or a cot  or a sin h 

x 2  a2 x = a sec  or a cosec  or a cos h 

By means of above substitution some standard integrals are


dx x x
 2 2
 sin1
a
or  cos1 c
a
a x

dx x
  sinh1   or log(x  x 2  a 2 )c
a
a2  x2

dx x
  co sh1   or log(x  x 2  a 2 )c
a
x2  a2

dx 1 x 1 x
 a2  x2 
a
tan1   or
a a
cot 1  c
a

1 1 x
 a 2  x 2 dx 
2
x a 2  x 2  a 2 sin1  c
2 a

1 1 x
 a 2  x 2 dx 
2
x a 2  x 2  a 2 sinh1  c
2 a

89
Vidyalankar : GATE – Engineering Mathematics

1 1 x
 x 2  a 2 dx 
2
x x 2  a 2  a 2 cosh1  c
2 a

dx 1 ax
 a2  x2 
2a
log
ax
c

dx 1 xa
 x2  a2 
2a
log
xa
c

Solved Example 27 : 1  sin 


cos x  sinx
I=  1  sin 
. 2 sin  cos  d
Integrate  1  sin2x
dx
1  sin 
Solution :  cos 
.2 sin  cos d

We know that   (2 sin   cos 2  1)d


2 2
1 + sin 2x = sin x + cos x + 2 sin x cos x
sin2
= (sin x + cos x)2 = 2 cos  +  +c
2
cos x  sin x
Let I =  1  sin2x
dx =  2 cos  + sin  cos    + c

= 2 1  sin2   sin  1  sin2     c


cos x  sinx
= dx
(sinx  cos x)2 = 2 1 x  x 1  x  sin1 x  c
Let cos x + sin x = u
= ( x 2) 1  x  sin1 x  c
 (cos x  sin x) dx = du
du 1 1
Then I =  u2 
u
c 
cos x  sinx
c Solved Example 29 :
Find the value of the integral
cos x  sinx 1
  1  sin2x dx  cos x  sin x  c sinx cos x dx
 a2 sin2 x  b2 cos2 x
Solution :
Solved Example 28 :
Put a2 sin2 x + b2 cos2 x = t
1 x 2(a2 sin x cos x) dx + 2b2 cos x (sin x) dx = dt
Evaluate  1 x
dx
dt
sin x cos x dx =
Solution : 2(a  b2 )
2

Put x = sin  or x = sin2  sinx cos xdx


 dx = 2 sin  cos  d
  a2 sin2 x  b2 cos2 x

90
Notes on Calculus

dt dt 1 t
  2(a2  b2 )t  I =  t2  ( 2)2

2
tan1
2
c

1 dt 1  1
2 
=  log t  c 1 x 
2
2(a  b ) t 2(a  b2 )
2
= tan  x c
2  
 2 
1
= log (a2 sin2x + b2 cos2x) + c
2(a 2  b2 ) 1  tan2   1
= tan1  c
2  2 tan  

Solved Example 30 :
d Solved Example 31 :
Evaluate  sin4   cos4  cos x
Solution :
Integrate  1  sin2 x dx
d sec 4  Solution :
I=  sin4   cos4  =  tan
4
1
d
Let sin x = z  cos x dx = dz
Dividing numerator and denominator by cos x dz
Now  dx =  1  z2
cos 4 1  sin2 x

sec 2  sec 2  = tan1 z + c


I=  d
1  tan4  = tan1 (sin x) + c
(1  tan2 )sec 2 
  1  tan4  d Solved Example 32 :
2
Put tan  = x so that sec  d = dx sin1 x
1
Integrate  1 x2
dx
1
(1  x 2 )dx x2
 I=    2 1 dx Solution :
1 x4 x  2
x Let x = sin   dx = cos  d
1
1 sin x sin1(sin )cos 
 x2
1
dx
 1 x2
dx =  1  sin2 
d
2
 1
x  x   2 2
  =  d  2
c
1  1
Put x  = t,   1  2  dx = dt (sin1 x)2
x  x  = c
2

91
Vidyalankar : GATE – Engineering Mathematics

Integration by parts
Integral of the product of two functions

 du 
 uvdxu vdx  dx  vdx dx

Rule to choose the factor of differentiation or the 1st function


Of the two functions the first function u is selected based on the following preference
order
1. Inverse function
2. Logarithmic function
3. Algebraic function
4. Trigonometric function
5. Exponential function

Solved Example 33 : x
  sin1 xdx  x sin1 x   dx
1 x2
Integrate  x cos xdx
Put 1  x2 = z,  2x dx = dz
Solution :
Here F. D. = x F. I. = cos x 1 dz
 sin 2 z
1
 xdx  x sin1 x 
Applying the rules of integration by parts.
= x sin1 x  z
 x cos xdx
 sin
1
 xdx  x sin1 x  1  x 2
 dx 
= x  cos xdx     cos xdx  dx
 dx 
Solved Example 35 :
= x sin x   1.sinxdx
Find  log xdx
= x sin x + cos x + c
Solution :
Solved Example 34 : We have  log xdx   1 log xdx
 sin
1
Integrate x dx
Integrating by parts,
Solution : d 
Here F. I. = 1 and F. D. = sin x 1  log xdx  log x  1dx    dx log x   1dx  dx
Now  1.sin
1
x  sin1 x   1dx
  1
= x log x    xdx
x
d 
   sin1 x) 1.dx  dx = x log x  x  c  x  log x  1  c
 dx 

92
Notes on Calculus

= x  log x  loge   c b  eax b 


  cosbx   eax sinbxdx 
a a a 
( 1 = log e)
eax sinbx
Solved Example 36 : =
a
e
ax
Evaluate sinbx dx b ax b2 ax
a2 
 e cosbx  e sinbxdx
Solution : a2

eax  a sinbx  b cosbx  b2


e
ax
I = sinbx dx I=  I
a2 a2
ax ax
e e b cosbx
= sinbx   dx  b2  eax  a sinbx  b cosbx 
a a  I 1 2 
 a  a2
eax sinbx b ax
=   e cosbxdx
a a eax  a sinbx  b cosbx 
 I=
a 2  b2
eax sinbx
=
a

2.8 Definite Integration


b
The integral a f(x)dx is called definite integral and is defined to be b    a) where (x)

is the indefinite integral of  f(x)dx . It is read as integral from a to b of f(x), a is called the
lower limit and b is called the upper limit.

b
x b
 f(x)dx    x  x  a    b     a 
a

This is also called the theorem of calculus.

Properties of definite integral


b b
 a f(x)dxa f(t)dt
b a
  f(x)dx  f(x)dx
a b

b c b
  f(x)dx f(x)dx f(x)dx
a a c

93
Vidyalankar : GATE – Engineering Mathematics

a a
  f(x)dx f(a  x)dx
0 0

a a
  f(x)dx2 f(x)dx
a 0
f(x) = f(x) i.e. it is an even function

=0 f(x) = f(x) i.e. it is an odd function


2a a
  f(x)dx2 f(x)dx
0 0
f(2a  x) = f(x)

=0 f(2a  x) = f(x)

Solved Example 37 : Solution :


/2
dx
Integrate 0 cos xdx We know that  x
 log x

b
Solution : dx b b
 a x
 log x a  logb  log a = log
a
We know that  cos xdx  sinx  c
/2 Solved Example 40 :
/2 
 0 cos xdx  sinx 0  sin
2
 sin   1 /2

0 1  cos 2x 
1/ 2
Evaluate dx

Solved Example 38 :
Solution :
1
dx
Integrate  /2

0 1  cos 2x 
2 1/ 2
0 1 x Let I = dx

Solution :
/2

We know that 
dx
 tan1 x  c
= 0 
1  1  2 sin2 x dx 
1 x2
/2 /2
1

dx 1
0 1  x2  tan x  c 0
1 = 0 2 sinxdx  2 0 sinx
/2
  tan1 1  c    tan1 0  c  = 2  cos x 0

   
= 0  = 2  cos   cos 
4 4  2

Solved Example 39 :  2 1  0   2

b
1
Evaluate a x dx

94
Notes on Calculus

Solved Example 41 : 1 3
= log  3  4y  02
/3
cos x 4
Evaluate the integral 0 3  4 sin x
dx
1  3 
= log 3  4  log 3 
Solution : 4  2 
Put sin x = y then cos x dx = dy 32 3
1
= log  
when x = 0, then y = 0 and 4  3
 
3
when x = /3 then y 
2
/3 3 /2
cos xdx dy
Let I = 0 3  4 sinx
 0 3  4y

Improper Integrals
b
 The integral a f(x)dx is said to be an improper integral of the first kind, if one or both

the limits of integration are infinite.


b
 The integral a f(x)dx is said to be improper integral of second kind, if f(x) becomes
unbounded at one or more points in the interval of integration [a, b].

Limit comparison Test for Convergence and Divergence of Improper Integrals :


If f(x) and g(x) are positive functions and if
f x  
lim  L, 0  L  , then  f  x  dx and  g  x  dx both converge or both diverge.
x  gx a a

 
dx 1 1 1
Note : 1.  1  x2 converges because
1 x 2

x 2
and  x2 dx converges.
1 1

 
1 1 1 1
2.  e2x dx converges because
e 2x

e x
and  ex dx converges.
1 1

 
dx 1 1 1
3.  x
diverges because
x

x
for x > 1 and  x dx diverges
1 1

 
1 1  1 1 1  1
4.   x  x2  dx diverges because  
x  x x 2 
and  x dx diverges.
1 1

95
Vidyalankar : GATE – Engineering Mathematics

Solved Example 42 :  1
= n1  n t  n  
1
1 x t
Solve 0 1 x
dx
1
dx  1
 lim   lim  n   
Solution :
t 0
t
x t 0  t

The integrand becomes infinite at x = 1. The given integral diverges.

1 x
The area under the curve y = f(x) =
1 x Solved Example 44 :
between the lines x = 0 and x = 1 is not 3
dx
well defined since the curve extends to
Solve   x  1
0
2/3

infinity as x tends to 1 from the left. Solution :


Nevertheless, we can define the area from
1
x = 0 to x = t where 0 < t < 1. The function f(x) = becomes
 x  1
2/ 3

1 t
Then the integral  f(x)dx  lim  f(x)dx
0
t 1
0
infinite at x = 1 which lies between the
limits of integration.
We also say that the improper integral For the integral to exist, we must have the
converges. convergence of the following integrals.
t t
1 x 1 x 1 3
lim  dx  lim  dx dx dx
t 1
0
1 x t 1
0 1 x
2   x  1
0
2/ 3
and   x  1
1
2/ 3

t
= lim  sin1 x  1  x 2  It can be easily verified the convergence of
t 1  0

= lim sin1 t  1  t 2  1


these two integrals are +3 and 3  2
3

t 1  
respectively.
 Hence the given improper integral
= sin1 1  0  1  1
2
converges.
Solved Example 43 :
1
dx Solved Example 45 :
Solve 0 x . 
dx

dx
Compare 1 x 2 and 1 1  x 2 with limit
Solution :
For some positive number comparison test.
1
dx Solution :
t x  n x t
1
t < 1,
1 1
f(x) = 2
and g(x) 
x 1 x2

96
Notes on Calculus

1 Solved Example 47 :
x 2 1 x2
lim  lim 2 
1
x  1
2
x  x Solve e 2x
 10e x
dx
1 x 1

 1  Solution :
= lim  2  1
x  x
  
1
= 1 a positive finite limit. e
1
2x
 10e x
dx converges because

 
dx dx 
1
Hence x
1
2
converges, therefore  1 x
1
2 e 2x
dx converges and
1

converges. 1 2x
lim
x  e2x

e  10e x 
Solved Example 46 :  10 
= lim  1  x   1 , a positive finite limit

3
x 
 e 
Solve e
1
x
5
dx

Solution : Solved Example 48 :




3

1 1
1 ex  5 dx converges because 1 ex dx Solve 
1 x
dx

converges Solution :
 
3 ex 3 1 1
Now lim x
x  e  5
  lim
1 x  1  5e  x

1 x
dx diverges because  x dx
1
diverges

= 3 a positive finite limit. 1 1


and  for x > 1
x x

2.9 Double Integrals


The double integral of a function f(x, y) over a region D is denoted by  f  x,y  dxdy .
D

Let f(x, y) be a continuous function defined on a closed rectangle


R = { (x, y) / a  x  b and c  y  d }
For continuous functions f(x, y) we have
d
d b
  b 
R  
f x,y dxdy  a  c   
f x,y dy dx     f  x,y  dx  dy
   a 
c

97
Vidyalankar : GATE – Engineering Mathematics

Note : 1.  dxdy represent the area of the region S.


S

2. In an iterated integral the limits in the first integral are constants and
if the limits in the second integral are functions of x then we must
first integrate w.r.t. y and the integrand will become a function of x
alone. This is integrated w.r.t. x.
3. If R cannot be written in neither of the above two forms we divide R
into finite number of subregions such that each of the subregions
can be represented in one of the above forms and we get the double
integral over R by adding the integrals over these subregions.

Solved Example 49 : 1
 1   3 x3 
  x    x    dx
2
=
12
 3   3  
   x  2  dydx
0
Evaluate I =
0 0 1
 1 4x 3 
  x
2
=    dx
Solution : 0
3 3 
1 1

  xy  2y    2x  4  dx
2 1
dx =  x3 x x4 
0
0
0
=    
 3 3 3 0
1
 2x 2 
  4  x 0
= 
1 1 1 1 1
=   
 2 0 3 3 3 3
=1+4=5
Solved Example 51 :
Solved Example 50 :
3 2
dxdy
1 1 Evaluate 
  x  dydx xy
2
Evaluate y 2
2 1
0 x
Solution :
Solution : 3 2 3 2
dxdy dy dx
1 1
2 1 xy = 2 y 1 x
  x 
2
 y 2 dydx
0 x
3
dy
 y log x 
2

1 1
 =
 
1
=    x 2  y 2 dy  dx 2

0 x  3
dy
1 =  log 2  log1 

1
y3  y
=   x 2 y   dx 2

0
3 x
= log 2 log y 2
3

98
Notes on Calculus

= log2 log3  log2 The region of integration 0  y  2 and 0 

3 y2
= log 2 log x can also be written as
2 2

0  x  2 and 2x  y  2
Solved Example 52 : 2  2 
y
x2
 I = 0   dy  dx
 e x  y  1 
2 2
Evaluate the double integral dxdy  2x
R
2 2
where the region R is given by =   x 2  y 2  1
 2x
dx
R : 2y  x  2 and 0  y  1. 0

2
Solution :
=   x 2  5   x  1  dx

Integrating with respect to y first, we get 0

2
 x / 2 x2  2
 x x2  5 5
 
x/2

0 ye  0 dx
x2
I = 0  0 e dy  dx = = 
2
 log x  x 2  5
2
  
2 2
1 1 2  1 4
 
2
2
2
1
=
20 xe x dy   ex  =
4 0 4
e 1   x  1 
2 0

Solved Example 53 :
= 3
5
2
 1
log5  log 5   9  1
2

Evaluate the integral 5
= log 5  1
y2 4
2 2
y
 x  y2  1
2
dxdy
0 0 Solved Example 54 :
Solution : The cylinder x 2  z 2  1 is cut by the
y planes y = 0, z = 0 and x = y. Find the
volume of the region in the first octant.
y=2 Solution :
(2, 2)
In the first octant we have z  1  x 2 . The
2
y = 2x projection of the surface in the x  y plane
is bounded by x = 0, x = 1, y = 0 and y = x.
0 x
1
x 
It would be easier to integrate it first with  V =  zdxdy     1  x 2 dy  dx
R 0 0 
respect to y.
1
1  x 2  y 0 dx
x
= 
0

99
Vidyalankar : GATE – Engineering Mathematics

1 In this region x varies from 0 to 2 when


= x
0
1  x 2 dx
0  x  1 for fixed x, y varies from 0 to x.

1  3/2 1
  1 cubic units. When 1  x  2, y varies from 0 to 2 x.
=
3 

1 x2   0 3  The region D can be subdivided into
2 regions D1 and D2 as shown.
Solved Example 55 :
 a cos 
  xydydx   xydydx   xydydx
 
D D1 D2
Evaluate I = r sin drd
0 0
In the region D1 for fixed x, y varies from
Solution : y = 0 to y = x and for fixed y, x varies
 a cos 
r  2
from x = 0 to x = 1.
I =  sin   2  d
0 0 Similarly for D2 the limit of integration for

1 2
 y is y = 0 to y = 2  x
2 0
= a cos2  sin d
 xydydx
D

a 2
=  cos2 d  cos   1 x 2 2x
2 0  
0 0
xydydx   
1 0
xydydx
[d(cos ) = sin  d]
x 2x
a 2 a2  xy 2 
1 2
 xy 2 
=

 cos3   = =   dx     dx
6 0 3 0
2 0 1
2 0
1 2
1 2 1
x dx   x  2  x  dx
2

2 0
Solved Example 56 : =
21
Evaluate I = D xydydx where D is the
1 2
 x3  1 x 4 4x 3 
=    2x 2   
region bounded by the curve x = y2,  6 0 2  4 3 1
x = 2  y, y = 0 and y = 1.
1 1 16 8  1  1 4 
=  2  4   4  2  
Solution : 6 2 4 3  2  4 3

1 4 11 9
=   
6 6 24 24
y2 = x
(1, 1)
Solved Example 57 :
Change the order of integration in the
D1 4 y
D2
integral I =   f  x, y  dxdy
1 y/2
(1, 0) (2, 0)

100
Notes on Calculus

Solution : Solved Example 58 :


The region of integration D is bounded by a a
xdxdy
x = y/2
Evaluate 0 y x2  y2
x=y
by changing the order of integration.
y=1
Solution :
y=4

y=a
y=4 (2, 4) (4, 4)
(a, a)
D3 y=x
y = 2x
y=x x=a
D2 R
(0.5, 1) D1 (2, 2)
y=1 (0, 0) (a, 0)
(1, 1)

(0, 0) Region R of integration


is bounded by
Here, x varies from 0.5 to 4
x=y
1 x=a
when  x  2 , y varies from 1 to 2x
2
y=0
when 1  x  2, y varies from x to 2x
y=a
when 2  x  4, y varies from x to 4
We have to change the order of integration
For changing the order of integration we
as dy dx.
must divide D into sub regions D1, D2, D3.
Here in the region R, x varies from 0 to a
 I =  f  x, y  dxdy
D
and for fixed x, y varies from 0 to x.
a a a x
xdxdy x
=  f  x,y  dxdy   f  x, y  dxdy 0 y x2  y2 = 0 0 x2  y2 dydx
D1 D2

  f  x, y  dxdy a x
1 1  y  
D3
= 0 x  x tan  x  dx
0
1 2x 2 2x

  f  x,y  dydx    f  x, y  dydx


a x
=  1  y  
1/ 2 1 1 x
= 0 tan  x  dx
0
4 4
   f  x,y  dydx a

0 tan 1  tan1 0  dx
1
2 x
=

a
  a a
dx   x 0 
4 0
=
4 4

101
Vidyalankar : GATE – Engineering Mathematics

Solved Example 59 :  32 128 8 2   1 1 1 1 


=        
4 2 5 21 3 3   5 21 3 3 
1   x 
2
Evaluate  y 2 dxdy by changing
1006
y
=
105
the order of integration.
Solution :
Solved Example 60 :
(2, 4) y=4 3 4y

Evaluate 0 1  x  y  dxdy by changing

y = x2 x=2
the order of integration.
(1, 1) y=1 Solution :
The region is bounded by

(0, 0) y=0
y=3
The region of integration is bounded by
x=1
y=1
x2 = 4  y
y=4
x2 = y
x=2 (1, 3) y=3
We change the order of integration as dy
dx. In the region x varies from 1 to 2 and
2 x=1 y = 4  x2
for fixed x, y varies from 1 to x .
4 2

1   x 
2
 y 2 dxdy
y
(0, 0) (1, 0) (2, 0)

2 x2
We have to change the order of integration
1 1  x 
2
=  y 2 dydx
as dydx.
x 2
In this region x varies from 1 to 2
2
 2 y3 
= 1 x y  3  dx and y varies from 0 to 4  x2
1
3 4y
 1 0 1  x  y  dxdy
2
x6
=   x4   x 2   dx
1
3 3
2 2 4  x2
 x5 x7 x3 x 
=      = 1 0  x  y  dydx
 5 21 3 3 1

102
Notes on Calculus

4  x2 We divide the region into two subregions



2
y2 
=   xy   dx R1 and R2
1
2 0
 R xydydx   xydydx   xydydx

  
2
2 4  x2 R R

=  x 4  x2    dx 1 2

 2  We have to change the order of integration


 
1

as dxdy in R1 and R2.


 x4
2

=    x 3  4x 2  4x  8  dx In region R1, y varies from a to 2a for a
1
2 
fixed y, x varies from 0 to 2a  y
2
x x5
4x 4
 3
In the region R2, y varies from 0 to a and
=     2x 2  8x 
 10 4 3 1 for a fixed y, x varies from 0 to ay .
241 a 2a  x
=
60  0  xydydx
x2 / a

Solved Example 61 : = 
R
xydydx   xydydx
R
1 2
a 2a  x
Evaluate 0  xydydx by changing the 2a 2a  y a ay
x2 / a = a 0 xydxdy  
0 0
 xydxdy
order of integration
2a  y ay
Solution : 2a
 x2 y   x2 y 
a
= a 
 2 0
 dy   
0
2 0
 dy

(0, 2a) 2a a
y ay 2
a 2  2a  y 
2
x2 = ay = dy   dy
0
2
R1
(0, a) (a, a) 2a
 2 y3 2 a 2
a

R2
= a  2a y 
2
 2ay 

dy 
2 0
y dy

y = 2a  x
2a a
 y 4 2ay 3   y3 
= a2 y 2    a 
(0, 0) (a, 0) (2a, 0)  8 3 a  6 0

5a 4 a 4 9a 4
=  =
24 6 24
The region R is bounded by
x=0
x=a Solved Example 62 :
1 1 x2
y = 2a  x
Evaluate 0 0 ydydx by interchanging the
x2
y=
a order of integration.

103
Vidyalankar : GATE – Engineering Mathematics

Solution : 1
 2
0 y  x 0
2 1 y
= dy
(0, 1)
1
x=1
0 y
2
= 1  y 2 dy
x=0
/2

0 sin
2
(0, 0) y=0 (1, 0) =  cos d

x2 + y2 = 1 Putting y = sin  
 when y = 0  = 0 
 
 y = 1  =  /2 
The region is bounded by
again put t = sin 
y = 0 (x  axis)
 dt = cos  d
y= 1  x 2 or x 2  y 2  1
When  = 0 t=0
x = 0 (y  axis)  = /2 t=1
x=1 1
1
 t3 
 0 t dt   3 
2
Here y varies from 0 to 1 and for fixed y
0

x varies from 0 to 1 y2
1 1 x 2
1
1 1 x 2 1 1 y
2  0 0 y 2 dydx 
3
 0 0 y 2 dydx = 0 0 y 2 dxdy

2.10 Triple Integrals


A triple integral of a function defined over a region R is denoted by

 f  x, y,z  dxdydz or  f  x,y,z  dV or  f  x, y,z  d  x,y,z 


R R

Evaluation of Triple Integrals :


If the region R can be described by
x1  x  x 2 , y1  x   y  y 2  x  , z1  x,y   z  z2  x, y 

then we evaluate triple integral as


x2 y 2  x  z2  x,y  x2  y2  x   z2  x,y   
   f  x,y,z  dzdydx = x  y x  z x,y f  x,y,z  dz  dy  dx
x1 y1  x  z1  x,y  1  1   1   

104
Notes on Calculus

Note : There are six possible ways in which a triple integral can be evaluated
(order of variables of integration). We choose the one which is simple to
use.

Solved Example 63 : x
1  xy 4 
a x a
1
2 0 0 2 0  4  0
3
2 3 2
= xy dydx =   dx
0 1 1 xy zdzdydx
2
Evaluate
a
1  x6 
a
1 5 a6
Solution : = 
80
x dx    
8  6  0 48
2
2 3 2
 xy 2 z2 
2 3

0 1 1 0 1  2  dydx
2
xy zdzdydx  Solved Example 65 :
1
 / 2 1
…(Integrating w.r.t. z keeping x, y constants)
0 0 0 r
2
Evaluate : I = sin drdd
2 3
 xy 2

0 1 2xy
2
=   dydx
2  Solution :
 /2 1
2 3 r3 
3
=   xy 2 dydx I= 0 0 sin    dd
 3 0
201
 /2 
1 1
  cos 0 d
3 /2

3 0 0 
3  xy 3 
2
= sin dd =
=   dx 30
2 0  3 1

1 1  
…(integrating w.r.t. y keeping x constant) =  d   0 
2
30 3 3
1
 27x  x  dx
2 0
=

2
Solved Example 66 :
= 13  xdx log a x x  y

0 0 0 e
xy z
0 Evaluate I = dzdydr
2
 x2 
= 13    26 Solution :
 2 0
log a x
xy
0 0 e  dydx
xy z
I=
Solved Example 64 : 0

a x y
log a x
0 0 0 xyzdzdydx 0 0  e 
Evaluate I = =
2 x  y 
 e x  y dydx

Solution : log a x
 1 2 x  y  
a x y
 xyz2 
= 0 2 e

 e x  y  dx
0
I= 0 0  2  dydx
0

105
Vidyalankar : GATE – Engineering Mathematics

log a
 1 4x 3 2x x 
1
0y 36  4x 2
= 0  2 e  2 e  e  dx
  3

log a 0x3
1 3 
=  e4x  e2x  e x 
8 4 0 2 9
x2 
 3 3 
 I =  0  36  4x  9y  dy  dx
2 2
1 3 3
= a4  a2  a  0 
8 4 8  

3 x2
Solved Example 67 :
0 4  9  x  y  3y
2 9
= 2 3  3
dx
0
Find the volume of the solid in the first
octant bounded by the paraboloid. 3
8 8 
0  3  9  x   
3/2 3/2
2
=  9  x2  dx
z  36  4x  9y 2 2
9 
Solution : 16
3

 
3/ 2

9 0
= 9  x2 dx
We have I=  dzdydx
R
Substituting x = 3 sin  we get
The projection of the paraboloid (in the first
/2
16
octant) in the x  y plane is the region in 0  27 cos    3 cos   d
3
I =
9
the first quadrant of the ellipse
/2
4x 2  9y 2  36
0 cos
4
= 144 d
 The region R is
3 1 
0  z  36  4x 2  9y 2 = 144      27 cubic units
4 2 2

2.11 Change of Variables in Double and Triple Integrals


and Jacobians
Consider the transformation given by the equation
x = x(u, v, w) ;
y = y(u, v, w) ;
z = z(u, v, w)
Where the functions x, y, z have continuous first order partial derivatives.

106
Notes on Calculus

The Jacobian J of the transformation is defined by


x x x
u v w
y y y
J=
u v w
z z z
u v w

  x, y,z 
The Jacobian is also denoted by J =
  u, v, w 

For a transformation in two variables x = x(u, v) and y = y(u, v) the Jacobian is given by a
  x, y 
determinant of order two. Hence J =
  u, v 

Solved Example 68 : Solution :


The transformation from cartesian Here 0     and 0    2
coordinates (x, y) to polar coordinates (r, )   x,y,z 
J=
is given by  r, ,  
x = r cos  y = r sin 
sin  cos  r cos  cos  r sin  sin 
Solution :   sin  sin  r cos  sin  r sin  cos 
x x cos  r sin  0
r  cos  r sin 
J=  r = r2 sin 
y y sin  r cos 
r  and  f  x, y,z  dxdydz
R
and
  f r sin  cos ,r sin  sin ,r sin  
R f  x,y  dxdy  R f r cos ,r sin   rdrd R

r 2 sin drdd
Solved Example 69 :
The transformation from Cartesian
Solved Example 70 :
coordinates (x, y, z) to spherical polar
The transformation from Cartesian
coordinates coordinates (x, y, z) to cylindrical
(r, , ) is given by coordinates
x = r sin  cos  (r, , z) is :
y = r sin  sin  x = r cos , y = sin , z = z
z = r cos 

107
Vidyalankar : GATE – Engineering Mathematics

Solution : Solution :
cos  r sin  0 The equations are
  x, y,z 
J=   sin  r cos 0 = r AB  x  y = 
 r, ,z 
0 0 1 BC  x + y = 3
and  f  x, y,z  dxdydz CD  x  y = 
R
DA  x + y = 
  f r cos ,r sin ,z  rdrddz
D(0, ) C(, 2)
R 
x  y = 

x+y=
Solved Example 71 : x + y = 3

If u = x2  y2 and v = 2xy xy=


A(,0) B(2,)
Prove that
x x Substitute y  x = u and y + x = v
  x, y  u v 2x 2y Then   u   and   v  3
 =  
  u,v  y y 2y 2x
v u v u
u v  x= and y 
2 2
= 4 x2  y2   x x
  x, y 
u v
Solution : and I= 
  u,v  y y

We have x 2  y 2  x 2  y 2   2xy     2
u v

 u2  v 2 1 1

2 2 1
= 
 x 2
 y 2  u2  v 2 1 1 2
2 2
  u,v 
  4 u2  v 2 1
  x,y   |J|=
2
  u,v  1
  x  y  cos2  x  y  dxdy
2
   I =
  x, y  4 u v 2 2 R

3 
1
  u
2
 cos2 vdudv
Solved Example 72 : 2
Evaluate the integral 3
3
 cos
2
= vdv
  x  y   x  y  dxdy , where R is the
2 2
cos 3
R
3
parallelogram with successive vertices at 3 4
=
6  1  cos 2v  dv  3
(, 0), (2, ), (, 2) and (0, ) 

108
Notes on Calculus

Solved Example 73 : 
 r 2  1 2

= 
20
e rdr =   e r d r 2
20 2
 
Evaluate I = R x 2  y 2 dxdy by changing

  1 r 2   1 
to polar coordinates, where R is the =   e    
2 2 0 2  2  4
region in the x  y plane bounded by the

circles x2 + y2 = 4 and x2 + y2 = 9.  I=
2
Solution :
Using x = r cos  Solved Example 75 :
y = r sin  Evaluate the integral  zdxdydz , where T is
T
We get dx dy = r dr d and
the hemisphere of radius a, x2 + y2 + z2 = a2,
2 3 2 3
r 
3
I=   r r dr d     3 
0 2 0
d z  0.
2
Solution :
2
19 38 Changing to spherical coordinates
=
3 0 d =
3
substitute
x  r sin  cos , y  r sin  sin , z  r cos 
Solved Example 74 :
0    2 and 0    /2
Evaluate the improper integral
We get,

 x2 dx dy dz = r2 sin dr d d
I= 0 e dx
2 / 2 a
 0 0  r cos   r
2
Solution : I = sin  dr d d

I2 = I  I 2 / 4
a4
  2    2     x2  y2  =
4 0 0 sin  cos dd
=   e x dx    e  y dy     e dxdy
0  0  00 2 / 2
a4
Put x = r cos  =
8 0 0 sin2dd
y = r sin 
2 /2
a4  cos 2 
Hence J = r =
8 0   2  d
 0
The region of integration is the entire first
2
quadrant. Hence r varies from 0 to  and  a4
=
8 0 d
varies from 0 to /2.
 /2 a 4
2
=
 I2 = 0 0 er rddr 4

109
Vidyalankar : GATE – Engineering Mathematics

Solved Example 76 : The projection of the given region on the


Using triple integral find the volume of the x  y plane is the triangle bounded by the
2 2 2 2
sphere x + y + z = a lines
Solution : x = 0;
V = 8  dxdydz where D is the region x y
y = 0 and  1 0xa
D a b
bounded by the sphere x2 + y2 + z2 = a2 in In this region x varies from 0 to a.
the first quadrant.
 x
For fixed x, y varies from 0 to  1   b.
Substitute x = r sin  cos , y = r sin  sin  a
, z = r cos   x
2
i.e. 0  y   1   b
Jacobian | J | = r sin   a
and the limits r = 0 to a; 0    /2 For fixed (x, y), z varies from 0 to
0    /2  x y
1 a  b  c
a /2 /2  
 V = 8  0 J dddr
  x y 
0  z   1  a  b  c 
0 0

a /2 /2    
= 8 0 0 r
2
sin dddr
0
 I = 
D
dxdydz

a /2

0 r   cos 0 ddr


/2  x  x x
= 8 2
 1 b  1   c
a  a  a b
0
= 0 0 0 dzdydx
a  /2 a
= 8  r 2  0 dr
/2
= 8  r d dr
2
 x
0 0 0  1 b
a  a
 x y
a
= c 0  1  a  b  dydx
a
r  3
0  
= 4 r 2 dr = 4  
0  3 0  x
 1 b

a
x y2  a

4a 3 = c   1   y   dx
= 0 
a 2b  0
3
a
x 
2 2
 x 1
= bc   1     1    dx
Solved Example 77 :  a  2  a  
0

Find by triple integral the volume of the


a 2
bc  x
2 0  a 
tetrahedron bounded by the planes x = 0, = 1 dx
x y z
y = 0, z = 0 and   1 a
a b c bc  a  x 
3
abc
=  1   =
Solution : 2  3  a   6
0

110
Notes on Calculus

Solved Example 78 : = a 2b2 c 2  uvwdudvdw


R
Evaluate I = 
R
xyzdxdydz where D is the
Now substitute
positive octant of the ellipsoid u = r sin  cos 
x 2 y 2 z2 v = r sin  sin 
  1
a 2 b2 c 2 w = r cos 
Solution : Then J = r2 sin 
Substitute x = au, 1 /2 /2

y = bv I  a 2 b2 c 2  0 0 r
5
sin3  cos  cos  sin dddr
0
z = cw 1 /2 /2
 a 2b2 c 2  r 5 dr 0 sin3  cos d  sin  cos d
a 0 0
  x,y,z  0 0
J=  0 b 0  abc 1 /2 /2
  u,v,w  1  1  1 
= a 2b2 c 2  r 6   sin4    sin2  
0 0 c 6 4 2
 0  0  0
Let R be the positive octant of the sphere
a 2 b2 c 2
2 2 2 =
u +v +w =1 48
 I=   abc uvw  abc  dudvdw
R

2.12 Application of Integration


Solved Example 79 : The latus rectum of the parabola y2 = 4ax
Calculate the area bounded by the is a line parallel to y  axis. The shaded
2
parabola y = 4ax and its latus rectum. portion is the desired area. As the
Solution : parabola is symmetric about the xaxis,
therefore area
y
L OLL = 2  Area OSL.
a a a
= 2  ydx  2  4axdx  4 a  x1/ 2 dx
0 0 0
S
0 (a, 0) x x3 / 2 8
a

= 4 a  a a3 / 2  0
3 / 2 x 0 3

8a 2
=
3

111
Vidyalankar : GATE – Engineering Mathematics

Solved Example 80 : 1 3
= 0 4xdx    3  x  dx
Find the area of the region bounded by the 1

graph of y = sin x and y = cos x between 1 3


 2   x2 
x = 0 and x = /4. = 2  x 3/ 2   3x  
 3 0  2 1
Solution :
4 10
= 2 = sq. units.
3 3

Solved Example 82 :
Find the area of the region bounded by the
two parabolas y = x2 and x = y2.

/4
 Required area = 0  cos x  sinx  dx
x = y2
P
/4
=  sinx  cos x  0 B
y = x2
2 2 A
=    0  1
2 2 0 M
=  
2  1 sq. units.

Solution :
Solved Example 81 :
Let us first find the points of intersection
Find the area above the xaxis bounded solving y = x2 and x = y2
2
by y = 4x and the line x + y = 3 y = y4
Solution :  y (y3  1) = 0
y
 y=0  x=0
(1, 2)
A y=1  x=1
For the curve x = y2
(3, 0)
1 1
0 C B Now the area OBPM =  ydx 
0
0 xdx

1
 x3 / 2 
 
 3 / 2 0
Required area = area OCA + area ACB 2
 sq.units
3

112
Notes on Calculus

For the curve y = x2 Solved Example 84 :


1 Find the area of one turn of the
Area OAPM = 0 ydx archimedan spiral r = a.
1 Solution :
1
 x3  1
  x dx     sq. units.
2

0  3 0 3
2 1 1
The required area =   sq. units.
3 3 3 O
A
Solved Example 83 :
Find the area bounded by the cardioid
2 2
1 2 1
r = a(1 + cos ) The area is A = 
20
r d  a 2  2 d 
2 0
Solution :
a 2  2 
3
4
=  3 a 2
2 3 3
O A

Solved Example 85 :
Find the area bounded by the curves :
y = 2x5 + 3 and y = 32x + 3
A cardioid is symmetrical about the initial
Solution :
line.
y

1 C
Hence the required area = 2  r 2 d since
0
2

 varies from 0 to  for the upper part.


(0, 3) y=3

0 a 1  cos   B
2 2
= d
x x
 /2

= 4a 2  cos4 d  4a 2 0 cos
4
t2dt A
0
2

 y = 32x + 3
(putting  t) y
2
/2 Two curves will intersect when
3 1
= 8a 2 0 cos4 tdt  8a 2
422 2x 5  3  32x  3  2x  x 4  16   0

3a 2  In the interval (2, 0) ; 2x 5  3  32x  3


=

113
Vidyalankar : GATE – Engineering Mathematics

and in the interval (0, 2) ; 32x + 3  2x5 + 3  x6 


0

2
x6 
=   16x 2    16x 2  
Thus the required area is given by  3  2  3  x 0

0 128 128 256


  2x  =   sq. units.
5
A =  3  32x  3 dx 3 3 3
2


  32x  3  2x 5  3 dx 
0

2.13 Partial and Total Derivatives


Functions of two variables
If three variables x, y, z are so related that the value of z depends upon the values of x
and y, then z is called the function of two variables x and y and this is denoted by z = f(x, y)

Partial derivatives of first order


Let z = f(x, y) be a function of two independent variables x and y. If y is kept constant and
x alone is allowed to vary then z becomes a function of x only. The derivative of z with
respect to x, treating y as constant, is called partial derivative of z with respect to x and is
denoted by
z f
or or fx
x x
z f(x  h, y)  f(x,y)
 lim
x h  0 h
Similarly the derivative of z with respect to y, treating x as constant, is called partial
derivative of z with respect to y and is denoted by
z f
or or fy
y y

z f(x,y  k)  f(x,y)
Thus  lim
y k 0 k

z z
and are called first order partial derivatives of z
x y

114
Notes on Calculus

Note : (i) If z = u + v where u = f(x, y) , v   (x, y) , then z is a function of x and y.


z u v z u v
   ;  
x x x y y y

(ii) If z = uv, where u = f(x, y) , v = (x, y)


z  v u
then  (uv)  u v
 x x x x
z  v u
 (uv)  u v
y y y y

u
(iii) If z = , where u = f(x, y), v =(x, y)
v
u v
v u
z  u x x
then  
dx x  v  v2
u v
v u
z  u y y
 
y y  v  v 2

(iv) If z = f(u) where u =  (x, y)


z dz u z dz u
then  . ;  .
x du x y du y

Partial derivatives of higher order


  z   2 z
 or fxx
x  x  x 2

  z   2 z
  or fyy
y  y  y 2

  z  2z
   or fxy
x  y  xy

  z  2z
   or fyx
y  x  yx

2z 2z
In general  or fxy = fyx
xy yx

115
Vidyalankar : GATE – Engineering Mathematics

Solved Example 86 : Solved Example 88 :


x
First order partial derivative of u = y is If u = log (tan x + tan y) show that
Solution : u u
sin 2x + sin 2y =2
u =y x x y
Treating y as constant Solution :
u u = log(tan x + tan y) [ u is symmetrical
= y x log y
x with respect to x and y]
treating x as constant u 1 
= . (tan x  tany)
u x tan x  tany x
= x yx1
y
sec 2 x
=
tanx  tany
Solved Example 87 :
u
Similarly
Find first order partial derivative of y
x2  y2 sec 2 y
u = tan1 =
xy tanx  tany
Solution :
u u
= sin 2x + sin 2y
u 1   x2  y2  x y
= .  
x  x 2  y 2  x  x  y 
2
1
1   = [sin 2x sec2x + sin 2y sec2y]
 xy  tanx  tany

(x  y)2 1
= =
(x  y)2  (x 2  y 2 )2 tanx  tany

 2   1 1 
(x  y)
x
(x  y 2 )  (x 2  y 2 ) (x  y)
x 2 sinx cos x. cos2 x  2 siny cos y cos2 y 
  
(x  y)2
(tan x  tany)
(x  y).2x  (x 2  y 2 ).1 =2 =2
= tanx  tan y
(x  y)2  (x 2  y 2 )2

u x 2  2xy  y 2
  Solved Example 89 :
x (x  y)2  (x 2  y 2 )2
3y 3 y
u y 2  2xy  x 2 If u = xy show that 
Similarly 
2
x y xyx
y (x  y)2  (x 2  y 2 )2
Solution :
u = xy
u
= xy log x
y

116
Notes on Calculus

2 x   u  1 2 y   y 
     yx y 1 log x  x y   x y 1  yx y 1 log x
xy x  y  x yx y  x 

= xy1 (y log x + 1) = xy1 (y log x + 1)

3 y   2 x   y 1  3u    3u   y 1
    [x (y log x  1)]      x (y log x  1)
2
x y x  xy  x xyx x  yx  x
...(1) …(2)
u  3u  3u
= y xy1 from (1) and (2) 
x x 2 y xyx

Euler’s theorem on Homogenous functions


If u is a homogenous function of degree n in x and y then

u u
x +y = nu
x y

Since u is a homogenous function of degree n in x and y it can be expressed as


u = xn f
u y  y   y 
 = n xn1f    xn f '   .  2 
x x x x 
u y y
x = n x n f    x n 1 y f '   ...(1)
x x x
u y 1 y
Also = xn f    . = xn1 f   
y x x x
u y
y = xn y f    …(2)
y x
Adding (1) and (2) we get
u u y
x +y = n xn f   = nu
x y x

Note : Euler’s theorem can be extended to a homogenous function of any


number of variables. Thus if u is a homogenous function of degree n in x,
y and z then
u u u
x +y +z = nu
x y z

117
Vidyalankar : GATE – Engineering Mathematics

Solved Example 90 : Solved Example 91 :


1 1 log x  log y x2 y2
If f(x, y) = 2
  If u= show that
x xy x2  y2 xy

f f  2u  2u u
show that x  y + 2f (x, y) = 0 x  y  2
x y x 2
x y x
Solution : Solution :
1 1 log x / y x2 y2
f(x, y) = 2
  2 u = is a homogenous function of
x xy x  y2 xy
tx degree 3 in x, y
log
1 1 ty
f(tx, ty) = 2 2  2  2 2 2 2  By Euler’s theorem we have
t x t xy t x t y
u u
x +y =3u
 x  x y
1 log 
= 2 1 1 y
t  2   2  Differentiating (1) partially with respect to x
 x xy x  y 2 
 2u u 2 y u
= t2 f (x, y) x  1.  y  3
x 2
x xy x
f (x, y) is a homogenous function of degree
 2u  2u u
2 in x and y or x  y  2
x 2 xy x
By Euler’s theorem, we have
f f
x y = 2 f (x, y)
x y

Composite functions
(i) If u = f (x, y) where x = (t), y = (t)
du
then u is called a composite function of (the single variable) t and we can find .
dt
(ii) If z = f(x, y) where x = (u, v), y = (u, v)
then z is called a composite function of (2 variables) u & v so that we can find
z z
&
u v

Total derivative of composite functions


If u is a composite function of t, defined by the relations
u = f(x, y) ; x = (t), y = (t) then
du u dx u dy
 .  .
dt x dt y dt

118
Notes on Calculus

Note : 1. If u = f (x, y, z) and x, y, z are functions of t then u is a composite


function of t and
du u dx u dy u dz
 .  .  .
dt x dt y dt z dt
2. If z = f (x, y) and x, y are functions of u and v, then
z  z x z  y
 .  .
u  x u  y  u

 z z x z y
 .  .
v x v y v

3. If u = f (x, y) where y = (x)


then since x = (x), u is a composite function of x
du u dx u dy
  .  .
dx x dx y dx

du u u dy
   .
dx x y dx

du
is called the total differential coefficient of u, to distinguish it from its
dx
u
partial derivative .
x

Solved Example 92 : = 2a3t3(5t + 8)


du Also u = xy2 + x2y = at2 . 4a2t2 + a2 t4. 2at
Find when u = x when u = xy2 + x2 y,
dt = 4a3t4 + 2a3 t5
x = at2, y = 2at. du
= 16a3 t3 + 10a3 t4 = 2a3 t3 (5t + 8)
Solution : dt
The given equations define u as a
composite function of t. Solved Example 93 :
du u dx u dy If u = sin1(x  y), x = 3t, y = 4t3 show that
  .  .
dt x dt y dt du 3
2 2
=
= (y + 2xy). 2at + (2xy + x ) . 2a dt 1 t2
= (4a2t2 + 2at2 . 2at) 2at Solution :
+ (2at2 . 2at + a2 t4).2a du u dx u dy
 .  .
= 8a3t3 + 8a3 t4 + 8a3t3 + 2a3 t4 dt x dt y dt

119
Vidyalankar : GATE – Engineering Mathematics

1 z z
= .3 = x y
1  (x  y) x y

1
 (1) . 12 t 2 Solved Example 95 :
1  (x  y) 2
If  (x, y, z ) = 0 show that
3(1  4t ) 2
3(1  4t ) 2
 y   z   x 
= 
1  (x  y) 2
1  (3t  4t 3 )2  z   x   y    1
  x   y  z

3(1  4t 2 ) Solution :
=
1  9t  24t  16t
2 4 6
The given relation defines y as a function

3(1  4t 2 ) of x and z. Treating x as constant.


=
(1  t 2 )(1  8t 2  16t 2 ) 
 y  z
3(1  4t 2 ) 3    
=   z  x
2
(1  t ) (1  4t ) 2 2
1 t2 y
The given relation defines z as a function
of x and y. Treating y as constant.
Solved Example 94 :
If z is a function of x and y 
 z  x
where x = eu + ev and y = eu  ev,    
 x  y
show that z

z z z z 
 x y  x 
u v x y y
Similarly   
 y z 
Solution : x
Here z is a composite function of u and v Multiplying we get the desired result.
z z x z y
  .  .
u x u y u
Solved Example 96 :
z u z If u = f (y  z, z  x, xy) prove that
= e  ( e  u )
x y
u u u
z z x z y   =0
and  .  . x y z
v x v y v
Solution :
z z
= (e v )  (e v ) Here u = f (X, Y, Z)
x y
where X = y z, Y = z  x, Z = xy
subtracting
 u is a composite function of X, Y and Z
z z z z
  (eu  e  v )  (e u  e v )
u v x y

120
Notes on Calculus

u u X u Y u Z u u X u Y u Z
 .  .  .  .  .  .
x X x Y x Z x z X z Y z Z z
u u u u u u
= (0)  (1)  (1) = (1)  (1)  (0)
X Y Z X Y Z
u u X u Y u Z Adding
 .  .  .
y X y Y y Z y u u u
  =0
u u u x y z
= (1)  (0)  (1)
X Y Z

2.14 Taylor's Series and Maclaurin's Series


Taylor's series is a representation of a function as an infinite sum of terms that are
calculated from the values of the functions derivatives at a single point.


f (a)(x  a) (x  a)2 f (n) (a)(x  a)n
f(x) = f(a) +
1!
f (a).
2!
..... =  n!
n 0

Note : Putting x = a + h, Taylor’s series becomes

h2 hn n
f  a  h   f  a   hf   a   f   a   ...  f a  …
2! n!

If the Taylor's series is centered at zero, that series is also called Maclaurin’s series.
If a = 0 in Taylor series of f(a + h), then the series becomes

h2 hn n
f  h   f  0   hf   0   f   0   ...  f  0   ...
2! n!

This is called as Maclaurin’s series about a = 0.

Taylor's series expansion of some functions


x x2 x3
 ex = 1   ....
1! 2! 3!
1 1 1
 e = 1   .... = 2.7182….
1! 2! 3!

x3 x5 x7 
(1)n
 sin x = x   ..... x 2n1
3! 5! 7! n 0 (2n  1)!

121
Vidyalankar : GATE – Engineering Mathematics

x2 x4 x6 
(1)n 2n
 cos x = 1   .... x
2! 4! 6! n  0 (2n)!

x3 2 5
 tan x = x  x ....
3 15

Taylor's series in two variables


Taylor's series of a function in two variables, f(x, y) about (a, b) is

f(x, y) = f(a, b) + (x  a) fx (a, b) + (y  b) fy (a, b)


1
+ (x  a)2 fxx (a,b)2(x  a)(y  b)fxy (a,b)(y  b)2 fyy (a,b) ....
2! 

Solved Example 97 : Solved Example 98 :


Expand cos z in a Taylor’s series about Obtain the Taylor’s series for sin x.
z/4 Solution :
Solution : f(x)  sin x
f (x)  cos x
f  z   cos z
f (x)   sinz
f   z    sinz
f (x)   cos z
f   z    cos z
By Maclaurin’s series
f   z   sinz
x2
 1 f(x)  f(0)  xf (0)  f (0)
f   2!
4 2
xn n
 1 ...  f (0)  ...
f     n!
4
  2
 1 x 2k 1
k
x3 x5
 1 sinx  x    ...   ...
f      3! 5!  2k  1 !
4 2
 1
f     Solved Example 99 :
4
  2
Obtain the Taylor’s series for cos hx.
By using Taylor’s series
Solution :
   
f  z   f     z   f   x2 x3
4
   4  4 ex  1  x    ...
2! 3!
z   / 4
2

 f     … x2 x3
2! 4 e x  1  x    ...
2! 3!
1    
2
 1  ex  e x x2 x4
= 1   z     z   ... coshx   1   ....
2   4  2!  4   2 2! 4!

122
Notes on Calculus

Solved Example 100 : h2 h3


f  x   f  2   hf   2   f   2   f   2 
Find the Taylor’s series expansion for the 2! 3!

function f  x   1  2x  2x 2  3x 3 about the h2 h3


= 29  h  42   40   18 
2 6
point x = 2.
Solution : f  x   29  42h  20h2  3h3

f  x   1  2x  2x 2  3x 3 f  2   29 where h = x  2
f   x   2  4x  9x 2 f   2   42
f   x   4  18x f   2   40
f   x   18

2.15 Fourier Series


Periodic function :
A function f(x) is said to be periodic if f(x + T) = f(x) for all values of x where T is the same
positive number.
For Example:
1. sin x = sin (x + 2)
= sin (x + 4) and so on.
sin x is periodic in x with period 2.
2. The period of tan x is .

Fourier series (General form)


Let f(x) be defined in (L, L) then Fourier series is given by


 nx nx 
f  x   a 0    an cos  bn sin
n 1  L L 

L
1
f  x  dx
2L L
where a0 

L
1 nx
f  x  cos
L L
an  dx
L
L
1 nx
bn   f  x  sin dx
L L L

123
Vidyalankar : GATE – Engineering Mathematics

Similarly if f(x) is defined in (0, 2L) then Fourier series is given by



 nx nx 
f(x)  a 0    an cos  bn sin
n 1  L L 
2L
1
a0 
2L  f(x)dx
0

2L
1 nx
an  
L 0
f(x)cos
L
dx

2L
1 nx
L 0
bn  f(x)sin dx
L

Odd and Even Functions :


Let f(x) be defined in (C, C)
1. If f(x) = f(x), for all x in (C, C) then f(x) is even function.
2. If f(x) = f(x), for all x in (C, C) then f(x) is an odd function.

Note : Unless ‘0’ is midpoint of the given interval, we cannot talk of the function
being even or odd.
For Example: f(x) = x2 in (0, 3) is neither even nor odd, since if x  (0, 3)
then x  (0, 3) and hence f(x) is not defined.

But if, f(x) = x2 in (3, 3) then f(x) is even function.

Graphical representation of even and odd function :


i) If f(x) is even, then its graph is symmetrical about f(x) axis
For Example: If f(x) =  x,  < x < 0
=x 0<x<
then,
f(x)

x
 

124
Notes on Calculus

ii) If f(x) is odd then its graph is symmetrical about origin. f(x)
For Example: if f(x) = a , if  < x < 0 (0, a)
=a , if 0 < x < 

x
 

(0, a)

Fourier expansion of an even function


Let f(x) be defined in (, ) and let f(x) be even function. Fourier series is given by

f(x)  a 0    an cosnx  bn sinnx 
n 1

 
1 1
where, a 0  
2 
f(x)dx   f(x)dx
0
f(x) is even

 
1 2
an   f(x)cosnxdx   0 f(x)cosnxdx
 
[ f(x) and cos nx are even functions]


1
bn   f(x)sinnxdx  0
 
[f(x) sin nx is odd]


f(x)  a 0   an cosnx [Half range cosine series in (0, )]
n 1

Fourier expansion of an odd function


Let f(x) be an odd function in (, )

f(x)  a 0   an cosnx  bn sinnx
n 1


1
ao   f(x)dx  0
2 
f(x) is odd function


1
an   f(x)cosnx dx  0
 
f(x) cos nx is odd

 
1 2
bn   f(x)sinnxdx =  0 f(x)sinnxdx
 

f(x)   bn sinnx dx {Half range sine series in (0, )}
n 1

125
Vidyalankar : GATE – Engineering Mathematics

Solved Example 101 :  sin2x sin3x 


Hence x    2  sin x    ...
 2 3 
Find the Fourier series representing

f(x) = x, 0 < x < 2 and draw its graph from


Solved Example 102 :
x = 4 to x = 4.
Find Fourier Series for f(x) = x2 in  ,  
Solution :
and show that

ao
f x  x     an cosnx  bn sinnx  1 1 1 2
2 n 1    .... 
12 22 32 12
where
Solution :
2 2
1 1 1 4 2
a0 
 0 f  x  dx   0 xdx 
 2
 2 We have f(x) = x2 is an even function in

2
 ,  
1
an =
  x cosnxdx
0 
nx
f  x   a 0   an cos
n 1 
1   x sinnx  
2 2
1
= 
   n  0
 0 n
sinnxdx  
1 2 2
 i) a0  
0
x dx 
3
2
1  cosnx 
=  x sinnx  =0
n  n  0 
2 2 nx
ii) an  
0
x cos

dx
2
1
bn =
  x sinnxdx
0 Integrating by parts

2
1  x cosnx sinnx  2 2  nx 
=  =  x  sin
  n n2  0    n  

1 2 2 
=     2 nx    3 nx  
 n n 2x  2 2 cos   2  3 3 sin 
n    n     0

2  2 3 n 4 2
 2 2  1   2 2  1
n
=
 n   n
4 2 0 2 4

126
Notes on Calculus

 1 2
n
 2 4 2 
 nx  1   cosnx x cosnx sinnx 
 f x 
3
 2


n 1 n2
cos 
  
 …(1) =
  n

n

n2  0

putting x = 0 in equation (1) we get 1  2 cos 2n  2


=
  n n

 1
n
 2 4 2 
0
3
 2


n 1 n2
Solved Example 104 :
2  1 1 1 1  Find a Fourier expansion for
   2  2  2  2  ...
12  1 2 3 4 
f(x) = x2 for  < x < 
1 1 1 1 2 Solution :
 2
 2  2  2  ... 
1 2 3 4 12
x2 is an even function
 
1 1 1 3 2
Solved Example 103 : a0  
2 
x 2 dx   x 2 dx 
0 3
 
3
Find a Fourier series to represent
 
f(x) =   x for 0 < x < 2. 1 2
an  
 
x 2 cosnxdx   x 2 cosnxdx
0
Solution :
2 2
The indefinite integral
1 1
a0 
 0 f  x  dx       x  dx 2  x 2 sinnx sinnx 
0 =   2xdx 
 n n 
1  x2  
2

 x 0    
2
= 2  x 2 sinnx 2x cosnx cosnx 
  2 0  =     2dx 
  n n n n2

1
= 2  2   0 2  x 2 sinnx 2x cosnx 2 sinnx 
 =    
 n n2 n3 
2
1
an =
  f  x  cosnxdx 2  x 2 sinnx 2x cosnx 2 sinnx 

0  an     
 n n2 n3  0
2
1
=
     x  cosnxdx 4 4
2 
0 =  cosn  2 cosn
n n
2
1  sinnx x sinnx cosnx  bn = 0
=   
  n n n2  0
4 4 4
Integrating by parts. a1  2
,a 2  2 ,a 3  2 etc
1 2 3
1 Hence
= 0  0

2 4 4 4
2 x2   2 cos x  2 cos 2x  2 cos 3x  ...
1 3 1 2 3
bn =
     x  sinnx dx
0

127
Vidyalankar : GATE – Engineering Mathematics

Solved Example 105: Solved Example 106 :


Find the Fourier series for f(x) = sin h ax in Find the Fourier series for
(, ) 2 x 2
f(x) =  in  < x <  and show
Solution : 12 4
Clearly f(x) is an odd function of x 1 1 1 2
that   .... 
f(x) = sin h a(x) =  sin h ax =  f(x) 12 22 33 12
 Solution :
Let, sin h ax = 
n 1
bn sinnx
Note that f(x) is an even function of x

2
 2 ( x)2  x 2
where, bn =  sinhax  sinnx dx  f(x)      f(z)
0 12 4 12 4

2 e
 ax
e  ax Let f(x) = a 0   an cosnx
 0
= sinnx dx n 1
2

1  2 2 
 0  12 4 
a0 =    dx
1  ax 
 

  e sinnx dx   e sinnx dx 
 ax
=
 0 0  
1  2 x3 
1 1 =  x  0
=  eax  a sinnx  ncosnx    12 12  0
  a 2  n2

 2  2 x 2 
 0  12 4 
1  an =    cosnxdx
 2 2
e ax  a sinnx  ncosnx  
a n 0
Integrating by parts
1 ea  n  1n  ne a  1n 
=

 a 2  n2   
=
2  2 x 2
 
  sinnx 
 
  12 4  n 
n  1
n

= e a
 e  a  


 a n 2 2
   x    cosnx   1   sinnx  
  2    2  3 
 2  n    n 0
2n  1 sinha 
n
e a  e  a 
=  sinha    2    (1)
n
(1)n1

 a 2  n2   2  =
  2n2
cosn  


n2

n2

2n  1
n 1
sinha 2 x 2 
(1)n1
=    cosnx

 a 2  n2  12 4 n1 n2
Put x = 0 we get
   1n   1 1n   1n1   1n1 
  2 
(1)n1  1 1 1 1 
   2  2  2  2  ...
2 sinha  n  1 sinnx
n 1
12 n1 n2 1 2 3 4 
 f(x) =


n 1 a 2  n2

128
Notes on Calculus

List of Formulae
 Limits d x
a  a x log e a
sinx dx
lim =1 …x in radians
x 0 x d
sin x = cos x
tanx dx
lim =1 …x in radians
x 0 x d
cosec x =  cosec x cot x
limcosx = 1 dx
x 0

d
x n  an cos x =  sin x
lim nan1 dx
x a x  a

d
ax  1 sec x = sec x tan x
lim = na (a > 0) dx
x 0 x
d
ex 1 tan x = sec2x
lim =1 dx
x 0 x
d
1 cot x =  cosec2x
lim1  x) = e x dx
x 0
d x 1
n(1  x) sin1 
lim 1 dx a a  x2
2
x 0 x
d x 1
cos 1 
dx a a  x2
2
 Continuity
The function f is said to be continuous d x a
tan1  2
at x = x0 if dx a a  x2

i) lim f(x)   exists and d x a


x  x0 cot 1  2
dx a a  x2
ii) lim f(x)  f(x 0 )
x  x0 d x a
sec 1 
dx a x x  a2
2
 Derivatives
d d x a
k(constant)0 cosec 1 
dx dx a x x2  a2
d n d
x nxn1 cos hx = sin hx
dx dx
d 1 d
log x  sin hx = cos hx
dx x dx
d x d 1
e  ex sin h1x =
dx dx x 12

129
Vidyalankar : GATE – Engineering Mathematics

d 1 dx 1 x
dx
cosh1x =
2
x 1
a 2
x 2
 tan1  c
a a

d 1 dx 1 x
(tanh1 x)  2 a 2
x 2

a
cot 1  c
a
dx x 1
dx 1 x
x x a2 2

a
s ec 1  c
a
 Integration
x n 1 dx 1 x
 x dx  n  1  c ,
n
(n  1) x x a2 2

a
cos ec 1  c
a

1
 x dx  log x  c  sinh x dx = cos h x + c

 cosh x dx = sin h x + c
 e dx  e
x x
c
dx
ax  = sin h1 x + c
 a dx 
x
+c x 12
log e a
dx
 cos xdx  sin x  c  x 12
= cos h1 x + c

 cos ecx cot xdx   cos ecx  c dx


x 2
1
= tan h1 x + c
 sin x dx   cos x  c
 tanxdx =  log (cos x) + c
 sec x tanx dx  sec x  c
 x
 sec x dx  tanx  c
2  sec xdx  logtan  4  2 c
 x
 cos ec x dx
2
= cot x + c
 cosec xdx log  tan 2  +c

 cot xdx = log sin x + c


dx x x
  sin1 or cos 1 c
a a
 tanxdx a x2 2
= log sec x + c
dx x
 sec xdx  log(sec x  tan x)c   sinh1  
a
a x2 2

 cosecxdx   log(cosecx  cot x)c or log(x  x 2  a 2 )c


1 x
  sin1 c dx x
a x
2 2 a  x a2
 cosh1  c
2
a
dx x
 2
a x 2
  cos 1
a
c orlog(x  x 2  a 2 )c

130
Notes on Calculus

dx 1 x Some Important Substitutions In


a 2
x 2
 tan1  
a a Integration
1 1  x  Rule 1: If the numerator is equal to or
or cot  c
a a a multiple of the derivative of the
1 denominator, then put the denominator
 a 2  x 2 dx 
2
x a2  x2
equal to some other variable say z. It
1 x is then reduced to standard form after
 a 2 sin1  c
2 a simplification.
1
 a 2  x 2 dx 
2
x a2  x2 Rule 2: If there are two factors in the
integrand one of which is of the type
1 x
 a 2 sinh1  c
2 a  [f(x)] and other of the type f (x) then
put f(x) = z.
1
 x 2  a 2 dx 
2
x x 2  a2
Rule 3: If the integrand is of the
1 x
 a 2 cosh1  c type a 2  x 2 , then put x = a sin  or
2 a
x = a cos
dx 1 ax
 a2  x2  2a log a  x c
Rule 4: If the integrand is of the type
dx 1 xa x2 + a2 or some power of it then put
 x2  a2  2a log x  a c
x = a tan  or x = a cot

Rule 5: If the integrand is of the


Integration by parts
 du  type x 2  a 2 , then put x = a sec  or
 uvdxu vdx  dx  vdx dx x = a cosec 
Of the two functions the first function u Rule 6: If the integrand is of the
is selected based on the following
ax
type , then put x = a cos 2 and
preference order ax
1. Inverse function
a2  x2
2. Logarithmic function if it is of the type , then put
a2  x 2
3. Algebraic function
x2 = a2 cos 2
4. Trigonometric function
5. Exponential function Rule 7: The presence of ax
suggests the substitution of a + x = t2
or x = a tan2 or x = a cos 2.

131
Vidyalankar : GATE – Engineering Mathematics

Rule 8: The presence of 2ax  x 2 2 tanA


tan 2A =
1  tan2 A
suggests the substitution
sin 3A = 3 sin A  4 sin3A
x = a (1  cos)
cos 3A = 4 cos3 A  3 cos A

 Important Trigonometric Identities 3 tanA  4 tan3 A


tan 3A =
1  3 tan2 A
sin2 A + cos2 A =1
sin (A + B) = sin A cos B + cos A sin B
 Definite Integration
cos (A + B) = cos A cos B  sin A sin B
b
x b
tan (A + B) =
tanA  tanB
 f(x)dx    x  x a
  b     a 
1  tanA tanB a

sin (A  B) = sin A cos B  cos A sin B


Properties of definite integral
cos (A  B) = cos A cos B + sin A sin B b b

tan A  tanB  f(x)dx f(t)dt


tan (A  B) = a a
1  tan A tanB
b a
sin2 A  sin2 B = sin (A + B) sin (A  B)  f(x)dx  f(x)dx
a b
cos2 A  sin2 B = cos (A + B) cos (A  B)
b c b
2 sin A cos B = sin (A + B) + sin (A  B)
 f(x)dx f(x)dx f(x)dx
a a c
2 cos A sin B = sin (A + B)  sin (A  B)
a a
2 cos A cos B = cos (A + B) + cos (A  B)
 f(x)dx f(a  x)dx
2 sin A sin B = cos (A  B)  cos (A + B) 0 0

a a
CD CD
2 sin
2
cos
2
= sin C + sin D  f(x)dx2 f(x)dx …f(x) = f(x)
a 0

CD CD =0 …f(x) = f(x)


2 cos sin = sin C  sin D
2 2 2a a

2 cos
CD
cos
CD
= cos C + cos D
 f(x)dx2 f(x)dx
0 0
…f(2a  x) = f(x)
2 2
=0 …f(2a  x) = f(x)
CD DC
2 sin sin = cos C  cos D
2 2
cos 2A = cos2 A  sin2 A = 1  2sin2 A  Euler’s theorem on Homogenous

1  tan2 A functions
= 2 cos2A  1 =
1  tan2 A u u
x +y = nu
x y
2 tanA
sin 2A = 2 sin A cos A =
1  tan2 A

132
Notes on Calculus

 Total derivative of composite  Maclaurin's Series


functions h2
f  h  f  0   hf   0   f   0   ...
du u dx u dy 2!
 .  .
dt x dt y dt hn n
 f  0   ...
n!
 Taylor's series
f (a)(x  a)  Fourier Series
f(x) = f(a) + 
1!

 nx nx 
f  x   a 0    an cos  bn sin
L 
2
(x  a) L
f (a).  .... n 1 
2!
L
1
f  x  dx
2L L

f (n) (a)(x  a)n where a0 
= 
n0 n!
L
f(x, y) = f(a, b) + (x  a) fx (a, b) 1 nx
an   f  x  cos dx
L L L
+ (y  b) fy (a, b)
L
1 nx
(x  a)2 f (a,b)  bn   f  x  sin dx
1  
xx
L L L
+ 2(x  a)(y  b)fxy (a,b)   ....
2!  
2
(y  b) fyy (a,b) 



133
Vidyalankar : GATE – Engineering Mathematics

Assignment  1
Duration : 45 Min. Max. Marks : 30

Q 1 to Q 10 carry one mark each 1


(A) 0 (B)
xsinx 2
1. Lt  is [ME  2014]
1cosx
x0 1
(C) (D) undefined
(A) 0 (B) 1 4

(C) 3 (D) not defined 6. If a continuous function f(x) does not


have a root in the interval [a, b], then
2. If a function is continuous at a point,
which one of the following statements
(A) the limit of the function may not
exist at the point is TRUE? [EE  2015]

(B) the function must be derivable at (A) f(a)f(b) = 0 (B) f(a)f(b)0

the point (C) f(a)f(b)0 (D) f(a) f(b)0


(C) the limit of the function at the point
a y
tends to infinity 7. The double integral  
0 0
f(x,y)dxdy is

(D) the limit must exist at the point and equivalent to [IN  2015]
the value of limit should be same x y

as the value of the function at that


(A)  
0 0
f(x,y)dxdy

a y
point [ME  2014] (B)  
0 x
f(x,y)dxdy
x
 1 a a
3. The value of lim  1  is [EC  2014]
x
(C)  
0 x
f(x,y)dydx
x 

a a
(A) n2 (B) 1.0 (D)  
0 0
f(x,y)dxdy
(C) e (D) ∞ n

 1
8. The value of  n   is _________.
1 n 0  2 


4. The series n 0
converges to
n!
[EC  2015]
[ME  2014]
loge (1  4x)
9. Lt is equal to
(A) 2 In 2 (B) 2 x 0 e3x  1
(C) 2 (D) e [ME  2016]
1
(A) 0 (B)
1cos(x ) 2 12
5. The value of limx 0  is
2x 4 4
(C) (D) 1
[ME  2015] 3

134
Assignment on Calculus

x 3  sin(x) 17. A political party orders an arch for the


10. The value of limx 0  is
x entrance to the ground in which the
[ME  2017]
annual convention is being held. The
(A) 0 (B) 3
(C) 1 (D) 1 profile of the arch follows the equation
y = 2x  0.1x2, where y is the height of
Q 11 to Q 20 carry two marks each
the arch in meters. The maximum
11. The minimum value of the function
possible height of the arch is
1
f(x) = x(x 2  3) in the interval [CS, ME-2012]
3
(A) 8 meters (B) 10 meters
100 ≤ x ≤ 100 occurs at
(C) 12 meters (D) 14 meters
x = ________. [EC  2017]
18. Consider the function f (x) = |x| in the
12. The maximum value of
interval 1  x  1. At the point x = 0,
f(x) = 2x3  9x2 + 12x  3 in the interval
f (x) is [ME  2012]
0  x  3 is _____. [EC  2014]
(A) Continuous and differentiable
13. Consider a spatial curve in three-
(B) Non-continuous and differentiable
dimensional space given in parametric
(C) Continuous and non-differentiable
form by [ME  2015] (D) Neither continuous nor
x(t) = cos t, y(t) = sin t, differentiable
2 
z(t) = t, 0  t  . 19. The maximum value of
 2
The length of the curve is __________ f(x) = x3  9x2 + 24x + 5 in the interval

2 [1, 6] is [EE  2012]


cos(1 x)
14.  dx = ______. [CS  2015] (A) 21 (B) 25
1 x2
(C) 41 (D) 46
15. The region specified by
  20. A parametric curve defined by
{(, , z): 3 ≤   5,  ,
8 4  u   u 
x = cos   , y = sin   in the
3  z  4.5} in cylindrical coordinates  2   2 
range 0 ≤ u ≤ 1 is rotated about the
has volume of _______ [EC  2016]
X-axis by 360 degrees. Area of the
16. The area between the parabola x2 = 8y surface generated is [ME  2017]
and the straight line y = 8 is _______. 
(A) (B) 
2
[CE  2016] (C) 2 (D) 4


135
Vidyalankar : GATE – Engineering Mathematics

Assignment  2
Duration : 45 Min. Max. Marks : 30

Q 1 to Q 10 carry one mark each (C) f(x0) = 0 and f(x0) < 0


 e2x  1  (D) f(x0) = 0 and f(x0) > 0
1. Lt 
x  0 sin(4x)
 is equal to [ME  2014]
 
7. A function f(x) = 1  x2 + x3 is defined
(A) 0 (B) 0.5 (C) 1 (D) 2
in the closed interval [1, 1]. The value
2. The value of the integral of x, in the open interval (1, 1) for
2
(x  1)2 sin(x1) which the mean value theorem is

0 (x1)2 cos(x1)
dx is
satisfied, is [EC  2015]
(A) 3 (B) 0 [ME  2014] (A) 1/2 (B) 1/3 (C) 1/3 (D) 1/2
(C) 1 (D) 2
8. Consider the function f(x) = 2x3  3x2
3. The maximum value of the function in the domain [1, 2]. The global
f(x) = In(1 + x)  x (where x > 1) minimum of f(x) is ______ [ME  2016]
occurs at x = _________. [EC  2014]
9. Given the following statements about a
x
4. Let f(x) = x e . The maximum value of function f:  , select the right
the function in the interval (0,∞) is option: [EC  2016]
1
(A) e (B) e [EE  2014] P: If f(x) is continuous at x = x0, then
1 1
(C) 1 e (D) 1+ e it is also differentiable at x = x0.

5. At x = 0, the function f(x) = |x| has Q: If f(x) is continuous at x = x0, then it

(A) a minimum [ME  2015] may not be differentiable at x = x0.

(B) a maximum R: If f(x) is differentiable at x = x0,

(C) a point of inflexion then it is also continuous at x = x0.

(D) neither a maximum nor minimum (A) P is true, Q is false, R is false


(B) P is false, Q is true, R is true
6. While minimizing the function f(x), (C) P is false, Q is true, R is false
necessary and sufficient conditions for (D) P is true, Q is false, R is true
a point, x0 to be a minima are:
1
dx
[CE  2015] 10. The integral  (1  x)
is equal to ____
0
(A) f(x0) > 0 and f(x0) = 0
[EC  2016]
(B) f(x0) < 0 and f(x0) = 0

136
Assignment on Calculus

Q 11 to Q 20 carry two marks each


17. At x = 0, the function f (x) = x3 + 1 has
11. The value of the integral is
2 x
(A) A maximum value [ME  2012]
 e
xy
dydx is [ME  2014] (B) A minimum value
0 0

1 1 2 (C) A singularity
(A) (e  1) (B) (e  1)2
2 2 (D) A point of inflection

1 2 1 1
2
 1  cos x 
(C) (e  e) (D)  e  18. lim  2  is [ME  2012]
2 2 e
x 0
 x 
(A) 1/4 (B) 1/2 (C) 1 (D) 2
12. The minimum value of the function
f(x) = x3  3x2  24x + 100 in the 19. Which one of the following functions is
continuous at x = 3? [CS  2013]
interval [3, 3] is [EE  2014]

(A) 20 (B) 28 (C) 16 (D) 32  2, if x3

13. Consider an ant crawling along the (A) f(x) =  x1, if x3
x  3
curve (x  2)2 + y2 = 4, where x and y are  , if x3
 3
in meters. The ant starts at the point
 4, if x3
(4, 0) and moves counter-clockwise with (B) f(x) = 
8x if x3
a speed of 1.57 meters per second. The
x  3, if x3
(C) f(x) = 
time taken by the ant to reach the point  x  4, if x3
(2, 2) is (in seconds) ____ [ME  2015] 1
(D) f(x) = , if x3
2
x 3 27
14. limx  x  x  1  x is [ME  2016]
20. Let I = c  xy 2 dxdy , where R is the
R
(A) 0 (B)  (C) 1/2 (D) 
region shown in the figure and
15. The area of the region bounded by the
4
c = 6 × 10 . The value of I equals
parabola y = x2 + 1 and the straight
_______. (Give the answer up to two
line x + y = 3 is [CE  2016]
decimal places) [EE  2017]
59 9 10 7
(A) (B) (C) (D)
6 2 3 6

16. Let f :[1, 1]   ,


where f(x) = 2x3  x4  10. The minimum
value of f(x) is _____. [IN  2016]



137
Vidyalankar : GATE – Engineering Mathematics

Assignment  3
Duration : 45 Min. Max. Marks : 30

Q 1 to Q 10 carry one mark each 6. Given, i = 1 , the value of the


1. Consider a function f(x,y,z) given by 2
cos xisin x
2 2 2 2
f(x,y,z) = (x + y  2z ) (y + z ) 2 definite integral, I = 
0
cos x  isinx
dx

The partial derivative of this function


is: [CE  2015]
with respect to x at the point, x = 2,
(A) 1 (B) 1
y = 1 and z = 3 is _____. [EE  2017]
(C) i (D) i
2. For 0  t < ∞, the maximum value of
7. The contour on the x-y plane, where
the function f(t) = et  2 e2t occurs at
the partial derivative of x2 + y2 with
(A) t = loge 4 (B) t = loge 2
respect to y is equal to the partial
(C) t = 0 (D) t = loge 8
derivative of 6y + 4x with respect to x,
[EC  2014]
is [EC  2015]
3. If z = xy In (xy), then [EC  2014] (A) y = 2 (B) x = 2

z z (C) x + y = 4 (D) x  y = 0
(A) x y =0
x y
8. The values of x for which the function
z z
(B) y x
x y x 2  3x  4
f(x) is NOT continuous
z z x 2  3x  4
(C) x y
x y are [ME  2016]
z z (A) 4 and 1 (B) 4 and 1
(D) y x =0
x y (C) 4 and 1 (D) 4 and 1

 xsinx 
4. lim 
x  equals to [CE  2014] 9. As x varies from −1 to + 3, which one
x 
 
of the following describes the
(A)  ∞ (B) 0
behaviour of the function
(C) 1 (D) ∞
f(x) = x3  3x2 + 1? [EC  2016]
  sin x 
5. The value of limx 0   (A) f(x) increases monotonically.
 2 sin xx cos x 
(B) f(x) increases, then decreases and
is ___________ [ME  2015]
increases again.

138
Assignment on Calculus

(C) f(x) decreases, then increases and 15. The angle of intersection of the curves
decreases again. x2 = 4y and y2 = 4x at point (0, 0) is
(D) f(x) increases and then decreases. [CE  2016]

10. The maximum value attained by the (A) 0 (B) 30

function f(x) = x(x  1) (x  2) in the (C) 45 (D) 90

interval [1, 2] is _____. [EE  2016]


16. The cost function for a product in a
Q 11 to Q 20 carry two marks each firm is given by 5q2, where q is the
11. The Taylor series expansion of amount of production. The firm can
3 sin x + 2 cos x is [EC  2014] sell the product at a market price of
x3 Rs. 50 per unit. The number of units to
(A) 2 + 3x  x2  + .....
2 be produced by the firm such that the
x 3
profit is maximized is [CS, ME  2012]
(B) 2  3x + x2  + .....
2 (A) 5 (B) 10
3
x (C) 15 (D) 25
(C) 2 + 3x + x2 + + .....
2
x3 17. The area enclosed between the
(D) 2  3x  x2 + + .....
2 straight line y = x and the parabola

x 1 y = x2 in the x  y plane is [ME  2012]
12. The expression lim0  is equal
 (A) 1/6 (B) 1/4
to [CE  2014] (C) 1/3 (D) 1/2
(A) log x (B) 0
(C) x log x (D) ∞ 18. Consider the function f(x) = sin(x) in

13. The maximum area (in square units) of the interval x[/4, 7/4]. The number

a rectangle whose vertices lie on the and location (s) of the local minima of

ellipse x2 + 4y2 = 1 is _________. this function are [CS  2012]

[EC  2015] (A) One, at /2


(B) One, at 3/2
1
2 D
14. The integral (x  y  10)dxdy , (C) Two, at /2 and 3/2
(D) Two, at /4 and 3/2
where D denotes the disc: x2 + y2  4,
evaluates to ________ [EC  2014]

139
Vidyalankar : GATE – Engineering Mathematics

19. The value of the definite integral 20. Consider the following definite integral:
e 1
(sin1 x)2

1
x l n(x)dx is [ME  2013] I=  dx [CE  2017]
0 1 x2
4 3 2 2 3 4
(A) e  (B) e  The value of the integral is
9 9 9 9
2 3
2 3 4 4 3 2 (A) (B)
(C) e  (D) e  24 12
9 9 9 9
3 3
(C) (D)
48 64



140
Assignment on Calculus

Assignment  4
Duration : 45 Min. Max. Marks : 30

Q1 to Q6 carry one mark each  2



ex   1  x  x 
1  cos x 6. lim  2 =
1. Lim equals to x 0 x3
x 0 x2
(A) 1/2 (B) 1/6
1 1
(A) (B)
3 2 (C) 1/3 (D) 1

1 1
(C) (D)
4 5 Q7 to Q18 carry two marks each
7. Which of the following is incorrect
sin 
2. What is lim equal to?
 0  statement ?

(A)  (B) sin  (A) A stationary value is always an

(C) 0 (D) 1 extreme value


(B) A maximum value is always a
x1/ 3  2
3. The value of lim is stationary value
x  8 (x  8)

(C) A minimum value is always a


1 1
(A) (B) stationary value
16 12
(D) A minimum value may be greater
1 1
(C) (D) than maximum value
8 4

ax  b 8. Which of the following is correct


4. Lim is
x  cx statement ?
(A) 0 (B) a (A) A stationary point on a graph is
a a dy
(C) (D) any point at which =0
b c dx

5. The discontinuity of the function (B) A stationary point on a graph is

ex  1 dy
f(x) at the point x = 0 is known any point at which =
x dx

as (C) A stationary point on a graph is


(A) first kind of discontinuity any point at which there are gaps
(B) second kind of discontinuity or breaks
(C) mixed discontinuity (D) None of these
(D) removable discontinuity

141
Vidyalankar : GATE – Engineering Mathematics

9. First rule for maximum and minimum 12. f (x) = (x  2)3 is


values of y = f(x) are as follows
(A) maximum at x = 2
2
dy d y dy (B) minimum at x = 2
Get and 2 . Solve  0 and
dx dx dx
(C) neither maximum nor minimum at
consider its roots. These are the value
x=2
dy (D) none of these
of x which make = 0. For each of
dx
these values of x, calculate the 13. The function sin x (1 + cos x) is
corresponding value of y, and examine maximum in the interval (0, ), when
d2 y  
the sign of . Now pick the correct (A) x = (B) x =
dx 2 4 2
statement from the following  2
(C) x = (D) x =
alternatives. 3 3

(A) If the sign is +, the corresponding


14. The absolute minimum of
value of y is a maximum
f (x) = 4x3  8x2 + 1 in [ 1, 1] is at
(B) If the sign is , the corresponding
(A) 1 (B) 3
value of y is a minimum
(C) 1 (D) 8
(C) If the sign is , the corresponding
value of y is a maximum 15. A spherical snowball is melting at rate
(D) None of these of 4 cubic centimeter per hour. How

10. Let f(x)  x 3  6x 2  24x  4. Then f(x) fast is the diameter changing when it is
20 centimeters ?
has
(A) 0.1 cm/hr (B) 0.02 cm/hr
(A) a maximum value at x = 2
(C) 0.04 cm /hr (D) 0.01 cm/hr
(B) a maximum value at x = 5
(C) a minimum value at x = 5 16. If a mothball evaporates at a rate
(D) neither maximum nor minimum at proportional to its surface area 4r2,
any point then its radius
11. Let a number 30 be divided into two (A) decreases at constant rate
parts such that cube of one multiplied (B) decreases logarithmically
with the other is maximum, then the (C) decreases exponentially
parts are (D) None of these
(A) 12.4, 17.6 (B) 13, 17
(C) 10.5, 19.5 (D) 22.5, 7.5

142
Assignment on Calculus

17. The absolute maximum of 18. The absolute minimum of


4 3 2
f(x) = x  2x  x  4x + 3 on the x3
f(x) = on the interval [ 1, 1] is
interval [0, 4] is. (x  2)
(A) 80 (B) 16 at
(C) 28 (D) None of these (A) 0 (B) 1
(C) 1/3 (D) None of these



143
Vidyalankar : GATE – Engineering Mathematics

Assignment  5
Duration : 45 Min. Max. Marks : 30

Q1 to Q6 carry one mark each cot 


6. The value of L im is
  (  / 2) ( / 2)  
1. L im ( x 2  1  x  1) equals to
x
(A) 1 (B) 2
(A) 0 (B) e
1
(C) 1 (D)  (C) 0 (D)
2
2. It is given that Q7 to Q18 carry two marks each
ax  b 7. If y = 7x  x3 and x increase at rate of
f(x) = ,L im f (x)  2 and
x 1 x0
4 units/sec how fast is slope of graph
L im f (x)  1 , then value of f ( 2) is changing when x = 3 to
x

(A) 0 (B) 1 (A) 72 (B) 80

(C) e (D)  (C) 64 (D) 64

3. The limiting value of 8. An object moves on the parabola


3y = x2 when x = 3, the x coordinate of
12 22 32 n2
   .............  as n  
n3 n3 n3 n3 the object is increasing at the rate of

is 1 foot/min. How fast is y coordinate


increasing of the moment
1
(A) 1 (B)
2 (A) 1 (B) 2

1 1 (C) 3 (D) 4
(C) (D)
3 4 9. A snowball is increasing in volume of
2n the rate of 10 cm3/h. How fast is the
 1
4. What is the value of lim  1   ?
n
 n  surface area growing at the moment

(A) 0 (B) e2 where radius of snowball is 5 cm ?


(A) 4 (B) 2
(C) e1/2 (D) 1
(C) 12 (D) 16
1  cos 2x
5. The value of L im is
x  (  / 2) (   2x)2 10. The height h and radius r of cylinder of
greatest volume that can be cut from a
1 1
(A) (B) sphere of radius b is
4 3
1 2b 2
(C) (D) 0 (A) h = ,r  b
2 3 3

144
Assignment on Calculus

/4
b 2
(B) h =
3
,r= b
3
14.  (1  tanx) / (1  tanx) dx evaluates to
0

b b (A) 0 (B) 1
(C) h = ,r 
2 3 6 (C) In 2 (D) ½ In 2
2b 1 15. The following plot shows a function y
(D) h = ,r  b
3 3
which varies linearly with x. The value
11. A pipe with length 3m and radius of 2
of the integral I   1
y dx is
10cm has an outer layer of ice that is y
melting at the rate of 2cm3 /min. How 3
fast is the thickness of ice decreasing 2
when ice is 2cm thick?
1
1 x
(A) cm / min
200 1 1 2 3
1 (A) 1.0 (B) 2.5
(B) cm / min
800 (C) 4.0 (D) 5.0
1 16. The value of the quantity P, where
(C) cm / min
1800 1

 xe dx, is equal to
x
1 P=
(D) cm / min 0
3600
(A) 0 (B) 1
12. The maximum area of any rectangle (C) e (D) 1/e
which may be inscribed in a circle of
17. The sum of the infinite series,
radius 1 is
1 1 1
(A) 1 (B) 2 1+    ....is
2 3 4
(C) 3 (D) 4
(A)  (B) infinity
13. A point on a curve is said to be an 2
(C) 4 (D)
extremum if it is a local minimum or a 4
local maximum. The number of distinct  
2 2

extrema for the curve


18. The value of the integral  
0 0
e x e y

4 3 2
3x  16x  24x  37 is dx dy is
(A) 0 (B) 1  
(A) (B)  (C)  (D)
(C) 2 (D) 3 2 4


145
Vidyalankar : GATE – Engineering Mathematics

Assignment  6
Duration : 45 Min. Max. Marks : 30

Q1 to Q6 carry one mark each 4. The maximum value of


1. The discontinuity of the function f (x) = 2 | x  1| + 3 | x  2 |
|x| for all x  R is
f (x) = , x0
x (A) 2 (B) 4
= 0, x = 0 (C) 1 (D) None of these
at the origin is known as
5. Pick out the correct statement
(A) Discontinuity of 1st kind
(B) Discontinuity of 2nd kind (A) logarithmic function is decreasing

(C) Mixed discontinuity wherever it is defined

(D) None of these (B) logarithmic function is increasing


whenever it is defined
2. The value of the coefficients ‘a’ such (C) logarithmic function is neither
that the function f defined as increasing nor decreasing
2
f (x) = ax  2 x  1 (D) None of these

= 1 x  1 6. f (x) = log sin x. This function is


continuous is
 
(A) increasing on  0,  and
(A) a = 1 (B) a = 2  2
(C) a = 0 (D) a = 3
 
decreasing on  ,  
2 
3. If f (x) = 1, if x < 3
 
= ax + b, if 3  x < 5 (B) increasing on  ,   and
2 
=7 if 5  x
 
Determine the value of ‘a’ and ‘b’ so decreasing on  0, 
 2
that f (x) is continuous.
(C) increasing on [0, 0] and
(A) a = 4, b =  7
decreasing on [, ]
(B) a = 3, b =  8
(D) increasing on [, 2] and
(C) a = 1, b =  9
decreasing on [0, 0]
(D) a = 2, b = 3

146
Assignment on Calculus

Q7 to Q18 carry two marks each 1 1 1


13. The series 1    +……… is
1 1 1 3 9 27
7. The series 1   2  3  .....is
2 2 2 (A) convergent (B) divergent
(A) convergent (B) divergent (C) oscillatory (D) None of these
(C) oscillatory (D) None of these
14. The series whose nth term is

8. The series 1 + 2 + 3 + …… is n3  1  n3 is
(A) convergent (B) divergent (A) convergent (B) divergent
(C) oscillatory (D) None of these (C) oscillatory (D) None of these

9. The series 1  2 + 3  4 + 5  6 + . . . . 1
15. The series whose nth term is sin is
n
is
(A) convergent (B) divergent
(A) convergent (B) divergent
(C) oscillatory (D) None of these
(C) oscillatory (D) None of these
16. The series
1 1 1
10. The series 1     .......is 2 3 4 5 n1
2 3 4     2  ........is
1 4 9 16 n
(A) convergent (B) divergent
(A) convergent (B) divergent
(C) oscillatory (D) None of these
(C) oscillatory (D) none of these
1 2 3 17. The interval in which Lagrange’s
11. The series 1
 2
 +….
1 2 1 2 1  23
theorem is applicable for function
is
f(x) = 1/x is
(A) convergent (B) divergent
(A) [3, 3] (B) [2, 2]
(C) oscillatory (D) None of these
(C) [ 2, 3] (D) [1, 1]
12. Which of the following to true?
18. For f(x) = x3 suppose
1 1 1
(A) 1  1/ 3  1/ 3  1/ 3 is convergent f(b)  f(a) = (b  a) f  (c) holds then c is
2 3 4
1 1 1 1 2 ab
(B) 1     is convergent (A) a  b2 (B)
2 3 4 2 2

1 1 1 a 2  b2  ab a 2  b2  ab
(C) 1    is convergent (C) (D)
22 32 42 3 3

1 1 1
(D) 1  k
 k  k  is divergent
2 3 4


147
Vidyalankar : GATE – Engineering Mathematics

Assignment  7
Duration : 45 Min. Max. Marks : 30

Q1 to Q6 carry one mark each 6. The maximum of f(x) = (x + 2)/(x  1) is


1. Function f (x) = e 1/ x ; x > 0 is a (A) 4 (B) 1

(A) increasing function (C) 2 (D) does not exist

(B) decreasing function Q7 to Q18 carry two marks each


(C) neither increasing nor decreasing 7. Use Lagrange’s mean value theorem
(D) none of these
for f(x) = ex . Which of the following is
1 true?
2. f (x) = x  is
x (A) 1 + x < ex < 1 + xex
(A) maximum at x = 1 (B) 1 + xex < ex < 1 + x
(B) maximum at x = 2 (C) ex < 1 + x ex < 1 + x
(C) maximum at x =  1 (D) none of these
(D) maximum at x =  2
8. x1/X is decreasing function if
3. f (x) =  x 2 + 2x + 1 is (A) x < e (B) x > e

(A) maximum at x = 1 (C) x = e (D) x > 1/e

(B) maximum at x = 2 9. The Maclaurin series of ex is


(C) maximum at x =  1 
x n 1 
xn
(D) maximum at x =  2
(A)  n  1 !
n 0
(B) 
n  0 n!

4. The maximum value of 5 cos  + 3 cos



xn
(C) 
n 1 n!
(D) none of these
 
   3  + 3 is
  10. Taylor series for sin x about /4 is
(A) 5 (B) 10
x    
n

1
(C) 11 (D)  1
(A) 
2 n 0 n!

 1  x   / 4 
n n

5. f (x) = x 3  12 x 2  45 x  11 is (B) 2
n 0 n!
(A) minimum at x = 5
1 1  x   / 4  x   / 4 
2
(B) minimum at x =  5 (C)   ...
(C) minimum at x = 4 2  1! 2! 

(D) minimum at x =  4 (D) none of these

148
Assignment on Calculus

11. The Maclaurin series for ln (1  x) is x


14. If f(x) =  2 n
xn , then f33(0) is
x 2 x 3 n 0
(A) x  .....
2 3 (A) (33!) 333 (B) (32!) 333

x2 x3 (C) (3!) 232 (D) (33!) 233


(B) x   .....
2 3

x 2 x 3 15. If f(x) = tan1x, then f99 (0) is


(C) x   .....
2 3 (A) 97! (B) 98!
(D) none of these (C) 99! (D) none of these

12. The Taylor series for 1/x about 1 is 16. The nth non zero term of Maclaurin

series for ln (1 + x2) is
  1  x  1
n 1 n
(A)
n0
 1
n 1
x 2n x 2n
 1
n
 (A) (B)
  1  x  1
n n
(B) n n1
n0
x 2n x 2n1
(C)  1 (D)  1
n 1 n

  1  x  1
n 1
(C)
n
n1 n
n 1

(D) none of these 1


17. If f(x) = 2
, then f36(0) is
x  x 1
13. The first non zero terms of Maclaurin
(A) 36! (B) 36!
series for sec x is
36
(C) 2 (D) none of these
1 5x 4
(A) 1  x 2 
2 24
18. The first three nonzero terms of
x 2 54 3
(B) x   x Maclaurin series for tan x are
3 16
x3 2 5
x 2 5x 4 4 (A) x   x  .....
(C) 1   x 3 15
2 16
x2 4 3
x 2 54 4 (B) 1   x  .....
(D) x   x 2 15
2 24
x2 2 5
(C) 1   x  .....
2 15
(D) none of these



149
Vidyalankar : GATE – Engineering Mathematics

Assignment  8
Duration : 45 Min. Max. Marks : 30

Q1 to Q6 carry one mark each 6. Evaluate lim e nlogn


x 0
1. For what value of k will f(x) = x  kx1
(A) 1 (B) 0
have a relative maximum of x = 2
(C)  (D) does not exist
(A) 2 (B) 1
(C) 4 (D) 8
Q7 to Q18 carry two marks each
7. The first three nonzero terms of
2. A wire 16 feet long is bent to form a
Taylors series for  n x around 2 is
rectangle. If the area of the rectangle
1
 x  2
2
is to be maximized, the dimensions of (A)  n2 + (x  2 ) 
8
the rectangle should be
1 1
(x - 2) -  x - 2 
2
(A) 2, 6 (B) 5, 3 (B) n2 +
2 8
(C) 4, 4 (D) None of these
1 1
(x - 2) +  x - 2 
2
(C) n2 -
2 4
|x2|
3. lim is equal to 1
x2  x - 2
x 2 2
(D) n2 + (x - 2) -
4
(A) 1 (B) 1
(C) 0 (D) does not exit
8. The Maclaurin series for 2x is

  n2  n   n2 
n n
x x
f x  2 (A)  x (B)  x n 1
4. If lim  0 then lim f  x  is equal n  0  n  1 ! n!
x 1 f x  2 x 1 n 0

  n2 
n
x
to
(C)  n!
xn (D) None of these
(A) 1 (B) 1 n 0

(C) 2 (D) 2
2
9. If f(x) = ex then f100 (0) is
5. Which of the following limit exist ?
(A) 100! (B) 2100
1
(A) lim e1/ x (B) lim 100!
x 0 x 0 1  e1/ x (C) (D) 100!  50!
50!
1
(C) lim (D) lim1/ x 2
x 0 x x 0

150
Assignment on Calculus

10. If f(x) satisfies f(0) = 2, /2


14. Evaluate I = 0
sin2x log tanx dx
f   0   1,f (0)  4,f (0)12
(A) 0 (B) 1
n
andf (0)  0 for n > 3 (C) /2 (D) /2 log 2
then f(x) is
(A) x + 2x3 15. The area bounded by curve x2 = 4y
(B) 2 + 2x2 and straight line x = 4y  2 is
2
(C) 2 + x + x (A) 1/8 (B) 3/8
2 3
(D) 2 + x + 2x + 2x (C) 9/8 (D) 2

11. The Taylor series for f(x) = 2x2 +4x  3 16. The area bounded by x axis,
about 1 is 8
2 y = 1+ ordinate at x = 2 and x = 4 is
(A) 3 + 8 (x 1) + 2 (x  1) x2
(B) 2 + 4x + 2x2 (A) 2 (B) 4
2
(C) 2 + 4 (x  1) + 4 (x  1) (C) 8 (D) 1
(D) None of these
17. The area bounded by curve
 sin2kx y = 2x + x2  x3 the x axis and lines
12. Evaluate I =  0 sinx
dx
x = 1 and x = 1
(A) 0 (B) 1/2 (A) 1/2 (B) 1
(C) 1 (D) /2 (C) 3/2 (D) 2


13. Evaluate I =  log 1  cos   d x  sin x
0 18. Evaluate I =  1  cos x dx

(A) log2 (B) log2 x x
2 (A) x  cot C (B)  x cot C
2 2
(C) log2 (D) /4 log 2
x x
(C) x cot C (D) x + tan C
2 2



151
Vidyalankar : GATE – Engineering Mathematics

Assignment  9
Duration : 45 Min. Max. Marks : 30
Q1 to Q6 carry one mark each 6. The limiting value of

 x 
2x
1 2 n
1. lim    .......  as n   is
x  1  x
 is n n n
 
(A) 2/3 (B) 1/2
1
(A) e (B) (C) 1/3 (D) does not exist
e2
1 Q7 to Q18 carry two marks each
(C) (D) 2e
e /2

tanx
7. I = 
0
cos4 x sin3 dx
 1
2. lim   is equal to 1 2
x 0
x (A) (B)
35 35
(A) 0 (B) 1
3 4
(C) 1 (D) 0 (C) (D)
35 35
lim  cos ecx  is equal to
x
3.
x 0 1 x2
8. Evaluate I =  1 x2
dx
(A) 1 (B) e
(C) r1 (D) 0 3 1 1
(A) sin x  x 1  x 2  C
2 2
1 2 3 1
4. lim  2  2  2  ......   1
n n
 n n n (B) x 1 x2  C
2
(A) 0 (B) 1
(C) sin 1 + 1 x2  C
(C) ½ (D) 
(D) none of these
 ax 2  b x2
 dx
5. The function f(x) =  2

x2 9. Evaluate I =  cos x  sinx
2ax  b x2
x 
is continuous at x = 2 if (A) log tan     C
2 8
(A) a = 0, b = 1/2
x
(B) a = 1/2 , b = 0 (B) log cot C
2
(C) a = 1/2, b = 0 (C) log cot x + C
(D) b = 1/2, a = 0 (D) log tan x + C

152
Assignment on Calculus

dx  nsinx 
10. Evaluate I =  1  3 sin 2
x
14. The value of lim 
x 0
 x 
 , where [.] is
1
(A) tan1  tan x   C the greatest integer function
2
(A) n + 1 (B) n
(B) tan1 (tan x) + C
(C) n  1 (D) None of these
1
(C) tan1  2 tanx   C
2 15. Let f   xy,  yz,z  and
(D) None of these

C : sin  2t  , t, t 2 . 
d
11. Evaluate I =  sin4
  cos4 
Then the value of the line integral

1  tan2   1  fd r from (0, 1, 1) to (0, 1, 1) is


(A) tan1   C

2  2 tan  
(A) 1 (B) 1
 tan   1  2
(C) 0 (D) None of these
(B) tan1  
 tan   

e
-x
16. If IN = xn-1logx dx , then
1  1 
(C) tan1  
0
2  2 tan  
(A) In+2 + (2n + 1) In+1 + n2In = 0
(D) None of these
(B) In+2  (2n+1) In+1  n2 In = 0
sin x (C) In+2 + (2n + 1) In+1  n2In = 0
12. I =  sin3x dx
(D) In+2  (2n+1) In+1 + n2 In = 0
 3  tan x 
(A) log  C
 3  tan x  17. The function f(x) = 1  1  x 2 is
 
1 3  tanx (A) not continuous of x = 0
(B) log C
2 3 3  tanx (B) is continuous but not differentiable

(C)
1
3
log  3  tan x  C  at x = 0
(C) differentiable at x = 0
1 3  (D) None of these
(D) log 
   C
2 3  3  tanx  18. The function
/2
 z = 5xy 4x2 + y2  2x  y + 5 has at
2
13. I = x sinx dx
0
1 18
2 x= ,y=
(A) (B) 2  2 41 41
2
(A) maximum (B) saddle point
(C)  + 2 (D)   2
(C) minimum (D) None of these


153
Vidyalankar : GATE – Engineering Mathematics

Test Paper  1
Duration : 30 Min. Max. Marks : 25
Q1 to Q5 carry one mark each Q6 to Q15 carry two marks each
sin3x  sinx 
1
1. Lim
x 0 sinx
is equal to 6. x
1
2
dx

(A) 0 (B) 2 (A) Converges


(C) 1 (D) 3 (B) Diverges
(C) Converges and diverges
1 (D) None of these
2. Lim sin is
x 0 x
(A)  (B) 0 
dx
(C) 3 (D) does not exist 7. 0 ex  e x is equal to

(A) /2 (B) /4


sin3x
3. Lim is equal to (C) /3 (D) 
x 0 5x
3 5
(A) (B) 
5 3 dx
8.  x n x
2
2
is equal to
1 1
(C) (D)
5 3 1 1
(A) (B)
 n4  n3

4. If [x] denotes the greatest integer not 2 1


(C) (D)
greater than x, then Lim[x] is  n2  n2
x 2

5
(A) 0 (B) 1 dx
9.  3
x2
is equal to
(C) 2 (D) does not exist 1

9 1
 
3
3
(A) (B) 3
9 1
n
(1 x)  1 2 2
5. Lim is equal to
x 0 x (C) 3  3
9 1 (D) None of these
(A) n (B) n  1
(C) 0 (D) does not exist

154
Test Paper on Calculus

x 13. The area between the curves y = 1/x


10. The area under curve y = for
1 x 2
1
and y = to right of line x = 1 is
0  x  1 is x 1

(A) 1/4 (B) 1/2 (A) n 3 (B) n 2

(C) 1 (D) 16 (C) 2 n 3 (D) 2 n 2

3
11. The volume of solid obtained by dx
14. x2
revolving area under y = e2x about 1

(A) converges
x axis is
(B) diverges
(A) /2 (B) /4
(C) both
(C) 2 (D) 
(D) neither of the above

9
dx
12.   x  1 2/3
is equal to 
dx
0 15.    nx 
e
p
is _________ for p  1

(A) 3 (B) 6
(A) convergent
(C) 7 (D) 9
(B) divergent
(C) convergent as well as divergent
(D) None of these



155
Vidyalankar : GATE – Engineering Mathematics

Test Paper  2
Duration : 30 Min. Max. Marks : 25

Q1 to Q5 carry one mark each (B) Both A and R are true and R is not
1. Let f (x) = 2x if x < 2 the correct explanation of A
=2 if x = 2 (C) A is true but R is false

= x2 if x > 2 (D) A is false but R is true

the discontinuity of f (x) at x = 2 is


4. If a function ‘f’ is defined on R by
known as
(A) first kind of discontinuity f (x) = 1 for x a rational number

(B) second kind of discontinuity =  1 for x an irrational number,

(C) mixed discontinuity then

(D) removable discontinuity at x = 2 (A) f (x) is continuous at x = 1


(B) f (x) is continuous at x =  1
2. If f (x) = x  1 1x2 (C) f (x) is continuous at x = 0

= 2x  3, 2x3 (D) f (x) is not continuous at any point

Tick the following alternative which is


appropriate for the above function. 5. If f (x) = 4x + 3, x4

(A) continuous at x = 1 = 3x + 7, x=4

(B) continuous at x = 2 the function ‘f’ is

(C) discontinuous at x = 1 (A) continuous at x = 4

(D) discontinuous at x = 2 (B) discontinuous at x = 4


(C) continuous at x =  4
3. Consider the following statement (D) discontinuous at x =  4
Assertion (A) :
The function f (x) = x  [x], x  z is Q6 to Q15 carry two marks each
discontinuous at x = 1 /2

Reason (R) :
6. I = 0 cot xdx is

Left L im f (x)  Right L im f (x)


x 1 x 1 (A) 2 (B) /4
Of these statements (C)  (D) None of these
(A) Both A and R are true and R is the
correct explanation of A

156
Test Paper on Calculus

7. The area in first quadrant under curve 11. The minimum value of x 2  5x  2 is
1
y= is (A) 5 (B) 0
x 2
 6x  10  (C) 1 (D) 2

(A) /2 (B)  2 tan1 3
4
12. The maximum value of
 
(C)  tan1 3 (D)  tan1 3 sinx  cos x
2 2 f(x)  is
2

(A) 1 (B) 2
1
8. The area under y =
x 2
 a2  (C) 1/ 2 (D) 3

for x  a + 1 is
13. The maximum value of 1/xx is
1 1
(A)  n  a  1 (B)  n  a  1 (A) e (B) ee
a 2a
(C) e1/e (D) e1/e
1
(C)  n  a  1 (D) None of these
2
14. The function f(c) = x5  5x4+5x3  1

9. For what values of k, with k  1 and has


1 (A) 1 minima and 2 maxima
1
k > 0 does  k dx converges ? (B) 2 minima and 1 maxima
0 x
(C) 2 minima and 2 maxima
(A) 2 (B) 1
(D) 1 minima and 1 maxima
(C) 3 (D) k > 1

15. The greatest and least value of


a
dx
10.  2 where a > 0 is x 3 3x 2
0 a  x2   2x in  0,2  are
3 2
1
(A) n a (B)  na  1 (A)
5
and
2
(B)
11
and
1
2
6 3 6 6
1
(C)  na  1 (D) None of these
(C)
8
and
2
(D)
13
and
1
2
3 3 3 3



157
Vidyalankar : GATE – Engineering Mathematics

Test Paper  3
Duration : 30 Min. Max. Marks : 25

Q1 to Q5 carry one mark each 5. The function


16
(x  1) 1
1. The Lim is f (x) = x 2 sin , if x  0
x 1 (x 5  1) x
= 0, if x = 0 is
5 2
(A) (B)
16 17 (A) differentiable at x = 0 and f ‘(0) = 0

16 17 (B) not differentiable at x = 0 since


(C) (D)
5 2 1
 , cos x  0
x
x2  4 (C) differentiable at x = 0 and the
2. L im equal to
x2
x  2  3x  2 derivative is continuous at x = 0
(A) 8 (B)  8 (D) not differentiable at any x since it
(C) 4 (D)  4 is not continuous for any x

a x  bx Q6 to Q15 carry two marks each


3. L im equals to
x0 x 6. The maximum point on curve x = exy is
a b (A) (1, e) (B) (1, 1/e)
(A) log   (B) log  
b a
(C) (e, 1) (D) (1/e, 1)
x b
(C) log   (D) log  
a x 7. If f(x, y) is such that fx = ex cos y and
fy = ex sin y then which of following is
|x| true
4. The function f (x) = at x = 0 has
x
(A) f(x, y) = ex + ysin (x + y)
(A) discontinuity of second kind
(B) f(x, y) = ex sin
(B) discontinuity of first kind
(C) f(x, y) does not exist
(C) mixed discontinuity
(D) None of these
(D) continuous

8. If f(x, y) = ln (x2 + y2) then


(A) fxx + fyy = 0 (B) fxx  fyy = 0
(C) fxx +2fyy = 0 (D) 2fxx + fyyt = 0

158
Test Paper on Calculus

9. If z = x4 + 3xy  y2 and y = sin x then z z


(C) x y
dz x y
is
dx z z
(D)  z
3
(A) (4x + 3 sin x) + (3x  2 sin x) cos x x y

(B) (2x2 3 sin x) + (3x  2 cos x)sin x f


13. Find the general solution of =1
(C) (4x3  3 sin x)  (3x + 2 sin x) cos x xy

(D) none of these is


(A) f(x, y) = A (x) + B(y) + xy

10. A point P is moving along curve of (B) f(x, y) = yA (x) + xB(y)


(C) f(x, y) = A (x) + B(y)
x2 y2
intersection of paraboloid  z
16 9 (D) none of these

and cylinder x2 + y2 = 5. If x is
increasing at the rate of 5cm/s how 14. The equation of tangent plane sphere

fast is z changing when x = 2m and phase x2 + y2 + z2 = 1 at point


y = 1 cm ? 1 1 1 
 , ,  is
25 125 2 2 2 
(A) (B)
36 36 (A) x + y + z = 2
144 18 (B) x + y + 2z=2
(C) (D)
25 25
(C) 2x  y + 32 = 5
(D) 2x + y = 2
2
 f
11. The general solution of = 0 is
x 2
15. The altitude h of right circular cone is
(A) f(x, y) = x g(y) + h (y)
decreasing at rate of 3 mm/s while
(B) f(x, y) = y g(x) + h (x)
radius is increasing at 2mm/s. How
(C) f(x, y) = x g(y) + h (x)
fast is volume v changing when
(D) f(x, y) = g(x) + h (y)
altitude and radius are 100mm and
50mm respectively ?
12. If z = xey/x then
2200 2500
(A) (B)
z z 3 3
(A) x y z
x y
12500 1600
(C) (D)
z z 3 3
(B) x y z
y z



159
Vidyalankar : GATE – Engineering Mathematics

Test Paper  4
Duration : 30 Min. Max. Marks : 25

Q1 to Q5 carry one mark each 5. Which of following is false ?


1. Maxima and minima of a continuous (A) If f  (a) = 0 and f  (a) >0 then f (a)
function occur is minimum value
(A) simultaneously (B) If f  (a) = 0 and f(a)  0 then f(a)
(B) once is extreme value
(C) alternately (C) a maximum value is greater than
(D) rarely greatest values
(D) f(a) is minimum value if f(a) is least
2. Let f(x) = | x | than
value of f(x) in immediate
(A) f  (0) = 0
neighborhood of point [a, f(a)]
(B) f(x) is maximum at x = 0
(C) f(x) is minimum of x = 0 Q6 to Q15 carry two marks each

(D) none of these 6. The area included between curves


y = 3x2  x  3 and y = 2x2 + 4x +7 is
3. Find the maximum speed with which a
(A) 10 (B) 45/2
snail moves if the equation of distance
(C) 15 (D) 30
covered (s) in terms of time (t) is
7. The area included between curve
t2
s = 3t   15 units ?
3
r = a (sec  + cos ) and its asymptote
2
r = a sec  is
1
(A) speed units
9 5a 2
(A) a2 (B)
4
1
(B) speed units
18 2a 2
(C) (D) 2a2
3
1
(C) speed units
27 8. The mass distributed over the area
(D) None of these bounded by curve 16y2 = x3 and line
2y = x assuming that density of a point
f(x)
4. The function g (x) = , x  0 has an of area varies as distance of point from
x
x axis. Take k as constant of
extreme value then
proportionality
(A) f  (x) = f (x) (B) g (x) = f(x)
(A) k (B) 1/3k
(C) f(x) = 0 (D) g(x) = f  (x)
(C) 2/3k (D) 2k

160
Test Paper on Calculus

9. The density at any point in circular  2


(C) a 3 log2  
lamina of radius a varies as its  3
distance from a fixed point on its (D) a3
circumference. Take k as proportional
constant, the mass of lamina is 13. The volume of solid formed by
1 3 revolution of the curve (a  x)y2 = a2x
(A) ka 2 (B) ka
9 about its asymptote
8 3 32 3 2 a 3
(C) ka (D) ka (A) (B) 2a3
9 9 2
3 2 a 3
2 2 2 2 (C) (D) 42a3
10. The volume of sphere x + y + z = a 2
cut off by plane z = 0 and cylinder
x2 + y2 = ax is 14. The surface of reel formed by
3 3
a 2a revolution of cycloid x = a ( + sin ),
(A) (B)
3 3
y = a(1 cos ) round a tangent at its
3 3
(C) a (D) 2a vertex is

11. The volume cut off from paraboloid a 2 2a 2


(A) (B)
3 3
1 2
x2 + y  z  1 by plane z = 0 is
4 16a 2 32a 2
(C) (D)
3 3
 
(A) (B)
4 2
3 15. The volume generated by revolving
(C) (D) 
4 area below the xaxis between curve

12. The volume of solid obtained by y8


 x  2 and x axis about line
2 2 x
revolution of loop of curve y = x
x + 5 = 0 is
ax
about x axis is (A) 18 (B) 32
ax
(C) 432 (D) 216
(A) 2a 3
 2
(B) 2a 3 log2  
 3



161
Vidyalankar : GATE – Engineering Mathematics

Test Paper  5
Duration : 30 Min. Max. Marks : 25

Q1 to Q5 carry one mark each 5. If z is defined as function of x and y by


1 z
1. The minimum value of x + ,x>0 xy  yz + xz = 0 then is
x x
(A) 1 (B) 3/2 yz yz
(A) (B)
(C) 5 (D) 2 yx yx

yx xz
2. The triangle of maximum area (C) (D)
yz xy
inscribed in a circle of radius r is
(A) a right angled  with hypotenuse 2r Q6 to Q15 carry two marks each

(B) an equilateral  6. If f(x) = ex and g (x) = ex then value of

(C) an isoceles  of height r c in interval [a, b] by Cauchy’s mean

(D) does not exist value theorem


ab
(A) (B) ab
log x 2
3. The maximum value of in (0,  )
x 2ab
(C) (D) none of these
is ab
(A) e (B) 1/e
7. If f and F be both continuous in [a, b]
(C) 1 (D) none of these
and are derivable in (a, b) and if

4. A manufacturer sells ‘x’ items at a f  (x) = F  (x) for all x in [a, b] then f(x)
and F (x) differ
 x 
price of Rs.  3  per item and it
 1000  (A) by 1 in [a, b]
(B) by x in [a, b]
x 
costs Rs.   200  to produce them. (C) by constant in [a, b]
 2 
Find the production level for maximum (D) none of these

profit. 8. If f(x) = 4x2 then the value of c in [1, 3]


(A) 1250
f  3   f  1
(B) 1500 for which f  (c) = is
4
(C) 1750
(A) 0 (B) 1
(D) Data insufficient
(C) 2 (D) 3

162
Test Paper on Calculus

 a 1 cos    a 1 cos  


9. I =  
0 0
r 4 cos drd 13. I =  
0 0
2r 4 sin3 dr d

5 21 5 2 24
(A) a 5 (B) a (A) a 5 (B) a 5
16 16 35 35
3 1 4 26 a 5 28 a 5
(C) a 5 (D) a (C) (D)
16 4 35 35
y
4 2
10. I =   2 x dx dy a a  a2  y 2 xy log  x  a 
 
2
2 y / 4 14. I = dx dy
x  a
0 0 2

24 36
(A) (B)
5 5 a2 a2
(A) log a (B) log a  1
72 108 2 8
(C) (D)
5 5 a2
(C) 8a log a (D) 2log a  1
b b 8
11. I =   y  2b  y  dx dy
0 0

a 2a  x
b4 2b4
(A) (B) 15. I =  
0 x2 / a
xy dx dy
3 3
3 4 5 4
4b4 (A) a (B) a
(C) b4 (D) 8 8
3
7 4
/4 2 sin  3 (C) a (D) a4
12. I = 2  
0 0
r dr d 8

3 3
(A) (B) 2
4 4
3 5
(C) 2 (D)
4 2



163
Vidyalankar : GATE – Engineering Mathematics

Test Paper  6
Duration : 30 Min. Max. Marks : 25

Q1 to Q5 carry one mark each Q6 to Q15 carry two marks each

R y dA
2
x 2 2 6. If I = where R is region
1. lim is
x 8 x8
bounded by y = 2x, y = 5x and x = 1.
0 1
(A) (B) Then I is equal to
0 2
1 1 39 39
(C) (D) (A) (B)
2 4
4 2 2
39 39
(C) (D)
x  sin x 8 16
2. lim is
x 0 x3
1
(A) 1/6 (B) 1/3 7. The value of I = R 2y  y 2
dA where

(C) 1/3 (D) none of these


R is region in the first quadrant
1 bounded by x2 = 4  2y is
3. lim  x  a  cos is
x a xa (A) 0 (B) 2
(A) 0 (B) a (C) 4 (D) 1
(C) cos a (D) does not exist
8. The volume V under plane z = 3x + 4y
1  cos x and over the rectangle
4. The function f(x) = (x  0) be
x2 R : 1  x  2, 0  y  3 is
made continuous at x = 0 by defining 49
(A) 25 (B)
f(0) to be 2
(A) 1 (B) 1/2 63
(C) (D) 16
(C) 0 (D) e 2

5. The differentiability of a function 9. The volume V in first octant bounded


continuity is by z = y2, x = 2 and y = 4 is
(A) sufficient 64 128
(A) (B)
(B) necessary 3 3

(C) sufficient and necessary 16 32


(C) (D)
(D) none of these 3 3

164
Test Paper on Calculus

10. The area of region R bounded by /4 a cos 2  r dr


xy = 1 and 2x + y = 3 is
13. I =  / 4 d0 a2  r 2
(A) n 2 (B) 2 + n 2    
(A) 2a 1   (B) 2a 2  
3  4  4
(C)   n2 (D) None of these
4
   
(C) 2a 1   (D) 2a  2  
 4  4 
11. The volume in first octant bounded by
2x + 2y  z + 1 = 0 y = x and x = 2 is 1 x 2x y

(A) 10 (B) 28
14. I = 0 x 0
2
dx dy dz

(C) 12 (D) 16 11 13
(A) (B)
30 30
1
12. The area of region enclosed by (C) (D) 16
8
cardioide r = 1 + cos  is
 3
1 y 1 x 
(A) (B) 15. I =
2 1 2
 y 2 dx dy
2 2
(C)  (D) 2 1 1
(A) (B)
3 3
2 2
(C) (D)
3 3



165
Chapter - 3 : Probability and Statistics
3.1 Basic Terms
Random Experiment
Consider an action which is repeated under essentially identical conditions. If it results in
any one of the several possible outcomes, but it is not possible to predict which outcome
will appear. Such an action is called as a Random Experiment. One performance of such
an experiment is called as a Trial.

Sample Space
The set of all possible outcomes of a random experiment is called as the sample space.
All the elements of the sample space together are called as ‘exhaustive cases’. The
number of elements of the sample space i.e. the number of exhaustive cases is denoted
by n(S) or N or n.

Event
Any subset of the sample space is called as an ‘Event’ and is denoted by some capital
letter like A, B, C or A1, A2, A3,.. or B1, B2, ... etc.

Favourable cases
The cases which ensure the happening of an event A, are called as the cases favourable
to the event A. The number of cases favourable to event A is denoted by n(A) or NA or nA.

Mutually Exclusive Events or Disjoint Events


Two events A and B are said to be mutually exclusive or disjoint if A  B = 
i.e. if there is no element common to A & B.

Equally Likely Cases


Cases are said to be equally likely if they all have the same chance of occurrence i.e. no
case is preferred to any other case.

Permutation
A permutation is an arrangement of all or part of a set of objects. The number of
permutations of n distinct objects taken r at a time is

n n!
Pr =
n  r  !

166
Notes on Probability and Statistics

Note: The number of permutations of n distinct objects is n! i.e., nPn = n!


The number of permutations of n distinct objects arranged in a circle
is (n  1)!
The number of distinct permutations of n things of which n1 are of one
n!
kind, n2 of a second kind … nk of a kth kind is
n1! n2! ......nk!

Combination
A combination is selection of all or part of a set of objects. The number of combinations of
n distinct objects taken r at a time is

n n!
Cr =
r!  n  r  !

Note: In a permutation, the order of arrangement of the objects is important.


Thus abc is a different permutation from bca.
In a combination, the order in which objects are selected does not matter.
Thus abc and bca are the same combination.

3.2 Definition of Probability


Consider a random experiment which results in a sample space containing n(S) cases
which are exhaustive, mutually exclusive and equally likely. Suppose, out of n(S) cases,
n(A) cases are favourable to an event A. Then the probability of event A is denoted by
P(A) and is defined as follows.

n(A) number of cases favourable to event A


P(A) = =
n(S) number of cases in the sample space S

3.3 Complement of an event


The complement of an event A is denoted by A and it contains all the elements of the
sample space which do not belong to A.
For example: Random experiment: an unbiased die is rolled.
S = {1, 2, 3, 4, 5, 6}
(i) Let A: number on the die is a perfect square

 A = {1, 4}  A = {2, 3, 5, 6}

167
Vidyalankar : GATE – Engineering Mathematics

(ii) Let B: number on the die is a prime number

 B = {2, 3, 5}  B = {1, 4, 6}

Note: P(A) + P( A ) = 1 i.e. P(A) = 1  P( A )

For any events A and B, P  A  = P  A  B  + P A  B  


3.4 Independent Events
Two events A & B are said to be independent if

P(A  B) = P(A).P(B)

Note: If A & B are independent then

A & B are independent

A & B are independent


A & B are independent

3.5 Theorems of Probability


Addition Theorem
If A and B are any two events then

P(AB) = P(A) + P(B) – P(AB)

Note: 1. A  B : either A or B or both i.e. at least one of A & B

A  B : neither A nor B i.e. none of A & B

AB & A  B are complement to each other

 P( A  B ) = 1 – P(A  B)

2. If A & B are mutually exclusive, P(A  B) = 0


P(A  B) = P(A) + P(B)

3. P  A1  A2  A3  = P  A1  + P  A2  + P  A3 P  A1  A2 

P  A2  A3   P  A3  A1  + P  A1  A2  A3 

168
Notes on Probability and Statistics

Multiplication Theorem
If A & B are any two events then

P(A  B) = P(A).P(B/A) = P(B).P(A/B)

1. Conditional probability of occurrence of event B given that event A has already


occurred.

P A  B
P(B/A) =
P A

2. Conditional probability of occurrence of event A given that event B has already


occurred

P A  B
P(A/B) =
P B 

Bayes’ Theorem
Suppose that a sample space S is a union of mutually disjoint events B1, B2, B3, ..., Bn,
suppose A is an event in S, and suppose A and all the Bi’s have nonzero probabilities. If
k is an integer with 1 ≤ k ≤ n, then

P  A / Bk  P Bk 
P(Bk / A) =
P  A / B1  P B1  + P  A / B 2  P B 2  + ... + P  A / Bn  P Bn 

Solved Example 1 : Solved Example 2 :


A single die is tossed. Find the probability A coin is tossed twice. What is the
of a 2 or 5 turning up. probability that at least one head occurs ?
Solution : Solution :
The sample space is S = {1, 2, 3, 4, 5, 6} The sample space is
1
P(1) = P(2) = … = P(6) = S = {HH, HT, TH, TT}
6
The event that either 2 or 5 turns up in Probability of each outcomes = 1/4
indicated by 2  5. Thus
Probability of atleast one head occurring is
1 1 1
P 2  5  P 2  P 5    1 1 1 3
6 6 3 P(A) =   
4 4 4 4

169
Vidyalankar : GATE – Engineering Mathematics

Solved Example 3 : 2 1 2 2 7
P  A  B     
A die is loaded in such a way that an even 9 9 9 9 9
number is twice as likely to occur as an 2
and P  A  B 
odd number. If E is the event that a 9

number less than 4 occurs on a single toss


Solved Example 5 :
of the die. Find P(E).
A mixture of candies 6 mints, 4 toffees and
Solution :
3 chocolates. If a person makes a random
S = {1, 2, 3, 4, 5, 6 }
selection of one of these candies, find the
We assign a probability of w to each odd
probability of getting (a) a mint, or
number and a probability of 2w to each
(b) a toffee or a chocolate.
even number. Since the sum of the
Solution :
probabilities must be 1, we have 9w = 1 or
(a) Since 6 of the 13 candies are mints,
w = 1/9.
the probability of event M, selecting
Hence probabilities of 1/9 and 2/9 are mint at random, is
assigned to each odd and even number 6
P(M) =
respectively. 13
 E = {1, 2, 3}
1 2 1 4 (b) Since 7 of the 13 candies are toffees
and P(E) =   
9 9 9 9 or chocolates it follows that
7
P  T  C 
Solved Example 4 : 13
In the above example let A be the event
that an even number turns up and let B be Solved Example 6 :

the event that a number divisible by 3 In a poker hand consisting of 5 cards, find

occurs. Find P(A  B) and P(A  B). the probability of holding 2 aces and

Solution : 3 jacks.

A = {2, 4, 6} and Solution :

B = {3, 6} The number of ways of being dealt 2 aces


from 4 is
We have,
4 4!
C2  6
A  B = {2, 3, 4, 6} and 2!2!
A  B = {6} The number of ways of being dealt 3 jacks
from 4 is
By assigning a probability of 1/9 to each
4 4!
C3  4
odd number and 2/9 to each even number 3!1!

170
Notes on Probability and Statistics

 Total number of ways n = 6.4 = 24 points. Since all sample points are equally
hands with 2 aces and 3 jacks. likely, we have P(A) = 1/6 and P(B) = 1/18.

The total number of 5card poker hands all The events A and B are mutually exclusive
of which are equally likely, is since a total of 7 and 11 cannot both occur
52! on the same toss.
N= 52
C5  = 2, 598, 960
5!47!  P  A  B   P  A   P B 
 The probability of event C of getting 2
1 1 2
aces and 3 jacks in a 5card poker =  =
6 18 9
hand is
Solved Example 9 :
24
P(C) =  0.9  105 If the probabilities are respectively 0.09,
2,598,960
0.15, 0.21 and 0.23 that a person
Solved Example 7 : purchasing a new automobile will choose
The probability that Paula passes the colour green, white, red or blue. What
mathematics is 2/3 and the probability that is the probability that a given buyer will
she passes English is 4/9. If the purchase a new automobile that comes in
probability of passing both courses is 1/4, one of those colours ?
what is the probability that Paula will pass Solution :
at least one of these courses ? Let G, W, R and B be the events that a buyer
Solution : selects, respectively, a green, white, red or
If M is the event “passing Mathematics” blue automobile. Since these four events are
and E the event “passing English” then mutually exclusive the probability is
P G  W  R  B
P(M  E) = P  M   P E   P  M  E 
 P  G   P  W   P R   P  B 
2 4 1 31  0.09  0.15  0.21  0.23 = 0.68
=   
3 9 4 36
Solved Example 10 :
Solved Example 8 :
If the probabilities that an automobile
What is the probability of getting a total of
mechanic will service 3, 4, 5, 6, 7 or 8
7 or 11 when a pair of dice are tossed ?
more cars on any given workday are
Solution :
respectively 0.12, 0.19, 0.28, 0.24, 0.10
Let A be the event that 7 occurs and B the
and 0.07. What is the probability that he
event that 11 comes up. Now a total of 7
will service at least 5 cars on next day at
occurs for 6 of the 36 sample points and a
work?
total of 11 occurs for only 2 of the sample

171
Vidyalankar : GATE – Engineering Mathematics

Solution : Solution :
Let E be the event that at least 5 cars are Suppose there are 100 flowers
serviced. Now Number of roses = 40; Number of
P(E) = 1  P(E), where E is the event carnations = 60
that fewer than 5 cars are serviced. 25% of 40 = 10 roses are red and
Since P(E) = 0.12 + 0.19 = 0.31 10% of 60 = 6 carnations are red

 P(E) = 1  0.31 = 0.69 Let A be the event that the flower is red
and B the event that the flower is a rose.
Solved Example 11 :  A  B is the event that the flower is a
In a garden 40% of the flowers are roses red rose.
and the rest are carnations. If 25% of the 16
n(A) = 16  P(A) =
roses and 10% of the carnations are red, 100
the probability that a red flower selected at 10
n(A  B) = 10  P (A  B) =
random is a rose. 100
(A) 5/6 (B) 1/4 P(B/A) = probability that a selected flower
(C) 4/5 (D) 5/8 is a rose red is colour
P  A  B 10 / 100 5
P(B/A) =  
PA 16 / 100 8

3.6 Random Variables


A random variable is a function that associates a real number with each element in the
sample space. It is a set of possible values from a random experiment.
For example: Suppose that a coin is tossed twice so that the sample space is S = {HH,
HT, TH, TT}. Let X represent the number of heads that can come up. With each sample
point we can associate a number for X as shown in the table.

SAMPLE POINT HH HT TH TT

X 2 1 1 0

Discrete Random Variable


A random variable is discrete if it can take only discrete or separate values. Value of a
discrete random variable is often obtained by counting.
For example: Number of heads appearing when three coins are tossed.

172
Notes on Probability and Statistics

Continuous Random Variable


A random variable is continuous if can take any value in a given range. Value of a
continuous random variable is often obtained by measuring.

For example: Time required to travel certain distance.

3.7 Probability Distribution Function


In general sense, probability distribution function may refer to:
 Probability mass function
 Probability density function
 Cumulative distribution function

Probability mass function


It is a function that gives probability that a discrete random variable is exactly equal to
some value.

Probability density function


It is used to specify the probability of a continuous random variable falling within a
particular range of values and the probability is given by the integral of the probability
density function over that range.

Cumulative distribution function


It is a probability that random variable X will take a value less than or equal to x.
i.e., F(x) = P(X ≤ x)

3.8 Expectation(Mean), Variance and Standard Deviation


Expected value for a discrete random variable is given by

n
E(X) = 
i=1
xi P(xi )

Expected value for a continuous random variable is given by

b
E(X) =  x f(x) dx
a

173
Vidyalankar : GATE – Engineering Mathematics

Variance of random variable is given by

Var(X) = E((X  )2) = E(X2)  E(X)2 = E(X2)  2

Standard deviation is square root of variance

= Var(X) 

Note: 1. Expected value  = E(X) is a measure of central tendency.


2. Standard deviation  is a measure of spread.

Solved Example 12 : 4, 5, 6, 7, 8, 9, 10, 10, 10, 10 each having


Thirteen cards are drawn simultaneously 1
the same probability so that
from a deck of 52 cards. If aces count 1, 13
face cards 10 and other count by their 1
p(x) = 1  2  ...  10  10  10  10 
denomination, find the expectation of the 13

total score on 13 cards. 85



Solution : 13

Let x denote the number corresponding to Hence the required expectation is 85.

the ith card. Then x takes the value, 1, 2, 3,

3.9 Standard Distributions


Binomial Distribution
Probability of exactly k successes in n trials is given by

P(X=k) = nCk pk (1  p)n  k

where p is the probability of success in each trial


Here the trial are Bernouilli trials, where each trial can have exactly two outcomes viz.
success and failure.
If p is the probability of success
then q = 1  p is the probability of failure.
Then

P(X=k) = nCk pk qn  k

174
Notes on Probability and Statistics

For Binomial Distribution,


Mean = np
Variance = npq = np(1  p)
Variance < Mean
k
Cumulative distribution function is, F(x) = 
i 0
n
Ci pi qni

Poisson Distribution
Probability of k events occurring in an interval is given by

 k e
P(k events) =
k!

where  is the average number of events per interval.


For a Poisson Distribution,
Mean = 
Variance = 
Variance = Mean
k
i
Cumulative distribution function is, F(x) = e  
i0 i!

Normal Distribution
Probability density function of normal distribution is

(x  )
1 
2 2
f(x) = e
 2

where  is the mean and 2 is the variance

Graph of f(x) is symmetric about x =  and is a bell shaped curve.

1  x   
Cumulative distribution function is, F(x) = 1erf  
2   2 

175
Vidyalankar : GATE – Engineering Mathematics

Note: Standard Normal Distribution


Special case with  = 0 and  = 1
x2

e 2
Probability density function is f(x) =
2
This graph is symmetric about x = 0 where it attains its maximum value
1
2

Uniform Distribution
Probability density function for uniform distribution is
1
a≤x≤b
ba
f(x) =
0 x < a or x > b

For Uniform Distribution,


ab
Mean =
2
(ba)2
Variance =
12
0 xa
xa
Cumulative distribution function is, F(x) = axb
ba
1 xb

Exponential Distribution
Probability density function for uniform distribution is

ex x≥0
f(x) =
0 x<0

For exponential distribution


1
Mean =

1
Variance =
2
Cumulative distribution function is, F(x) = 1  ex

176
Notes on Probability and Statistics

Solved Example 13 : 8

The probability of getting exactly 2 heads


b) P(3  X 8) =  15 Ci (0.4)i (0.6)15i
i 3

in 6 tosses of a fair coin is = 0.8779


Solution : c) P (X = 5) = 15
C5 (0.4)5 (0.6)15  5
2 62 = 0.1859
 1  1
P (X = 2) = 6 C2    
2 2 Solved Example 16 :
2 62
6!  1   1  15 The probability is 0.02 that an item
= 
2!4!  2   2  64 produced by a factory is defective. A
shipment of 10,000 items are sent to its
Solved Example 14 :
warehouse. Find the expected number E
The probability that a certain kind of
of defective items and the standard
component will survive a given shock test
deviation.
is 3/4. Find the probability that exactly 2 of
the next 4 components tested survive. Solution :

Solution :  = np = 10000  0.02 = 200

Assuming the tests are independent and = npq  10000    
p = ¾ for each of the 4 tests we get = 196 = 14
2 2 2
4  3  1 4! 3 27
C2     = 4
= Solved Example 17 :
4 2 2!2! 4 128
Ten percent of the tools produces in a
Solved Example 15 : certain manufacturing turn out to be
The probability that a patient recovers from defective. Find the probability that in a
a rare blood disease is 0.4. If 15 people sample of 10 tools chosen at random
are to have contracted this disease, what exactly two will be defective.
is the probability that Solution :
a) at least 10 survive We have  = np = 10 (0  1) = 1
b) from 3 to 8 survive Then according to the Poisson distribution
c) exactly 5 survive e  x
P (X = x ) =
Solution : x!

Let X be the number of people that survive. 12 e1


or P(X = 2) =  0.1839 or 0.18
a) P(X  10) = 1  P(X  10) 2!
In general the approximation is good if
9
= 1   15 Ci (0.4)i (0.6)15 i P  0.1 and  = np  5
i 0

= 1  0.9662 = 0.0338

177
Vidyalankar : GATE – Engineering Mathematics

Solved Example 18 :  x e 
P (X = x) =
If the probability that an individual suffers a x!
bad reaction from injection of a given where  = np = 2000 (0.001) = 2
serum is 0.001, determine the probability 23 e2
a) P (X = 3) =  0.180
that out of 2000 individuals 3!
a) exactly 3
b) P (X > 2)
b) more than 2, individuals will suffer a
= 1  P  X  0   P  X  1  P  X  2  
bad reaction.
Solution :  20 e2 21e 2 22 e2 
= 1    
Let X denote the number of individuals  0! 1! 2! 

suffering a bad reaction. X is poisson = 1  5e2 = 0.323


distributed.

3.10 Mean, Median, Mode and Standard Deviation


Mean
Mean of a data generally refers to arithmetic mean of the data. However, there are two
more types of mean which are geometric mean and harmonic mean. The different types
of mean for set of values and grouped data are given in the following table.

Set of Values Grouped Data

 xi  fi xi
Arithmetic Mean
n  fi

  f log xi 
Geometric Mean (x1 . x 2 . x 3 ....xn )1/n Antilog  i 
  fi 

n
 fi
Harmonic Mean  1 1 1 1
 + +    fi xi
 x1 x 2 x 3 xn 

178
Notes on Probability and Statistics

Median
For an ordered set of values, median is the middle value. If the number of values is even,
median is taken as mean of the middle two values.
For a grouped data, median is given by

N 
 2  C
Median = L +   h
f

where, L = Lower boundary of the median class


N =  fi
C = Cumulative frequency upto the class before the median class
h = Width of median class
f = Frequency of the median class
Mode
For a set of values, mode is the most frequently occurring value. For a grouped data,
mode is given by

 fi  fi-1 
Mode = L + h
 2fi  fi-1  fi+1 
where, L = Lower boundary of the modal class
fi = Frequency of the modal class (Highest frequency)
fi1 = Frequency of the class before the modal class
fi+1 = Frequency of the class after the modal class
h = Width of the modal class

Note: Relation between Mean, Median and Mode :


Mean  Mode = 3(Mean  Median)

Standard Deviation
Standard deviation is given by,

  xi  x 
2
2
 = For a Set of Values
n
 fi  xi  x 
2
2
 = For a Grouped Data
 fi

179
Vidyalankar : GATE – Engineering Mathematics

3.11 Correlation and Regression Analysis


Correlation
Correlation analysis deals with finding linear association between two variables i.e. if the
two variables are linearly related to each other.
Correlation coefficient between variables x and y can be found as follows.

 (xi  x) (yi  y) n x i y i   x i  y i
r= =
2 2
 (xi  x)  (yi  y) n xi2  (  xi )2  n yi2  (  yi )2 

where, x = mean of x
y = mean of y

Regression
Regression analysis involves identifying the relationship between a dependent variable
and one or more independent variables.
If X and Y are the variables then equation for regression line of Y on X is given by

Y = a + bX
n xy   x  y
where, b=
n  x 2  (  x)2
and a can be found by using
y = a + bx

Note: Correlation coefficient measures linear association between two variables.


r can vary between 1 to 1 i.e. 1 ≤ r ≤ 1.
If there is no linear association between the two variables, r will be zero.

180
Notes on Probability and Statistics

List of Formulae
 Permutation and Combination  Bayes’ Theorem

n n! 
P(Bk )P(A / Bk )
Pr = P(Bk/A) =
  r !
n n
 P(Bi P(A/Bi 
i 1
n n!
Cr =
r!  n  r  !  Expectation(Mean), Variance and
 Probability Standard Deviation
n
n(A) E(X) =  xi P(xi )
P(A) = i 1
n(S)
…. For Discrete Random Variable
b
 P(A) + P( A ) = 1
E(X) =  x f(x) dx
a

 
P  A  = P  A  B + P A  B  ….. For Continuous Random Variable

Var(X) = E((X  )2) = E(X2)  E(X)2


 P(AB) = P(A) + P(B) – P(AB)
= E(X2)  2

 P  A1  A 2  A 3  = P  A1  + P  A 2 
= Var(X)
+P  A 3   P  A1  A 2 P  A 2  A 3 
 Binomial Distribution
P  A 3  A1  + P  A1  A 2  A 3  P(X=k) = nCk pk (1  p)n  k
Mean = np
 Two events A & B are said to be
Variance = npq = np(1  p)
independent if
Variance < Mean
P(A  B) = P(A).P(B)
Cumulative distribution function is,
 P(A  B) = P(A).P(B/A) = P(B).P(A/B) k
F(x) = 
i 0
n
Ci pi qni

 Conditional probability of occurrence  Poisson Distribution


of event B given that event A has
k e 
already occurred P(k events) =
k!
P(A  B) Mean = 
P(B/A) =
P(A)
Variance = 
Variance = Mean

181
Vidyalankar : GATE – Engineering Mathematics

Cumulative distribution function is, Cumulative distribution function is,


k
 i
F(x) = 1  ex
F(x) = e  
i0 i!
 Mean
 Normal Distribution
Arithmetic Mean
(x  )
1 
2 2  xi
f(x) = e = …. for set of values
 2 n
Mean =   fi xi
= …. for grouped data
Variance =  2
 fi

Cumulative distribution function is, Geometric Mean

1  x    = (x1.x2.x3….xn)1/n …. for set of values


F(x) = 1erf  
2   2    f log xi 
= Antilog  i 
  fi 
 Uniform Distribution
…. for grouped data
1
a≤x≤b
ba Harmonic Mean
f(x) =
0 x < a or x > b n
=
 1 1 1 1
ab        
Mean = x
 1 x 2 x 3 x n 
2
…. for set of values
(ba)2
Variance =  fi
12 = …. for grouped data
 fi xi
Cumulative distribution function is,
0 xa  Median
xa For an ordered set of values, median
F(x) = axb
ba is the middle value. If the number of
1 xb
values is even, median is taken as
 Exponential Distribution mean of the middle two values.
e x
x≥0 For a grouped data,
f(x) =
0 x<0 N 
 2 C 
Median = L   h
1 f
Mean =

1
Variance =
2

182
Notes on Probability and Statistics

where, fi = Frequency of the modal class


L = Lower boundary of the median class (Highest frequency)
N = fi fi1 = Frequency of the class before the
C = Cumulative frequency upto the modal class
class before the median class fi+1 = Frequency of the class after the
h = Width of median class modal class
f = Frequency of the median class h = Width of the modal class

 Relation between Mean, Median and


 Mode
Mode :
For a set of values, mode is the most
Mean  Mode = 3(Mean  Median)
frequently occurring value.
For a grouped data,  Standard Deviation
 fi fi1    xi x 
2
Mode = L  h 2 = For a Set of Values
 2fi fi1fi1  n
where,  fi  xi x 
2
2
 = For a Grouped Data
L = Lower boundary of the modal class  fi



183
Vidyalankar : GATE – Engineering Mathematics

Assignment  1
Duration : 45 Min. Max. Marks : 30

Q 1 to Q 10 carry one mark each 5. Among the four normal distributions


1. A sample of 15 data is as follows: 17, with probability density functions as
18, 17, 17, 13, 18, 5, 5, 6, 7, 8, 9, 20, shown below, which one has the
17, 3. The mode of the data is lowest variance? [ME  2015]
IV
[ME  2017]
(A) 4 (B) 13
III
(C) 17 (D) 20
II
2. In a housing society, half of the I
families have a single child per family,
2 1 0 1 2
while the remaining half have two
(A) I (B) II
children per family. The probability that
(C) III (D) IV
a child picked at random, has a sibling
is _______. [EC2014] 6. A random variable X has probability
density function f(x) as given below:
3. An unbiased coin is tossed an infinite
number of times. The probability that a  bx for0x1
f(x) = 
the fourth head appears at the tenth  0 otherwise

toss is [EC2014] If the expected value E[X] = 2/3, then

(A) 0.067 (B) 0.073 Pr[X < 0.5] is _______. [EE2015]

(C) 0.082 (D) 0.091 7. Consider a Poisson distribution for the


tossing of a biased coin. The mean for
4. The probability density function of
this distribution is . The standard
evaporation E on any day during a
year in a watershed is given by deviation for this distribution is given
by [ME2016]
1
 0E5mm / day
f(E) =  5 (A)  (B) 2
 0 otherwise
(C)  (D) 1/
The probability that E lies in between 2
and 4 mm/day in a day in the
watershed is (in decimal) _________
[CE2014]

184
Assignment on Probability and Statistics

8. The probability of getting a “head” in a 12. The number of accidents occurring in


single toss of a biased coin is 0.3. The a plant in a month follows Poisson
coin is tossed repeatedly till a “head” is distribution with mean as 5.2. The
obtained. If the tosses are independent, probability of occurrence of less than 2
then the probability of getting “head” for accidents in the plant during a
the first time in the fifth toss is _______ randomly selected month is[ME2014]
[EC2015] (A) 0.029 (B) 0.034
9. X and Y are two random independent (C) 0.039 (D) 0.044
events. It is known that P(X) = 0.40 and
13. Parcels from sender S to receiver R
P(X  YC) = 0.7. Which one of the
pass sequentially through two post-
following is the value of P(X  Y)?
offices. Each post-office has a
[CE2016]
1
(A) 0.7 (B) 0.5 probability of losing an incoming
5
(C) 0.4 (D) 0.3
parcel, independently of all other
10. Suppose that a shop has an equal
parcels. Given that a parcel is lost, the
number of LED bulbs of two different
probability that it was lost by the
types. The probability of an LED bulb
second post-office is ____. [EC2014]
lasting more than 100 hours given that
it is of Type 1 is 0.7, and given that it is 14. A traffic office imposes on an average
of Type 2 is 0.4. The probability that 5 number of penalties daily on traffic
an LED bulb chosen uniformly at violators. Assume that the number of
random lasts more than 100 hours is penalties on different days is
________. [CS2016] independent and follows a Poisson
Q 11 to Q 20 carry two marks each distribution. The probability that there
11. In the following table, x is a discrete will be less than 4 penalties in a day is
random variable and p(x) is the ________. [CE2014]
probability density. The standard
deviation of x is [ME2014]

x 1 2 3

P(x) 0.3 0.6 0.1

(A) 0.18 (B) 0.36


(C) 0.54 (D) 0.6

185
Vidyalankar : GATE – Engineering Mathematics

15. The chance of a student passing an 18. The variance of the random variable X
exam is 20%. The chance of a student with probability density function
passing the exam and getting above 1
f(x) = | x |e|x| is _____. [EC2015]
90% marks in it is 5%. GIVEN that a 2
student passes the examination, the 19. Two random variables ܺ and ܻ are
probability that the student gets above distributed according to
90% marks is [ME2015] FX,Y(x, y)

1 1 (x  y), 0  x  1, 0  y  1,
(A) (B) = 
18 4 0, otherwise.

2 5 The probability P(X + Y  1) is ______.


(C) (D)
9 18 [EC2016]
16. The probability density function of a
random variable, x is 20. An urn contains 5 red and 7 green
x balls. A ball is drawn at random and its
f(x) = (4x 2 ) for 0  x  2
4
colour is noted. The ball is placed back
= 0 otherwise
into the urn along with another ball of
The mean, x of the random variable is the same colour. The probability of
___________. [CE2015] getting a red ball in the next draw is
17. Suppose a fair six-sided die is rolled [IN2016]
once. If the value on the die is 1, 2, or 65 67
(A) (B)
3, the die is rolled a second time. What 156 156

is the probability that the sum total of 79 89


(C) (D)
values that turn up is at least 6? 156 156

[EC2015]
(A) 10/21 (B) 5/12
(C) 2/3 (D) 1/6



186
Assignment on Probability and Statistics

Assignment  2
Duration : 45 Min. Max. Marks : 30

Q 1 to Q 10 carry one mark each 1


(A) 0 (B)
1. A group consists of equal number of 2

men and women. Of this group 20% of 4 1


(C) (D)
the men and 50% of the women are 5 5

unemployed. If a person is selected at


5. Three vendors were asked to supply a
random from this group, the probability
very high precision component. The
of the selected person being employed
respective probabilities of their
is_______ [ME2014] meeting the strict design specifications
are 0.8, 0.7 and 0.5. Each vendor
2. Vehicle arriving at an intersection from
supplies one component. The
one of the approach roads follow the
probability that out of total three
Poisson distribution. The mean rate of
components supplied by the vendors,
arrival is 900 vehicles per hour. If a
at least one will meet the design
gap is defined as the time difference
specification is ________. [ME2015]
between two successive vehicle
arrivals (with vehicle assumed to be 6. Consider the following probability
points), the probability (up to four mass function (p.m.f.) of a random
decimal places) that the gap is greater variable X:
than 8 second is _______. [CE2017]  q ifX0

p(x, q) = 1  q ifX1
3. Let X be a zero mean unit variance  0 otherwise

Gaussian random variable. E[|X|] is
If q = 0.4, the variance of X is ______.
equal to _______. [EC2014]
[CE2015]

4. A fair (unbiased) coin was tossed four 7. The area (in percentage) under
times in succession and resulted in the standard normal distribution curve of
following outcomes: (i) Head, (ii) Head, random variable Z within limits from
(iii) Head, (iv) Head. The probability of −3 to +3 is __________ [ME2016]
obtaining a ‘Tail’ when the coin is
tossed again is [CE2014]

187
Vidyalankar : GATE – Engineering Mathematics

8. Types II error in hypothesis testing is Q 11 to Q 20 carry two marks each


(A) acceptance of the null hypothesis 11. Consider an unbiased cubic dice with
when it is false and should be opposite faces coloured identically and
rejected each face coloured red, blue or green
(B) rejection of the null hypothesis such that each colour appears only
when it is true and should be two times on the dice. If the dice is
accepted thrown thrice, the probability of
(C) rejected of the null hypothesis obtaining red colour on top face of the
when it is false and should be dice at least twice is ____. [ME2014]
rejected
12. Let X be a real-valued random variable
(D) acceptance of the null hypothesis
with E[X] and E[X2] denoting the mean
when it is true and should be
values of X and X2, respectively. The
accepted [CE2016]
relation which always holds true is
9. A voltage V1 is measured 100 times (A) (E[X])2 > E[X2] [EC2014]
and its average and standard deviation
(B) E[X2]  (E [X])2
are 100 V and 1.5 V respectively. A
(C) E[X2] = (E[X])2
second voltage V2, which is
(D) E[X2] > (E[X])2
independent of V1, is measured
13. A fair coin is tossed n times. The
200 times and its average and
probability that the difference between
standard deviation are 150 V and 2 V
the number of heads and tails is
respectively. V3 is computed as:
V3 = V1 + V2. Then the standard (n  3) is [EE2014]

deviation of V3 in volt is ________. (A) 2n (B) 0

[IN2016] (C) Cn3 2n


n
(D) 2n + 3

10. Suppose p is the number of cars per 14. An observer counts 240 veh/h at a
minute passing through a certain road specific highway location. Assume that
junction between 5 PM and 6 PM, and the vehicle arrival at the location is
p has a Poisson distribution with mean Poisson distributed, the probability of
3. What is the probability of observing having one vehicle arriving over a
fewer than 3 cars during any given 30second time interval is _________.
minute in this interval? [CS2013] [CE2014]
3 3
(A) 8/(2e ) (B) 9/(2e )
3
(C) 17/(2e ) (D) 26/(2e3)

188
Assignment on Probability and Statistics

15. Two players, A and B, alternately keep 18. The probability that a screw
rolling a fair dice. The person to get a six manufactured by a company is
first wins the game. Given that player A defective is 0.1. The company sells
starts the game, the probability that A screws in packets containing 5 screws
wins the game is [EE2015] and gives a guarantee of replacement
(A) 5/11 (B) 1/2 if one or more screws in the packet are
(C) 7/13 (D) 6/11 found to be defective. The probability

16. The probability that a thermistor that a packet would have to be

randomly picked up from a production replaced is _______. [ME2016]

unit is defective is 0.1. The probability


that out of 10 thermistors randomly 19. If a random variable X has a Poisson

picked up, 3 are defective is distribution with mean 5, then the

[IN2015] expectation E[(X + 2)2] equals

(A) 0.001 (B) 0.057 _________. [CS2017]

(C) 0.107 (D) 0.3


20. Consider the following experiment.
17. Let X  {0, 1} and Y  (0, 1) be two Step 1. Flip a fair coin twice.
independent binary random variables. Step 2. If the outcomes are (TAILS,
If P(X = 0) = p and P(Y = 0) = q, the HEADS) then output Y and stop.
P(X + Y  1) is equal to [EC2015] Step 3. If the outcomes are either
(A) pq + (1  p) (1  q) (HEADS, HEADS) or (HEADS,
(B) pq TAILS), then output N and stop.
(C) p(1  q) Step 4. If the outcomes are (TAILS,
(D) 1  pq TAILS), then go to Step 1.
The probability that the output of the
experiment is Y is (up to two decimal
places_______. [CS2016]



189
Vidyalankar : GATE – Engineering Mathematics

Assignment  3
Duration : 45 Min. Max. Marks : 30

Q 1 to Q 10 carry one mark each 4. If {x} is a continuous, real valued


1. A nationalized bank has found that the random variable defined over the
daily balance available in its savings interval (∞, + ∞) and its occurrence is
accounts follows a normal distribution defined by the density function given as:
with a mean of Rs. 500 and a standard 1 x a 
2

1  
2 b 

f(x) = e where 'a' and 'b'
deviation of Rs. 50. The percentage of 2  b
savings account holders, who maintain are the statistical attributes of the
an average daily balance more than random variable {x}. The value of the
Rs. 500 is________ [ME2014] 1  xa 
2
a 1   

2 b 
integral e dx is

2  b
2. Let X be a random variable which is
uniformly chosen from the set of [CE2014]

positive odd numbers less than 100. (A) 1 (B) 0.5

The expectation, E[X], is _________. 


(C)  (D)
2
[EC2014]

3. Ram and Ramesh appeared in an 5. If P(X) = 1/4, P(Y) = 1/3, and


interview for two vacancies in the P(X  Y) = 1/12, the value of P(Y/X) is
same department. The probability of [ME2015]
Ram's selection is 1/6 and that of 1 4
(A) (B)
Ramesh is 1/8. What is the probability 4 25
that only one of them will be selected? 1 29
(C) (D)
[EC2015] 3 50

(A) 47/48 (B) 1/4


(C) 13/48 (D) 35/48

190
Assignment on Probability and Statistics

6. Suppose A and B are two independent Q 11 to Q 20 carry two marks each


events with probabilities P(A)  0 and 11. A machine produces 0, 1 or 2

P(B)  0. Let A and B be their defective pieces in a day with

complements. Which one of the associated probability of 1/6, 2/3 and

following statements is FALSE? 1/6, respectively. The mean value and

[EC2015] the variance of the number of


defective pieces produced by the
(A) P(A  B) = P(A) P(B)
machine in a day, respectively, are
(B) P(A|B) = P(A)
[ME2014]
(C) P(A  B) = P(A) + P(B)
(A) 1 and 1/3 (B) 1/3 and 1
(D) P(AB) = P(A)P(B)

(C) 1 and 4/3 (D) 1/3 and 4/3
7. The second moment of a Poisson-
distributed random variable is 2. The 12. A box contains 4 red balls and 6 black
mean of the random variable is _____. balls. Three balls are selected
[EC2016] randomly from the box one after

8. The spot speeds (expressed in km/hr) another without replacement. The

observed at a road section are 66, 62, probability that the selected set

45, 79, 32, 51, 56, 60, 53, and 49. The contains one red ball and two black

median speed (expressed in km/hr) is balls is [EC2014]

______ (Note: answer with one 1 1


(A) (B)
20 12
decimal accuracy) [CE2016]
3 1
9. A probability density function on the (C) (D)
10 2
interval [a, 1] is given by 1/x2 and
outside this interval the value of the 13. Let X be a random variable with
function is zero. The value of a is probability density function
______. [CS2016] 0.2, for| x |1

10. Let U and V be two independent zero f(x) 0.1,for1| x |4
 0, otherwise.
mean Gaussian random variables of 

1 1 The probability P(0.5 < x < 5) is


variances and respectively. The
4 9 ________. [EE2014]
probability P(3V  2U) is [ME2013]
(A) 4/9 (B) 1/2
(C) 2/3 (D) 5/9

191
Vidyalankar : GATE – Engineering Mathematics

14. The probability of obtaining at least 18. Three cards were drawn from a pack
two "SIX" in throwing a fair dice 4 of 52 cards. The probability that they
times is [ME2015] are a king, a queen, and a jack is
(A) 425/432 (B) 19/144 [ME2016]
(C) 13/144 (D) 125/432 16 64
(A) (B)
5525 2197
15. Two coins R and S are tossed. The 4
3 8
(C) (D)
joint events HRHS, TRTS, HRTS, TRHS 13 16575
have probabilities 0.28, 0.18, 0.30,
0.24, respectively, where H represents 19. Probability density function of a
head and T represents tail. Which one random variable X is given below.
of the following is TRUE? [EE2015]
0.25 if1x5
f(x) = 
(A) The coin tosses are independent  0 otherwise
(B) R is fair, S is not
P(X  4) is [CE2016]
(C) S is fair, R is not
3 1
(D) The coin tosses are dependent (A) (B)
4 2
1 1
16. The probability density function of a (C) (D)
4 8
random variable X is pX(x) = ex for
x  0 and 0 otherwise. The expected
20. Consider the random process
value of the function gX(x) = e3x 4 is X(t) = U + Vt
where U is a zero-mean Gaussian
___________. [IN2015]
random variable and V is a random
variable uniformly distributed between
17. A fair die with faces {1, 2, 3, 4, 5, 6} is
0 and 2. Assume that U and V are
thrown repeatedly till '3' is observed for
statistically independent. The mean
the first time. Let X denote the number
value of the random process at t = 2 is
of times the die is thrown. The
________. [EC2017]
expected value of X is __________.
[EC2015]



192
Assignment on Probability and Statistics

Assignment  4
Duration : 45 Min. Max. Marks : 30
Q 1 to Q 6 carry one mark each Q 7 to Q 18 carry two marks each
1. Probability of getting an even number 7. Probability than a leap year selected at
in a single throw with a die is random will have 53 Sundays.
(A) 1/2 (B) 2/3 (A) 2/5 (B) 1/8
(C) 1/4 (D) 1/3 (C) 2/7 (D) 1/7
8. A card is drawn from an ordinary pack
2. Probability of getting tail in a throw of a
of playing cards and a person bets that
coin is
it is a spade or an ace. Then the odds
(A) 1 (B) 1/3
against his winning this bet is
(C) 1/4 (D) 1/2
(A) 9 to 4 (B) 8 to 5
3. A bag contains 6 white balls, 9 black (C) 7 to 6 (D) none of above

balls. The probability of drawing a black 9. In a horse race the odds in favour of
ball is four horses H1, H2, H3, H4 are 1 : 3,
(A) 2/5 (B) 3/5 1 : 4, 1 : 5; 1 : 6 respectively not more
(C) 1/5 (D) 4/5 than one wins at a time. Then the
chance that one of them wins is
4. Probability of a card drawn at random
(A) 320/419 (B) 319/420
from an ordinary pack of cards to be
(C) 419/520 (D) 520/519
club card is
10. In a garden 40% of the flowers are
(A) 3/4 (B) 2/4
roses and the rest are carnations. If
(C) 1/4 (D) 1/5
25% of the roses and 10% of the
5. Probability of throwing a number carnations are red, the probability that a

greater than 3 with an ordinary die is red flower selected at random is a rose.

(A) 1/2 (B) 1/3 (A) 5/6 (B) 1/4

(C) 1/4 (D) 2/3 (C) 4/5 (D) 5/8


11. Find the chance of throwing more than
6. Probability of getting a total of more
15 in one throw with 3 dice
than 10 in a single throw with 2 dice
(A) 1/54
(A) 1/6 (B) 1/8 (B) 17/216
(C) 1/12 (D) 2/3 (C) 5/108
(D) cannot be determined

193
Vidyalankar : GATE – Engineering Mathematics

12. In a race where 12 horses are running, 15. A man and his wife appear for an
the chance that horse A will win is 1/6, interview for two posts. The probability
that B will win is 1/10 and that C will of the husband's selection is 1/7 and
win is 1/8. Assuming that a dead heat that of the wife’s selection is 1/5. The
is impossible the chance that one of probability that only one of them will be
them will win selected is
(A) 1/390 (B) 47/120 (A) 2/7 (B) 4/5
(C) 3/20 (D) 1/54 (C) 4/35 (D) 6/35

13. There are two bags, one of which 16. There are two bags. One bag contains
contains 5 red and 7 white balls and 4 white and 2 black balls. Second bag
the other 3 red and 12 white balls. A contains 5 white and 4 black balls.
ball is to be drawn from one or other of Two balls are transferred from first bag
the two bags, find the chance of to second bag. Then one ball is taken
drawing a red ball. from the second bag. The probability
(A) 37/120 (B) 1/10 that it is white is
(C) 1/13 (D) 1/96 (A) 42/165 (B) 95/165
(C) 5/165 (D) 48/165
14. What is the probability of a particular
person getting 9 cards of the same suit 17. In a single throw of two dice find the

in one hand at a game of bridge where probability that neither a doublet

13 cards are dealt to a person ? (same number on the both dice) nor a
13 total of 9 will appear.
C9  4
(A) 39 (A) 5/1 (B) 1/9
C4
(C) 13/18 (D) 1/4
13
C9 13 C9 13 C2  4
(B) 52
C9 18. A speaks truth in 75% and B in 80% of
13 the cases. In what percentage of
C9  39 C4  4
(C) 52
C13 cases are they likely to contradict each

13
other narrating the same incident ?
C9  4
(D) 52 (A) 75% (B) 80%
C13
(C) 35% (D) 100%



194
Assignment on Probability and Statistics

Assignment  5
Duration : 45 Min. Max. Marks : 30

Q 1 to Q 6 carry one mark each 5. 4 coins are tossed. Then the probability
1. A die is rolled. The probability of that at least one head turns up is
getting a number 1 or 6 on the upper (A) 15/26 (B) 1/16
face is (C) 14/16 (D) 15/16
(A) 1/3 (B) 1/2
6. In a throw of 3 dice the probability that
(C) 2/3 (D) 1/4
at least one die shows up 1 is
2. The probability of the horse A winning (A) 5/6 (B) 1/6
the race is 1/5 and the probability of (C) 91/216 (D) 90/215
the horse B winning the same race is
Q 7 to Q 18 carry two marks each
1/6, then the probability of one of the
7. Match the following :
horse to win the race is
List  I
(A) 11/29 (B) 11/28
(a) P(),  is the empty set
(C) 11/30 (D) 11/31
(b) P(A/B)P(B)
3. A card is drawn from a pack of 52
cards and then a second is drawn.
(c) P A  
Probability that both the cards drawn 
(d) P A  B 
are queen is
(e) P (A  B)
(A) 1/219 (B) 1/13
List  II
(C) 1/17 (D) 1/221
(i) 1  P(A)
4. A bag contains 5 white and 3 black
(ii) P(A  B)
balls. Two balls are drawn at random
(iii) 1  P(A  B)
one after the other without
(iv) 0
replacement. The probability that both
(v) P(A) + P(B)  P(A  B)
the balls drawn are black is
(A) 3/28 (B) 3/8 (A) a  iv, b  ii, c  i, d  iii, e  v

(C) 2/7 (D) 3/29 (B) a  iii, b  ii, c  i, d  iv, e  v


(C) a  ii, b  iii, c  i, d  iv, e  v
(D) a  i, b  ii, c  iii, d  iv, e  v

195
Vidyalankar : GATE – Engineering Mathematics

8. Three machines A, B and C produce 12. If 10 coins are tossed 100 times, how
respectively 60%, 30% and 10% of the many times would you expect 7 coins
total number of items of a factory. The to fall head upward ?
percentages of defective output of (A) 12 (B) 11
these machines are respectively 2%, (C) 10 (D) 9
3% and 4%. An item is selected at
13. If the probability of a defective bolt is
random and found defective. Find the
0.1, the mean and standard deviation
probability that the item was produced
for the distribution of defective bolts in
by machine C.
a total of 500 are
(A) 2/25 (B) 1/25
(C) 4/25 (D) 3/25 (A) 50, 6.7 (B) 40, 7.3
(C) 20, 3.1 (D) none of these
9. A fair die is tossed 180 times. The
expected number of sixes is 14. Two cards are drawn with replacement
(A) 40 (B) 25 from a well shuffled deck of 52 cards.
(C) 180 (D) 30 The mean and standard deviation for
the number of aces are
10. In a population having 50% rice
consumers, what is the probability that X 0 1 2
three or less out of 10 are rice
144 24 1
consumers? P(x)
169 169 169
(A) 17% (B) 10% Probability distribution table
(C) 40% (D) 50%
2 1
(A) ,0.377 (B) ,0.277
13 13
11. The chances of a person being alive
who is now 35 years old, till he is 75 3
(C) ,0.477 (D) none of these
13
are 8 : 6 and of another person being
alive now 40 years old till he is 80 are 15. In a Binomial distribution, the mean
4 : 5. The probability that at least one and standard deviation are 12 and 2
of these persons would die before respectively. The values of n and p are
completing 40 years hence is respectively.
(A) 8/14 (B) 16/63 1
(A) 12, 2 (B) 9,
(C) 4/9 (D) 47/63 3
2 1
(C) 18, (D) , 18
3 3

196
Assignment on Probability and Statistics

16. For a biased die the probabilities for 18. Aishwarya studies either computer
different faces to turn up are given science or mathematics everyday. If
below. she studies computer science on a

Face Probability day, then the probability that she

1 0.10 studies mathematics the next day is


0.6. If she studies mathematics on a
2 0.32
day, then the probability that she
3 0.21
studies computer science the next day
4 0.15
is 0.4. Given that Aishwarya studies
5 0.05
computer science on Monday, what is
6 0.17
the probability that she studies
The die is tossed and you are told that computer science on Wednesday ?
either face 1 or face 2 has turned up. (A) 0.24 (B) 0.36
The probability that it is face 1 is (C) 0.4 (D) 0.6
(A) 2/11 (B) 3/21
(C) 11/13 (D) 5/21

17. There are three events A, B and C,


one of which must occur and only one
can happen, the odds are 8 to 3
against A, 5 to 2 against B; find the
probability of C.
(A) 41/57 (B) 3/11
(C) 34/77 (D) 2/7



197
Vidyalankar : GATE – Engineering Mathematics

Assignment  6
Duration : 45 Min. Max. Marks : 30

Q1 to Q6 carry one mark each (A) 1 / 16 (B) 7 / 663


1. The probability that the birthday of a (C) 8 / 663 (D) none of these
child is Saturday or Sunday is
Q7 to Q18 carry two marks each
(A) 1 / 2 (B) 2 / 7
7. The probability that a man aged 50
(C) 1 / 3 (D) 1 / 7
years will die within a year is 0.01125.
2. The probability of getting a multiple of The probability that out of 12 such
2 in the throw of a die men at least 11 will reach their fifty first
(A) 1 / 3 (B) 1 / 4 birthday.
(C) 1 / 6 (D) 1 / 2 (A) 1 (B) 0

3. Probability that the sum of the score is (C) 0.9923 (D) 0.8823

odd in a throw of two dice is 8. The probability that a certain beginner


(A) 1 / 4 (B) 1 / 5 at golf gets a good shot if he uses the
(C) 1 / 2 (D) 1 / 3 correct club is 1/3 and the probability

4. A committee of 5 students is to be of a good shot with an incorrect club is

chosen from 6 boys and 4 girls. The 1/4. In his bag are 5 different clubs,

probability that the committee contains only one of which is correct for the

exactly 2 girls is shot in question. If he chooses a club

(A) 9 / 20 (B) 1 / 6 at random and takes a stroke, the

(C) 10 / 21 (D) 1 / 9 probability that he gets a good shot is


(A) 2 / 3 (B) 4 / 15
5. A bag contains 7 red, 5 blue, 4 white
(C) 5 / 9 (D) 7 / 36
and 4 black balls. Then the probability
that a ball drawn at random is red or 9. From a pack of cards two are drawn,
white is the first being replaced before the
(A) 11 / 20 (B) 1 / 7 second is drawn. The probability that
(C) 1 / 11 (D) 3 / 20 the first is a diamond and the second
is a king is
6. Two cards are drawn at random from a (A) 1 / 52 (B) 4 / 13
pack of 52 cards. The probability that (C) 57 / 64 (D) 11 / 52
one may be a jack and other an Ace is

198
Assignment on Probability and Statistics

10. One of the two mutually exclusive 15. Find the probability of drawing one
events must occur, if the chance of rupee coin from a purse with two
one is 2/3 of the other, then odds in compartments one of which contains 3
favour of the other are fifty paisa coins and 2 onerupee
(A) 1 : 1 (B) 1 : 2 coins and the other contains 2 fifty
(C) 2 : 3 (D) 3 : 2 paisa coins and 3 one rupee coins.
1 (A) 1 / 2 (B) 2 / 5
11. Given P(A  B ) = and
3
(C) 3 / 5 (D) none of these
2
P(A  B) = then P(B) is
3 16. If A and B are two events such that
(A) 2 / 3 (B) 1 / 5
P(A  B) = 5/6 . P(A  B) = 1/3 P( B )
(C) 1 / 3 (D) 4 / 5
= 1/ 2 then the events A and B are
12. There are 64 beds in a garden and 3
(A) dependent
seeds of a particular type of flower are
(B) independent
sown in each bed. The probability of a
(C) mutually exclusive
flower being white is 1/4. The number
(D) none of these
of beds with 3, 2, 1, and 0 white
flowers is respectively 17. A determinant is chosen at random from

(A) 1, 9, 27, 27 (B) 27, 9, 1, 27 the set of all determinants of order 2 with

(C) 27, 9, 1, 1 (D) 27, 9, 9, 1 elements 0 or 1 only. The probability that


value of the determinant chosen is
13. A sample space has two events A and
positive is
B such that probabilities
(A) 2 / 17 (B) 1 / 17
P(A  B) = 1/2 , P(A) = 1/3,
(C) 1 / 16 (D) 3 / 16
P(B) = 1/3. What is P(A  B) ?
18. One hundred identical coins each with
(A) 11/12 (B) 10/12
probability p of showing up heads are
(C) 9/12 (D) 8/12
tossed. If 0 < p < 1 and the probability
14. A man takes a step forward with
of heads showing on 50 coins is equal
probability 0.4 and backward with
to that of the heads showing in 51
probability 0.6. The probability that at
coins, then the value of p is
the end of eleven steps he is one step
(A) 1 / 2 (B) 49 / 101
away from the starting point.
(C) 50 / 101 (D) 51 / 101
(A) 0.3678 (B) 0.25
(C) 11c2(0.24)3 (D) none of these



199
Vidyalankar : GATE – Engineering Mathematics

Test Paper  1
Duration : 30 Min. Max. Marks : 25

Q1 to Q5 carry one mark each 5. From a bag containing 4 white and


1. In a shooting competition, the 5 black balls a man draws 3 at
probability of hitting the target by A is random. Then the probability that all 3
2/5, by B is 2/3 and by C is 3/5. If all are black is
of them fire independently at the same (A) 5/42 (B) 1/42
target, calculate the probability that (C) 1/20 (D) 1/15
only one of them will hit the target. Q6 to Q15 carry two marks each

(A) 2/5 (B) 2/3 6. Urn A contains 6 red and 4 black balls

(C) 3/5 (D) 1/3 and urn B contains 4 red and 6 black
balls, one ball is drawn at random from
2. A box contains 6 white balls and
urn A and placed in urn B. Then one
3 black balls and another box contains
ball drawn at random from urn B and
4 white balls and 5 black balls. The
placed in urn A. If one ball is now
probability that a ball selected from
drawn from urn A, the probability that it
one of the box again selected at
is found to be red is
random is a white ball.
(A) 3 / 11 (B) 1 / 17
(A) 6/18 (B) 4/18
(C) 32 / 55 (D) 31 / 56
(C) 1/2 (D) 5/9
7. Two persons each make a single
3. Two sisters A and B appeared for an throw with a dice. The probability they
Audition. The probability of selection get equal value is P1. Four persons
of A is 1/5 and that of B is 2/7. The each make a single throw and
probability that both of them are probability of three being equal is P2.
selected is Then
(A) 2/35 (B) 1/5 (A) P1 = P2 (B) P1 < P2
(C) 2/7 (D) none of these (C) P1 > P2 (D) none of these
4. Two balls are to be drawn from a bag 8. A bag has 13 red, 14 green and 15
containing 5 red and 7 white balls. Find black balls. The probability of getting
the chance that they will both be white exactly 2 blacks on pulling out 4 balls is
(A) 5/108 (B) 5/7 P1. Now the number of each colour ball
(C) 7/22 (D) 2/35 is doubled and 8 balls are pulled out.

200
Test Paper on Probability and Statistics

The probability of getting exactly 4 13. A student appears for tests I, II and III.
blacks is P2. Then The student is successful if he passes
(A) P1 = P2 (B) P1 > P2 either in tests I and II or tests I and III.
(C) P1 < P2 (D) none of these The probabilities of the student
passing in tests I, II and III are p, q and
9. If A and B are arbitrary events, then
1/2 respectively. If the probability that
(A) P ( A  B)  P(A) + P(B) 1
the student is successful is 1/2. Then
(B) P(A B)  P(A) + P(B)  1
(A) p = q = 1 (B) p = q = 1/2
(C) P(A  B) = P(A) + P(B)  1
(C) p = 1, q = 0 (D) p = 1, q = 1/2
(D) none of above

14. The figure below shows the network


10. One mapping is selected at random
connecting cities A, B, C, D, E and F.
from all the mappings from the set
The arrows indicate permissible
S = 1,2,3,...n into itself. The probability
directions of travel. If Deepak from city
that the selected mapping is one to one
A wants to visit city F, what is the
is
probability he will pass through city C
(A) 1/nn (B) 1/n!
B D
(C) n/n! (D) none of these
A F
11. For two events A and B P(A  B) is
(i) not less than P(A) + P(B) 1 (ii) C E
not greater than P(A) + P(B)
(A) 3/5 (B) 4/5
(iii) equal to P(A) + P(B)  P(A  B)
(C) 1/2 (D) 3/4
(iv) equal to P(A) + P(B) + P(A  B)
15. Mean of 200 observations was found
(A) i, ii, iii (B) i, iv
to be 90. However, when the
(C) i, iii (D) i, ii, iii, iv
observations were being made, two
12. Two persons A and B have observations were wrongly taken as
respectively n + 1 and n coins, which 15 and 80 instead of 40 and 87. Then
they toss simultaneously. Then the the correct mean is
probability that A will have more heads (A) 90 (B) 91.16
than B is (C) 90.16 (D) 91.6
(A) 1 / 2 (B) > 1 / 2
(C) < 1 / 2 (D) none of these


201
Vidyalankar : GATE – Engineering Mathematics

Test Paper  2
Duration : 30 Min. Max. Marks : 25

Q1 to Q5 carry one mark each 5. A bag contains 3 black and 5 white


1. What is the chance of throwing a balls. One ball is drawn from the bag.
number greater than 4 with an ordinary What is the probability that the ball is
die whose faces are numbered from not black ?
1 to 6 ? (A) 3/8
(A) 1/6 (B) 1/3 (B) 5/8
(C) 1/4 (D) none of these (C) 2/7
(D) none of the these
2. Find the chance of throwing at least
one ace in a simple throw with two
Q6 to Q15 carry two marks each
dice numbered 1 to 6.
6. A single letter is selected at random
(A) 1/11 (B) 1/36
from the word “PROBABILITY”. The
(C) 11/18 (D) 11/36
probability that it is a vowel is
3. What is the probability that a digit (A) 1/3 (B) 2 / 21
selected at random from the logarithmic (C) 0 (D) 2 / 9
table is 1 ?
7. A and B throw a coin alternately till
(A) 1/2 (B) 1/5
one of them gets a ‘head’ and wins the
(C) 1/3 (D) 1/10
game. Their respective probabilities of

4. A problem in mathematics is given to winning are

three students Dayanand, Ramesh (A) 1/3, 2/3 (B) 1/2, 1/2

and Naresh whose chances of solving (C) 1/4, 3/4 (D) 2/3, 1/3

1 1 
it are , , respectively. The 8. Two dice are thrown the probability of
2 3 4
getting an odd number on the one and
probability that the problem is solved is
a multiple of 3 on the other is
(A) 1/4 (B) 1/2
(A) 11/36 (B) 2/17
(C) 3/4 (D) 1/3
(C) 10/36 (D) 12/36

202
Test Paper on Probability and Statistics

9. In a certain college, 4% of the men 13. If A and B are two events such that
and 1% of the women are taller than P(A) = 0 and P(B)  1, then P( A / B ) is
1.8m. Further 60% of the students are equal to
woman. If a student is selected at
(A) 1  P(A/ B ) (B) 1  P( A /B)
random and is taller than 1.8m, the
(C) 1  P(A / B) (D) P(A)
probability that the student is women is P(B) P(B)
(A) 3/11 (B) 2/11
14. Ten points are marked on a straight
(C) 4/11 (D) 1/11
line and 11 points are marked on
10. A and B throws two dice; if A throws 9 another straight line. If any three
then B’s chance of throwing a higher points are chosen, what is the
1 probability that it forms a triangle?
number is , state whether the above
6 11 21
(A) (B) C3
statement is true or false. 14  10
C3  11C3 
(A) True
6 10
C3  11C3
(B) False (C) (D)
7 1045
(C) Can not say
(D) Data insufficient 15. Three machines A, B and C produce

11. A pack of cards contains 4 aces, identical items of their respective

4 queens and 4 jacks. Two cards are output 5%, 4% and 3% items are

drawn at random. The probability faulty. If A produces 25% of the total

that at least one of them is an ace is output, B produces 30% and C the

19 3 remainder and an item selected at


(A) (B)
33 16 random is found to be faulty, what is

1 1 the probability that it was produced by


(C) (D)
6 9 machine C?

12. x is a random variable with mean  (A) 0.155 (B) 0.255


(C) 0.355 (D) 0.455
and S. D.  > 0
x
x = ; then var(x) = _______

(A) 0 (B) 2
(C) can’t say (D) 1



203
Vidyalankar : GATE – Engineering Mathematics

Test Paper  3
Duration : 30 Min. Max. Marks : 25

Q1 to Q5 carry one mark each 5. A bag contains 8 green and 10 white


1. An urn contains 25 balls numbered 1 balls. Two balls are drawn. What is the
through 25. Two balls are drawn from probability that one is green and the
the urn with replacement. The other is white ?
probability of getting at least one odd 80 10
(A) (B)
is 153 17
(A) 144/625 (B) 312/625 8 81
(C) (D)
(C) 12/25 (D) 481/625 17 157

2. Four persons are chosen at random Q6 to Q15 carry two marks each
from a group containing 3 men, 6. Fifteen coupons are numbered
2 women and 4 children. The chance 1, 2, 3, …….., 15. Seven coupons are
that exactly 2 of them will be children selected at random one at a time with
is replacement. The probability that the
(A) 10/21 (B) 5/8 largest number appearing on selected
(C) 13/32 (D) 3/32 coupon is 9, is
6 8
 9   7 
1 (A)   (B)  
3. Let A and B be events with P(A) = ,  16   15 
2
7
1 1 2
P(B) = and P(A  B) = . Then (C)   (D) none of these
3 4 5

P(A/B) is
(A) 1/4 (B) 1/3 7. A bag contains 2 white and 3 black

(C) 1/2 (D) 3/4 balls. A ball is drawn 5 times, each


being replaced before another is
4. How many different words can be drawn. Find the probability that exactly
formed from the letters of the word 4 of the balls drawn are white.
Ganeshpuri when all the letters taken 2 48
(A) (B)
(A) 2! (B) 10! 5 625

(C) 9! (D) 5! 31 3
(C) (D)
725 5

204
Test Paper on Probability and Statistics

8. From a pack of 52 cards two are 12. For a Binomial distribution, the mean
drawn at random. The probability that is 6 and the standard deviation is 2 .
one is a king and the other a queen is The Binomial distribution is
(A) 3/ 8 (B) 8 / 663 9 9
 1 2  1 1
(C) 7/ 663 (D) 9 / 663 (A)    (B)   
3 3 2 2
9 9
9. In three throws of two dice, the  6 1  4 1
(C)    (D)   
7 7 5 5
probability of throwing doublets not
more than two times is
13. If 3 squares are chosen at random on
(A) 1/216 (B) 1/6
a chess board the probability that they
(C) 4/216 (D) 215 / 216
should be in a diagonal line is

10. If the sum of the mean and the (A) 5/744 (B) 3/744

variance of a Binomial distribution for (C) 7/744 (D) 9/744

18 trials is 10, find the distribution.


18 14. A man has 9 friends (4 men and
n  1 2
(A) (p + q) (B)    5 women). In how many ways can he
3 3
invite them, if there have to be exactly
18 11
 1 1  1 2
(C)    (D)    3 women in the invitees ?
2 2 3 3
(A) 320 (B) 160
(C) 80 (D) 200
11. If the sum of the mean and the
variance of a Binomial distribution for
15. Given two events A and B and
5 trials is 1.8 find the distribution.
P(A) = 1/4, P(B/A) = 1/2, P(A/B) = 1/4.
5 5
 1 2  1 1
(A)    (B)    Which of the following is true?
3 3 2 2
(A) P(A B) = 1/2
9 5
 6 1  4 1
(C)    (D)    (B) A is subevent of B
7 7 5 5
(C) P(A/B) + P(A B) = 1

(D) None of these



205
Solutions  Linear Algebra

Answer Key on Assignment  1


1. (A) 2. (D) 3. 0.99 to 1.01 4. (A)

5. (A) 6. (B) 7. (A) 8. 2

9. 6 10. (C) 11. (B) 12. 2.9 to 3.1

13. (A) 14. (B) 15. (D) 16. (A)

17. 6 to 6 18. (C) 19. (B) 20. (D)

Answer Key on Assignment  2


1. (A) 2. (D) 3. 199 to 201 4. 23 to 23

5. 88 to 88 6. 2 7. 4.49 to 4.51 8. 16.5 to 17.5

9. (D) 10. (C) 11. 48.9 to 49.1 12. 2.0 to 2.0

13. (D) 14. (D) 15. 2 to 2 16. (B)

17. (B) 18. (D) 19. (A) 20. 0 to 0

Answer Key on Assignment  3


1. (D) 2. (D) 3. (B) 4. (A)

5. (A) 6. (C) 7. 2.0 to 2.0 8. (C)

9. (A) 10. 0.95 to 1.05 11. (B) 12. (A)

13. 5.0 to 5.0 14. 0 15. 2.9 to 3.1 16. (B)

17. (A) 18. (B) 19. (A) 20. (A)

206
Solutions  Linear Algebra

Answer Key on Assignment  4


1. (C) 2. (B) 3. (A) 4. (B)
5. (D) 6. (A) 7. (A) 8. (C)
9. (A) 10. (A) 11. (B) 12. (D)
13. (B) 14. (C) 15. (A) 16. (D)
17. (B) 18. (B)

Answer Key on Assignment  5

1. (B) 2. (C) 3. (A) 4. (B)


5. (B) 6. (A) 7. (B) 8. (A)
9. (B) 10. (A) 11. (C) 12. (B)
13. (C) 14. (A) 15. (B) 16. (C)
17. (D) 18 (D)

Answer Key on Assignment  6


1. (B) 2. (C) 3. (C) 4. (C)
5. (B) 6. (C) 7. (B) 8. (B)
9. (B) 10. (D) 11. (C) 12. (C)
13. (C) 14. (A) 15. (C) 16. (A)
17. (C) 18. (B)

Answer Key on Assignment  7

1. (C) 2. (D) 3. (B) 4. (B)


5. (D) 6. (A) 7. (A) 8. (C)
9. (A) 10. (A) 11. (B) 12. (C)
13. (B) 14. (A) 15. (B) 16. (A)
17. (A) 18. (B)

207
Vidyalankar : GATE – Engineering Mathematics

Answer Key on Assignment  8


1. (C) 2. (C) 3. (A) 4. (C)
5. (C) 6. (C) 7. (A) 8. (A)
9. (B) 10. (C) 11. (C) 12. (C)
13. (C) 14. (C) 15. (C) 16. (B)
17. (B) 18. (A)

Answer Key on Assignment  9


1. (B) 2. (B) 3. (C) 4. (A)
5. (C) 6. (B) 7. (C) 8. (A)
9. (C) 10. (B) 11. (C) 12. (C)
13. (C) 14. (C) 15. (B) 16. (C)
17. (D) 18. (D)

Answer Key on Assignment  10


1. (D) 2. (B) 3. (B) 4. (C)
5. (B) 6. (C) 7. (A) 8. (A)
9. (A) 10. (B) 11. (C) 12. (B)
13. (C) 14. (C) 15. (D) 16. (C)
17. (D) 18. (C)

Answer Key on Assignment  11


1. (C) 2. (C) 3. (C) 4. (D)
5. (C) 6. (C) 7. (C) 8. (C)
9. (A) 10. (B) 11. (D) 12. (B)
13. (C) 14. (D) 15. (B) 16. (C)
17. (D) 18. (D)



208
Solutions  Linear Algebra

Model Solution on Assignment  1


1. (A) 1 2 4
1
2 6 0 1 3 0   0 2 3
2
 4 12 8 = (2)3   2 6 4 1 1 1
2 0 4 1 0 2 1
= 1[2
 3]2[03]4[02]
= 8  (12) = 96 2
1 3
2. (D) = 1  6  8
2 2
We know that the Eigen vectors
corresponding to distinct Eigen values 6. (B)
of real symmetric matrix are
7. (A)
orthogonal.
 x1   y1 
 x  y  = x y + x y + x y = 0 8. 2
 2  2 1 1 2 2 3 3
 x 3   y 3  x  2y + 3z = 1
x  3y + 4z = 1
3. 0.99 to 1.01 2x + 4y  6z = k
Given, A2 = I Arugmented matrix (A/B) is given by
We know that above equation is
 1 2 3 1
satisfied by identity matrix. (A/B) =  1 3 4 1 
1 0 0 0  1 0 0 0  2 4 6 k 
0 1 0 0  0 1 0 0 
i.e.,   =I Now applying row reduction technique 
0 0 1 0  0 0 1 0
    R2  R2  R1
0 0 0 1 0 0 0 1
R3  R3 + 2R1
Eigen value of identity matrix is 1.
We get
4. (A)  1 2 3 1 
 0 1 1 2 

5. (A)  0 0 0 k  2 
Area of triangle For infinite solution
x1 x 2 x3 (A/B) = (A) = r
1
=  y1 y 2 y3 
2  k2=0
1 1 1
 k=2

209
Vidyalankar : GATE – Engineering Mathematics

9. 6  max = 1 and min = 3


The given matrix is max 1 1
   =   =
4 5  min 3 3
A=  
2 1
13. (A)
Inorder to find eigen values of above
 1 5 3 7 
matrix. A=   B=  
6 2  8 4 
|A  I| = 0
 1 5  3 8  38 28 
4 5 ABT =    =  
   =0 6 2  7 4  32 56 
2 1 

 (4  ) (1  )  10 = 0 14. (B)

 4  5 + 2  10 = 0 10 5  J 4 
Let A =  x 20 2 
 2  5  6 = 0
 4 2 10 
 2  6 + 1  6 = 0
Given that all eigen values of A are real.
 (  6) + 1(  6) = 0
 A is Hermitian
  + 1 = 0 or 
 
T
  = 1 or  = 6  A = A i.e. A =A

 Larger eigen value of matrix is 6.


 10 x 4  10 5  J 4 
 5  j 20 2  =  x 20 2 
10. (C)   
 4 2 10   4 2 10 
11. (B)
 x=5j
12. 2.9 to 3.1
Characteristic equation is given by 15. (D)
|A  I| = 0 The characteristic equation is 2 
0 1 1 (sum of diagonal elements)  + |A| = 0
 6 11   6  =0
 2  (2 + p)  + (2p  1) = 0
6 11 5
(2  p) (44pp2 )4(2p  1)
0 1 1  =
2
  6 11   6  =0
6 11 5 (2  p) (84pp2 )
=
2
 3 + 62 + 11 + 6 = 0
(2  p)  (84pp2 ) 3
  = 1, 2, 3 are the eigen values  
(2  p) (84pp ) 2 1
of A

210
Solutions  Linear Algebra

2 (84pp2 ) 2 Multiply by ‘A’, We get,


 
2(2  p) 4 A3 = 5A2  6A
= +5 [5A + 6I]  6A
 2 84pp2 = (2 + p)
3
A = 19A + 30I
 4(8  4p + p2) = (4 + 4p + p2)
20. (D)
 3p2  20p + 28 = 0
 1 1 
 (3p  14) (p  2) = 0  0 
 2 2
 p = 2, 14/3 P=  0 1 0 
 
 1 0
1 
16. (A)  2 2 

17. 6 to 6 1  1  1  1 
|P| =  0 0  
2 2  2 2
18. (C)
1 1
2 1 1  =  1
0 1 1 2 2
 
 1 1 0   1 1   1 1 
 0   0 
r < n. Hence, infinite number of  2 2  2 2
P.PT =  0 1 0  0 1 0 
solutions.    
 1 0
1   1
0
1 
19. (B)  2 2   2 2 

|A  I| = 0 …characteristic equation  1 0 0


 5 3   0  = 0 1 0 
     0 0 0 1
 2 0  0  
 P is an orthogonal matrix
5   3
   0 (A) Is correct
2 
 Inverse of P is its transpose only
(5 + ) + 6 = 0
 (B) and (C) both are correct
2 + 5 + 6 = 0
 (D) is incorrect
 By Cayley Hamilton’s Theorem,
A2 + 5A + 6I = 0
 A2 = 5A  6I


211
Vidyalankar : GATE – Engineering Mathematics

Model Solution on Assignment  2


1. (A) 5. 88 to 88
dx 0 1 2 3
Given that = 3x  5y 1
dt 0 3 0 
A= 
dy 2 3 0 1
= 4x + 8y  
dt 3 0 1 2

d x  3 5   x  1 3 0 1 0 0
Matrix form   =   
dt y  4 8   y   = 1 2 0 1 2 2 3 1
3 1 2 3 0 2
2. (D)
1 0 3
3. 199 to 201 3 2 3 0
3 0 1
4. 23 to 23 = 1[1(1)  3(4  3)] + 2[1(6)]
Given Matrices,  3[1(3) + 3(9)]
3 2 1  1 = 1[ 1  3] + 12  3[3  27]
J = 2 4 2  and K = 2
  = 4 + 12  3 [24]
 1 2 6   1
= 4 + 12 + 72 = 88.
KT = 1 2 1

 3 2 1  1  6. 2
 K JK = 1 2 12 4 2  2 
T
4 2 
 1 2 6   1 Let A=  
 1 3
 1 Characteristic equation of A is
=  3  4  1 2  8  2 1  4  6   2 
A  I  0
 1
4 2
 1    =0
1 3
=  6 8 1  2  = 6 + 16 + 1
 1 2  7 + 10 = 0   = 2, 5

KTJK = 23

212
Solutions  Linear Algebra

7. 4.49 to 4.51  2x = 12
2 3 : 5   x=6
[A : B] =  
3 p : 10   (6, 2)
2C2  3C1
2 3 : 5 10. (C)
0 2p9 : 5 
  Given : M4 = I
There is no solution if  M4k = (M4)k = Ik = I
rank (A : B)  rank(A). (A) M4k + 1 = M
Now, rank (A : B) = 2. So rank(A) = 1 (B) M4k + 2 = M2

only if 2p  9 = 0 i.e. p = 9/2 = 4.5 (C) M4k + 3 = M4k M4 M1 = M1

8. 16.5 to 17.5 11. 48.9 to 49.1

AX = X a b 
Let A =  
4 1 2  1  1  c d
 P 2 1  2  =  2 
      a + d = 14 and bc ≥ 0
14 4 10  3  3  Now |A| is maximum when ad is

 12   maximum and bc = 0 i.e.


 P  7  = 2  when a = d = 7
   
 36  3   Maximum value of |A| = 7 × 7 = 49
  = 12 ….(1)
12. 2.0 to 2.0
2 = P + 7 ….(2)
The given matrix is
and 3 = 36
6 0 4 4 
 = 12
A =  2 14 8 18 

i.e.,
 Equation (2) gives P + 7 = 24 14 14 0 10 
 P = 17 The matrix is of order 3  4
Hence (A)  3
9. (D)
2x + 5y = 2 ….(1) 6 0
4
2 8 = 6(0 + 112)
14
4x + 3y = 30 ….(2)
14 14 0
2 × (1) + (2) gives 13y = 26
+ 4(28  196)
 y = 2
= 672  672 = 0
 2x  10 = 2

213
Vidyalankar : GATE – Engineering Mathematics

0 4 4  eigen values are 1 and 2


and  14 8 18   smallest is 1 and largest is 2.
14 0 10
14. (D)
= 4(140 + 252) + 4(112)
Sum of Eigen values = 1 + a
= 4(112) + 4 (112) = 0
 6=1+a
Hence (A)  2.
 a=5
6 0
Now   = 84  0 and product of Eigen values = det(A)
2 14
7 = a  4b
Hence (A) = 2
 7 = 5  4b
 Rank of given matrix is 2
 7  5 = 4b

13. (D)  b=3

The characteristic equation is


15. 2 to 2
(3  ) 2 2
It is an upper triangular matrix. Hence
 4 (4  ) 6   0 ;
eigen values are the diagonal
2 3 (5   )
elements i.e. 2, 2, 3. Hence there are
C2 = C2 + C3 gives
2 linearly independent eigen vectors.
(3  ) 0 2
 4 (2  ) 6  =0 16. (B)
2 (2  ) (5   )
17. (B)
(3   ) 0 2
 (2  )  4 1 6  = 0; 1 1 1 : 6 
 146:20 
2 1 (5   )
1 4  :  
R3 = R3  R2 gives
R3  R3  R2
(3  ) 0 2
(2  ) 4 1 6  =0 1 1 1 : 6 
2 0 (1  )  14 6 : 20 

0 0   6 :   20 
 (2  ) {(3  )[(1  )  0]  0
+ 2[0  (2)]} = 0 For  = 6,   20 System is

 (2  ) {3  2 + 2 + 4} = 0 inconsistent

 (2  ) (2  2 + 1) = 0  It has no solution.

 (2  ) (  1)2 = 0

214
Solutions  Linear Algebra

18. (D) 19. (A)


1 1  If matrix B is obtained from matrix A by
A 
1 1 replacing the lth row by itself plus k

32 0  times the mth row, for /  m then


A 5 .A 5   
 0 32  det(B) = det(A). With this property
given matrix is equal to the matrices
4 0 
A4   
0 4  given in options (B), (C) and (D).

128 128 
A10 .A 5    20. 0 to 0
128 128 
 50 70 
A=  
4 4   70 80 
A5   
 4 4 
Eigenvectors are
512 512 
A15 .A 4     70    2  80 
512 512  X1 =  ;X2  
 1  50   70 
512 512 
 A19    where 1, 2 = Eigenvalues of A
512 512 
   80 
512   512 X1T X2  70 1  50  2 
  0  70 
512 (512   )
= 70(2  80) + (1  50) 70

   512   
2 2
   512  2
0 = 702  5600 + 701  3500

 2 = 2  (512)2 = 70(1 + 2)  9100


= 70(130)  9100
 = 2 (512)
= 9100  9100 = 0
If eigen values of A are 1, 2,…, then
eigen values of Ak are  sum of eigenvalues  1   2 
 
 Trace5080130 
k1 , k2 ,…(k > 0)



215
Vidyalankar : GATE – Engineering Mathematics

Model Solution on Assignment  3


1. (D) 5. (A)
 5 2 
Eigen values of the matrix  
 9 6  6. (C)
Characteristic equation is
are 4, 3
1 1 5
 The eigen vector corresponding to
 0 5 6  0
eigen vector  is Ax = x (verify the
0 6 5
options)
 (1  ) [(5  )2 + 36] = 0
1
  is eigen vector corresponding to  = 1; 2  10 + 61 = 0
1

eigen value  = 3 10  100  224


=
2
2. (D) 1012j
= 5j6
MN is not always equal to NM. 2

3. (B)
7. 2.0 to 2.0
 1 2 2 : b1 
[A:B] =    0 1 2 
 5 1 3 : b2  P + Q =  8 9 10 
R2  R2  5R1  8 8 8 

1 2 2 : b1   8 9 10 
 0 9 7 : b  5b 
 2 1 R1R2  0 1 2 
 1 1 1 
 (A) = (A/B) < number of unknowns,
for all values of b1 and b2. 8 9 10 
 The equations have infinitely many 8R3 R10 1 2 
0 1 2 
solutions, for any given b1 and b2.
8 9 10 
R3  R2 0 1 2 
4. (A)
 215 650 795  0 0 0 
Given, Matrix [M] =  655 150 835 
 Rank is 2
 485 355 550 

Sum of the eigenvalues


= Trace of matrix
= 215 + 150 + 550 = 915

216
Solutions  Linear Algebra

8. (C) 11. (B)


 1 tanx 
A=  12. (A)
  tanx 1 
|P| = (16 + 9)  (1) = 24
 1
T  tanx 
A =  1  4  3i
1  1 i 
 tan x P1 = adj(P) 
|P | 24  i 4  3i
1
Now A1 = (Adj(A))
|A| 13. 5.0 to 5.0
1  1  tanx  Since one eigenvalue of M is 2
A= 
2
sec x  tan x 1   23  4(2)2 + a(2) + 30 = 0
 |AT.A1| = 1  a = 11
 Characteristic polynomial is
9. (A)
3  42  11 + 30 = 0
Given systems
(  2) (  5) ( + 3) = 0
3x1 + 2x2 = c1
  = 2, 5, 3
4x1 + x2 = c2
Largest absolute value of '' is 5
 3 2   x1   c1 
Matrix Form is    
 4 1   x 2  c 2  14. 0
AX = B The given matrix is
Characteristic equations of above  3 4 45 
system is A =  7 9 105 
|A  I| = 0 13 2 195 

3 2 Now according to question


  0
4 1  (i) R2  R2 + R3 (Adding third row to
2
By expanding   4  5 = 0 the second row)
 3 4 45 
10. 0.95 to 1.05
 B =  20 11 300 
Matrix A has zero as an eigen value if 13 2 195 
|A| = 0
(ii) C1  C1  C3 (Subtracting third
 3[(63 + 7x) + 52]  2[(81 + 9x)
column from first column)
+ 78] + 4[36 + 42] = 0
 42 4 45 
 3(7x  11)  2(9x  3) + (4 × 6) = 0
 C =  280 11 300 
 21x  33  18x + 6 + 24 = 0  182 2 195 
 3x = 3  x = 1

217
Vidyalankar : GATE – Engineering Mathematics

Now |C| = 42(2145  600)  17. (A)


4(54600 + 54600) + 45(560 + 2002) 5 1
Let A =  
|C| = 42(1545)  4(0) 4 1 
+ 45(1442) Characteristic equations is 2  6 + 9
= 64890 + 64890 = 0 = 0   = 3, 3
Eigen value 3 has multiplicity 2.
15. 2.9 to 3.1 Eigen vectors corresponding to  = 3
sum of diagonal elements is (A  3I) X = 0
= sum of eigen values  5  3 1   x   0 
   
 a + b + 7 = 14  4 1 3   y   0 
a+b=7  2 1  x   0 
   
(a  b)2 = (a + b)2  4ab = 49  40 = 9  4 2   4   0 
 |a  b| = 3  2 1  x   0 
R2  R2  2R1     
0 0  4 0
16. (B) e(A) = 1
 1 2 3 : a  Number of linearly independent
[A : B] =  2 3 3 : b  eigenvectors corresponding to
 5 9 6 : c 
eigenvalue  = 3 is n  r = 2  1 = 1
R2  R2  2R1
where n = no. of unknowns,
 1 2 3 : a 
 [A : B] =  0 1 9 : b  2a 
 r = rank of (A  I)
 5 9 6 : c   One linearly independent
R3  R3  5R1 eigenvector exists corresponding
 1 2 3 : a 
to  = 3
 [A : B] =  0 1 9 : b  2a 

 0 1 9 : c  5a  18. (B)
R3  R3  R2 5 3
 1 2 3 : a  |A  I| = 0  =0
 1 3
 [A : B] =  0 1 9 : b  2a 
 0 0 0 : c  b  3a   (5  ) (3  ) = 3 = 0
 Equation are consistent if  2  8 + 15  3 = 0
c  b  3a = 0 i.e. 3a + b  c = 0  2  8 + 12 = 0   = 2,  = 6
(A  2I)  X = 0
At,  = 2

218
Solutions  Linear Algebra

 3 3   x1   0  Therefore atleast one of the Eigen


 1 1 x   0  value is ‘0’
   2  
 x1 + x 2 = 0 As the choices are non negative, the
minimum Eigen value is ‘0’
 x1 = x2
 1 
  20. (A)
2
Hence the required vector is 
 1  Characteristic equation is
 
 2  1 0
19. (A) 0  1 
0 3 4  
3 5 2   5 5 2
 5 12 7  
C1  C3 12 12 7 
      (4 + 2 + 3) = 0
 2 7 5   7 7 5 
 ( + 1) ( + 3) = 0
 determinant = 0,   = 0, 1, 3 are the eigenvalues.
So the matrix is singular



219
Vidyalankar : GATE – Engineering Mathematics

Model Solution on Assignment  4


1. (C) c1  c1 + c2 + c3, we get

2. (B) 1    2  2
If [A] 3  4 & [B] 4  5  =  1    2 2 1
2
AB will exist, BA does not exist 1    1 

3. (A) 0  2
Every square, real symmetric matrix is =  0 2 1 =0
Hermitian and therefore all its 0 1 
eigenvalues are real.
as 1 +  + 2 = 0
4. (B)
8. (C)
5. (D)
For a matrix rank is equal to no. of
[A] x ( x + 5) [B] y  (11 y)
independent vectors.
AB exists  x+5=y …..(1)
9. (A)
BA exists  x = 11  y …..(2)
We find relation between than as
Solving (1) and (2) gives x = 3, y = 8
1x1 + 2x2 + 3x3 = 0
6. (A)
1 (3, 2, 7) + 2 (2, 4, 1)
A is singular matrix means | A | = 0
+ 3 (1, 2, 6) = 0
3x 2 2
which gives 31 + 22 + 3 = 0
  2 4x 1  =0
21 + 42  23 = 0
2 4 1  x
71 + 2 + 63 = 0
R2  R2 + R 3
which gives 1 = 1, 2 = 1,3 = 1
3x 2 2
 x1  x2  x3 = 0 is linear equation
 0 x x  = 0
2 4 1  x  They are dependent

 x(3  x) (x  3) = 0 10. (A)

 x = 0, 3 11. (B)

7. (A) By performing

1  2 a h g x 
ABC = [x y z]  h b f  y 
 
 2 1
2  g f c  z 
 1 

220
Solutions  Linear Algebra

ax  hy  gz  We have
= [x y z] hx  by  fz   1 1 1   x   3 
gx  fy  cz  AX = 3 1 2   y    2  = B
= [ax2 + by2 + cz2 + 2hxy 2 4 7  z  7 

+ 2gzx + 2fyz] Augmented matrix


12. (D)  1 1 1 : 3 
As | A | = 0 [ A B ] ~ 3 1 2 : 2 
So inverse does not exist. 2 4 7 : 7 

13. (B) We reduce [A B] to Echelon form by


0 2 2 applying successively.
Let A =  7 4 8 
R2  R2  3R1, R3  R3  2R1
 7 0 4 
transformations
R2  R2 + R 3
 1 1 1 : 3 
0 2 2
 [A : B] ~ 0 2 5 : 7 
A =  0 4 4 
0 2 5 : 13 
 7 0 4 
Apply R3  R3 + R2
 | A | = 0 ….. rank < 3
 1 1 1 : 3 
0 2 
Now minor   [A : B] ~ 0 2 5 : 7 
7 4  0 0 0 : 20 
0 2
   = 14  0  Rank [A : B] = 3
7 4
(No. of non-zero rows in Echelon form.)
 rank = 2
1 1 1 
14. (C)
 A ~ 0 2 5 
Eigen values are obtained by
0 0 0 
|AI| =0
Rank A = 2
1  0 0
 2 3 1  =0 Since Rank A  Rank [A : B]
0 2 4  Inconsistent equations.
which gives 1 = 1, 2 = 2, 3 = 5 16. (D)
15. (A)
17. (B)
Given equations are AX = B
18. (B)
It will be consistent if
Rank A = Rank [A : B]


221
Vidyalankar : GATE – Engineering Mathematics

Model Solution on Assignment  5


1. (B) R3  R3  2R1, R4 R4  R1
1 1
As we know A A = I = A A and subsequently.
 B must be inverse of A R2  R2  R3, R3  R3  6R2 ,
R4  R4 3R2 ,
2. (C)
1 1
3. (A) R3  R3, R4  R4 &
5 2
4. (B) R4  R4  R3.
Consequently we get
5. (B)
 1 2 1 : 3
6. (A) 0 1 0 : 4 
[A : B] = 
 p q  r s  0 0 1 : 4
PQ =     
 q p   s r  0 0 0 : 0

 pr  qs ps  qr  which is Echelon form with rank


 
 qr  ps qs  pr  Number of nonzero rows = 3
1 2 1
7. (B) 0 1 0 
Matrix is involutory if it satisfies condition Also AI = 
0 0 1
2
that A = I and given matrix satisfies this  
0 0 0
condition.
Here rank = 3
8. (A)
As rank [A : B] = rank A
adj A
A1 = we calculate inverse using  It has solution, which is unique
|A|
These equations are equivalent to
this formula.
x + 2y  z = 3

9. (B)  y = 4
z = 4
10. (A)
which gives x = 1, y = 4, z = 4
By writing augmented matrix
11. (C)
 1 2 1 :3 
3 1 2 :1 Eigen values of given matrix = 2, 6

2 2 3 :2  For  = 6,
 
 1 1 1 : 1 1 101
eigen vector =  or 
performing R2  R2  3R1 , 1 101

222
Solutions  Linear Algebra

12. (B) 15. (B)


For triangular matrix eigen values are Augmented matrix
same as diagonal elements  1 2 3 : 6
1 = 1, 2 = 3, 3 = 2. [A : B] = 3 2 1 : 2 
4 2 1 : 7 
13. (C) By performing R2  R2  3R1 ,
1 1 1 1
Explanation: Given matrix is, 1 1 1 R3  R3  4R1 and  R2
8
1 1 1
1 2 3 : 6 
0 1 1 : 2 
1    1 1

Now  1 1    1  0 0 6 11 : 17 
1 1  
1 
Performing R3  R3 + 6R2
3   3   3   1 2 3 : 6 
or  1 1    1  =0 0 1 1 : 2 
 
1 1 1    0 0 5 : 5 

1 1 1 we get x = y = z = 1
or  3    1 1    1  0
1 1 1    16. (C)

1 0 0
or  3    1  0 0 17. (D)
1 0 

 2 (3  ) = 0 18. (D)
Hence eigenvalues are 0, 0, 3.

14. (A)
Reducing to Echelon form,
R4  R4  R1
We have number of nonzero rows = 3
 rank = 3



223
Vidyalankar : GATE – Engineering Mathematics

Model Solution on Assignment  6


1. (B) 7. (B)
Characteristic equation is
1   2 8. (B)
  =0
5 4   Characteristic Equation:
(1  ) (4  )  10 = 0 |A  I| = 0
2
4  5 +   10 = 0 (1   ) 1 0
2
  5  6 = 0   0 (1   ) 1  =0
0 0 1 
(  6) (  + 1) = 0
 1 = 6, 2 = 1  (1  )3 = 0
1  3 + 32  3 = 0

2. (C)  I  3A + 3A2  A3 = 0
…. Cayley-Hamilton Theorem
3. (C)  I  3A + 3A.A  A2A = 0
Post multiplying by A1,
4. (C) I.A1  3AA1 + 3AAA1  A2AA1 = 0A1
 A1  3I + 3AI  A2I = 0
5. (B)  A1  3I + 3A  A2 = 0
5 2   A1 = A2  3A + 3I
A=   ,| A | = 1
3 1
adjA 9. (B)
 A1 =
|A| For given matrix of rank = 1
 1 2   = 1  rank = 1
 3 5 
  =  1 2 
1  3 5 
  10. (D)
As | A |  0 for unit matrix
6. (C)  rank equal to number and order
As A A1 = I
2x 0   1 0
  0 2x   0 1
   
 2x = 1
x = 1/2

224
Solutions  Linear Algebra

11. (C) 15. (C)


 3 1 1 | AT A | = | I | = | AT | . | A | = 1
A = 15 6 5   |A|.|A| =1
  2 2 
 (|A|)2 =1
For nontrivial solution P(A) < 3
 |A| =1
 | A | = 0  6 +  (0 + 1) = 0
 =6 16. (A)

A.A 1  I
12. (C)
 1
Solving characteristic equation, 2 10  . 1/ 2 a    1 0 
| A  I | = 0    0 b  0 1
0 3 
3 2
  =0  b
6 5    1 2a  10    1 0 
  0 1
Gives  = 1  2 7 0 3b  

comparing the corresponding terms,


13. (C) we get
As determinant of A i.e. |A|  0 1 b b
b , 2a  0  2a 
3 10 10
14. (A)
b 1
The sum of eigen values of a matrix is  a  a 
20 60
the sum of diagonal elements
1 1 1  20 21 7
1 + 2 = 12.  a  b    
60 3 60 60 20
Only option (A) has sum of diagonal
elements of 12. 17. (C)

18. (B)



225
Vidyalankar : GATE – Engineering Mathematics

Model Solution on Assignment  7


1. (C) 6. (A)
As per theorem for inverse matrix, that 8 8 2
eigen values of inverse matrix are A =  4 3   2  = 0
inverse of eigen values of the matrix 3 4 1 

1 R1  R1  (R2 + R3)


i.e. if  is eigen value of A, then is

1   1   1  
eigen value of A1
 4 3   2  =0
2. (D) 3 4 1 
For non zero matrix rank must be 1 1 1
greater than or equal to 1, but not (1  ) 4 1   2  = 0
equal to zero. 3 4 1  

3. (B) (1  ) [(1  ) (4  ) + 2] = 0

1 0 (1  ) (2  5 + 6) = 0
As determinant   =1
0 1  (1  ) (  2) (  3) = 0

 rank equal to no. of rows = 2 Roots of equation are 1, 2 and 3.


As the eigen values of the matrix A are
4. (B)
all distinct hence A is similar to diagonal
For given equations
matrix.
 1 2 x  1 
3    y   3  7. (A)
     
Adj A
By using formula A1 =
R2  R2  3R1 |A|

1 2  x  1   1 1 2 
0   6   y   0 
      We find A =  0 1/ 2 1/ 2 
1
 1 1/ 2 3 / 2 
i.e. (  6) y =0
i.e. 6 =0 8. (C)
|AI| =0
 = 6 will give no solution to have
 0 
unique solution   6
 0  0  = 0  3 = 0
5. (D) 0 0 

Adj A   =0
As A1 =
|A|

226
Solutions  Linear Algebra

x  13. (B)
Hence if x =  y  is eigen vector then, By solving,
z 
 3 2 1  x  4 
AX =  X  1 1 1  y   2  ,
     
0 0    x  0   2 0 2  z  5 
  0 0 0   y   0 
      We get the values only one for each
0 0 0  z  0 
parameter, so it has unique solution.
 z=0
14. (A)
  0 
 The vectors must be 0  or   a h g  x 
0  0   x y z  h b f y 

 g f c   z 
x 
As the vectors of the form  y  = ax2 + by2 + cz2 + 2hxy + 2fyz + 2gzx
0  i.e. XT AX gives,
9. (A) ax2 + by2 + cz2 + 2hxy + 2fyz + 2gzx
Hint : After taking the common from  a11 a12 a13 
each column, we get matrix having all  If A = a 21 a 22 a 23 
values = 1 and we get after reducing a 31 a 32 a 33 

all rows and columns to zero the Then for


matrix of rank 1 only ax2 + by2 + cz2 + 2hxy + 2fyz + 2gzx
10. (A) a11 = a a22 = b a33 = c
For characteristic equation 1
a12 = a21 = h = × (co-efficient of xy)
 h g 2
A =  h  f  1
g f  a23 = a32 = f = × (co-efficient of yz)
2
= 3  (f2 + g2 + h2)  2fgh = 0 1
a31 = a13 = g = × (co-efficient of zx)
11. (B) 2
5 4 15. (B)
By solving,   =0
1 2 Hint : The property of the unitary
We get  = 1 and  = 6 matrix
12. (C)
16. (A)
Hint : By reducing the given matrix to
17. (A)
echelon form, we come to know that,
18. (B)
the rank of given matrix is 3.


227
Vidyalankar : GATE – Engineering Mathematics

Model Solution on Assignment  8


1. (C) 7. (A)
Here P diagonals matrix B by pre & As the rows are similar if common is
post multiplication such that taken from each row
1
B = P AP 8. (A)
2. (C) Hint : AX = X
Hint : As per the properties of the A(AX) = A(X)
matrices.  A2X = (AX) =  (X) = 2X
3. (A) A2X = 2 X

1 bc  ca  ab a(b  c) 9. (B)
 = 1 bc  ca  ab b(c  a) Given vectors are
1 bc  ca  ab c(a  b)
1x1 + 2x2 + 3x3 =0
1 1 a(b  c) Which gives
= (ab + bc + ca) 1 1 b(c  a) 1x1 + 2x2 + 23x3 = 0
1 1 c(a  b)
1 x1 + 22x2 + 33x3 =0
= (ab + bc + ca ) 0 = 0 1x1 + 32x2 + 43x3 = 0

4. (C) 31x1 + 42x2 + 93x3 = 0


As per the property of multiplication of which on solving gives 1 = 2 = 3 = 0
matrix.  They are not linearly dependent

5. (C) 10. (C)


6. (C)  1 2 1 : 6
Now 2 1 2 : 6 
8 4 5 
4 1 2     1 1 1 : 5 
0 5 3  1 0 2 
  3 4 7 
  Now, R3  R1

17 29 32  1 2 1 : 6 
  2 1 2 : 6 
 4 12 11   
0 1 0 : 1
17 24 32 
  4 12 11
  R2  2R1

8x  3y 6z 3z  1 2 1 : 6 
 0 3 0 : 6 
 4 12 26x  5y    
0 1 0 : 1
 x = 1, y = 3, z = 4

228
Solutions  Linear Algebra

R2 + 3R3 14. (C)


1 2 1 : 6  We have
 0 0 0 : 3 
  1 a a2 a3 1 a a2 bcd
0 1 0 : 1
2 3 2
1 b b b 1 b b cda
Now the left part of dashed line have =     
1 c c2 c 3
1 c c2 dab
rank 2 and right part have all elements
1 d d2 d3 1 d d2 abc
present so, it does not have solutions.
= 1 + 2 (say)
11. (C)
Multiplying R1, R2, R3, R4 of 2 by
Hint : If the matrix is in the form of
a, b, c, d
upper / lower triangular matrix , then
the diagonal elements are the eigen a a2 a3 abcd
2 3
values of the element. 1 b b b abcd
Now 2 =  
abcd c c 2 c 3
abcd
12. (C)
2
d d d3 abcd
 2 1  2i
1  2i 2  have eigen values as
  a a2 a3 1
2   1  2i b b2 b3 1
  =0 =  
1  2i 2   c c2 c 3
1
2
(  2) ( + 2)  (1 + 2i) (1  2i) = 0 d d d3 1

4  4  (1 + 4) = 0
1 a a2 a3
2
 9=0 1 b b2 b3
=   =  1
 The eigen values are  = 3,  = 3 1 c c2 c3
13. (C) 1 d d2 d3
2 2 0 6   = 1 + (1) = 0
4 2 0 2
  =0
1 1 0 3 15. (C)
1 2 1 2 For triangular matrix eigen values are
Rank < 4 same as major principal diagonal
Now minor of order 3 element i.e. 18, 7, 8.

4 2 0 16. (B)
1 1 0 = 1(4  2) = 6  0
17. (B)
1 2 1
18. (A)
Rank = 3


229
Vidyalankar : GATE – Engineering Mathematics

Model Solution on Assignment  9


1. (B) A + A
As per the definition of unitary square 1 0 5 3   1 2 3 4
 2 1 6 1 0 1 2 4 
matrix.
=  +
3 2 7 1 5 6 7 2 
   
2. (B)  4 4 2 0  3 1 1 0

As per the definition of Hermitian


 2 2 8 7 
matrix  2 2 8 3 
=  
 8 8 14 1
 
3. (C)  7 3 1 0 

4. (A) 7. (C)
a  ic b  id
A=   =1
5. (C) b  id a  ic
cos  sin 
  = cos2  + sin2  = 1  a2 + b 2 + c 2 + d 2 = 1
 sin  cos 
T
 cos   sin  
& Adjoint of A =   8. (A)
  sin  cos  
a h g  x 
cos   sin  
=  
 x y z  h b f y 

 sin  cos    g f c   z 
cos   sin   = ax2 + by2 + cz2 + 2hxy + 2fyz + 2gzx
 A1 =  
 sin  cos   i.e. XT AX gives,
ax2 + by2 + cz2 + 2hxy + 2fyz + 2gzx
6. (B)  a11 a12 a13 
1 0 5 3  If A = a 21 a 22 a 23 
 2 1 6 1 a 31 a 32 a 33 
A=  &
3 2 7 1
  Then for
 4 4 2 0
ax2 + by2 + cz2 + 2hxy + 2fyz + 2gzx
 1 2 3 4 a11 = a a22 = b a33 = c
0 1 2 4 
A =  1
5 6 7 2  a12 = a21 = h = × (co-efficient of xy)
  2
3 1 1 0

230
Solutions  Linear Algebra

1 11. (C)
a23 = a32 = f = × (co-efficient of yz)
2 | A | 33 = 5
1  Rank of A is 3.
a31 = a13 = g = × (co-efficient of zx)
2
Extension of this method 12. (C)
Hint : Eigen values are the values of
9. (C)
diagonal for diagonal matrix. So
As the number of equations &
reduce given matrix to diagonal matrix.
unknowns are same and they are
Or sum of the eigen values of a matrix
equal to zero, so their values are also
= trace of the matrix
zero.
Or | A | ≠ 0 13. (C)
 the system will have trivial solution. Characteristic Equation:
|A  I| = 0
10. (B)
(3  ) 1
a h g  x     =0
1 (2  )
 x y zh b f y 
 g f c   z   (3  ) (2  )  (1) = 0

= ax2 + by2 + cz2 + 2hxy + 2fyz + 2gzx  6  5 + 2 + 1 = 0

i.e. XT AX gives,  2  5 + 7 = 0

ax2 + by2 + cz2 + 2hxy + 2fyz + 2gzx Cayley-Hamilton Theorem,

 a11 a12 a13  A2  5A + 7I = 0


 If A = a 21 a 22 a 23 
a 31 a 32 a 33  14. (C)
 cos  sin  
Then for A=  
  sin  cos  
ax2 + by2 + cz2 + 2hxy + 2fyz + 2gzx
a11 = a a22 = b a33 = c  cos  sin    cos  sin  
A2 =   
  sin  cos     sin  cos  
1
a12 = a21 = h = × (co-efficient of xy)
2  2 2 cos  sin  
cos   sin   sin  cos  
1 = 
a23 = a32 = f = × (co-efficient of yz)   sin  cos  
2 cos 2   sin2  

  sin  cos  
1
a31 = a13 = g = × (co-efficient of zx)
2  cos 2 sin2 
=  
  sin2 cos 2 

231
Vidyalankar : GATE – Engineering Mathematics

As cos2  sin2  = cos 2 a  (p  1)d a  (q  1)d a  (r  1)d


& 2 sin  cos = sin 2  p q r
1 1 1
Similarly , we get
R2  R2  R3
 cosn sinn 
An =   a  (p  1)d a  (q  1)d a  (r  1)d
  sinn cosn 
 p 1 q 1 r 1
1 1 1
15. (B)
a  (p  1)d a  (q  1)d a  (r  1)d
As (ABC)T = CT BT AT 1
 (p  1)d (q  1)d (r  1)d
d
 (B) is wrong. 1 1 1
…..(d0 )
16. (C) R1  R1  R2

Tp Tq Tr a a a
1
Let   p q r   = (p  1)d (q  1)d (r  1)d
d
1 1 1 1 1 1

Here, Tp, Tq and Tr are pth qth and rth 1 1 1


a
term of an AP. = (p  1)d (q  1)d (r  1)d
d
Let the first term of the AP be a and 1 1 1

common difference of the AP be d.  =0


Then
Tp = a +(p  1)d 17. (D)
Tq = a +(q  1)d
Tr = a + (r  1)d 18. (D)



232
Solutions  Linear Algebra

Model Solution on Assignment  10


1. (D) 6. (C)
As R2 = (2)  R1 By performing R4  R4  R3  R2  R1
 | A| 3  3 = 0 & R2 R2  2R1, R3  R3  3R1.
 rank  0 Sequentially we get
3 2 1 2 3 0
| A | 22 =   =  12 + 12 = 0 0 0 3
6 4 2 
A~ 
0 4 8 3
4 18  
  = 144  144 = 0 0 0 0 0
8 36
Now | A | 4  4 = 0 … As E transform
| A | 22 = 0
does not change rank of matrix
 | A | 11 = | 3 | = 3
 rank  4
| A | 11  0
1 2 3
 rank = 1
Now 0 0 3 = 12 i.e.  0
Or R2 = 2R1 0 4 8
R3 = 4R1
 Rank = 3
 rank = 1
7. (A)
2. (B) A matrix is said to be orthogonal if
This is like a property of triangular AT A = I
matrix.  1 1   2 1 
 0   0 
 2 2  6 3
3. (B)  2 1 1   1 1 1 
   
 6 6 6  2 6 3
4. (C)  1 1 1   1 1 1 
   
The diagonal matrix is  3 3 3  2 6 3

1 0 0   x1   1 0 0
= [x1 …….xn]  0  2 . 0  : 
   = 0 1 0  = I
..
 0 0 n   xn   0 0 1

= x12 + 2 x22 + …… + nxn2  A is orthogonal

8. (A)
5. (B)
For triangular matrix the eigen values
Q is obtained by interchanging C1 and C2
are same as principal diagonal
| Q | =  | P | = 8
elements.

233
Vidyalankar : GATE – Engineering Mathematics

9. (A) 6 2 0
a h g  x  (2  )  4 4   0 = 0
 x y zh b f y  2 1 1
 g f c   z 
(2  ) . [(6  ) . (4  )  8] = 0
2 2 2
= ax + by + cz + 2hxy + 2fyz + 2gzx [(2  ) (2  10 + 16) ] = 0
i.e. XT AX gives, (2  ) (  2) (  8) = 0
2 2 2
ax + by + cz + 2hxy + 2fyz + 2gzx   = 2, 2, 8
 a11 a12 a13  Or sum of the eigen values of a matrix
 If A = a 21 a 22 a 23  = trace of the matrix
 a 31 a 32 a 33 

Then for 11. (C)


2 2 2
ax + by + cz + 2hxy + 2fyz + 2gzx As R4 = 3R1
a11 = a a22 = b a33 = c  | A |44 = 0
1 1 4 8
a12 = a21 = h = × (co-efficient of xy)
2
Now 0 0 3
1 4 2 3
a23 = a32 = f = × (co-efficient of yz)
2
= 1(0  6)  4(0  12) + 8(0)
1
a31 = a13 = g = × (co-efficient of zx) = 6 + 48 = 42 i.e.  0
2
 rank = 3
10. (B)
Characteristic equation of A is
12. (B)
| A  I | = 0
| A | = 37
6 2 2
 2 3 1  = 0  1 4 6 
2 1 3 Adj. A =  4 16 13 

 6 13 1 
by C3  C3 + C2
 1 4 6 
1 
6 2 0 A1 = 4 16 13 
 2 3   2   = 0 37 
 6 13 1 
2 1 2  
13. (C)
R2  R2  R3 As A is not symmetric matrix.

234
Solutions  Linear Algebra

14. (C) x x x
1 2 3 c bx a 0
4 5 8 b a cx
3 2 1 1 1 1
= 1(5  16)  2(4  24) + 3(8  15) x c bx a 0
b a cx
= 11 + 40  21
= 8 i.e.  0  x = 0 is one root of the given

 rank = 3 equation.

15. (D) 17. (D)

As R3 = R1 ax b c
 Value of determinant = 0 0 bx a 0
0 0 cx

16. (C) bx a b c


 (a  x) 0
0 cx 0 cx
ax c b
c bx a 0 b c
0 0
b a cx bx a

R1  R1 + (R2 + R3)  (a  x) (b  x)(c  x)  0   0  0  0


ax cb ba  (a  x)(b  x)(c  x) = 0
c  b x  a c  x
 x = a, b, c are the roots.
0
c bx a
18. (C)
b a cx

ab ab ab


c  x c  x c  x

0
c bx a

b a cx

 a+b+c=0


235
Vidyalankar : GATE – Engineering Mathematics

Model Solution on Assignment  11


1. (C) 8. (C)
7 0 
2. (C) X+Y=   ….(i)
2 5 
3. (C)
3 0 
XY=   ….(ii)
4. (D) 0 3 
Addition of (i) and (ii) gives
5. (C)
10 0 
2 3 2X =  
  = 12  12 = 0  2 8
4 6
5 0 
2 3   X=  
  4 6  is not invertible.  1 4
 
7 0  5 0 
 Y=    
6. (C) 2 5   1 4 

7. (C) 2 0 
 Y=  
A = (aij) is 3  2 matrix  1 1

Elements are given as


aij = 2i  j i>j 9. (A)

= 2j  I ij Check with options

Elements of A will be a11, a12, a21, a22,


a31, a32 10. (B)

Elements having i > j = a21, a31, a32  1 2 3 


 
 a21 = 2(2)  (1) = 3 Let A  0 1 2 
 0 0 1 
a31 = 2(3)  (1) = 5
a32 = 2(3)  2 = 4 1 2 3
A  0 1 2
Elements having i  j ; a11, a12, a22
0 0 1
 a11 = 2(1)  (1) = 1
1 2 2 3 2 3
a12 = 2(2)  (1) = 3 =1 0 0
0 1 0 1 1 2
a22 = 2(2)  (2) = 2
A  1 0
 1 3
   A1 exists
 Matrix A  3 2 
 5 4 

236
Solutions  Linear Algebra

Cofactors :  1 2 7 
1
1 2   0 1 2  .....( A  1)
A11  (1)11 1 2
0 1  0 0 1 

0 2  1 2 7 
A12  (1)1 2 0
0 1  A 1
  0 1 2 
 0 0 1 
0 1
A13  (1)1 3 0
0 0
11. (D)
2 3
A 21  (1)2 1  2
0 1
12. (B)
22 1 3
A 22  (1) 1
0 1
13. (C)
23 1 2
A 23  (1) 0 a1, a2, ……. are in GP
0 0
am 1 am  2 a m  3
2 3         k(say)
A 31  (1) 3 1
7 am a m 1 a m  2
1 2
am+1 = amk
1 3
A 32  (1) 32
 2 am+2 = am+1k = amk2
0 2
am+3 = am+2k = amk3
1 2 and so on
A 33  (1)3  3 1
0 1
log am logam 1 logam  2
 1 0 0  = logam  3 logam  4 logam  5 
 
 Cofactor Matrix =  2 1 0  logam  6 logam  7 logam  8
 7 2 1
log am log am
logam
Adj A = Cofactor matrix 
T
 logk 2logk
T
 1 0 0 logam logam log am
  =  
=  2 1 0  3logk 4logk 5logk
 7 2 1

 1 2 7  logam logam log am


  6logk 7logk 8logk
Adj A = 0 1 2 
 0 0 1 

adjA
 A 1 
A

237
Vidyalankar : GATE – Engineering Mathematics

R2  R2  R1 ; R3  R3 R1 = 1  cos2(  )  cos2(  )  cos2(  )


+ 2{cos(  ) cos(  )} cos(  )
logam logam
logam
 logk 2logk = 1  cos2(  )  cos2(  )
 cos2( ) + cos{(  )  (  )}
=  
3logk 3logk 3logk + cos{(  ) + (  )} cos(  )
= 1  cos2(  )  cos2(  )  cos2(  )
6logk 6logk 6logk
+ cos(  ) cos(  2 + )
log am logam
log am + cos(  ) cos (  )
 logk 2logk
= 1  cos2 (  )  cos2 (   )
= 36  
logk logk logk cos ( )(2 ) 
 
cos (  )(2 )
logk logk logk 
2
=0
= 1  cos 2 (  )  cos 2 (   )

cos(2  2)cos(2 2)


14. (D) 
2
Given determinant, 
= 1cos 2 (  )cos 2 ( )
1 cos(  ) cos(   )
cos2(  )cos 2(   )
= cos(  ) 1 cos(   ) 
2
cos(   ) cos(   ) 1
22 cos2 (  )2 cos2 ( ) 
1 cos(  ) cos(   )  2

2 cos (  )1 
= cos(  ) 1 cos(   )  2 
 2 cos (   )1 
cos(   ) cos(   ) 1 =
2
.... Since cos() = cos
=0
= 1(1  cos2(  ))
cos(  )  15. (B)
 cos(  )  
cos(   ) cos(   )
0 2 4
cos(  ) cos(   ) A   1 1 2 
+ cos(  )  
cos(   )   2 0 5 

= 1  cos2(  )  cos2(  ) Characteristic Equation of A is


+ cos(  ) cos(  ) cos(  ) A  I  0
+ cos(  ) cos(  ) cos(  )
 cos2(  )

238
Solutions  Linear Algebra

0 2 4   = 2, 2, 3
 1 1  2  0 For  = 2
2 0 5
(3  2) 10 5   x1  0 
 2 (3  2) 4  x 2  =  0 
  
 2 4 
i.e. 1 1  2  0  3 5 (7  2)  x 3   0 
2 0 5
 1 10 5   x1  0 
  2 5 4  x  =  0 
1  2 1 2    2  
  2
 3 5 5   x 3   0 
0 5 2 5  

1 1  5
4 0  2  satisfies the above equation.
2 0  
 5 
  (1  )(5  )  0  2  5    4

4  0  2(1   )  0
17. (D)
  5  6     2 1   
2
2 22 23
4  2  2   0 1 = 3 32 33 
4 42 43
 5  6 2   3  2  2  8  8  0
1 2 22
 3 + 62  1 + 6 = 0
= 234 1 3 32 
3 2
i.e.   6 + 1  6 = 0 1 4 42

1 2 4
16. (C)
= 24 1 3 9 
Characteristic equation: |A  I| = 0 1 4 16
(3  ) 10 5
1 = 24
  2 (3   ) 4  = 0
3 5 (7   )
18. (D)
gives 3  72 + 16  12 = 0



239
Vidyalankar : GATE – Engineering Mathematics

Answer Key on Test Paper  1


1. (B) 2. (C) 3. (C) 4. (B)
5. (C) 6. (D) 7. (C) 8. (B)
9. (A) 10. (C) 11. (D) 12. (A)
13. (B) 14. (C) 15. (C)

Answer Key on Test Paper  2


1. (B) 2. (B) 3. (A) 4. (A)
5. (B) 6. (B) 7. (D) 8. (B)
9. (B) 10. (D) 11. (C) 12. (B)
13. (C) 14. (A) 15. (A)

Answer Key on Test Paper  3


1. (C) 2. (C) 3. (D) 4. (A)
5. (C) 6. (A) 7. (B) 8. (D)
9. (D) 10. (C) 11. (C) 12. (B)
13. (B) 14. (C) 15. (A)

Answer Key on Test Paper  4


1. (B) 2. (C) 3. (C) 4. (C)
5. (B) 6. (A) 7. (A) 8. (C)
9. (B) 10. (B) 11. (A) 12. (A)
13. (B) 14. (A) 15. (B)

Answer Key on Test Paper  5


1. (A) 2. (B) 3. (B) 4. (B)
5. (C) 6. (C) 7. (B) 8. (A)
9. (B) 10. (D) 11. (B) 12. (A)
13. (B) 14. (C) 15. (D)

Answer Key on Test Paper  6


1. (B) 2. (B) 3. (B) 4. (A)
5. (A) 6. (C) 7. (C) 8. (B)
9. (C) 10. (D) 11. (A) 12. (B)
13. (B) 14. (B) 15. (C)



240
Solutions  Linear Algebra

Model Solution on Test Paper  1


1. (B) 31  52 3 = 0
41 + 22 + 33 = 0
2. (C) 21 + 22 + 23 = 0
x+3=0  x = 3 By solving these equations we get
similarly y, z & a can be calculated. 1 = 1, 2 = 1, 3 = 2
 Equation is x1 + x2  2x3 = 0
3. (C)

8. (B)
4. (B) Suppose these points are collinear on
line ax + by + c = 0
5. (C) Then ax1 + by1 + c = 0 ……(i)
Characteristic equation is ax2 + by2 + c = 0 …….(ii)
4 1 ax3 + by3 + c = 0 ……(iii)
  =0
1 4 Eliminate a, b, c, between (i),
2
(4)  1 =0 (ii) and (iii)
16  8 + 2  1 = 0 x1 y1 1
2
  8 + 15 = 0 x 2 y 2 1 = 0
x3 y3 1
(  5) (  3) = 0
 1 = 3, 3 = 5 x1 y1 1
Thus rank of matrix x 2 y 2 1 is
Or following property can be used
x3 y3 1
Sum of the eigen values = trace
less than 3
Conversely if rank of matrix A is less
6. (D)
than 3, then

7. (C) x1 y1 1
x 2 y 2 1 = 0
Relationship obtained by using
x3 y3 1
1 x 1 + 2 x 2 + 3 x 3 = 0
 Area of triangle with vertices
 1(1, 3, 4, 2) + 2(3, 5, 2, 2)
(x1 , y1) (x2, y2) (x3, y3) is equal to 0
+ 3(2, 1, 3, 2) = 0
 They are collinear points.
gives 4 equations
i.e. 1 + 32 + 23 = 0

241
Vidyalankar : GATE – Engineering Mathematics

9. (A) 10. (C)


We have equation as By taking product
1 1 1   x  9  c b   a ab ac 
2
0
2 5 7   y   52   
      AB =  c 0 a  ab b2 bc 
 2 1 1 z  0   b a 0   ac bc c 2 
 
AX = B
0 0 0 
Now augmented matrix
= 0 0 0 
1 1 1 : 9  0 0 0 
[A : B] =  2 5 7 : 52 
 2 1 1 : 0  11. (D)
f(x) = x2  5x + 6
We reduce it in Echelon form
f(A) = A2  5A + 6 I
R2  R2  2R1 R3  R3  2R1
 3 1  3 1  3 1
1 1 1 : 9  =     5 
 0 3 5 : 34   1 2   1 2   1 2 
 
 0 1 3 : 18   1 0
6  
 0 1
R2
R2 
3 which gives
 1 0 
1 1 1 : 9  f(A) =  
 0 1 5 / 3 : 34 / 3   0 1
 
 0 1 3 : 18 
R3  R3 + R 2 12. (A)

1 1 1 : 9  A matrix is said to be idempotent if


 0 1 5 / 3 : 34 / 3  A2 = A
 
 0 0 4 / 3 : 20 / 3 
 2 2 4   2 2 4 
Here,  1 3 4  1 3 4 
 Rank A = Rank [A : B] = 3    
 Equations are consistent.  1 2 3   1 2 3 

 2 2 4 
gives 1 3 4 
 1 2 3 

which is same as A.
 It is idempotent

242
Solutions  Linear Algebra

13. (B) 14. (C)


cos   sin  0  We have (A  I ) = 0
A =  sin  cos  0  1  1 1
 0 0 1 1 1  1 = 0
 |A| = 1 1 1 1 

adj A  (1  ) [(1  )2 1] 1[(1  )  1]


 A1 =
|A| + 1[1  (1  )] = 0
T
Adj A = [cofactor matrix A]  (1  ) [(1  )2 1] 1[(1  )  1]
cos   sin  0  + 1[1  (1  ) ] = 0
=  sin  cos  0   (1  ) (2  2) + 2 = 0
 0 0 1
 2 (3  ) = 0
 cos  sin  0    = 0, 0, 3
=   sin  cos  0 
 0 0 1
15. (C)
 cos  sin  0  1
 A1 =   sin  cos  0  We have C1 
8
C1
 0 0 1
 1 1 3 6
 0 3 2 2
 
 1 1 3 4 

Now R3  R3 + R1
1 1 3 6 
0 3 2 2  which is in Echelon
 
0 0 0 10 

form so rank is equal to no. of nonzero


rows
Rank = 3



243
Vidyalankar : GATE – Engineering Mathematics

Model Solution on Test Paper  2

1. (B) 6. (B)
The given system of equation is
2. (B)
equivalent to the single matrix
3. (A) equation
2 6 0  x 
4. (A)
AX = 6 20 6  y 

2 3 4  0 1 2  0 6 18  z 
2A + 3B = 2   +3  
 1 2 5  2 4 6   11
4 9 12    3  = B
=  
8 8 28   1 

We reduce this matrix into triangular


5. (B)
matrix by sequentially performing
1 a bc
 = 1 b ca R2  R2  3R1,
1 c ab R3  R33R2, we get

Performing R2  R2  R1 , 2 6 0   x   11
0 2 6   y   30 
R3  R3  R1      
0 0 0  z   91
1 a bc
0 b  a ca  bc =0 We have 0x + 0y + 0z = 91, this
0 c  a ab  bc shows systems is not consistent

 (b  a) (ab  bc) 7. (D)

 (c  a) (ca  bc) = 0 Using

(b  a). b(a  c) cos  cos   sin  sin  = cos ( + )

 (a  c) c(a  b) = 0 cos  sin  + sin  cos  = sin ( + )

(a  c) (b  a) (b  c) = 0 cos  cos   sin  sin  = cos ( + )

 (a  b) is a factor of   cos(  ) sin(  ) 


we get  
  sin(  ) cos(  )

8. (B)
As A3 = A . A . A = 0
where A is given matrix
 It is nilpotent matrix

244
Solutions  Linear Algebra

9. (B) 2 3 5   1 9 17 
R3  R3 + R1 , R2  R2 + 2R1 gives = 0 3 1  0 4 5 
0 0 4  0 0 9 
 1 2 3 
0 0 5 
  3 12 22 
0 0 10  = 0 7 6 
The determinant of above matrix is 0 0 0 13 
Checking for non-zero minor of order 2 Now for triangular matrix eigen values
1 2 are same as principal diagonal
  =0
2 4 element.

2 3  1 = 3, 2 = 7, 3 = 13
  = 2  12 = 10  0
4 1 are eigen values.

 rank = 2 13. (C)

10. (D) cos   sin   a 0 


Let A =   B=  
 sin  cos   0 b 
11. (C)
Here AB = BA if a = b
As it is not a singular matrix
14. (A)
12. (B)
R4 = 3R1
1 3 5 
Hence rank < 4 and
A = 0 2 1
0 0 3  1 4 8
0 0 3 = 48  0
 I + A + A2
4 2 3
 1 0 0  1 3 5 
 Rank = 3
= 0 1 0   0 2 1
0 0 1 0 0 3 
15. (A)
1 3 5  1 3 5 
 0 2 10 2 1
0 0 3  0 0 3 



245
Vidyalankar : GATE – Engineering Mathematics

Model Solution on Test Paper  3


1. (C) 10. (C)
As for triangular matrix eigen values The given equation in Matrix Form:
are principal diagonal elements 2 3 5   x  9 
7 3 2  y  =  8 
    
2. (C) 2 3    z   
As per definition of rank of matrix
AX = B
2 3 5 : 9 
3. (D)
[A : B] = 7 3 2 : 8 
As columns of A  rows of B
2 3  :  
So product is not possible
R3 R3 R1
4. (A) 2 3 5 : 9 
7 3  2 : 8 

5. (C)  0 0 (  5) : (  9)
As these are properties of skew
If rank(A)  rank (A : B), system has no
symmetric matrix.
solution.

6. (A)   = 5,   9

1 1
e.g. A =   for 2  2 matrix 11. (C)
1 1
Just by the definition of the rank of
Rank of A = 1
matrix we find that the rank of given

7. (B) matrix A is 3.

12. (B)
8. (D)
Characteristic equation of A is
As per property of idempotent matrix if
|A  I| = 0
A & B are idempotent implies AB = BA.
1  0 2
9. (D)  0 2 1  =0
The determinant of 3  3 element of 2 0 3
given matrix is not equal to zero or  (1  ) [(2  ) (3  )  0]
when reduced to echelon form, it gives + 2  2 (2  ) = 0
3 rowed matrix.  3  62 + 7 + 2 = 0

246
Solutions  Linear Algebra

13. (B) 15. (A)


By solving the above equations as R3  R3  (R1 + R2), we get
follows x2 2x  3 3x  4
2 1 2 : 1 2x  3 3x  4 4x  5 = 0
1 1 0 : 0 0 1 3x  8
1 k 6 : 3
By R2  R2  R1 & R1  R1 + R3
 That is, after reducing to Echelon x  2 2x  4 6x  12
form, if we want unique solution, x  1 x 1 x 1  = 0
then K  2 0 1 3x  8

14. (C) 1 2 6
(x + 1) (x + 2) 1 1 1  =0
Hint : By just the definition
0 1 3x  8
i.e. transpose of all elements' cofactor
R1  R1  R2
matrix.
OR [cofactor Matrix ]T 0 1 5
(x + 1) (x + 2) 1 1 1  =0
0 1 3x  8

Now,
(x +1) (x + 2) [(3x + 8  5)] = 0
3 (x + 1) (x + 2) (x + 1) = 0
 x = 1, 1, 2



247
Vidyalankar : GATE – Engineering Mathematics

Model Solution on Test Paper  4


1. (B) 1
a12 = a21 = h = × (co-efficient of xy)
As per the property of the skew 2
Hermitian matrix 1
a23 = a32 = f = × (co-efficient of yz)
2
2. (C)
1
a31 = a13 = g = × (co-efficient of zx)
As per the property of the eigen values 2
for orthogonal matrices. i.e. for Extension of this method
orthogonal matrix, the eigen value is
reciprocal of the original eigen value. 6. (A)
When reduced to Echelon form, non-
3. (C) zero rows obtained is 1.
Hint : The transpose of the matrix Or R2 = 6R1 & R3 = 5R1
does not change the rank of the
matrix. 7. (C)

4. (C) 8. (C)
As per the defitnition of nilpotent  2 1 1 
matrix for index K. A =  1 2 1
 1 1 2 
5. (B)
Characteristic equation of matrix
a h g  x  2 1 1
 x y zh b f  y   1 2 1  = 0
 g f c   z 
1 1 2
= ax2 + by2 + cz2 + 2hxy + 2fyz + 2gzx
 3  62 + 9 + 4 = 0
T
i.e. X AX gives,
ax2 + by2 + cz2 + 2hxy + 2fyz + 2gzx 9. (B)
 a11 a12 a13 
 If A =  a 21 a 22 a 23  10. (B)
 a 31 a 32 a 33 
11. (A)
Then for
Inverse of matrix A is,
ax2 + by2 + cz2 + 2hxy + 2fyz + 2gzx
1
a11 = a a22 = b a33 = c A1 =  Adj A
|A|

248
Solutions  Linear Algebra

12. (A) 13. (B)


0 is an eigen value of A   = 0 R2 = 3R1
satisfies the equation | A  I | =0 As one of the row is repeated, so the

 | A | = 0  A is singular no. of rows for considering rank

Conversely  | A | = 0 reduces to 2. Now, for given row


matrix, determinant is non zero and
 = 0 satisfies the equation
hence the matrix is 2.
|AI|=0
 0 is eigen value of A 14. (A)

15. (B)



249
Vidyalankar : GATE – Engineering Mathematics

Model Solution on Test Paper  5


1. (A) 8. (A)
T
For symmetric matrix A = A For triangular matrix eigen values are
same as major principal diagonal
2. (B)
elements.
We have, a + 3 = 2a + 1
 a = 2 9. (B)
Similarly, b = 1, 2 1x1 + 2x2 + 3x3 + 4x4 = 0
1(1, 2, 4) + 2(2, 1, 3) + 3 (0, 1, 2)
3. (B)
 ab b2  + 4 (3, 7, 2) = 0
A=  
2 1 + 22  34 = 0
 a ab 
21  2 + 3 + 74 = 0
 ab b2   ab b2 
A.A=    2  41 + 32 + 23 + 24 = 0
 a 2 ab   a ab 
By solving these equations we get
0 0 
=   1 = 9 , 2 = 12, 3 = 5, 4 = 5
0 0 
 relation is 9x1  12x2 + 5x3  5x4 = 0
 A2 = 0  K=2
 Index = 2 10. (D)

4. (B)
11. (B)
It is the definition of eigen values.
|A|=1
5. (C) T
 a  ib c  id 
For diagonal matrix, eigen values are adj A =  
 c  id a  ib 
different.
Adj A
A1 =
6. (C) |A|
7. (B) a  ib c  id
=  
C2 = 2C1  c  id a  ib 
 | A |3  3 = 0
 rank < 3
minor of order 2
3 2
  = 12  4 = 8  0
2 4

 rank = 2

250
Solutions  Linear Algebra

12. (A) 14. (C)


Idempotent matrix is one which determinant
2
satisfies A = A. But given matrix not 6 0 0
satisfy this relation 1 8 2 0 = 1[6 [80] ] = 48
 It is not a idempotent matrix 1 4 4

13. (B)
15. (D)
| A | 33= 0
 Rank < 3
1 1
Minor of order 2,   = 2
0 2

i.e.  0
 rank = 2



251
Vidyalankar : GATE – Engineering Mathematics

Model Solution on Test Paper  6


1. (B) 6. (C)
| A | = a [b (d   )] = abd  As per theorem,
1 a 1 1 1
2. (B) 1 1 b 1 1
 
1 1 1 c 1
As [A] 3  3 [B] 3  2
1 1 1 1 d
 [B] 3  2 [A] 3  3
BA is not possible as  1 1 1 1
= abcd  1     
 a b c d
columns of B  rows of A
Now here a = b = c = d = 4

3. (B)  = (4)4  (0) = 0

A is obtained by substituting i by i
7. (C)

4. (A) By R4  R4  R3  R2  R1 , R1  R2,

3 1 2 R2  R2  2R1, R3  R3  3R1
| A | 3  3 = 6 2 4 = 0  1 1 2 4 
3 1 2 0 5 3 7 
Subsequently,  
 0 4 9 10 
3 1  
| A | 22 =   =0 0 0 0 0 
6 2
1 1 2
Similarly other minors of order 2 are 0
We see that 0 5 3  = 33  0
 | A | 11 = | 3 |  0
0 4 9
 rank = 1
 Rank = 3

5. (A) 8. (B)
Characteristic equation are Hint : Adjoint A = [ Cofactor matrix A]T
| A  I| = 0
9. (C)
5 3
  =0 Characteristic roots are obtained by
3 3  
| A  I | = 0
(5  ) (3  )  9 = 0
 1 2
15 + 2 + 2  9 = 0
1  1 = 0
2  2  24 = 0 2 1 
  = 6, 4

252
Solutions  Linear Algebra

 (2 1) 1( + 2) + 2(1 + 2) = 0 14. (B)


3
 + 6  4 = 0  3 4 
2 1 2 
3 + 02  6 + 4 = 0 AB =  1 1  
 1 3 4 
 2 0  
By synthetic division

2 1 0 6 4  2 9 10 
  3 4 6 
0 2 4 4
 4 2 4 
1 2 2 0
 2 1
 3 1 2
 =2 B A =  1 3  
4 1 0 
2 + 2  2 = 0  2 4  

  = 1  3  2 3 4
 roots are 2, 1  3   9 4 2 
 10 6 4 

10. (D)
As for triangular matrix eigen values 15. (C)
are same as principal diagonal The given equations in matrix form
element.
1 a a  x  0 
b 1 b  y  =  0 
11. (A)     
 c c 1  z   0 

12. (B) For non trivial solution,


As C4 = 2C1 |A| = 0
 | A | 44 = 0 1 a a
 rank < 4  b 1 b = 0
c c 1
1 2 1
Minor of order 3 1 3 1 1(1  bc)  a(b  bc) + a(bc  c) = 0
2 4 3  1  bc  ab + abc + abc  ac = 0
= 1( 9  4)  2(3  2) + 1(4  6)  ab + bc + ac  abc = 1 + abc ….(i)
=1 0 a b c
 
 rank = 3 1 a 1 b 1 c

a(1  b)(1  c)b(1a)(1c) 


 
 c(1a)(1b) 
13. (B) =
(1a)(1b)(1c)

253
Vidyalankar : GATE – Engineering Mathematics

a(1bcbc)  abcabc2(1abc)
  =
b(1acac)  1abc(1abc)
c(1a bab) 
=   ….From (i)
(1a)(1bcbc)
abcabc22abc
=
aabacabcbabbc  1abc1abc
 
abccacbcabc
=  
=
2  a  b  c  abc
1bcbcaabacabc 2  a  b  c  abc
abcabc  = 1
 
 2(abbcacabc)
=
1 a  b  c(abbcacabc)



254
Solutions  Calculus
Answer Key on Assignment  1
1. (A) 2. (D) 3. (C) 4. (D)

5. (C) 6. (C) 7. (C) 8. 2

9. (C) 10. (D) 11. 100.01 to 99.99 12. 5.9 to 6.1

13. 1.85 to 1.87 14. 1 15. 4.66 to 4.76 16 85.0 to 85.5

17. (B) 18. (C) 19. (C) 20. (C)

Answer Key on Assignment  2


1. (B) 2. (B) 3. 0.01 to 0.01 4. (A)

5. (A) 6. (D) 7. (B) 8. 5.1 to 4.9

9. (B) 10. 2.0 to 2.0 11. (B) 12. (B)

13. 1.9 to 2.1 14. (C) 15. (B) 16 13 to 13

17. (D) 18. (B) 19. (A) 20. 0.99 to 1.01

Answer Key on Assignment  3


1. 40 to 40 2. (A) 3. (C) 4. (C)

5. 0.35 to 0.30 6. (C) 7. (A) 8. (C)

9. (B) 10. 0.0 to 0.0 11. (A) 12. (A)

13. 0.95 to 1.05 14. 18 to 22 15. (D) 16 (A)

17. (A) 18. (D) 19. (C) 20. (A)

255
Vidyalankar : GATE – Engineering Mathematics

Answer Key on Assignment  4


1. (B) 2. (D) 3. (B) 4. (D)

5. (D) 6. (B) 7. (A) 8. (A)

9. (C) 10. (D) 11. (D) 12. (C)

13. (C) 14. (C) 15. (B) 16 (A)

17. (D) 18. (B)

Answer Key on Assignment  5


1. (D) 2. (A) 3. (C) 4. (B)

5. (C) 6. (A) 7. (A) 8. (B)

9. (A) 10. (A) 11. (D) 12. (B)

13. (B) 14. (D) 15. (B) 16. (B)

17. (B) 18 (D)

Answer Key on Assignment  6


1. (A) 2. (A) 3. (B) 4. (D)

5. (B) 6. (A) 7. (A) 8. (B)

9. (C) 10. (A) 11. (B) 12. (C)

13. (A) 14. (A) 15. (B) 16. (B)

17. (C) 18. (C)

256
Solutions  Calculus

Answer Key on Assignment  7


1. (B) 2. (C) 3. (A) 4. (B)

5. (A) 6. (D) 7. (A) 8. (B)

9. (B) 10. (C) 11. (A) 12. (B)

13. (A) 14. (D) 15. (B) 16. (A)

17. (B) 18. (A)

Answer Key on Assignment  8


1. (C) 2. (C) 3. (D) 4. (D)

5. (D) 6. (D) 7. (B) 8. (C)

9. (C) 10. (D) 11. (A) 12. (A)

13. (B) 14. (A) 15. (C) 16. (B)

17. (C) 18. (B)

Answer Key on Assignment  9


1. (B) 2. (B) 3. (A) 4. (C)

5. (B) 6. (D) 7. (B) 8. (A)

9. (A) 10. (C) 11. (A) 12. (B)

13. (D) 14. (C) 15. (A) 16. (D)

17. (B) 18. (B)



257
Vidyalankar : GATE – Engineering Mathematics

Model Solution on Assignment  1


1. (A) 5. (C)
xsinx 0 1  cos(x 2 ) 2 sin2 (x 2 2)
lim =   lim = lim
x  0 1  cos x
0 x 0 2x 4 x 0 2x 4
Applying L. Hospital Rule, sin2 (x 2 2) 1
= lim 
1cos x 0 x  0 (x 2 2)2 4
lim =  
x 0 sinx 0 2
1  sin(x 2 2) 
Once again, L. Hospital rule = lim 
4 x 0  (x 2 2) 
sinx 0
lim =   =0 1 1
x 0 cos x  1 = (1)2 =
4 4
2. (D) Alternate Method
We know that f(x) is continuous at x = a,
1  cos(x 2 ) 0
lim 
if limf(x) exists and equal to f(a). x 0 2x 4 0
x a

Using L Hospital Rule


3. (C)
(sin x 2 )2x 0
 1
x
lim 3

lim  1   x 0 8x 0
x 
 x
(cos x 2 )2x2x  (sin x 2 )2
1 lim
Let h x 0 24x 2
x
as x  , h  0 (cos x 2 )4x 2  2sin x 2
lim
x
x 0 24x 2
 1 1
 lim  1    lim 1  h  h ( sinx 2 )4x 2  cos x 2 (8x)
x 
 x h 0
 2

=e 2(cos x )2x 
= lim
x 0 48x
4. (D)
2 3
(cos x 2 )2x 4x 2

  

a a a
We have e = 1a  .... (sin x )  8x  
2
2! 3!  
(sin x )  2x  8x   12 cos x 
2 2

an  
=  n! 2
( sinx )(2x)4x 
n0 = lim
x 0 48
Put a = 1
12 1

1 = =
 
n 0 n!
e1e 48 4

258
Solutions  Calculus

6. (C) 9. (C)
Given: f(x) does not have a root in  4 
loge (1  4x)  1  4x  4
[a, b]. That is from x = a to x = b, then lim 3x
lim  3x
 
curve f(x) does not cross the X-axis.
x 0 e 1 x  0 3e 3

Hence the curve is entirely above


10. (D)
X-axis or entirely below it.
x 3  sinx
 f(a) > 0 and f(b) > 0 OR f(a) < 0 limx  0 
x
and f(b) < 0
 sin x 
= (limx 0 x 2 ) limx 0
 f(a) . f(b) > 0
 x 

7. (C) = 0  1 = 1
The ranges are given by 0 < y < a and
11. 100.01 to 99.99
0 < x < y i.e. 0 < x < y < a.
1 x3
This can also be written as 0 < x < a f(x) = x(x 2  3) = x
3 3
and x < y < a
3x 2
f(x) = 1 = x2  1
8. 2 3

 n  x2  1 = 0
 1

n 0
n.   = ?
2 x=1
This can be solved using Z-transform f(x) = 2x
and also using algebra as arithmetic- f(1) = 2 > 0
geometric series. We will use algebra.  at x = 1, f(x) has local minimum.

 1
n
1  1
2
f(1) = 2 < 0

n 0
n.   = 0  1 1 2    ...
2 2 2  at x = 1, f(x) has local maximum
[a + (a + d)r + (a + 2d)r2 + …..infinite For x = 1, local minimum value
terms] 1 2
f(1) = 2
a rd 3 3
=  (1 < r < 1)
1  r (1  r)2 Finding f(100) = 333433.33

1 f(100) = 333233.33
For above series, a = 0, d = 1, r = .
2 ( x = 100, 100 are end points of
n

 1 0 (1 2)1) interval)
    .n
n 0  2 
=
1
 2
=2
 Minimum occurs at x = 100
1  1
2  1 2 
 

259
Vidyalankar : GATE – Engineering Mathematics

12. 5.9 to 6.1 14. 1


3 2
f(x) = 2x  9x + 12x  3 2/
 1  1
 2
f(x) = 6x  18x + 12
I=   2 cos  x dx
2/  x 

 f(x) = 12x  18  1
2
Put  y
f(x) = 0 gives 6(x  3x + 2) = 0 x
 x = 1, x = 2  1
   2 dxdy
f(1) = 12  18 = 6 x 

and f(2) = 24  18 = 6 Further, if x = 1/ then y =  and


 There is local maxima at x = 1 and if x = 2/ then y = /2
local minima at x = 2 /2 

Now, f(0) = 3,


I=  cos y(dy) =
/2
 cos ydy
f(1) = 2  9 + 12  3 = 2 and = sin()  sin(/2) = 0  1 = 1
f(3) = 54  81 + 36  3 = 6
15. 4.66 to 4.76
5 /4 4.5

13. 1.85 to 1.87 Volume = 


 3

 / 8
 dzdd
z3
The length of the curve
5 /4
 /2
 dx   dy 
2
 dz 
2 2 =   (1.5)dd
=   dt    dt    dt  dt
     
 3  / 8

0
5

 /2
2
2 =  1.5( / 8)d

2 2
= ( sin t)  (cos t)    dt  3

0 
= 1.5 × (/8) × 8
 /2
4 = 1.5 = 4.7124
=  sin2 t  cos2 t 
 2
dt
0 16. 85.0 to 85.5
 /2
4 17. (B)
=  1
2
dt
0 y = 2x  0.1x2

4  /2 dy
= 1 .t 0 = 2  0.2x
 2 dx

4  d2 y
= 1 x = 1.8622 0
2 2 dx 2
 y maximizes at 2  0.2x = 0
 x = 10
 y = 20  10 = 10 m

260
Solutions  Calculus

18. (C) 20. (C)


Given function is f(x) = |x|  u 
Given x = cos   ,
|x| is continuous at x = 0 but not  2 
differentiable  u 
y = sin   0 ≤ u ≤ 1
 2 
19. (C)
dx   u 
To find maxima  sin  
du 2  2 
We differentiate f(x) w.r.t. ‘x’ & equate
dy  u
to zero  cos
du 2 2
df(x) We know that surface area when the
  3x 2  18x  24  0
dx
curve revolved about X-axis of a
 x2  6x + 8 = 0
parametric curve is
(x  4) (x  2) 2 2
1
 dx   dy 
x = 2 or x = 4 = 2  y     du
0  du   du 
Now,
1
 u 
d2 f(x) = 2  sin  
 6x  18   6  0 (maxima)  2 
dx 2 x  2 0

2 2
d2 f(x)    u      u  
 6  0 (minimum)  sin     cos    du
dx 2 x  4  2  2    2  2 

1
 f(x) x  2 = (2)3  9(2)2  24(2)  5 u 2
= 2 sin  du
0
2 4
= 8  36 + 48 + 5 = 25
1
But since interval [1, 6] i.e. inclusive,  u
= 2  sin dx
20 2
we have to find
1
f(1) = 1  9 + 24 + 5 = 21  u 
 1cos
  
2
f(6) = (6)3  9(6)2 + 24(6) + 5 2
=   
  
= 216  324 + 144 + 5 = 41  2 0
Thus, in the interval maximum value is
2  
41. = 2   cos cos 0 
 2 
= 2[cos0 1] = 2



261
Vidyalankar : GATE – Engineering Mathematics

Model Solution on Assignment  2


1. (B) 1+x=1
e2x 1 0 x=0
lim  is  
x  0 sin4x
0  There is a maxima at x = 0
So, Applying L-Hospital Rule,  Maximum value = f(0) = n(1)00
2x
2e 2
lim = = 0.5 4. (A)
x 0 4cos4x 4
In order to find maximum value of the
2. (B)
function we have
2

 (x  1) sin(x  1) f(x) = 0
2

Let I = 0
dx  x ex + ex = 0
(x  1)2 cos(x  1)
 ex(1  x) = 0
We know that,
 x=1
2
(2x  1)2 sin(2x 1)
0 (2x1)2 cos(2x1)dx And f(x) < 0 at x = 1
 Maximum value is f(1) = (1) e1 = e1
2 2
(1x) sin(1x)
=  (1x) cos(1x)dx
0
2 5. (A)

2
For negative value of x, f(x) will be
(x  1)2 sin(x  1)
=  2
dx = I positive
0 (x  1) cos(x  1)
For positive values of x, f(x) will be
 I+I=0
positive
 2I = 0
 minimum value of f(x) will occur at
 I=0
x=0
3. 0.01 to 0.01
6. (D)
f(x) = n(1  x)x
7. (B)
1
 f(x) =  1 and f(x) The given function is
1 x
F(x) = 1  x2 + x3,
1
=  0 where x  [1, 1]
(1  x)2
 F(x) = 2x + 3x2
1
 Maxima f(x) = 0 gives 10
1 x By mean value theorem

1 F(1)F(1)
 1 F(x) =
1 x 1  (1)

262
Solutions  Calculus

Now F(1) = 1  (1)2 + (1)3 = 1 9. (B)


2 3
and F(1) = 1  (1) + (1)
10. 2.0 to 2.0
= 1  1  1 = 1 1
1
 F(x) =
1(1)
=
2
=1
I= 
0 (1  x)
dx ;
1  (1) 2
a a
 F(x) = 1 Use:  f(x)dx f(ax)dx
0 0
 2x + 3x2 = 1
1
 3x2  2x  1 = 0 1
I= 
0 (1  (1  x))
dx
2
 3x  3x + x  1 = 0
1 1
 3x(x  1) + 1(x  1) = 0 1
 x
1/ 2
= dx = dx
x
 (3x + 1) = 0 0 0

1
or x1=0  x1/ 2 
=   =2
1  1/ 2  0
 x= 
3
11. (B)
or x=1
2 x
1
 e
xy
Now  lies between (1, 1) dydx
3 0 0

0 e  e 0 e dy dx


2 x
1 = x x y
 x= 
3
2 2

 e  e  dx
x
 e (e 1)dx
x y x x
8. 5.1 to 4.9 = =
0
0 0
f(x) = 2x3  3x2 where x  [1, 2] 2
2
 e2x 
=  (e e )dx = 
2
 f(x) = 6x  6x and f(x) = 12x  6 2x x
e x 
0  2 0
 f(x) = 0 gives 6x2  6x = 0
e4 1 e4 1
 6x(x  1) = 0  x = 0, 1 = e2  1 = e2 
2 2 2 2
f(0) = 6 < 0
1 4
 There is a maxima at x = 0 = (e 2e2 1)
2
f(1) = 12  6 = 6 > 0
1 2
= (e 1)2
 There is a minima at x = 1 2
f(1) = 2  3 = 1; f(1) = 2  3 = 5;
f(2) = 16  12 = 4
 Global minimum value of f(x) = 5

263
Vidyalankar : GATE – Engineering Mathematics

12. (B)
= lim
x 2
 x  1 x2 
f(x) = x3  3x2  24x + 100 x 
x2  x  1  x
where x  [3, 3]  1
x 1 
 f(x) = 3x2  6x  24 and  x 1
= lim =
x   1 1  2
f(x) = 6x  6 x  1   2  1
 x x 
f(x) = 0 gives 3(x2  2x  8) = 0
 (x  4) (x + 2) = 0  x = 2, 4 15. (B)

As x = 4 > 3, consider x = 2  [3, 3]


16. 13 to 13
f(2) = 12  6 < 0
f(x) = 2x3  x4  10 where x  [1, 1]
 There is a maxima at x = 2
 f(x) = 6x2  4x3 and
f(3) = 27  27 + 72 + 100 = 118
f(x) = 12x  12x2
f(3) = 27  27  72 + 100 = 28
f(x) = 0 gives 2x2(3  2x) = 0
 Minimum value of f(x) = 28
 x = 0. 1.5
13. 1.9 to 2.1 As x = 1.5 > 1, consider x = 0  [1, 1]
1 f(0) = 0  There is a neither maxima
x circumference
4 nor minima at x = 0
1 f(1) = 2  1  10 = 13
 xx4
4
f(1) = 2  1  10 = 9

time = = 2 sec  Minimum value of f(x) = 13
1.57
17. (D)
The function f(x) = x3 + 1 has a point of
inflection at x = 0, since in the graph
sign of the curvature (i.e., the
concavity) is changed.

14. (C)

lim x2  x  1 x
x 

= lim  x  x  1 x 
2
 x2  x  1  x 
x 
 x2  x  1  x

264
Solutions  Calculus

18. (B) 1  24992 


= 32  32
2 sin  
x 2 3  5 
 1  cos x  2
lim  2   lim 2 24992
x 0
 x  x 0 x =
15
x
sin2  
2 2
 2 lim 2    C  xy 2 dxdy = (24992)10 4
x 0  x  5
 4
R

 2  = 0.99968  1
1 1
 2  OR
4 2
5  2x 
19. (A)  R
xy 2 dxdy =   1 y 0
xy 2 dy dx

x 
lim f(x)  lim (x  1)  2  f(3)
x 3  x 3  2x
5
 y3 
x3 = 1  3  dx
x
lim f(x)  lim    2  f(3)
x 3  x 3 
 3  0

 f(x) is continuous at x = 3 5
8
3 1
=  x(x 3 )dx

20. 0.99 to 1.01 5


8  x5 
=   

2
xy dxdy 3  5 1
R

 R xy dxdy R xy dxdy
2 2
= 8 24992
 (3124)
1
15 15
5 2 5 2x
24992
   xy 2 dxdy    xy dx
2
=  C xy 2 dxdy = 10 4 6
x 1 y  0 x 1 y 2 R
15
5
 x2   y3  5  y3 
2 2x
2
=      x   dx = 2.4992
5
 2 1  3 0 1  3 2
5
= 0.9968  1
8 1
= (12)   x(8x 3  8)dx
3 3 1

1   x5   x2  
5 5

= 32 8   8   


3   5 1  2 1 



265
Vidyalankar : GATE – Engineering Mathematics

Model Solution on Assignment  3


1. 40 to 40 4. (C)
f  x  sinx 
((y 2  z2 )(2x)) Given, lim  
x x   x 
at x = 2, y = 1, z = 3 = (1 + 9) (4) = 40 1
Put x = and x   or h  0
h
2. (A)
1 1  1
t
f(t) = e  2e 2t
where 0 ≤ t <   hsin h   1hsin h 
Now, lim   = hlim  
h 0
 1  0  1 
 f(t) = e  2e2t(2) = 4e2t  et
t
 h 
and f(t) = et  8e2t
10
f(t) = 0 gives 4e2t  et = 0 = =1
1
 et (4et  1) = 0
5. 0.35 to 0.30
 et = 0 or et = 1/4
  sin x  0 
 t =  or et = 4  t = , t = n(4) lim    form 
x 0  2 sinx  x cos x   0 
f() = 0
  cos x 
 Neither maxima or minima at t =  = lim  
x 0  2 cos x  cos x  x sin x 

1 1 1 (L  Hospital Rule)
f (n4) = 8 =  0
4 16 4
1
 Maxima at t = n(4) =
3

3. (C) 6. (C)
/2
z = xyn(xy)xyn(x)xyn(y)  cos x  isinx 
I= 0  cos x  isinx dx
z
 = y[1n(x)]yn(y)
x /2
 eix  /2

0  eix dx 0 e
2ix
= = dx
= yyn(xy)

z  e2ix 
/2
1 i
and = xn(x)x[1n(y)] =  = [e e0 ]
y 
 2i  0 2i
= xxn(xy)
1
= [cos()i sin()1]
z z 2i
 x y
x y i
= [ 101]i
2i2

266
Solutions  Calculus

7. (A) 12
2 2 = 1 = 1  0.5774
The partial derivative of x y with 6
respect to y is 0 + 2y  2y. = 0.4226, 1.5774
The partial derivate of 6y + 4x with Now, x = 0.4226 < 1.
respect x is 0 + 4 = 4. Consider x = 1.5774
Given that both are equal. f(1.5774) = 6(1.5774  1) > 0
 2y = 4  Minima at x = 1.5774
 y=2 f(1) = 0 and f(2) = 0
 Maximum value is f(x) = 0
8. (C)
f(x) is discontinuous when 11. (A)

denominator = 0 3 sin x + 2 cos x

 (x2 + 3x  4) = 0 replacing sin x and cos x by its series,

 (x + 4) (x  1) = 0  x3 x5 
= 3  x  
 3! 5! 
 x = 4, 1
 x2 x4 
2 1  .....
9. (B)  2! 4! 
f(x) = x3  3x2 + 1 3x 3 3x 5
= 3x  .....
 f(x) = 3x2  6x = 3x(x  2) 3! 5!

 Between 1 & 0, f(x) > 0 i.e. f(x) is 2x 2 2x 4


2  ....
increasing 2! 4!

Between 0 & 2, f(x) < 0 i.e. f(x) is x3


= 23xx 2  .....
2
decreasing
above 2, f(x) > 0 i.e. f(x) is increasing 12. (A)
x  1  0 
lim   form 
10. 0.0 to 0.0  0  0 
f(x) = x(x  1) (x  2) = x3  3x2 + 2x Hence applying L Hospital rule
where x  [1, 2] Let y = x
 f(x) = 3x2  6x + 2 and f(x) = 6x  6  log y =  log x
2
 f(x) = 0 gives 3x  6x + 2 = 0 1 dy  d
 = log x
6  36  24 6  12 y dx x dx
x= =
6 6 dy  d 
 = x   log x
dx x dx 

267
Vidyalankar : GATE – Engineering Mathematics

Now applying L Hospital rule 1


 x=
 d  2
x   logx 
lim   x dx  = log x For x = 0,
 0 d
1
dx y2 =
4
1
13. 0.95 to 1.05  y=
2
1
and for x=
2
1
1
2
y = 2 = 1
4 8
1
 y=
8
Here rectangle is inscribed in ellipse.
Let the length and breadth of rectangle Now F(x) = 8  48 x2

be 2x and 2y respectively.  F(x) < 0 for

Now area of rectangle = (2x)  (2y) 1


x=
= 4xy 2

Let F = (Area of rectangle)2  F(x) is maximum at x =


1
= (4xy)2 2

= 16 x2 y2 The maximum area of rectangle


inscribed in ellipse is
 1 x 2 
= 16x 2  
 4   1  1 
4xy = 4   
 2  8 
 2 1x 2 
y  fromequationofellipse  4
 4  = =1
4
= 4x2 (1  x2)
Now F(x) = 0 14. 18 to 22
 x(1  2x2) = 0 1
2 
I =  (xy10)dxdy where D
 x=0 D

or 1  2x2 = 0 denotes x2 + y2 ≤ 4

 2x2 = 1 Use polar co-ordinates. Put x = r cos ,

1 y = r sin  dx dy = r dr d
 x2 =
2

268
Solutions  Calculus

2 2
1
2 r 0  (r cos r sin 10)rdrd
I= 
 0

2
1
 r(r sin r cos 100 dr
2

2 r 0
=

2
1
2 r 0
=  (10r2)dr

2
r2  18. (D)
= 10   = 10 × 2 = 20
 2 0
19. (C)
e
15. (D)
1
x In (x) dx

e
 3   3 
16. (A)   1 x2 
x2 
= In(x)        dx
P = 50q  5q2 3
 x 2 
 2 1  3 
dp d2p
 50  10q; 2  0
dq dq  3 3

e

= In(x)  x 2  2  4  x 2 
 p is maximum at 50  10q = 0 or,  3 9 1
q=5 2 3 4
= e 
Else check with options 9 9
20. (A)
Given
17. (A) 1
(sin1 x)2
The given curves are y = x and y = x2 I=  dx
0 1 x2
solving (1) and (2) we have 1
(sin1 x)3  f n1(x) 
=  f n (x)f(x)dx 
x = 0, x = 1 3 0 
n1 
1 1
 (x  x
2
Area = )dx = (sin1 )3 sin1 0 
0
3
1    
1 3
 x2 x3  1 1 3
=      =   0  =
3  2   24
 2 3 0 2 3

1
= sq units.
6



269
Vidyalankar : GATE – Engineering Mathematics

Model Solution on Assignment  4


1. (B) 4. (D)

1  cos x  sin(x / 2)  1
2
ax  b a b
lim = lim  lim = lim 
2  x  cx x  c x
x 0 x x 0
 (x / 2)  2
1 a a
= = 0 =
2 c c

 sin  
 xlim  1
5. (D)
 0  
ex  1
L im f (x) = L im =1
2. (D) x0 x0 x
Applying L’ Hospitals rule, we have 0
But f (0) = , which is not defined.
0
cos 
lim0  cos 0  1
1 Since L im f (x)  f (0) therefore f (x)
x 0

has removable discontinuity at x = 0.


3. (B)
x1/ 3  2
L = lim 6. (B)
x  8 (x  8)

 x2 
Let x = a3  a2 ex   1  x  
 2 
(a  2) lim
 L = lim
x 0 x3
a 2 a3  8
 x2 x3 
Now by partial fractions,  1  x    ...  
 2! 3! 
(a3  8) = (a  2) (a2 + 2a + 4)  
  1  x  x 
2

1 1   2 
 

 L = lim
a 2 2

a  2a  4 12 = lim  
x 0 x3
Alternate Method
1 x x2 1
= lim    ... 
x an n x 0 6 4! 5! 6
Since lim = nan1
x a xa Else, use L'Hospital's rule
1/ 3
x 2
L = lim 7. (A)
x 8 (x  8)
2
1 3 1 8. (A)
= (8) =
3 12
9. (C)

270
Solutions  Calculus

10. (D) 12. (C)

f (x) = x 3  6x 2  24 x  4 f (x) = (x  2)3

f ‘(x) = 3x 2  12 x  24 f(x) = 3(x  2)2

f ‘’(x) = 6x  12 f(x) = 0 for maximum and minimum.


For maxima and minima, f ‘(x) = 0 or x = 2
or 3x 2  12 x  24 = 0 When x > 2, f ‘(x) = 3(x  2)2  0

or x = 2  2i When x < 2, f ‘(x) > 0


 f (x) has no maximum and no Hence neither maximum nor minimum
minimum at any real value of (x). at x = 2.

11. (D) 13. (C)


Two parts are 30 and 30  x y = sin x (1 + cos x)

f (x) = x 3 .(30  x) dy
= cos x  sin2 x  cos 2 x
dx
f (x) = 30 x 3  x 4
= cos x + cos 2x
f(x) = 90 x  4 x
2 3 2
d y
=  (sin x + 2 sin 2x)
dx 2
f (x) = 180 x  12 x 2
dy
For maximum and Minimum f ‘(x) = 0 For maximum and minimum, =0
dx
or 90 x 2  40 x 3 = 0
cos x + cos 2x = 0
90 45 or 2 cos2 x + cos x  1 = 0
or x = 0, x = =
4 2
1
But, x = 0 is impossible, hence or cos x = , cos x =  1
2
45
consider only x = 
2  x = or x = 
3
2
 45  45  45 
f ''   = 180  2  12  2  But

 (0, ) and   (0, )
 2   
3
 45 
f ''   <0 d2 y
 0 at x =

 2  2
dx 3
45
 f (x) is maximum at x = = 22.5
2
 Two parts of 30 are 22.5 and 7.5

271
Vidyalankar : GATE – Engineering Mathematics

14. (C) 16. (A)


3 2
f(x) = 4x  8x + 1 in [1, 1] dV
 4r 2
f(x) = 12x2  16x dt

Equating f(x) to 0 4 3
V= r
3
 12x2  16x = 0
dV 4  2 dr  dr
3x2  4x = 0    3r   4r 2
dt 3  dt  dt
x(3x  a) = 0
dr
4  4r 2  4r 2
 x = 0, x= dt
3
dr
4 i.e. 4r 2  k 4r 2
Now, x [ 1,1]
 dt
3
dr
So x = 0   k
dt
f(x) = 6x  4
17. (D)
 f(0) = 4 < 0
f  (x) = 4x3  6x2  2x  4
 f(x) in not minimum at x = 0.
= 2 (x  2) (2x2 + x + 1)
Checking at endpoints :
Thus only critical number is x = 2
f(1) = 4(1)3  8(1)2 + 1
So f (0) = 3
=  4  8 + 1 = 11
f (2) = 9
f(1) = 4(1)3  8(1)2 + 1
f (4) = 99
= 4  8 + 1 = 3
18. (B)
 f(x) has minimum at x = 1 in [1, 1]
 x  2   3x 2   x 3
15. (B) f  (x) =
 x  2
3

4  dv dr
v= r   4r 2 2x 2  x  3 
3 dt dt  0
 x  2
2

dr
 4  = 4r2
dt x = 0 and x = 3 are critical.
dr 1 1 However x = 3 does not lie in interval
    0.01
dt r 2 100 so we list 0 and end points 1 and 1.
Since diameter = 2  radius f(0) = 0, f (1) = 1, f (1) = 1/3
Hence diameter is decreasing at the
rate of 0.02 cm /hr.



272
Solutions  Calculus

Model Solution on Assignment  5


1. (D) But L im f (x) = 1 (Given)
x
2
Lim ( x  1  x  1)  a = 1 (From eq. (2))
x 

x2
( x 2  1  x  1)  f (x) =
= L im( x 2  1  x  1) x 1
x 
( x 2  1  x  1)
2 2
(x 2  1)  (x  1) f (2) = = 0
= L im  2 1
x
( x2  1  x  1)

x (x  1) 3. (C)
= L im
x  1 1 1  Here
x  1 2  1 
 x x x   12 22 32 n2 
L im  3  3  3  .............  3 
(x  1)
n 
n n n n 
= L im
x   1 1 1   12  2 2  3 2  .....  n 2 
 1 2  1  = L im  
 x x x  n  n3
 
  n(n  1)(2n  1) 
= =  = L im 
01 
n
 6n3 

 2 2 2 2 n(n  1)(2n  1) 
2. (A) 1  2  3  ....  n  6 
 
ax  b  2n  3n  1
2
L im f (x) = L im = L im  
x0 x0 x 1 n
 6n2 
L im ax  b 1  3 1
 2  n  n2 
x0
= = b ….(1) = L im
L im x  1 n 6  
x0

2 1
But L im f (x)  2 (Given) = =
x0 6 3
 b=2 (from eq. (1))
4. (B)
ax  b
Also, L im f (x) = L im 2
x x x  1
 1
2n
  1 
n

L im  1     L im  1     e 2
n  n  n  n  
a  (b / x)  
= L im
x 1  (1/ x)

= a ….(2)

273
Vidyalankar : GATE – Engineering Mathematics

5. (C) 8. (B)
  dy dx
Put x  = t, so that, x = t 3  2x
2 2 dt dt
or 2x =  + 2t dx
When x = 3,  1foot
 / min
 dt
when x  then t  0
2 dy dy
3 6  2ft / min
1  cos 2x dt dt
Now L im
x  (  / 2) (  2x)2
9. (A)
1  cos(  2t)
= L im 4 3
t0 ( 2t)2 A = 4r2. v= r
3
1  cos 2t
= L im dA dr dv dr
t0 4t 2  8r  4r 2
dt dt dt dt
2 sin2 t
= L im dr 1 dA
t0 4t 2 10 = 4r 2  r 
dt 2 dt
2
 sin t  1 1 When r = 5,
= L im   . =
t0
 t  2 2
1 dA
10 = 5
6. (A) 2 dt
  dA
Put   = y, as   , y  0   4cm2 / h
2 2 dt
cot 
 L im
  (  / 2) ( / 2)   10. (A)
b2 = r2 + (h/2)2
tan(( / 2)  )
= L im
  (  / 2) ( / 2)   v = r2 h =  (b2  h2 /4) h
dv
= L im
tan y
dh
 
  b2  3h2 / h  0  h  2b / 3
y0 y
=1 d2 v 3h d2 v 
2
  2
dh 2 dh  2b / 3
7. (A)
3 2b
dy   b  ve
 7  3x 2 hence rate of change of 2 3
dx
Hence there is relative maxima at
d  dy  dx
slope is    6x
dt  dx  dt h = 2b/ 3
=  6  3 4 =  72 r=b 2/3

274
Solutions  Calculus

11. (D)  x (x2  4x +4) = 0


Let x be thickness of ice  x (x2)2 = 0
v = 300 [  (10 + x)2  100]  x = 0, 2, 2
dv  dx  d2 y
 300  x)  = 36x2  96x + 48
dt  dt  dx 2

dv At x = 0,
 2
dt
d2 y
= +48
dx 1 dx 2
 
dt 300 10  x  At x = 2,
1 1 d2 y
  = 36  4  96  2 + 48 = 0
300 10  2  3600 dx 2
12. (B) so only 1 local minima.
Let length be 2x and breadth be 2y either x = 0 or complex root
2 2
x + y = 1  dy/dx = x/y so only value = 0

A = (2x) (2y) = 4xy


14. (D)
dA 4
  
x 2  y 2  0  /4
1  tan x 
dx y 0 1  tan x  dx
 x=y=1/ 2
/4
cos x  sin x
d A 2
4x
 3
I = 0 cos x  sin x
dx
2
dx y
Let t = cos x + sin x
2
d A 4  1 2
   8  dt = (sin x + cos x) dx
dx 2 1
 
3
2,1 2 1 2
dt cos x  sinx
 = dx
t cos x  sinx
d2 A
 = 16 /4
dx 2 1 dt
 lnt 0
/4
2,1 2
 I= 0 t
1 1
 Area = 4xy = 4   2
 ln cosxsinx 0
/4
2 2

13. (B)  ln cos / 4sin / 4 


4 3 2
Given : y = 3x  16x + 24x + 37
For maximum value or minimum
 ln  2   ln2 1/ 2 1
 ln2
2
dy
= 12x3  48x2 + 48x = 0
dx

275
Vidyalankar : GATE – Engineering Mathematics

15. (B) 18. (D)


If 2 points on line are A1(0, 1) and 
 x2 2

A2(1, 0)
Let I = 0 0 e e  y dxdy

y  y 1 y1  y 2 
 Equation of line is   (x 2  y 2 )
x  x 1 x1  x 2  I= 0 0 e dxdy

y 1 1 0
 Put x = r cos 
x  0 0  (1)
y = r sin 
y 1
1 Then J = r
x
Now 0  x   and 0 ≤ y  
y=x+1
2 2
 region of integration is the first
I   y dx =  (x  1)dx quadrant.
1 1
 0r, 0    /2
2
 x2  1 
=   x  = (2  2)    1 /2 
r 2
2 1 2   I= 0 0 e  rdrd

3 5
= 4 = = 2.5 /2  
2 2  r 2 2r
0 d 0 e 0 e
r
= rdr = dr 
2 2

16. (B)  r 2  2 
=  e  2r dr =  er 
1 1 1 1 40 4 0
0 e dx = x 0 e dx 0 1 0 e dx
x x x


=  0  (1)
4
 xe   e 
1 1
x x
= (e  0)  (e  1) = 1
0 0 
 I=
4

17. (B)



276
Solutions  Calculus

Model Solution on Assignment  6


1. (A) 3. (B)
Here f (0) = 0 Here f (x) = 1
Right Hand Limit Also, Left Hand Limit
 |x|  L im 1 = 1
lim f (x) = lim   f (x)  3 
x 0 x 0  x 
Right Hand Limit
x
= lim   = 1 L im (ax  b) = 3a + b
x 0  x  x 3 

|x|  x if x  0  Also, f (3) = 1


   x if x  0 
 Since f (x) is continuous, therefore
Left Hand Limit 3a + b = 1 ….(1)
 |x|  Also, f (s) = 7
lim f (x) = lim  
x 0 x 0  x  and L im  f (x) = L im (ax  b)
f (x)  5 x 5

 x 
= lim   =  1 = 5a + b
x 0  x 
Since f (5) is continuous at x = 5,
 R.H.L.  L.H.L.
therefore
The function f (x) is discontinuous at
5a + b = 7 ….(2)
x = 0 and the discontinuity at x = 0 is
Solving eq. (1) and eq. (2), we get
of first kind because R.H.L. and L.H.L.
a = 3, b =  8.
both exist finitely but are unequal.
4. (D)
2. (A) The given function is of straight line
Here f (1) =  1 hence no maximum or minimum.
Also, Right Hand Limit
5. (B)
L im f (x) =  1
x 1 y = log x

Left Hand Limit dy 1


=
dx x
L im f (x) = a  2
x 1
Since log x is defined only for +ve
Now, L.H.L. = R.H.L. values of x.
 a2=1  a=1 1 dy
So, >0  > 0  y = log x is
x dx
increasing.

277
Vidyalankar : GATE – Engineering Mathematics

6. (A) 8. (B)
y = log (sin x) The series 1 + 2 + 3 + ….. is an A.P
dy 1 d 1
= (sin x) Sn  n  n  1
dx sinx dx 2
1
=
cos x
= cot x lim Sn  lim n  n  1  
n n 2
sin x
hence the series is divergent.
 
For the interval  0,  , cot x is +ve
 2 9. (C)

dy   The series 1  2 + 3  4 +  6 + . ……
  0 on  0,  .
dx  2 is an alternatory series

  Sn = (1  2) + (3  4 ) + (5  6) +. . . .+
 log (sin x) is increasing on  0, 
 2 [(n  1)  n]
Again cot x is negative on the interval 1
= (1) + (1) + ........... + (1) = n
  2
 2 , 
   1 
lim sn  lim   n   
dy
n n
 2 
 < 0
dx Series can also be written as
 log (sin x) is decreasing. Sn = 1  (2  3)  (4  5)  (6 7)……..
 ((n1)  n))
7. (A)
n1
1 1 1 = 1 + 1 + 1 + 1 ………..+ 1 =
The series 1   2  3  ..... is a G.P 2
2 2 2
1
lim Sn  lim n  1  
11   1/ 2   2   1n 
n n n 2
Sn =    1
 
 1  3   2   Since limit does not exist because the
1  
 2 sum of infinite forms of series is not
unique
2   1 
n

lim S n  lim 1     
n 3
  2  
n
10. (A)

2 2 The series is alternatory series


 1  0 
3 3 1 1 1
1 >   .......
Hence the series is convergent 2 3 4
1
lim 1n  lim 0
n n n
here the series is convergent

278
Solutions  Calculus

11. (B)  
 
n 1
Here Un = = lim  
n  
1  2 n 1
 1 3/ 2  1 
 n 
n
lim Un  lim 0
n n 1  2 n 1 1
 
1 1 2
hence the series is divergent.
Hence it is convergent.
12. (C)
15. (B)
The infinite series
1
1 1 1 here Un = Sin
 p
 p  p  .....is n
n 1 2
3
i) convergent of p > 1 1 1  1 1 1
    ...   3  .....
n 3n n 6n
ii) divergent of p  1
1
Take Vn =
13. (A) n

1 1 1 Vn 1 1  n
The series 1    ......is a G.P. lim  lim   3  ...  
3 9 27
n Vn n 
 n 6n  1

Where a = 1 ,  1 
 lim  1  2  ...   1
1
n
 6n 
r=  Which lie between 1 and 1
3 series Vn n a divergent. Hence  Un
Hence the given series is convergent. is also divergent.

14. (A)
16. (B)
Un = n3  1  n3
n1 n 1
Un = , Take Vn = 2 
1 n2
n n

n3  1  n3
  n  1 
1 Vn  2 
n 
Take Vn = lim  lim   1
n3/ 2 n V
n
n  1 
 n 
 1   
U  
3 3
By comparison test Un &  Vn behave
lim n  lim  n  1  n 
A  V n  1 
n alike.
 n3/ 2 
 
1
Now Vn =  is a divergent series
n
hence Un is also divergent.

279
Vidyalankar : GATE – Engineering Mathematics

17. (C) 18. (C)


Since f(x) = 1/x is not continuous in f(x) = x3  f  (x) = 3x2
[ 3, 3] [4, 2 ] or [1, 1] the point of f (b)  f(a) = (b  a) f  (c)
discontinuity is 0. only in [ 2, 3] the b3 a3 = (b a)  3c2
function is continuous and differentiable.
b  a   a2  ab  b2 
 3c2 =
ba

a 2  b2  ab
c=
3



280
Solutions  Calculus

Model Solution on Assignment  7


1. (B) 4. (B)

f (x) = e (1/ x)  
f()  5 cos   3 cos      3
  1  3 
f ‘(x) = e (1/ x)  2 
x   
(1/ x) 2
f()  5 sin   3 sin    
Since e > 0 and x > 0,  3
for all x  0  
f()  5 cos   3 cos    
 f ‘(x) < 0, for all x  0, x > 0  3

 f (x) is decreasing in the interval Equating f() to 0


[0, ]  
5 sin   3 sin      0
 3
Hence f (x) = e 1/ x , x > 0 is a
decreasing.  
 5 sin   3 sin      0
 3
2. (C)
 1 3
1  5 sin   3 sin    cos   0
f (x) = x  2 2 
x 
1 2
f (x) = 1  2 ; f(x) = 3 3 3
x x  5 sin   sin   3 cos   0
2 2
for maximum and minimum, f(x) = 0
13 3 3
1  sin   cos   0
or 1  2 = 0 2 2
x
3 3
or x = 1  tan   
13
f( 1) =  2 < 0
 3 3 
 f (x) is maximum at x =  1.    tan1 
 13 
 
3. (A)
  21.79
f (x) =  x 2  2x  1
f(21.79) < 0
f ‘(x) =  2x + 2; f ‘’(x) =  2
 maximum at  = 21.79
f ‘(x) = 0
f(21.79) = 5 cos (21.79)
[for maximum and minimum]
+ 3 cos (21.79 + 60)
 2x + 2 = 0
f(21.79) = 10
x = 1
f (x) =  2 < 0
 f (x) is maximum at x = 1

281
Vidyalankar : GATE – Engineering Mathematics

5. (A) 9. (B)

f (x) = x 3  12 x 2  45 x  11 let f(x) = ex. Then fn(x) = ex.


Hence fn (o) = 1 for all n  0
f (x) = 3x 2  24 x  45

f n (0) n 
xn
f (x) = 6x  24  Maclaurin series 
n  0 n!
x  
n  0 n!

f (x) = 0 or 3x 2  24 x  45  0
10. (C)
x = 3, x = 5 Let f(x) = sin x.
f (5) = (6  5)  24
Then f(/4) = 2 / 2.f    / 4 
=6 > 0
 2
 f (x) is minimum at x = 5.  2 / 2f    / 4  
2

6. (D) f    / 4  
  2  .Thus
2
7. (A) Taylor series for sin x about /4 is
Using lagrange’s mean value theorem
2  x   / 4  x   / 4
2

f  x   f  0 1  
e 1x
2  1! 2!
f  (c) = or ec = ….(i) 
x0 x
 x   / 4 
3
0 c x
Now 0 <<< x  e < e < e .…(ii)   ......
3! 
From equation (i) and (ii)
ex  1 11. (A)
1  ex
x Let f(x) = ln (1x). Then f(0) = 0.
x x
 x <e 1 < xe f (0)  1,f (0)  1,f   0   12
x x
 1 + x < e < 1 + xe
f(4) = 1.2.3 and fn (0) =  (n 1) !

8. (B) Thus for n  1 fn (0)/n! = 1/n and


x2 x3 x4
x1/x is decreasing function then Maclaurin series is x    ...
2 3 4
 1  loge x 
f  (x) = x1/x   12. (B)
 x2 
1
f(x) is decreasing if f  (x) < 0 Let f(x) = Then
x
 1  log x  1 2 2.3
i.e. x1/x  x 2   0 f   x   2 ,f   x   3 ,f   x   4
x x x
234
 1  loge x < 0  loge x > 1 f4  x  andingeneral
x5
x>e
n!
f n  x    1 sof n 1   1 n!
n n
n 1
x

282
Solutions  Calculus

Thus Taylor series is Hence


x f 1
n x x4 x6
  x  1    1  x  1 ln (1 + x2) = x 2 
n n n
  .....
n0 n! n 0 2 3

13. (A) x 2n
  1
n 1
.....
Let f(x) sec x. Then n

f   x   Secxtan x, f   x  x 2n
Thus term is  1
n 1

n

 sec x 1  2 tan2 x  17. (B)
f   x   sec x tan x(5  6 tan x) 2
1 x
f(x) =
1 x3
 f(0)  1f   0   0,f   0   1

1
from   xn
f (0)  0,f  5 4
1  x n 0
The Maclaurin series is 1 1
&sof  x   x
1 2 5 4 1 x3 1 x3
1+ x  x  ....
2 24  
=  x 3n
  x 3n1
14. (D) n 0 n 0

In general, From Maclaurin expansion,


f 0
n
f 36  0 
an = .so f 33  0   33!a 33  33!233  a 36  1
n! 36!
15. (B) Hence f36(0) = 36!

x 2n1
tan1 =   1
n
 18. (A)
n0 2n  1
Let f(x) = tan x . then
fn  0 
Sincean  ,f 99
 0   99!a99 f (x)  sec 2 x,f   x   2 tanx sec 2 x
n!
1 1 f (x)  2(sec 4 x  2 tan2 x  tan4 x)
But a99 = (1)49 
99 99
So f(0) = 0,
Thusf 99
 0     99! / 99  98! f   0   1,f   0   0,f   0   2
16. (A)
Thus Maclaurin series is
2 3
x x 1 3 2 5
ln (1 + x) = x    ..... x x  x  ...
2 3 3 15
xn
  1
n 1
 ...for | x | 1
n


283
Vidyalankar : GATE – Engineering Mathematics

Model Solution on Assignment  8


1. (C) Right Hand Limit :
2
f  (x) = 1 + k x h h
lim   lim …  h  h for h  0 
For x = 2 to be critical h 0 h h 0 h

f  (2) = 1 + k (2)2 = 0 =1

 k = 4 h h
 lim   lim 
Hence h 0 h h 0 h

4 8 h
f  (x) = 1  ,f (x)  3  lim does not exist.
x 2
x h 0 h

f  (2) = 1 4. (D)
Hence relative maximum at x = 2 f x  2
lim  0  lim f  x   2
2. (C) x 1 f x  2 x 1

Let x and y be dimension  f(x) = 2x


2x + 2y = 16  A = x(8  x) = 8x  x2 hence lim f  x   2
x 1
2
dA d A
 8  2x, 2  2
dx dx 5. (D)

d2 A  6. (D)
Critical number is x = 4    2
dx 2  4
1
Let A = lim nlogn
here x = 4 and y = 4 n 0 e
1
3. (D)  loge A  lim logee
n 0 nlogn
x2
lim 1/ n 1/ n2
x 2 x2 = lim  lim
n 0 logn n 0 1/ n
Put x  2 = h
1
as x  2, h0  lim  
n 0 n

x2 h
 lim  lim 7. (B)
x 2 x2 h 0 h
Let f(x) = nx . then
Left Hand Limit :
1 1
h h f  x   ,f   x    2 ,f   x 
lim  = lim x x
h 0 h h 0 h

2 4 2.3
…  h  h for h  0   3
f  x   4 & in general
x x
= 1

284
Solutions  Calculus

n  1 ! 11. (A)
fn(x) = (1)n+1 xn
f   x   4x  4,f   x 
So f(2) = n2 ,
 4 & f n  x   0forn  2
n  1 !
n n+1
f (2) = (1) 2n Thus f(1) = 3,
f (1)  8,f  1  4 & f  x 
 fn  2
  x  2
n
Thus Taylor series is
 3  8  x  1  2  x  1
2
n 0 n!

x2 1
  8  x  2   .......
2
  n2  
 2  12. (A)

8. (C) 13. (B)



2x = exn2 I= 0 log 1  cos     d

xn
Now, ex=  .
n  0 n!
2I =

0 log 1  cos   d

  n2    log 1  cos   d
n

Therefore x
2 = e xn2
= 
n 0 n!
x n 0

0 log 1  cos   d

2
=
9. (C)


1  2  log sin d
ex =  xn 0

n  0 n!
  
= 4   log2 
Hence   
1 2n f  0 
 100
1 I = log 2
f x   x ,  G100 
n 0 n! 100  50 !
14. (A)
100 100!
Hence f (0) =
50! /2    
I= 0 sin2   x  log tan   x  dx
 2   2 
n! 2
f n (0) = for f(x) = ex
(n 2)! /2 sin2x log(tanx) 
2I = 0   dx
 sin2x logcot x 
10. (D) /2
= 0 sin2x log  tanx cot x  dx
The Maclaurin series for f(x) is the
/2

polynomial 2  x 
4 2 12 3
x  x = 0 sin2x log(1)dx  0
2! 3!
Thus f(x) = 2 + x + 2x2+ 2x3

285
Vidyalankar : GATE – Engineering Mathematics

0  2x  x 
15. (C) 1
= 2
 x 3 dx
N D
1 2x  x 
C 0
B M 2  2
 x 3 dx
x = 4y
1 0
A O E x3 x4 x3 x4 3
= x2    x2   =
3 4 0
3 4 1
2
Required area
2  x  2 x 2  18. (B)
BOD =  1
 4 4
 dx
 x  sin x
2
I=  1  cos x dx
1  x2 x3  9
=   2x    x sinx
4 2 3  1 8  dx   dx
1  cos x 1  cos x
16. (B) x x
2 sin cos
The given curve passes through the x 2 2
=  dx  
2 sin2 x / 2 2 sin2 x / 2
3
point (2, 3) and (4, )
2 x x x
 2 cos ec dx   cot dx
2
=
2 2
4  8 
Required area = 2  1  x2  dx x
=  x cot C
2
4
 8
= x    4
 x 2

17. (C)
Required area

1,0 0, 0 x =1 (2,0)
x = 1



286
Solutions  Calculus

Model Solution on Assignment  9


1. (B) log  cosecx 
 lim
2x 2x x 0 1
 x   1 x 
let A = lim    lim   x
x  1  x
  x 
 x 
2x
ByLHospitalrule
 1
= lim  1  
1
x 
 x   cosecx cot x 
= lim cosecx
2
 1 
x x 0 1

  lim  1    x2
 x   x  
x2
1  lim = lim 2x / sec 2 x
= [e]2 = x  0 tan x x 0
e2
ByLHospitalrule
2. (B)  lim 2x cos2 x
x 0
tan x
 1 loge A = 0  A = e0 = 1
A = lim  
x  x
 
 loge A  lim tan x log1/ x 4. (C)
x 0

log1/ x 1 2 1 n  n  1 / 2
= lim lim  2  2  ...    lim
x 0 cot x n n n n  n n2


 lim

x 1/ x 2  1 1
= lim  1 
x   cos ec x 2
n 2 n
ByLHospitalrule 1
 1  0   1/ 2
2
sin2 x
= lim
x 0 x
5. (B)
2 sinxcos x
 lim 0 L.  lim f  x   lim 2a  2  h  b  4a  b
x 0 1 x 2 h 0

ByLHospitalrule R. lim f  x   lim a  2  h   b  4a  b


x 2 h 0
2

loge A = 0  A = e0 = 1
L. lim f  x   R lim f  2 
x 2 x 2

3. (A) 4a  b = 2 and 4a + b = 2
A = lim  cosecx 
x
 a = 1/2, b = 0
x 0

loge A = lim x log  cosecx 


x 0

287
Vidyalankar : GATE – Engineering Mathematics

6. (D) 1
=  cos ec  x   / 4  dx
 1  2  ....  n  n  n  1 2
lim    nlim
n
 n   2n 1 x 
 log tan     C
n1 2 2 
 
2
10. (C)
7. (B)
1 sec 2 x dx
/2 I =  1  3 sin2 x  sec2 x  3 tan2 x

I= 0 cos4 x sin3 x dx

/2
(Divide numerator and denominator by
= 0 cos4 x sin2 x sin xdx
cos2x)

  sec 2 xdx
/2
 0 cos4 x 1  cos2 x sinxdx =  1  tan2 x  3 tan2 x
let cos x = t
1 sec 2 xdx
4   1 2

1 t 1  t  dt
0
I= 4 2
2
 2   tan x
 
1
 t5 t7  2
    Put tan x = t
 5 7  0 35
1 dt
= 
4 1/ 2 2  t 2
8. (A)
Put x = sin  tan1 2t
 C
1  sin2  2
I =  1  sin2 
cos d
tan1  2 tan x 
= C
2
=  1  sin2  d 
11. (A)
 1 sin2
=+  C
2 2 2 sec 4 
I =  tan4  1d
3 1 1
= sin x  x 1  x 2  C
2 2 1  tan  sec
2 2

  1  tan 4
d
9. (A)
Put tan  = x
1 dx
I=
2
 1 1 1  x  dx  1  1/ x  dx
2 2
cos sinx
2 2 I =  1 x4  1
2

1 dx x  x   2
 
 
2 sin  x   / 4 
1  1
Put x   t   x  2  dx  dt
x  x 

288
Solutions  Calculus

dt 14. (C)
I = 
 2
2
2
t  For x  (0, /2), 0 < sin x < x
sinx nsin x
1 t  x  1/ x  0  1 0   n …(1)
 tan1  tan  C x x
2 2  2 
 nsin x 
  x   n 1 …(2)
1  tan2   1  
= tan1  C
2  2 tan   Divide by x
sin x
1 0 …(3)
12. (B) x
sin xdx nsin x
I =  3 sinx  4 sin 3
x
 0
x
n

dx  nsin x 
   x   n 1 …(4)
3  4 sin2 x  
dx (2) & (4)
  3 cos 2
x  sin2 x  nsinx 
lim    n1
2
sec xdx x 0
 x 
=  3  tan2
x
Put tan x = t 15. (A)

 t sin  2t  dx   t  dy  t dz


3 2
dt 1  3 t I=
I =   log  C
3t 2
2 3  3  t  C
 
dx
 2 cos  2t 
1  3  tanx  dt
= log  C
2 3  3  tanx 
  dx  2 cos  2t  dt

Since dy = dt and dz = 2tdt


13. (D)
 t sin2t   2 cos2t 
1

Integrating by parts I =  t 3


 2t 3
 dt
1  
x
2
I= sinxdx 1 1
= 2  t sin  4t  dt   t 3 dt
  x 2 cos x   2x cos xdx 1 1

= 1
= x2 cos x + 2x sin x  2  sin x dx
/ 2
=  x2 cos x + 2x sin x +2 cos x + C 0

=2

289
Vidyalankar : GATE – Engineering Mathematics

16. (D)
1 1 x2

= lim
e  x log x xn1 x 0
x 1 1 x2
In = 
0
u dv
dx

dx x2
= lim
 x 0
 xn  x 1 1 x2
In = e  x log x 
 n 0 x
= lim ,
x 0 2

e x
x n x 1 1 x
   e x log x  dx
0
x  n  xifx  0
because x 2
 xifx  0
1 n  1 !
=0+ In+1 
n n 1
=
 2
 e x xn1dx = (n  1)!
1
0
But f(0) = 
2
1 n  1 !
 In = In+1  ….(1)  F is not differentiable of x = 0.
n n
Replace
18. (B)
n  1 !
n  n + 1 : (n+1) In + 1  .…(2)
n z z
 5y  8x  2 ,  5x  2y  1
x y
(1) & (2) : In + 2 – (2n+1)In+1 + n2In = 0
z z 1 18
 0, 0 x= ,y=
x y 41 41
17. (B)
f(0) = 0 Here r = 8, s = 5, t = 2

lim f  h   0  rt  s2 < 0
x 0
 1 18 
 f(x) is continuous at x = 0   ,  is a saddle point.
 41 41 
1 1 x2  0
f(0+) = lim
x 0 x



290
Solutions  Calculus

Answer Key on Test Paper  1


1. (B) 2. (D) 3. (A) 4. (D)

5. (A) 6. (A) 7. (B) 8. (D)

9. (B) 10. (C) 11. (B) 12. (D)

13. (B) 14. (B) 15. (B)

Answer Key on Test Paper  2


1. (D) 2. (B) 3. (A) 4. (D)

5. (A) 6. (D) 7. (C) 8. (D)

9. (D) 10. (D) 11. (B) 12. (A)

13. (D) 14. (D) 15. (A)

Answer Key on Test Paper  3


1. (C) 2. (D) 3. (A) 4. (B)

5. (A) 6. (B) 7. (C) 8. (A)

9. (A) 10. (B) 11. (A) 12. (A)

13. (A) 14. (B) 15. (C)

Answer Key on Test Paper  4


1. (C) 2. (C) 3. (D) 4. (D)

5. (C) 6. (B) 7. (B) 8. (C)

9. (D) 10. (A) 11. (D) 12. (B)

13. (A) 14. (D) 15. (C)

291
Vidyalankar : GATE – Engineering Mathematics

Answer Key on Test Paper  5


1. (D) 2. (B) 3. (B) 4. (A)

5. (A) 6. (A) 7. (B) 8. (B)

9. (B) 10. (C) 11. (D) 12. (B)

13. (C) 14. (D) 15. (A)

Answer Key on Test Paper  6


1. (C) 2. (A) 3. (A) 4. (B)

5. (D) 6. (B) 7. (C) 8. (C)

9. (B) 10. (C) 11. (A) 12. (B)

13. (C) 14. (A) 15. (C)



292
Solutions  Calculus

Model Solution on Test Paper  1


1. (B) 5. (A)
3 sinx  4 sin3 x  sin x Put 1 + x = y, so that x  0  y  1
Lim
x 0 sinx (1 x)n  1 yn  1
=
= Lim(2  4 sin2 x) = 2 x y 1
x 0

(1 x)n  1 yn  1
2. (D)  L im = L im
x0 x y 1 y  1

1
Limsin does not exist because = n(1) n  1
x 0 x
1 =n
sin oscillates.
x
6. (A)
3. (A)
7. (B)
sin3x sin3x 3x
Now, L im = L im .
x0 5x x0 3x 5x
8. (D)
sin3x 3
= L im . 
1

1
2  nx 
2
x 0 3x 5 2 x n2 x   dx
x
3 3
=1× = 
=     nx  
1
5 5
 2
4. (D)
1
=    n      n2  
1 1
Left Hand Limit,
 n2
L im [x] = L im [2  h]
x 2 h 0
9. (B)
= L im 1
h 0 There is discontinuity at x = 2. Thus
=1 5
dx
2
dx
5
dx
Right Hand Limit,

1
3
x2
 
1
3
x2
3
2 x2
L im [x] = L im [2  h] 2 5
3 2/ 3  3 2/3 
x 2 h 0
   x  2  +   x  2 
2 1  2 2
= L im 2
h 0

=2 
3
2
 3
9 1 
 L.H.L. L im [x]  R.H.L. L im [x]
x  x 2

293
Vidyalankar : GATE – Engineering Mathematics

10. (C) 13. (B)


1 1 
x 1 2x 1 1 
A= 0 1 x 2
dx  
2 0 1  x 2
dx A=   x  x  1 dx
1


1/ 2 1    n x   n x  1 1
  1 x2
    =1
 0
= n2

11. (B) 14. (B)


By Stoke’s formula There is a discontinuity at x = 2
 

 
2 3 2 3
V =  e 2x dx  e4x dx dx dx dx
0 0
So 1 x  2  1 x  2  2 x  2
v
    n x  2 1    n x  2 2
v 2 3
 1
=  lim  e 4x dx   lim   e 4x  
v 
0
v 
 4 0
= neither limit exists.
 
=  lim e 4v  1 
4 v  4
  Therefore integral diverges.

15. (B)
12. (D) By successive application of L'Hospital
There is discontinuity at x = 1. Then
  nx 
p

9
dx
1
dx
9
dx rule, lim 0.
x
  x  1   x  1  x 
=
 x  1
2/3 2/3 2/3
0 0 1
  nx 
p

Hence  1 for sufficiently large


1/ 3 1 9
x
= 3  x  1   3  x  11/ 3 
0 1
x. Thus for some x0.
=3+6=9
1 1
 n x
p
If x  x0  x,  .
  nx 
p
x

 
dx dx
So   nn x 
e
p
  nn x   .
e

Hence the integral is divergent.



294
Solutions  Calculus

Model Solution on Test Paper  2


1. (D) L im f (x)  f (2)
x2
Left Hand Limit,
 f (x) is continuous at x = 2.
L im f (x) = L im 2 x = 4
x 2 x 2
3. (A)
Right Hand Limit,
Left Hand Limit,
L im f (x) = L im x 2 = 4
x 2 x 2 L im f (x) = L im(1 h)  [1 h]
x 1 h0

Thus, L im f (x) = 4
x2 = L im (1  h)  0 = 1
h0

But,f (2) = 2  L im f (x)


x2 Right Hand Limit,
Thus the given function is L im f (x) = L im (1 h)  [1 h]
x 1 h0

discontinuous at x = 2.
= L im (1 h)  1
But if we change f (x) = 4 at h0

x = 2, then the given function f = L im h = 0


h 0

becomes continuous at x = 1.
L.H.L. L im f (x)  R.H.L. L im f (x)
x 1 x 1
Hence 2 is the removable
discontinuity. f (x) = x  [x] is discontinuous at x = 1.

4. (D)
2. (B)
At x = a (say), where ‘a’ is any rational
Left Hand Limit,
number.
L im f (2  h) = L im (2  h  1)
h0 h0
Given, f (x) = 1, when x is rational.
=1 ….(1) Left Hand Limit,
Right Hand Limit, L im f (x) = L im f (a  h)
xa ha
L im f (2  h) = L im  [2(2  h)]  3 
h0 h0 = L im ( 1)
h0
=1 ….(2)
(because a  h is an irrational number)
From eq. (1) and eq. (2),
Right Hand Limit,
L.H.L. = R.H.L.
L im f (x) = L im f (a + h)
L im f (x) exists. x a h0
x2

= L im ( 1) =  1
f (2) = value of f (x) h0

=2.23=1 But, f (a) = 1

295
Vidyalankar : GATE – Engineering Mathematics

L.H.L. L im f (x) = R.H.L. L im f (x)  f (a)


xa x a
= tan1    3   tan1  0  3 

 f (x) is discontinuous at x = a, 
  tan1 3
2
where ‘a’ is a rational number.
Similarly we can prove that at x = b (an 8. (D)
irrational number) f (x) is discontinuous. 
dx 1 xa

 f (x) is discontinuous at every real


A= a 1 x2  a2  2a  n x  a a 1
point.
1
=  n 2a  1
2a
5. (A)

Left Hand Limit, 9. (D)


1 1
L im f (4  h) = L im 4(4  h)  3 1 1
h0 h0
x
0
k
dx  lim
v 0  x
v
k
dx
= 19 ….(1)
1
Right Hand Limit, 1 
 lim x  k 1 
v  0  k  1
0
L im f (4  h) = L im 4(4  h)  3
h0 h0
1  1 
= lim 1  k 1  .
= 19 ….(2) v 0  1  k  v 
From eq. (1) and eq. (2),
If k > 1 this limit is + whereas if k < 1
 L im f (x) = 19
x 4 1
the limit is
Also, f (h) = 3 . 4 + 7 1  k 
= 19
L im f (x) = f (4) = 19 10. (D)
x4
a a
 f (x) is continuous at x = 4. dx 1 ax
0 a2  x 2 = 2a  n a  x 0
6. (D) 1
=  n a  x
/2
/2 2a
I= 0 cot xdx  ln  sinx  0 =+
a
1 
  n  a  x 
2a 0
7. (C)
=+
 
dx dx
A=  2   x  3 2 Thus the integral diverges.
0 x  6x  10 0 1

 tan1  x  3  
0

296
Solutions  Calculus

11. (B) 14. (D)


The value of mod function cannot be f(x) = x5  5x4 + 5x3  1
less than 0 f  (x) = 5x4  20x3 + 15x2
f(x) = | x2  5x + 2 | = 0 f  (x) = 20x3  60x2 + 30x
f  (x) = 60x2  120x + 30
12. (A)
for maxima or minima f  (x) = 0
 sinx  cos x  x4  4x3 + 3x2 = 0  x = 0, 1, 3
let f(x) =  sin  x    4 
2
f  (0) = 0, f  (0) = 30  0
since maximum value of sin  is 1
f  (1) =  10 < 0
 maximum is 1
f  3 = 90 > 0

13. (D) Hence f (x) is neither maximum and


minimum at x = 0 f (x) is maximum at
let y = xx
x = 1 and minimum at x = 3
log y =  x log x
Hence 1 maxima and 1 minima
dy/dx = 1/xx (1 + log x)
d2y/dx2 =  [ y/x + (1 + log x)
15. (A)
{  y (1 + log x)}]
f(x) = x3/3  3x2/2 + 2x
for maxima and minima dy/dx = 0
f  (x) = x2  3x+ 2
 1/xx (1 + log x) = 0  x = 1/e
f  (x) = 2x  3
 
d2 y  1  2 1 For maxima minima
2 
 0  
dx  x  1  1 1/e  1 f  (x) = 0 x3  3x + 2 = 0
e e  e 
   (x  1) (x  2) = 0
e1/e
 e   0  x = 1, 2

Hence maximum at x = 1/e f  (1) = 2  3 = 1 < 0

1 f  (2) = 2  23 = 1 > 0


Maximum value = 1/e
 e1/ e
 1 f(1) = 1/3  3/2 + 2 = 5/6
e
  f(2) = 8/3  6 + 4 = 4/6 = 2 / 3



297
Vidyalankar : GATE – Engineering Mathematics

Model Solution on Test Paper  3


1. (C) a x  bx
 L im
x 16  1
x0 x
L im
x 1 x5  1 a x 1 b x 1
= L im  L im
x 0 x x 0 x
 x 16  1 x 5  1 
= L im    a
x 1
 x 1 x 1 = log a  log b = log  
b
x 16  1 x5  1
= L im  L im 4. (B)
x 1 x 1 x 1 x  1

Left Hand Limit,


= 16  5
|0  h|
16  xn  an  L im f (x) = L im
=  L im  na n  1  x 0 h0 0 h
 x 0
5  xa 
h
= L im =1
h0 h
2. (D)
Right Hand Limit,
x2  4
L im |0  h| h
x0
x  2  3x  2 L im f (x) = L im = L im = 1
x 0 h0 0h h0 h

x2  4 x  2  3x  2  L.H.L. L im f (x)  R.H.L. L im f (x)


= L im x0 x0
x 2 x  2  3x  2 x  2  3x  2
Hence f (x) is discontinuous function of
(x  2)(x  2) ( x  2  3x  2)
= L im first kind at x = 0.
x2 (x  2)  (3x  2)

(x  2)(x  2) ( x  2  3x  2)
= L im 5. (A)
x2  2 (x  2)
Left Hand Limit,
(x  2)( x  2  3x  2)
= L im f (0  h)  f (0)
x2 2 f ‘(0) = L im
h0 h
4 ( 4  4) 1
= =8 = L im h sin =0
2 h0 h
Right Hand Limit,
3. (A)
f (0  h)  f (0)
ax  bx (a x  1)  (b x  1) f ‘(0) = L im
Now, = h0 h
x x
1
h2 sin 0
a x 1 b x 1 h
=  = L im
x x h0 h

298
Solutions  Calculus

1 2y
= L im h sin = 0 fy = ,
h0 h x  y2
2

 L.H.L. f ‘(0) = R.H.L. f ‘(0) = 0


fyy 
x 2
 y2  2   2y  2y   2  x  y 
2 2

x  y  x  y 
2 2
2 2 2 2
6. (B)
Hence fxx + fyy = 0
x
y= x
e
9. (A)
log y = log x  x
By chain rule
1 1 dy 1  dz dx dy
dy / dx   1   y   1  0
y x dx x   fx  fy
dx dx dx
x=1
2
 
 4x 3  3y   3x  2y  cos x
2
1 d y 1  dy  1
 2
y dx 2 y 2  dx  x 10. (B)

1 d2 y  d2 y  dz z dx z dy
     
 0 1   e  0 dt x dt y dt
e dx   x 1 dx 2  x 1

Hence maxima at x = 1 x dx 2y dy dx
  since 5
8 dt 9 dt dt
1
Maximum value  ymax = and
e
dy
5x + y 0
dt
7. (C)
sowhenx  2andy  1,
Assume that there is such a function.
Then fxy and fyx will be continuous dy / dt  10and

everywhere. Hence fxy = fyx Thus  ex dz 2 2 125


  5    10  cm / s
x
sin y = e sin y = 0 for all y which is dt 8 9 36

false
11. (A)

8. (A) f k
Let k (x, y) = , then 0
x x
2x
fx = , then k (x, y) = g(y).
x  y2
2

f
Hence = g(y) then
fxx 
x 2
 y2   2   2x  2x   2  y  x 
2 2
x

x  y  x  y 
2 2
2 2 2 2
f(x,y) = xg(y) + h(y) for a suitable
function h

299
Vidyalankar : GATE – Engineering Mathematics

12. (A) 14. (B)


If z = x f(y/x) then A normal vector to tangent plane will

x
z
y
z 
be (2x, 2y, 2z) = 1,1, 2 
x y
Hence equation of tangent plane is
 
 x  xf   y / x  y / x 2  f  y / x  
   1  1  1 
 x 2    y  2   2 z  0
xyf   y / x  1/ x      2

 xf  y / x   z Or x + y + 2z  2

15. (C)
13. (A)
1  dh
let c (x, y) = f (x, y) xy, V = r h
3 dt
2c
then  1  1  0 then V dr v dh
xy  
r dt h dt
c (x, y) = A (x) + B (y)
2 1
 f (x, y) = A (x) + B(y) + xy  rh  2   r 2  3 
3 3

4 
=    5000   2500 
3 
12500
 mm3 / s
3


300
Solutions  Calculus

Model Solution on Test Paper  4


1. (C) 7. (B)
/2 a  sec  cos  
2. (C) A=2 0 a sec  rdrd
3. (D) /2 a  sec  cos  

t2
 0 r 2 
a sec 
d
3
s = 3t   15
2 = a2  0
/2
 sec   cos  2

 sec 2  d
ds
speed 9t 2  t /2

= a2 0  2  cos   d
dt  2

d2 s d2 s 1
 18t1  0 t 5a 2
dt 2 dt 2 18 =
4
d3 s
again;  18  0, hence speed 8. (C)
dt 3
4 x/2
has maxima. Mass =k 0 dx x 3/2
ydy
4
ds
Maximum speed = x/2
dt t  1 4  y2 
18 =k  0
  3 / 2 dx
 2 x
2
 1 1 4
 9   
18
  18 4
k  x3 x4  2
     k
1 1 2  12 64  0 3
=  
36 18
9. (D)
2 1
=
36 Equation of circle r = 2acos 
/2 2a cos 

1
speed units. Mass = k   / 2
d 
0
r 2 dr
36
k /2 2acos
4. (D) =
3  / 2
d r 3 
0

5. (C)
8a 3k  / 2
3 0
2 cos3 d
6. (B)
2 2x 2  4x  7 32 3
A =  
1 3x 2  x  3
dx dy =
9
ka

 
2
= 5  dx,  x 2  x  2
1

2
 x3 x2  45
= 5    2x  =
 3 2  1 2

301
Vidyalankar : GATE – Engineering Mathematics

10. (A) dy a2

/2 a cos  dx 2 x  a  x 3 / 2
v=  / 2 d0 a 2  r 2 rdr
 a2 x 
Putt 2  a 2  r 2  fromcurvey 2
 
 ax
a sin 
a sin  2 t3 
=  t dt     ax
a 3 a V = a2 
0
x
dx

1 3
= a 1  sin3   Put x = a sin2 
3 
/2 2 a 3
11. (D)
V = 2a3  0
cos2  d 
2
2 1 x 2
1  x 
1
14. (D)
1 dx  2
v =  y 2 / 4 dy
2 1 x 2
 ds
2 1 x2
S = 2  yds  2  y d
1  y3 
 d
=   1 x2 y  
1

12 2
 ds
 
1 x2
 4  y d
0 d
8
 3 1  x 
1 3/2
2
= dx   1/ 2
ds  dx   dy  
1 2 2

        2a cos   2
d  d   d  
12. (B)
ax   
0
v =   y 2 dx   x 2
0
dx S = 4  2a sin2 2a cos d
a a ax
0 2 2

put z = a  x so 32a 2
= .
3
 a  z   2a  z 
2
2a
v= 0 z
dz
15. (C)
2a  2a 3 
    5a 2  4az  z2  dz 4 0
0
 z  V = 2
2 x  x  2   8
 5  x  dx dy
= 2a3 [log2  2/3] 4
= 2 2  5  x  8  x  x  2 dx
13. (A) 4
2 40  18x  3x  x 3  dx
2
= 2

V = 2    a  x  dy
2
0
= 2  216 = 432 
a dy
 2   a  x 
2
dx
0 dx


302
Solutions  Calculus

Model Solution on Test Paper  5


1. (D) 2  3 1
 f  (e) =  3 0
1 1 e3 e
Let f(x) = x +  f  (x) = 1  2
x x f(x) is maximum at x = e
 f   x   2 / x 3
loge 1
maximum value f (e) = 
e e
For maxima and minima f  (x) = 0
1
 1 =0x= 1 4. (A)
x2
2  x  x2
f   x   3  2  0 Total SP = x  3   3x
 1000  1000
1
f(x) is minimum at x = 1 x
Total CP =  200
2
1
minimum value = f (1) = 1 + =2
1 Profit,f(x)SP CP

5 x2
 x  200
2. (B) 2 1000
Let  be semi vertical angle of 5 2x
 f '(x) = 
inscribed triangle 2 1000

Let S be the area 2


f ''(x) 0
1 1000
Then s =  2x sin 2  (r + r cos 2)
2 5 2x
  0 x1250
dS 2 1000
= 2r2 (cos 4 + cos 23) = 0
d 1
 f''(1250) negative
  = /6 500

S is maximum at  =  / 6 So, profit is maximum when x = 1250

vertical angle = 2 = /3


5. (A)

3. (B) By implicit differentiation w.r.t. x


z z
1  log x y y x z 0
f  (x) = x x
x2
z z y  z
 1  log x = 0  x = e   x  y   (y  z) 
x x y  x
2log x  3
f  (x) =
x3

303
Vidyalankar : GATE – Engineering Mathematics

6. (A) 10. (C)


By Cauchy’s mean value theorem, y
2
4 2

f b   f  a  f  c 
I=  x dxdy   2
dy 
y2 / 4
xdx

g  b   g  a  g  c  y / 22
4  x2 
Choosing f(x) = e , g(x) = c x x = 2  2  2
dy
y /4

eb  ea ec ab
 b a
 c
c 1  y3 y5  72
4
e e e 2 =   y 2  4y   
2  12 80  2 5

7. (B)
11. (D)
f   x  = F  (x)
b b

let (x) = f(x)  F(x)


I= 0 0 y  2b  y dx dy
(x) = f  (x)  F  (x) = 0 b  2 y3 
b

 (x) = constant
= 0 by  3  dx
0

b  3 b3  4b4
8. (B)
= 0 
 b  
3 
dx 
3

9. (B) 12. (B)


/4 2 sin 
 r 4 cos drd I=2 0  r 3 drd

 a 1 cos   4 /4


 0 cos 0 r drd =8 0 sin4 d

a5 
1  cos 2 
2

0 1  cos 
5
= cos d 8
 
d
5  4
32a 5     /2
 0 1  cos  d ....    2
2
 cos10  2 cos2  1 d =
5 0 2 2 

64 5  / 2  1   3
= a   2cos12   cos10  d =  2    2
5 0  2 2  

 
…   
2 
21 5
= a (by reduction formula)
16

304
Solutions  Calculus

13. (C) 15. (A)


 a 1 cos   a 2a  x
I= 0 0 2r 4 sin3  dr d 0
dx  2
x /a
xy dy

2a 5  1
a
 x4 
 1  cos  
5

2 0
= sin3  d  x dx  (2a  x)2
 
5 0
 a2 
2a 5
  2z 
2
1   x5 
6 a
=  z7 dz

2 3
5 0   4a x  4ax  x  2  dx
20 a 
….(z = 1 + cos)
1 4 1 a4 
2 6   2a 4  a 4  a 4  
= a 5 2 3 4 6
35
3a 4

14. (D) 8
x2
y
x log  x  a   y 2 
a
a a
I= 0
dx
x  a  2 
2   (a, a)
2ax  x 2

y=2ax
1 a x log  x  a  2
=  dx  a  2ax  x 2 
x  a 
2
2 0

Integrating with parts, we get


1 a
x log  x  a  dx
2 0
=

a2
= 2loga  1
8


305
Vidyalankar : GATE – Engineering Mathematics

Model Solution on Test Paper  6


1. (C) 7. (C)
2 4  2y
1
2. (A)
I=  
0 0 2y  y 2
dxdy

4  2y
3. (A)  2
x 
=   dy
0 
2
1  2y  y
lim  x  a  cos
0
x a x a 2
2 2y
1
  y 2y
dy
 limhcos [x  a  h] 0
h 0 h
2
= 0  some finite a + y = 0 = 2  2y1/ 2   4
0

8. (C)
4. (B)
3 2
V=   3x  4y  dA     3x  4y  dxdy
5. (D) R 0 1

3 2
3 2 
6. (B) = 0  2 x  4yx 1 dy
1 5x

  y dydx
2
I= 9
3
 9  63
3

0 2x =    4y  dy   y  2y 2  
0
2  2 0 2
5x
1 
1 1
1
3  2x 30

=  y 3  dx   125x 3  8x 3 dx  9. (B)
0
2 4

 y dA    y dydx
2 2
1 V=
39 4  39
= x   R 0 0
4 0 4
2 4 2
1  64
   y 3  dx   dx
0
3 0 0
3
5
64 128
= 2 
3 3

10. (C)
1 3  2x
A  1dA    dydx
1 R 1/ 2 1/ x

1
 1
   3  2x  x  dx
1/ 2

306
Solutions  Calculus

1
=  3x  x 2   nx 
1/ 2 13. (C)
3 1 1 a cos 2 
  3  1 0      n 
/4
I=  d     r 2 
2 4 2 /4  0

 
/4
5 
= 2     n2 
4
  / 4
d a 1  cos 2  a
 
/4
 2 cos   1d
3
=   n2
=a  / 4  
4  
= a  2 sin   
 
11. (A)
 
 a  2   / 2   2a 1  
V   2x  2y  1 dA
R
 4
2 x
    2x  2y  1 dydx
0 0
14. (A)
1 x 2x y

2
x
I=  0
dx  2 dy 
x 0
dz
  2xy  y  y  dx 2

dx  2 dy  z 0
0 1 x 2x y
0 =  0 x
2
 1  1 x
=  x3  x2  =  dx  2  2  x  y  dy
 2 0 0 x

= 8  2 = 10 
1 x x4 
= 0   2  x  x    2  x  x 2
 dx
2 2

12. (B) 11
=
2  1 +cos 30
A=  dA    rdrd
R 0 15. (C)
1 cos 
x 
2 2 1
1 2
 dy  2
I=  y 2 dx
  2r
0

0
d 1 y 1

y
2  x3 


1 = dy   y 2 x 
=
2 0

1  2 cos   cos2  d  1
 3  y 1

1  y  1 
3

1  1  cos 2  2
   1  2 cos    d =    y2   y 2  y  1  dy
2  2 
1 3 3 
 
1  3  3 2
=  2   y 2y 3  y  14 y 4  2
2  2  2 =     =
 3 3 12 4 3
1


307
Solutions  Probability and Statistics
Answer Key on Assignment  1
1. (C) 2. 0.65 to 0.68 3. (C)

4. 0.4 to 0.4 5. (D) 6. 0.25

7. (A) 8. 0.07 to 0.08 9. (A)

10. 0.55 to 0.55 11. (D) 12. (B)

13. 0.43 to 0.45 14. 0.26 to 0.27 15. (B)

16. 1.06 to 1.07 17. (B) 18. 6

19. 0.32 to 0.34 20. (A)

Answer Key on Assignment  2


1. 0.64 to 0.66 2. 0.1276 to 0.1372 3. 0.79 to 0.81

4. (B) 5. 0.96 to 0.98 6. 0.23 to 0.25

7. 99.6 to 99.8 8. (A) 9. 2.5 to 2.5

10. (C) 11. 0.25 to 0.27 12. (B)

13. (B) 14. 0.25 to 0.28 15. (D)

16. (B) 17. (D) 18. 0.39 to 0.43

19. 54.0 to 54.0 20. 0.33 to 0.34

Answer Key on Assignment  3


1. 49 to 51 2. 49.9 to 50.1 3. (B)

4. (B) 5. (C) 6. (C)

7. 0.9 to 1.1 8. 54.49 to 54.51 9. 0.5 to 0.5

10. (B) 11. (A) 12. (D)

13. 0.35 to 0.45 14. (B) 15. (D)

16. 4 17. 6 18. (A)

19. (A) 20. 2.0 to 2.0

308
Solutions  Probability and Statistics

Answer Key on Assignment  4


1. (A) 2. (D) 3. (B)

4. (C) 5. (A) 6. (C)

7. (C) 8. (A) 9. (B)

10. (D) 11. (C) 12. (B)

13. (A) 14. (C) 15. (A)

16. (B) 17. (C) 18. (C)

Answer Key on Assignment  5

1. (A) 2. (C) 3. (D)

4. (A) 5. (D) 6. (C)

7. (A) 8. (C) 9. (D)

10. (A) 11. (D) 12. (A)

13. (A) 14. (A) 15. (C)

16. (D) 17. (C) 18. (C)

Answer Key on Assignment  6


1. (B) 2. (D) 3. (C)

4. (C) 5. (A) 6. (C)

7. (C) 8. (B) 9. (A)

10. (D) 11. (C) 12. (A)

13. (B) 14. (A) 15. (A)

16. (B) 17. (D) 18. (D)



309
Vidyalankar : GATE – Engineering Mathematics

Model Solution on Assignment  1


1. (C) Alternative
We know that mode is the value of the Suppose there are total 10 families.
data which occurred most of 10

 17 is mode.
One child per  5 5 Two child
2. 0.65 to 0.68 family per family
N Number of children in Number of children in
If there are N families, then have these 5 families is 5 these 5 families is 10
2
N  Total children = 15
single child per family and have two
2 10 2
Required probability = = = 0.67
children per family. 15 3
N 3. (C)
1 1
P(Head) =  P(Tail) =
Single child N N Two children 2 2
per family 2 2 per family
P(Fourth Head in tenth toss)
3 6
9  1  1 1
= C3       0.082
N N 2 2 2
 Total children = 2 =N
2 2
N 4. 0.4 to 0.4
 Total number of children = N
2
Given, probability density function of
3
= N evaporation E is
2
1
If a child picked at random should  , 0E5mm / day
f(E) =  5
have a sibling, then that child should  0, otherwise
come from the family which has 2
As, the probability has to be find
child. There are N such children.
between 2 and 4 mm/day, therefore
N 2
 Required probability = = 4 4 1
3
N
3 P(2 < E < 4) = 
2
f(E)dE = 
2 5
dE
2
1 4 4 2 2
= 0.67 = E 2 = =
5 5 5
 P(2 < E < 4) = 0.4

310
Solutions  Probability and Statistics

5. (D) a b 2
 + =
We have Probability distribution 2 3 3
function of Normal Distribution  3a + 2b = 4 ….(2)
(x  )2 Solving (1) & (2) we get, a = 0, b = 2
1 2 2
f(x) = e … (1) 0.5 0.5
 x P(X < 0.5) = 
0
f(x) dx = 
0
2x dx
2
Variance =  is lowest
0.5
  also lowest  x2 
=  2   = (0.5)2 = 0.25
 2 0
 If  decreases  f(x) increases
( from (1))
 Curve will have highest peak 7. (A)

8. 0.07 to 0.08
6. 0.25
P(Head) = 0.3  P(Tail) = 0.7
As f(x) is a pdf,
P (getting Head first time in fifth toss)
1
= 0.7  0.7  0.7  0.7  0.3
 f(x) dx = 1
0
= 0.072
1
 
0
(a + bx) dx = 1 9. (A)
P(X) = 0.40
1
 x2  P(X  Yc) = 0.7
  ax  b  = 1
 2 0  X and Y are independent events

b P(X  Y) = P(X) . P(Y) ….(1)


a+ =1
2 c
 X and Y are also independent
 2a + b = 2 ….(1) P(X  Yc) = P(X) . P(Yc) ….(2)
2 Now,
Also, E(X) =
3 P(X  Yc) = P(X) + P(Yc)  P(X  Yc)
1
2 P(X  Yc) = P(X) + P(Yc)  P(X) P(Yc)
  x f(x) dx =
3
0  0.7 = 0.4 + P(Yc)  0.4 P(Yc)
1
2  0.7  0.4 = P(Yc) [1  0.4]
 0
x (a + bx) dx =
3  0.3 = 0.6 P(Yc)

 x2
1 0.3
x3  2 P(Yc) = 0.5
 a  b  = 0.6
 2 3 0 3
 P(Yc) = 1  P(Y)

311
Vidyalankar : GATE – Engineering Mathematics

 0.5 = 1  P(Y) E(x2) = x 2


P(x)
 P(Y) = 1  0.5 = 0.5
= 1  0.3 + 4  0.6 + 9  0.1
Then,
= 0.3 + 2.4 + 0.9 = 3.6
P(X  Y) = P(X) + P(Y)  P(X  Y)
Variance v(x) = E(x2)  2
= P(X) + P(Y)  P(X) P(Y)
= 3.6  (1.8)2
= 0.4 + 0.5  0.4 × 0.5
S.D() =  v(x)
 P(X  Y) = 0.7
=  3.6(1.8)2 = 0.36
10. 0.55 to 0.55
= 0.6
There are equal number of bulbs of
two different types. 12. (B)
Probability that the bulb selected is of Given  = 5.2
1 Let x be random variable which follows
Type 1 P(T1 )   0.5
2 Poission's distribution
Probability that the bulb selected is of P(x < 2) = P(x = 0) + P(x = 1)
1 e  0 e  1
type 2 P  T2    0.5 =  
2 0! 1!
Let E be the event that a bulb selected = e5.2 (6.2)
at random lasts more than 100 hours.
= 0.0055 × 6.2 = 0.034
P(E/T1) = 0.7
P(E/T2) = 0.4 13. 0.43 to 0.45
By total probability Parcels pass sequentially through two
P(E) = P(T1) P(E/T1) + P(T2) P(E/T2) post offices.
= 0.5  0.7 + 0.5  0.4 Probability that parcel reaches first
 P(E) = 0.55 post office
P(01) = 1
11. (D)
Probability that parcel is lost by first
Given
post office
x 1 2 3
p(x) 0.3 0.6 0.1 1
P(L/01) =
5
mean () = exp(x) Parcel will reach second post office if it
= 1  0.3 + 2  0.6 + 3  0.1 is not lost at first post office.
= 0.3 + 1.2 + 0.3 = 1.8 Probability that parcel reaches second
post office

312
Solutions  Probability and Statistics

4 (out of 100) are considered, of the 5


P(02) =
5 students get more than 90%
Probability that parcel is lost by  5 out of 20 is the probability
second post office Or 1/4 is the answer.
1 Alternative Method:
P(L/02) =
5 Let the student pass the examination
By Total Probability, be A and student pass the examination
Probability that parcel is lost and got about 90% marks be B
P(L) = P(01) P(L/01) + P(02) P(L/02) Now P(A) = 20% and P(A  B) = 5%
1 4 1 9  B  P(AB) 5% 1
= 1   =  P    
5 5 5 25
A P(A) 20% 4
By Baye's Theorem, knowing that the
16. 1.06 to 1.07
parcel is lost, probability that it was
2
lost by second post office Mean = 0 x f(x) dx

P(02)P(L 02) 4 25 4
P(02/L) =  
P(L) 9 25 9 1  x3 x5 
2

= 4  
P(02/L) = 0.444 4  3 5 0

14. 0.26 to 0.27 1  32 32 


=  = 1.0666
4  3 5 
Given, mean  = 5
The probability that there will be less 17. (B)
than 4 penalties in a day is
P(y < 4) = P(y = 0) + P(y = 1)
+ P(y = 2) + P(y = 3)
e5 50 e 5 51 e5 52 e5 53
=   
0! 1! 2! 3!

 50 51 52 53 
= e5     
 0! 1! 2! 3! 
= 0.006737 [1 + 5 + 12.5 + 20.833]
= 0.006737  39.33 = 0.2649
 Required probability
15. (B) 1 2 1 3 1 4 1
=      
6 6 6 6 6 6 6
Given that the student is passing
15 5
exam, i.e., if only 20 students  
36 12

313
Vidyalankar : GATE – Engineering Mathematics

18. 6 I) Red ball in first draw


2 2
Var(x) = E(x )  (E(x)) 5
P(red in first draw) =
 1 12
E(x) =  
x. | x | e|x| dx
2 When the red ball is placed back with
1 0 2 x 1  another red ball, there will be total 13
=  x e dx  x 2 .e x dx
2  2 0 balls with 6 balls of red colour.
=0 6
P(red/red in first draw) =
 1 13
E(x)2 =  
x 2 . | x |e |x| dx
2 II) Green Ball in first draw
1   7
= 2   x 2 .x.e  x dx  P(green in first draw) =
2 0
 12
 When the green ball is placed back
= 
0
x 3 .e x dx
with another green ball, there will be

x
e  total 13 balls with 5 balls of red colour.
= x 3 . (3x 2 )(e x )
1 0 0
5
P(red/green in first draw) =
  13
6x(e x ) 6e  x
0 0
5 6 7 5
=6  P(red) =   
12 13 12 13
 Var(x) = 6  0 = 6
30 35
= 
156 156
19. 0.32 to 0.34 65
 P(red) =
156
20. (A)
5 red and 7 green balls
Total 12 balls.
A ball is drawn and placed back along
with another ball of same colour, this
can be done in two ways:



314
Solutions  Probability and Statistics

Model Solution on Assignment  2


1. 0.64 to 0.66 1 2
Put x =y  x dx = dy
Let M  men 2

W  women 1
 E( X ) = 2  0 2
e  y dy
E  employed
U  unemployed 1
=2 (1)
Given P(M) = 0.5 2
P(W) = 0.5 = 0.7979

P U M  = 0.20
4. (B)
P U W  = 0.50 The probability of getting 'tail' when the

By Total probability, 1
coin is tossed again is
2
 U M P(W)P
P(U) = P(M)P  U M 
5. 0.96 to 0.98
= 0.5 × 0.20 + 0.5 × 0.50 = 0.35
P(at least one will meet specification)
Required probability = P(E)
= 1  P(none will meet specification)
= 1  P(U)
= 1  (1  0.8) x (1  0.7) x (1  0.5)
= 1  0.35 = 0.65
= 1  0.2 x 0.3 x 0.5
2. 0.1276 to 0.1372
= 1  0.03 = 0.97
By Poission's distribution
6. 0.23 to 0.25
p(h≥8) = e8
E(X) = (0q) + (1p) = p
900
= = 0.25
3600 E(X2) = (02q) + (12p) = p

p(h≥8) = e80.25 0.1354 V(X) = E(X2)  [E(X)]2


= p  p2 = p(1  p) = pq
3. 0.79 to 0.81
= 0.60.4 = 0.24

1

2
E( X ) = x e x /2
dx
 2 7. 99.6 to 99.8

1 2 For SN distribution, percentage of area
= 2   | x | e  x /2
dx
0 2 from 3 to 3 is 99.74

….(even function)
8. (A)

1 2
=2 0 x e x 9. 2.5 to 2.5
/2
dx
2

315
Vidyalankar : GATE – Engineering Mathematics

10. (C) Here, n  3 = 1 is not possible


P(p < 3) = P(p = 0) + P(p = 1) Taken n=3
+ P(p = 2)  S = {HHH, HHT, HTH, HTT, THH,
     
e  e  e  THT, TTH, TTT}
=  
0! 1! 2! and X = 3, 1, 1, 2
(where µ = 3) Here n  3 = 0 is not possible
3
e 9 Similarly, if a coin is tossed n times
= e  3  e 3  3 
2 then the difference between heads
 9  17 and tails is n  3 is not possible.
= e 3  1  3    3
 2  2e
 Required probability is 0.
11. 0.25 to 0.27
14. 0.25 to 0.28
2 1
p= = Given: Poisson distribution,
6 3
1 2 e  x
q = 1 = p(x) = ; x = 0, 1, 2, ...
3 3 x!

Using Binomial distribution  = 240 per hour = 4 per minute

p(x ≥ 2) = 2 per 30 seconds


2 1 3 0 e 1
 1 2  1 2 P(X = 1) = = 2  e  2 = 0.2706
= 3 C2 3   3C3 3   1!
  3   3
15. (D)
6 1 7
=  =
27 27 27 Player A Starts the game
Player A can with the game in the
12. (B)
following manner :
Variance = E[X2]  [E[X]]2
A wins by getting 6 in the first try)
We know that, variance  0
A loses, B loses, A wins
 E[X2]  [E[X]]2  0
A loses, B loses, A loses, B loses, A
E[X2]  [E[X]]2
wins and so on,
P(A winning)
13. (B)
1 5 5 1 5 5 5 5 1
Let X = difference between the number =          .....
6 6 6 6 6 6 6 6 6
of heads and tails.
1  25 25 25 
= 1    ....
Taken x=2 6  36 36 36 
 S = {HH, HT, TH, TT} Which forms a G.P.
and X = 2, 0, 2

316
Solutions  Probability and Statistics

 P(A winning) From above table,

  P{X + Y + Z}  P < Z  B
1  1   9  P{Z  1} = P{X = 0 and Y = 1}
=   ....  Sn 
6  25   1  r 
1 + P{X = 1 and Y = 0}
 36 
+ P{X = 1 and Y = 1}
1 1
=  = 1  P{X = 0 and Y = 0}
6 11
36 = 1  pq
6
 P(A winning) =
11 18. 0.39 to 0.43
Use Binomial distribution:
16. (B) p(x) = nCx px q n  x
Use Binomial distribution: Here n = 5, p = 0.1, q = 0.9
n x nx
p(x) = Cx p q P(packet is replaced)
Here n = 10, p = 0.1, q = 0.9 = P(X  1) = 1  p(0)
P(3 out of 10 are defective)
= 1  qn = 1  (0.9)5
10 3 7
= C3(0.1) (0.9) = 0.0574
= 0.40951

17. (D) 19. 54.0 to 54.0


P{x = 0} = P  P{x = 1} = 1  p E(X) = 5
P{y = 0} = q  P{y = 1} = 1  q  E(X2) = 30, where X  P(),  = 5
Let Z = X + Y  E[(X + 2)2] = E(X2) + 4E(X) + 4

X Y Z = 30 + 20 + 4 = 54

0 0 0 ( V(X) = E(X2)  (E(X))2)

0 1 1
20. 0.33 to 0.34
1 0 1
1 1 2



317
Vidyalankar : GATE – Engineering Mathematics

Model Solution on Assignment  3


1. 49 to 51 P(only one) = P(Ram)  P(not Ramesh)
Given M = 500 + P(Ramesh)  P(not Ram)
 = 50 1 7 1 5
=   
P(X > 500) = ? 6 8 8 6
12 1
= =
48 4

4. (B)

where X follows normal distribution 5. (C)

We know that standard normal 1


P(XY) 12 1
variable P(Y/X) = = 
P(X) 1 3
x M 4
z=

6. (C)
500500
z= =0 Here A and B are two independent
50
events
 (X > 500) = P(z > 0) = 0.50
 P(A  B) = P(A) P(B)
(see figure)
and P(A/B) = P(A)
2. 49.9 to 50.1 and P(B/A) = P(B)
Let X be a positive odd number less Also if A and B are independent event
than 100 then A and B are also independent
 X = 1, 3, 5, ..., 99 each with event.

probability
1
50
  
P AB = P(A)P(B)

Hence option (A), (B) and (D) are


1
 E(X) = (1 + 3 + 5 + ... + 99)
50 correct.

1 Now,
= 10025 = 50
50 P(A  B) = P(A) + P(B)  P(A  B)

3. (B) = P(A) + P(B)  P(A) P(B)

1 1  Option (C) is false.


P(Ram) = ; P(Ramesh) =
6 8
7. 0.9 to 1.1

318
Solutions  Probability and Statistics

8. 54.49 to 54.51 W ~ N (0, 2) ie., W has mean  = 0


Observed speeds (in km/hr) : 66, 62, and variance, 2  2
45, 79, 32, 51, 56, 60, 53 and 49.
W 0
 p(W  0) = p  
Number of observations = 10    
 Median will be the mean of middle = p(Z  0), Z is standard normal variants
two values in the ordered set.
1
= 0.5 =
Arranging in ascending order : 32, 45, 2
49, 51, 53, 56, 60, 62, 66, 79 11. (A)
53  56 Let 'x' be no. of defective pieces.
 Median Speed = = 54.5
2
x 0 1 2
9. 0.5 to 0.5 1 2 1
P(x)
1 6 3 6
Given: f(x) = ;a<x<1
x2 Mean () = E(x) = (x) P(x)
1
 1  1  1

a
f(x) dx = 1 =  0  1  2 
 6  3  6
1 2 1
= 0  = 1
 a x  2 dx = 1 3 3

1
E(x2) =  x2 P(x)
x  1
   =1  1  2   1
 1  a =  0  1  4 
 6  3  6
1 1 1 2 2 4
 1 + =1  =2 a= = 0  =
a a 2 3 3 3
10. (B) Variance, V(x) = E(x2)  (E(x))2
p(3V  2U) = p(3V  2  0) = p(W  0), 4 1
= 1 =
W = 3V  2U 3 3

U, V are independent random 12. (D)

 1 Re d Black
variables and U ~ N  0,  Given,
 4 4 6

 1 The selection will be RBB or BBR of


V ~ N  0,  
 9 BRB

 1 1 Probability of selecting RBB


 W = 3V  2U ~ N  0,9   4  
 4 9  4 6 5
=  
10 9 8

319
Vidyalankar : GATE – Engineering Mathematics

Probability of selecting BBR 16. 4


6 5 4 
=  
10 9 8
E(e 3X/4) = 0 e 3x / 4 e  x dx

Probability of selecting BRB 

=
6 4 5
 
= 0 e  x / 4 dx = 4
10 9 8
P(Red = 1) = sum of above three
17. 6
probabilities = 0.5
Let S: getting 3 on a die
13. 0.35 to 0.45 1 5
p = P(S) = , q = 1  p =
P(0.5 < x < 5) 6 6
X = number of times die is thrown to obtain
5
S for the first time
=  f(x)dx
0.5
X 1 2 3 4 ….
1 4 5
p(X) p qp q2p q3p ….
=  0.2 dx  1 0.1 dx  4 0 dx
0.5
E(X) =  x (qx  1 p) = p  x q x  1
= 0.2[x]10.5  0.1[x]14  0 1 1
=p = =6
1  q 
2
= 0.2  0.5 + 0.1  3 p

 P(0.5 < x < 5) = 0.4


18. (A)
14. (B)
1
n = 4; p = 19. (A)
6
4
1 5
 q = 1 = P(X  4) =  0.25 dx
6 6 1

p(x  2) = 0.25(4  1) = 0.75


= 1  p(x < 2)
= 1  [p(x = 0) + p(x = 1)] 20. 2.0 to 2.0
Given X(t) = U + Vt
 0
 1  5 
4
 1  5  
1 3
= 1  4 C0      4 C1      X(2) = U + 2V
 6 6  6   6   E[X(2)] = E[U + 2V]

19 = E(U) + 2E[V] = 0 + 2 × 1 = 2

144

15. (D)


320
Solutions  Probability and Statistics

Model Solution on Assignment  4


1. (A) 5. (A)
The possible cases in the throw of a Sample space = 6
die are six i.e. 1, 2, 3, 4, 5, 6 4, 5, 6 are greater than 3
Cases of even numbers 2, 4, 6 are 3 1
 Probability = 
three. 6 2
 Probability of getting an even 6. (C)
3 1 Sample space = 62 = 36
number = 
6 2 Favourable cases are (5, 6), (6, 5),
(6, 6)
2. (D)
When we toss a coin, there are two 3 1
Probability = 
36 12
possible outcomes i.e. Head or Tail.
7. (C)
 Number of possible cases = 2
There are 366 days in a leap year and it
The outcome of Tail is favourable
has 52 complete weeks and 2 days
event
extra.
 Probability of getting a tail = 1/2
They are
3. (B) (1) Sunday and Monday
Total number of equally likely and (2) Monday and Tuesday
exhaustive cases = n = 6 + 9 = 15 (3) Tuesday and Wednesday
(4) Wednesday and Thursday
Number of favourable cases
(5) Thursday and Friday
= 9 C1  9
(6) Friday and Saturday
[ number of black balls = 9]
(7) Saturday and Sunday
Probability of drawing a black ball
sample space = 7
9 3
=  Number of favourable cases = 2
15 5
 Probability = 2/7
4. (C) 8. (A)
Number of clubs in a pack = 13 In a pack of 52 cards, 1 card can be
13 1 drawn in 52 ways
Probability of getting a club = 
52 4
Since there are 13 spades and 3 aces
(one ace is present in spade)

321
Vidyalankar : GATE – Engineering Mathematics

 Number of favourable case Let A be the event that the flower is


= 13 + 3 = 16 red and B the event that the flower is a
Sample space = 52 rose.
Probability of getting a spade or an ace  A  B is the event that the flower is
16 4 4 a red rose.
= = 
52 13 9  4 16
n(A) = 16  P(A) =
Odds against winning the bet are 100
9 to 4. 10
n(A  B) = 10  P (A  B) =
100
9. (B)
P(B/A) = probability that a selected
Events are mutually exclusive
flower is a rose red is colour
odd in favour of horse H1 = 1/3
P  A  B 10 / 100 5
1 1 P(B/A) =  
P(H1) =  PA 16 / 100 8
1 3 4
1 1 11. (C)
Similarly P(H2) =  ,
1 4 5 A throw amounting to 18 must be
1 1 made up of 6, 6, 6 and this can occur
P H3   
1 5 6 in 1 way. 17 can be made up of 6, 6, 5
1 1 which can occur in 3 ways. 16 may be
P(H4) = 
1 6 7 made up of 6, 6, 5 and 6, 5, 5 each of
P H1  H2  H3  H4   P H1   P H2  which arrangements can occur in 3

P H3   P H4  ways.


 The number of favourable cases is
1 1 1 1
= + + + = 1 + 3 + 3 + 3 = 10 and the total
4 5 6 7
number of cases is 63 = 216
319
=
420 10 5
 Required chance = 
216 108
10. (D)
Suppose there are 100 flowers
12. (B)
Number of roses = 40; Number of
Probability that A wins (P1) = 1/6
carnations = 60
and that B wins (P2) = 1/10
25% of 40 = 10 roses are red and
and that C wins (P3) = 1/8
10% of 60 = 6 carnations are red
As a dead heat is impossible, these
are mutually exclusive events, so the

322
Solutions  Probability and Statistics

chance that one of them will win the The probability that wife is not selected
race is 1 4
=1 
1 1 1 47 5 5
P1  P2  P3 i.e.   
6 10 8 120 Probability that only husband is
1 4 4
13. (A) selected =  
7 5 35
The chance of choosing the first bag is
Probability that only wife is selected
1/2 and if the first bag be chosen the
1 6 6
chance of drawing a red ball from it is =  
5 7 35
5/12 hence the chance of drawing a
Probability that only one of them is
red ball from the first bag is
4 6 10 2
1 5 5 selected =   
  35 35 35 7
2 12 24
16. (B)
Similarly the chance of drawing a red
ball from the second bag is There are 3 mutually exclusive and

1 3 1 exhaustive way in which 2 balls are


  . Hence, as these events
2 15 10 transferred form first bag to second bag.
are mutually exclusive, the chance First Way :
5 1 37 Two white balls are transferred from
required is  
24 10 120 first bag to second bag so that
4
14. (C) C2
probability for that is = 6
. In the
C2
9 cards of a suit can be selected in
13 second bag we have 7 white and 4
C9 ways, 4 cards can be selected
black balls and the probability of
39
from the remaining in C4 ways.
7
getting a white ball is =
The suit can be selected in 4 ways 11
 Required probability  Required probability
13
C9  39 C4  4 4
C2 7 6 7 42
= 52
=    
6 11 15 11 165
C13 C2

15. (A) Second way :


Two black balls have been transferred
The probability that husband is not
from first bag to the second bag so
1 6
selected = 1   2
7 7 C2 1
that probability for that is 6

C2 15

323
Vidyalankar : GATE – Engineering Mathematics

In the second bag we have five white Number of cases with a total of 9 = 4
and 6 black balls and probability of  5,4  ,  4,5  ,  3,6 ,  6,3 
5  Total number of cases in which a
getting a white ball is .
11
doublet or a total of 9 appear = 6 + 4
 Required probability
= 10
1 5 5
=   P(a doublet or a total of 9) = P(A)
15 11 165
10 5
Third way : = 
36 18
One black and one white ball have
P(neither a doublet nor a total of 9)
been transferred from first bag to the
= 1  P(A)
second so that the probability for this
5 13
is = 1 
18 18
4
C1  2 C1 8 18. (C)
6

C2 15
Let P(A), P(B) be the probability of A &
In the second bag we have 6 white B speaking the truths, then
and 5 black balls and the probability of 75 3 80 4
P(A) =  , P(B) = 
drawing a white ball is 6/11. 100 4 100 5
 Required probability
 
P A  P ( A tells a lie) = 1  P(A)
8 6 48
=   3 1
15 11 165 = 1 
4 4
Since these three cases are mutually
exclusive,  
P B = P(B tells a lie) = 1  p(B)

 The required probability of drawing 4 1


=1 
a white ball 5 5
42 5 48 95 Now P(A and B will contradict)
=   
165 165 165 165
 
= P  A  P B  P B  P A  
17. (C) 3 1 4 1 7
=      35%
Total number of cases in a throw of 4 5 5 4 20

two dice = 36
Number of cases with doublets = 6

1,1 ,  2,2  ,  3,3  ,  4,4  ,  5,5  ,  6,6 




324
Solutions  Probability and Statistics

Model Solution on Assignment  5


1. (A) 4. (A)
Probability of 1 appearing on upper P(A) and P(B/A) denote the probability
1 of drawing a black ball in the first and
face =
6 second attempt
1  Probability of drawing a black ball in
Probability of 6 appearing =
6 the first attempt is
 The probability 1 or 6 appear 3  3 
P(A) = 5  3
1 1 1 8  
=  
6 6 3
Probability of drawing the second
2. (C)
black ball given the first ball drawn is
Probability of the winning of the horse
black
1
A= and the probability of the horse 2  2 
5 P(B / A)  5  2
7  
1
B= Probability that both the balls drawn
6
are black is
 P(A + B) = P(A) + P(B)
P(AB) = P(A) P(B/A)
1 1 11
=  
5 6 30 3 2 3
=  
8 7 28
3. (D) 5. (D)
First draw : Probability of getting a Probability of getting a head in a toss
4 1 1
queen =  of a coin = ;
52 13 2
Second draw : After drawing the first Probability of getting tail in each case
queen we are left with 51 cards with 3
1
=
queens 2
 Probability of getting a queen in Probability of getting a tail in all four
3 1 1 1 1 1 1
second draw =  cases =    
51 17 2 2 2 2 16
 Probability of both the cards are  Probability of getting at least one
1 1 1 1 15
queen =   head = 1  
13 17 221 16 16

325
Vidyalankar : GATE – Engineering Mathematics

6. (C) 10. (A)


P = probability of getting 1 in a throw Probability that any person selected at
of a die = 1/6 1
random is a rice eater = . Probability
1 5 2
 q=1p=1 
6 6 1
of a non rice eater = . Let A0, A1, A2,
Probability of getting at least one die 2
3 A3 denote the event that none, one,
5 91
= 1  q3 = 1     two or three persons respectively are
6
  216
rice eaters out of 10. Then
7. (A)
0 10
P(A0) = 10C0  1   1  ,
8. (C) 2 2
Let X = defective items. We seek P 1 9
(C/X) the probability that an item is P(A1) = 10C2  1   1 
2 2
produced by machine C, given that the
2 8
item is defective P(A2) = 10C2  1   1  ,
2 2
By Bayes’ theorem,
3 7
P CP  X / C  1  1
P(C/X) = P(A3) = 10C3    
P  A  P  X / A   2 2
 
 P B  P  X / B   Now, the required probability
 
 P  C  P  X / C   = P(A0 + A1 + A2 + A3)
= P(A0) + P(A1) + P(A2) + P(A3)
 0.10  0.04 
= 1
 0.60  0.02   = [10C0 + 10C1 + 10C2 +10C3]
  210
  0.30  0.03  
  1
  0.10  0.04   = [ 1 + 10 + 45 + 120]
210
4
= 176
25 = = 0.17 = 17 % (approx)
1024
9. (D)
11. (D)
Probability of six occurring in one toss
Probability that the first person lives till
= 1/6
8
Probability of six occurring in 180 toss he is 75 years =
14
1
= 180  = 30 Probability that the second person
6
4
lives till he is 80 years =
9

326
Solutions  Probability and Statistics

Probability of the compound event that  Variance = npq


both the persons live 40 years hence = 500  0.1  0.9 = 45
8 4 32 16  Standard deviation = 45 = 6.7
=   
14 9 126 63
Probability that at least one of them 14. (A)
would die without living 40 years  =  pi xi
16 47 144 24 1
hence = 1   =0.  1.  2.
63 63 169 169 169
26 2
12. (A)  
169 13
The probability of getting a head in a
2 =  pixi2  ()2
single toss of a coin is 1 / 2
144 24 1 4
1 1 = 0.  1.  4. 
 p = 1/2, q = 1 p = 1  = 169 169 169 169
2 2
28 4 24
Also we are given n = 10, N = 100 =  
169 169 169
and r =7
24 4.9
The required frequency  =  = 0.377
169 13
= N nCr pr qnr
15. (C)
7 3
10  1  1
= 100  C7     np = 12 npq = 2
2 2
10
 npq = 4
10!  1 
100.
7!3!  2 
npq 4 1 1
  q=
np 12 3 3
10
10  9  8  1  375
= 100   1 2
3  2  1  2  32 p=1q =1 =
3 3
= 11.7 = 12
Also np = 12
2
13. (A) n = 12
3
Let (q + p)n, q + p =1, be the binomial
2
distribution  n = 18 & p=
3
P = 0 1, n = 500
Mean = np = 0.1  500 = 50
Now p = 0.1
 q = 1  p = 1  0.1 = 0.9

327
Vidyalankar : GATE – Engineering Mathematics

16. (D) 18. (C)


The probability that face 1 or face 2 Combinations are CMC, CMM, CCC,
turn up = 0.10 + 0.32 = 0.42 CCM.
The probability that face 1 turns up Probability of
= 0.10 CMC = 0.6  0.4
 The required probability = 0.24  Favourable
0.10 10 5 CMM = 0.6  0.6 = 0.36
=  
0.42 42 21 CCC = 0.4  0.4
= 0.16  Favourable
17. (C)
CCM = 0.4  0.6 = 0.24
Since odds against A are 8 : 3
Total Probability = 1
3 3
P(A) =  Favourable = 0.24 + 0.16 = 0.4
83 11
So P = 0.4/1
2 2
P(B) = 
52 7
P(A) + P(B) + P(C) = 1
{mutually exclusive}
3 2
Or  + P(C) = 1
11 7
3 2 34
 P(C) = 1   
11 7 77
 odds against C are 77  34 = 34
i.e. 43 : 34



328
Solutions  Probability and Statistics

Model Solution on Assignment  6


1. (B) 1,2  , 1,4 , 1,6 ,  2,1 ,  2,3 ,
S = {Sun, Mon, Tue, Wed, Thu, Fri, Sat}  2,5  ,  3,2  ,  3,4  ,  3,6  ,  4,1 ,
A=
n(s) = 7  4,3  ,  4,5  ,  5,2  ,  5,4  ,  5,6  ,
Let A be the event that the child is born  6,1 ,  6,3  ,  6,5 
on Sunday or a Saturday
n(A) = 18
 n(A) = 2
nA  18 1
nA   PA   
 P(A) = 
2 n S 36 2
nS 7
4. (C)
2. (D) 5 students can be selected from 10 by
S = {1, 2, 3, 4, 5, 6 } 10
C5 ways
n(S) = 6
10 10!
Let A be the event that the die shows a  n(S) = C5 =
5!5!
multiple of 2
10 × 9 × 8 × 7 × 6
 A = {2, 4, 6} = = 252
5×4×3×2
 n(A) = 3 Let A be the event that the committee
nA 3 1 includes exactly 2 girls and 3 boys. The
P(A) =  
nS 6 2 4
two girls can be selected in C2 ways
3. (C) and the three boys can be selected in
S= 1,1, 1,2  , 1,3  , 1,4 , 1,5 , 1,6  6
C3 ways

 2,1 ,  2,2  ,  2,3  ,  2,4  ,  2,5  ,  2,6  n(A) = 4 C2 × 6 C3 = 6 × 20 = 120


 3,1 ,  3,2  ,  3,3  ,  3,4  ,  3,5  ,  3,6  nA  120 10
P(A) =  
 4,1 ,  4,2  ,  4,3  ,  4,4  ,  4,5  ,  4,6  nS 252 21

5. (A)
 5,1 ,  5,2  ,  5,3  ,  5,4  ,  5,5  ,  5,6 
Let A be an event that ball drawn is red
 6,1 ,  6,2  ,  6,3  ,  6,4  ,  6,5  ,  6,6  or white
n(S) = 36  n(A) = 7 C1  4 C1  7  4  11
Let A be the event that the score on the
11
dice is odd  PA 
20

[ n  S   20 C1  20 ]

329
Vidyalankar : GATE – Engineering Mathematics

6. (C) 11. (C)


52 1
Two cards can be selected in C2 P(A  B ) = P(A)  P(A  B) =
3
ways
52 P(A  B) = P(A) + P(B)  P(A  B)
 n(S) = C2  26  51
P(A  B) = [P(A)  P(A  B)] + P(B)
Let A be the event that 2 cards selected
P(A  B) = P(A  B ) + P(B)
is Jack and an Ace.
2 1
Jack can be selected in 4 C1 ways and = + P(B)
3 3
the Ace can also be selected in 4 C1 2 1 1
 P(B) =  =
ways. 3 3 3

 n(A) = 4 C1  4 C1  16 12. (A)


1
nA 16 8 Probability P of a white flower =
 PA    4
n  S  26  51 663
1 3
q = 1 P = 1  =
7. (C) 4 4

The required probability n=3 N = 64


r 3 r
 12 C11  (0.99875)11 (0.01125) + 12C12  1  3 
p(r) = 3Cr    
4 4
(0.99875)12 = 0.9923
8. (B) Number of beds with zero white flowers
0 3
Required probability  1  3 
= 64  3 C0    
1 1 4 1 4 4 4
= .  . 
5 3 5 4 15 27
 64  = 27
9. (A) 64
13
C1 4
C1 1 Beds with 1 white flower
P(A) = 52
 51

C1 C1 52 1
 1  3 
2
= 64  3 C1     = 27
10. (D) 4 4

2 Beds with 2 white flowers


Probability of first event = P
3 2
 1  3  9
= 64  3 C2      64  =9
Mutually exclusive events 4 4 64
2 3 Beds with 3 white flowers
P+   P=1 Or P = =3:5
3 5 3 0
 1  3  1
Hence odds in favor of the other are = 64  3 C3      64  =1
4 4 64
3 : (5  3) i.e. 3 : 2

330
Solutions  Probability and Statistics

13. (B) 16. (B)


1 1 1 1 1
P (A  B) = ; P(A)  ; P(B)  P(B) = 1  P( B)  1  
2 3 3 2 2
1 2 and P(A  B) = P(A) + P(B)  P(A  B)
Now, P(A) = 1  P(A)  1  
3 3 5 1 1
or = P(A) + 
1 2 6 2 3
P(B) = 1  P(B)  1  
3 3 2
or P(A)
 P(A  B) = P(A)  P(B)  P(A  B) 3

2 2 1 2 1 1
=    P(A) P(B) =   = P(A  B)
3 3 2 3 2 3
Hence A and B are independent.
4 1 83
  
3 2 6
5 5 2 10 17. (D)
=   
6 6 2 12 n(s) = 24 = 16 since each of the four
places in a determinant of order 2 is to
14. (A)
filled by 0 or 1 favourable number of
To be one step away from the starting
ways is 3.
point the Man is to take 6 steps forward
and 5 steps backward or 5 steps 1 0 1 0 1 1
  or   or  
0 1 1 1 0 1
forward and 6 steps backward
 The required probability 3
Hence P =
= 11C6 (0.4)6 (0.6)5 + 11C5 (0.4)5 (0.6)6 16

= 11C5 (0.4)5 (0.6)5 {0.4 + 0.6}


18. (D)
= 11C5 (0.24)5
100
C50 p50 (1  p)50 = 100
C51 p51 (1  p)49
15. (A) 1 p 100! 50! 50! 50
or   
Required probability p 51! 49! 100! 51

1 2 1 3 1 or 51  51p = 50 p
=    
2 5 2 5 2
51
giving P=
101



331
Vidyalankar : GATE – Engineering Mathematics

Answer Key on Test Paper  1


1. (D) 2. (D) 3. (A)

4. (C) 5. (A) 6. (C)

7. (C) 8. (B) 9. (A)

10. (D) 11. (A) 12. (A)

13. (C) 14. (A) 15. (C)

Answer Key on Test Paper  2


1. (B) 2. (D) 3. (D)

4. (C) 5. (B) 6. (A)

7. (D) 8. (A) 9. (A)

10. (A) 11. (A) 12. (D)

13. (A) 14. (A) 15. (C)

Answer Key on Test Paper  3


1. (D) 2. (A) 3. (D)

4. (B) 5. (A) 6. (D)

7. (B) 8. (B) 9. (D)

10. (B) 11. (D) 12. (A)

13. (C) 14. (B) 15. (D)



332
Solutions  Probability and Statistics

Model Solution on Test Paper  1


1. (D) Let C be the event that both are
2 3 selected
P(A) = P(A) =
5 5  C be the event that both are
2 1 selected.
P(B) = P(B) =
3 3  C =AB
3 2  P(C) = P(A  B)
P(C) = P(C) =
5 5
 P(C) = P(A)  P(B) as A and B are
Probability that only one of them hits
1 2 2
the target independent events =  
5 7 35
= probability that A hits the target but
4. (C)
not B and C + probability that B hits
12
the target but not A and C Two balls are selected from 12 in C2
+ probability that C hits the target ways. Two white balls can be selected
but not A and B in 7 C2 ways.
= P  A  B  C   P  A   B  C  7
C2
 Required probability =
P  A   B  C 
12
C2

2 1 2 2 3 2 3 3 1 21 7
=          
5 3 5 3 5 5 5 5 3 66 22

4 12 9 25 1 5. (A)
    
75 75 75 75 3
Three balls can be selected from 9 in
2. (D) 9
C3 ways
Probability of selecting any bag is 1/2 3 black balls can be selected from 5 in
Probability of getting a white ball 5
C3 ways
6 4
1 C 1 C 5
= 9 1 9 1 5
C3
2 C1 2 C1 9  Required probability = 9
C3

3. (A) 10 5
 
84 42
Let A be the event that A is selected
and B be the event that B is selected
 P(A) = 1/5 and P(B) = 2/7

333
Vidyalankar : GATE – Engineering Mathematics

6. (C) 8. (B)
R stands drawing a red ball and B for 15
c 2 27 c 2
P1 = 42
drawing black ball. Then required c4
probability is 15  14  27  26  1 2  3  4

= RRR + RBR + BRR + BBR 1 2  1 2  42  41 40  39
 6 5 6   6 6 5  27
=        
 10 11 10   10 11 10  82
 4 4 7   4 7 6  30
c 4 54 c 4
         P2 =
 10 11 10   10 11 10  84
c8
6400 32 30.29.28.27.54.53.52.51.8!
=  
1100 55 4!4!84.83.82.81.80.79.78.77
17.29.45.53
=
7. (C) 11.79.82.83
6 1
P1 =  P1 27 11.79.82.83 33.79.83
36 6   
P2 82 17.29.45.53 29.53.85
[ out of total of 36 ways both the
216381
persons can throw equal values in = >1
130645
6 ways]
Hence P1 > P2
To find p2 the total number of ways
n  64 and the favorable number of
9. (A)
ways, M =15  8 = 120
P(AB) = P(A) + P(B)  P(A  B)
Since any two numbers out of 6 can
P(A  B)  1
be selected in 6c2 i.e. 15 ways and
 P(A) + P(B)  P(A  B)  1
corresponding to each of these ways,
P(A) + P(B)  1  P(A  B)
there are 8 ways e.g. corresponding to
P(A  B)  P(A) + P(B)  1
the numbers 1 and 2 the eight ways
are (1, 1, 1, 2), (1, 1, 2, 1), (1, 2, 1, 1),
10. (D)
(2, 1, 1, 1) , (2, 2, 2, 1) (2, 2, 1, 2) (2,
We first find the total number of cases.
1, 2, 2) (1, 2, 2, 2).
For each element of set A with n
120 5
Hence P =  element, the number of possible
64 54
images is n.
1 5
Since  , we have P1 > P2
6 54  The total number of cases = nn.

334
Solutions  Probability and Statistics

When next find the number of There are 6 way to go to city F


favourable cases. For the first element through C.
we have n choices. For the seconds Total no. of paths = 10
element we have (n  1) choices and  Hence, required probability
so on. = 6/10 = 3/5

 The number of favourable cases


15. (C)
= n  n  1 n  2  ...2  1  n!
x
Hence the required probability Mean x =
N
n!  n  1 ! x
=  n1  90 =
nn n 200
 x = 200 × 90 = 18000
11. (A)
 Sum of observations is 18000
12. (A)
Of these, two wrong observations are
13. (C) 15 and 80.

14. (A) Subtracting 15 and 80 from 18000, we


get 17905
Distinct paths from A to F are given
Adding the correct observations to
below.
17905,
1) ABDF
i.e. 17905 + 40 + 87 = 18032
2) ACEF
3) ABF  Correct sum of 200 observation is

4) ABEF 18032

5) ACDF 18032
 Correct mean is = 90.16
200
6) ABCDEF
7) ACDEF
8) ABDEF
9) ABCDF
10) ABCEF



335
Vidyalankar : GATE – Engineering Mathematics

Model Solution on Test Paper  2


1. (B)  The probabilities of Dayanand,
There are 6 possible ways in which the Ramesh, Naresh not solving the
die can fall and of these two are problem are
favourable events required. 1 1 1 2 1 3
1  ;1   ;1  
2 1 2 2 3 3 4 4
 Required chance = 
6 3 respectively.
2. (D)  The probability that the problem is
The possible number of cases is not solved by any one of them is
6  6 = 36 1 2 3 1
  
An ace on one die may be associated 2 3 4 4

with any of the 6 numbers on the other  The probability that the problem
die and the remaining 5 numbers on will be solved by at least one of
the first die may be associated with the 1 3
them = 1  
ace on the second die, thus the 4 4

number of favorable cases is 11. 5. (B)

11 The probability of drawing a black ball


 Required chance =
36 from the bag = 3/8

3. (D) The probability that the drawn ball is

Various Digits in the log table are 0, 1, 3 5


not black = 1 
8 8
2, 3, 4, 5, 6, 7, 8, 9
6. (A)
i.e. a total of 10 digits are used.
In “PROBABILITY” there are
The number of favourable cases for
O A I : 3 distinct vowels
getting 1 out of the 10 all equally likely
P R B L T Y : 6 distinct consonants
cases is one
Total number of distinct letters = 9,
 Probability of getting 1 is 1/10.
3 1
Required probability = 
4. (C) 9 3
The probabilities of Dayanand, 7. (D)
Ramesh and Naresh solving the Let P(A), P( A ) be probabilities of A’s
problem are getting the head and not getting the
1 1 1 1
, , respectively head respectively, then P(A) =
2 3 4 2

336
Solutions  Probability and Statistics

1 1 8. (A)
 P( A ) = 1 P(A) = 1  =
2 2 n(S) = 36
1 1 The outcomes which have odd number
Similarly, P(B) = and P( B ) =
2 2 and multiple of 3 are
Let A start the game. He can win it in (1, 3) (1, 6) (3, 1) (3, 3) (3, 5) (3, 6)
the 1st throw 3rd throw 5th throw, 7th (5, 3) (5, 6) (6, 1) (6, 3) (6, 5)
throw and so on. Probability of A’s favourable cases = 11
1 11
winning in 1st throw = P(A) = Probability =
2 36
Probability of A’s winning in 3rd throw
9. (A)
= P( A ) P( B ) P(A) Let A = The event that students are
3
1 1 1  1 taller than 18m
= . . =  
2 2 2 2 P(w /A) = Probability that a student is
th
Probability of A’s winning in 5 throw a woman given A
= P( A ) P( B ) P( A ) P( B ) P(A) P(w)P(A / w)
=
5 P(w)P(A / w)  P(M)P(A / M)
1 1 1 1 1  1
= . . . . =  
2 2 2 2 2 2 (0.60)(0.01) 3
= 
(0.60)(0.01)  (0.40)(0.04) 11
Probability of A’s winning in 7th throw
= P( A ) P( B ) P( A ) P( B ) P( A )
10. (A)
P( B ) P(A)
n = total number of ways = 6  6 = 36
7
1 1 1 1 1 1 1  1 The numbers higher than 9 are 10, 11,
= . . . . . . =  
2 2 2 2 2 2 2 2
12 in the case of two dice
Mutually exclusive cases
 m = favourable number of ways
 Probability of A’s winning the
=3+2+1=6
game first is
M 6 1
1 Hence P =  
3 5 7 n 36 6
1  1  1  1 2 2
          ....   11. (A)
2 2 2 2 1 3
1
4 Total number of cards =12
 Probability of B winning the game
Probability of at least one are
2 1
first = 1   = 1 probability no are
3 3
8
C2 14 19
= 1 12
 1 
C2 33 33

337
Vidyalankar : GATE – Engineering Mathematics

12. (D) 21 20  19


= = 7  10  19
Var (x) = 1 3  2 1
 Required probability
13. (A) 1045 209
= =
7  10  19 14  19
The probability that event A occurred,
11 19 11
provided that B took place will be = 
14  19 14
denoted byP( A B ). Now the probability
that event A occurred, provided that B
15. (C)
took place will be denoted by
Probability that the item is produced by
P(A/ B ).The event A is the non machine A, P(A) = 0.25
occurrence of the event A. Probability that the item produced by A
 We have P(A/ B ) + P( A / B ) = 1 is faulty, P(F/A) = 0.05
Similarly,
Hence P( A / B ) = 1  P(A / B )
P(B) = 0.30
P(F/B) = 0.04
14. (A)
P(C) = 1  0.25  0.3 = 0.45
Two points are always collinear so we
P(F/C) = 0.03
can say a triangle consists of two
By total probability,
collinear points and one point which is
Probability that the item selected is
not collinear to both of them
faulty
simultaneously.
= P(A) P(F/A) + P(B) P(F/B)
So here, we have no. of triangles
+ P(C) P(F/C)
= No. of triangles having 2 points on
= 0.25 × 0.05 + 0.30 × 0.04 + 0.45
line 1 and one point on line 2
× 0.03
+ No. of triangles having 2 points
= 0.038
on line 2 and one point on line 1
By Baye's Theorem,
10
= C2  11  11C2 = 45  11 + 55  10
Probability that the faulty item was
= 1045 produced by machine C,
Total number of points = 10 + 11 = 21
P(C)F(F / C) 0.450.03
P(C/F) = =
 No. of 3 points group P(F) 0.038
21
= C3  P(C/F) = 0.355



338
Solutions  Probability and Statistics

Model Solution on Test Paper  3


1. (D) 4. (B)
Probability of getting both the balls When all the letters are taken they can
even numbered be arranged in 10p10 ways or 10 !
P(EE) = P(E)P(E)
5. (A)
12 12 144
=   Here the balls are drawn without
25 25 625
replacement. Let P(W) and P(G)
P(at least one odd) = 1  P(EE)
denote the probability of drawing white
144
=1 ball and green ball respectively.
625
P(W and G) = P(W) P(G) + P(G) P(W)
481
 10 8 8 10 80
625 =    
18 17 18 17 153
2. (A)
The total number of ways in which four 6. (D)
persons can be selected out of 9 The largest number on the selected
persons is 9 C4 . coupon is 9, and hence the selection is

For favourable cases, we want that 2 to be made from the coupons

out of the four selected should be numbered 1 to 9. Since there are 15

children. Two children can be selected possible cases for selecting a coupon

4 and seven coupons are selected the


out of 4 in C2 ways. The other two
total number of possible cases = 157
are to be selected out of 5 persons (3
The number of favorable cases
men and 2 women). Two persons can
= 97  87. Observe that out of 97 cases
5
be selected out of 5 in C2 ways.
87 cases do not contain the number 9
 The number of favourable cases The required probability
4
C2  C25
Number of favourable cases
=
Total number of cases
 Required probability
4
C2  5 C2 10 97  87
=  =
9
C4 21 157

3. (D)

 A  P  A  B  1/ 4 3
P    
B P B  1/ 3 4

339
Vidyalankar : GATE – Engineering Mathematics

7. (B) 10. (B)


2 3 Here np + npq = 10 , n = 18
P= q= n=5
5 5 18p + 18pq = 10  18p(1 + q) = 10
5 r r
3 2  p + q = 1  p = 1 q
P(r) = nCr . qn  r pr = 5
Cr   . 
5 5 18(1  q) (1 + q) = 10
 The required chance 10
1  q2 =
= P(4) 18

3 2
4 10 8
= 5 C4 . .    q2 = 1 
5 5 18 18

48 48 48 8 4
= 5 =  q2 = 
55 54 625 18 9
2
q =
8. (B) 3
Total number of ways in which two 2
 q= (ve sign rejected),
cards are drawn out of 52 is 52
C2 . 3

A king can be drawn in 4 ways and 1


p=
3
Queen can be drawn in 4 ways.
Hence Binomial distribution is
44 8
Required probability = 52  18
C2 663  1 2
3  3 {(p +q)n}
 
9. (D)
In a single throw of two dice, the 11. (D)
probability of throwing a doublet is
6 1 12. (A)

36 6
 The probability of throwing three 13. (C)
doublets in three throws is
1 1 1 1 14. (B)
. . 
6 6 6 216
Out of the 5 women, 3 women can be
 Hence the required probability 5
invited in C3 ways. Nothing is
1 215
=1  mentioned about the number of men
216 216
that he has to invite. He can invite one,
two, three, four or even number of

340
Solutions  Probability and Statistics

men. Out of 4 men, he can invite them 1


P(AB) =
4
in the said manner in (2) ways. Thus, 8

P(A B) 18 1
the total number of ways is 5 C3  (24 ) P(A B) = = 
P(B) 12 4
= 10  16 = 160.
1
 P(A B) = is false.
2
15. (D) If A  B , A  B = A
P(A) = 1/4
then P(A  B) = P(A)
P(A B) 1
P(B/A) = = 1 1
P(A) 2 but P(A  B) = and P(A) =
8 4
1  This is false.
 P(A  B) =
8
1 1 3
P(AB) 1 P(A B)P(A B) =  =
P(A/B) = = 4 2 4
P(B) 4
 P(A B)P(A B) = 1 is false
1
 P(B) =  None of the options (A), (B) or (C)
2
is true.
P(AB) = P(A)  P(A  B)

1 1
= 
4 8



341

You might also like